btstudy.com 으로 오세요. 수능/내신 변형, 퀴즈를 무료로 공개합니다.

블루티쳐학원 | 등록번호: 762-94-00693 | 중고등 영어 | 수강료: 30(중등), 33(고등), 3+4(특강)

THE BLUET

728x90
반응형

2062 | Since 2005 임희재 | 블루티쳐학원 | 01033383436 | 200710 22:53:52

 

순서배열

 

1. 2062-18

 

We at the Future Music School have been providing music education to talented children for 10 years.


(A) That's why we want to ask you to perform at the opening event of the festival. It would be an honor for them to watch one of the most famous violinists of all time play at the show. 1

(B) We hold an annual festival to give our students a chance to share their music with the community and we always invite a famous musician to perform in the opening event. Your reputation as a world‒class violinist precedes you and the students consider you the musician who has influenced them the most. 0

(C) It would make the festival more colorful and splendid. We look forward to receiving a positive reply. 2


① A-C-B ② B-A-C ③ B-C-A ④ C-A-B ⑤ C-B-A

 


2. 2062-19

 

It was time for the results of the speech contest.


(A) She tore open the envelope to pull out the winner's name. My hands were now sweating and my heart started pounding really hard and fast. "The winner of the speech contest is Josh Brown" the announcer declared. As I realized my name had been called, I jumped with joy. 1

(B) "I can't believe it. I did it" I exclaimed. I felt like I was in heaven. Almost everybody gathered around me and started congratulating me for my victory. 2

(C) I was still skeptical whether I would win a prize or not. My hands were trembling due to the anxiety. I thought to myself, 'Did I work hard enough to outperform the other participants?' After a long wait, an envelope was handed to the announcer. 0


① A-C-B ② B-A-C ③ B-C-A ④ C-A-B ⑤ C-B-A

 


3. 2062-20

 

We all have set patterns in life.


(A) We like to label ourselves as this or that and are quite proud of our opinions and beliefs. We all like to read a particular newspaper, watch the same sorts of TV programs or movies, go to the same sort of shops every time, eat the sort of food that suits us, and wear the same type of clothes. 0

(B) You have to see life as a series of adventures. Each adventure is a chance to have fun, learn something, explore the world, expand your circle of friends and experience, and broaden your horizons. Shutting down to adventure means exactly that ―you are shut down. 2

(C) And all this is fine. But if we cut ourselves off from all other possibilities, we become boring, rigid, hardened― and thus likely to get knocked about a bit. 1


① A-C-B ② B-A-C ③ B-C-A ④ C-A-B ⑤ C-B-A

 


4. 2062-21

 

Over the centuries various writers and thinkers, looking at humans from an outside perspective, have been struck by the theatrical quality of social life.


(A) Some people are better actors than others. Evil types such as Iago in the play Othello are able to conceal their hostile intentions behind a friendly smile. 1

(B) Others are able to act with more confidence and bravado — they often become leaders. People with excellent acting skills can better navigate our complex social environments and get ahead. 2

(C) The most famous quote expressing this comes from Shakespeare: "All the world's a stage, / And all the men and women merely players; / They have their exits and their entrances, / And one man in his time plays many parts." If the theater and actors were traditionally represented by the image of masks, writers such as Shakespeare are implying that all of us are constantly wearing masks. 0


① A-C-B ② B-A-C ③ B-C-A ④ C-A-B ⑤ C-B-A

 


5. 2062-22

 

Personal blind spots are areas that are visible to others but not to you.


(A) That truck you don't see? It's really there! So are your blind spots. 1

(B) The developmental challenge of blind spots is that you don't know what you don't know. Like that area in the side mirror of your car where you can't see that truck in the lane next to you, personal blind spots can easily be overlooked because you are completely unaware of their presence. They can be equally dangerous as well. 0

(C) Just because you don't see them, doesn't mean they can't run you over. This is where you need to enlist the help of others. You have to develop a crew of special people, people who are willing to hold up that mirror, who not only know you well enough to see that truck, but who also care enough about you to let you know that it's there. 2


① A-C-B ② B-A-C ③ B-C-A ④ C-A-B ⑤ C-B-A

 


6. 2062-23

 

A child whose behavior is out of control improves when clear limits on their behavior are set and enforced.


(A) Too many limits are difficult to learn and may spoil the normal development of autonomy. The limit must be reasonable in terms of the child's age, temperament, and developmental level. To be effective, both parents (and other adults in the home) must enforce limits. 1

(B) Otherwise, children may effectively split the parents and seek to test the limits with the more indulgent parent. In all situations, to be effective, punishment must be brief and linked directly to a behavior. 2

(C) However, parents must agree on where a limit will be set and how it will be enforced. The limit and the consequence of breaking the limit must be clearly presented to the child. Enforcement of the limit should be consistent and firm. 0


① A-C-B ② B-A-C ③ B-C-A ④ C-A-B ⑤ C-B-A

 


7. 2062-24

 

Many inventions were invented thousands of years ago so it can be difficult to know their exact origins.


(A) For many years archaeologists believed that pottery was first invented in the Near East (around modern Iran) where they had found pots dating back to 9,000 BC. In the 1960s, however, older pots from 10,000 BC were found on Honshu Island, Japan. There is always a possibility that in the future archaeologists will find even older pots somewhere else. 2

(B) Sometimes scientists discover a model of an early invention and from this model they can accurately tell us how old it is and where it came from. However, there is always the possibility that in the future other scientists will discover an even older model of the same invention in a different part of the world. 0

(C) In fact, we are forever discovering the history of ancient inventions. An example of this is the invention of pottery. 1


① A-C-B ② B-A-C ③ B-C-A ④ C-A-B ⑤ C-B-A

 


8. 2062-25

 

The graph above shows the amount of the electric car stock in five countries in 2014 and 2016.


(A) All five countries had more electric car stock in 2016 than in 2014. In 2014, the electric car stock of the United States ranked first among the five countries, followed by that of China. 0

(B) In the Netherlands, the electric car stock was not more than three times larger in 2016 than in 2014. 2

(C) However, China showed the biggest increase of electric car stock from 2014 to 2016, surpassing the United States in electric car stock in 2016. Between 2014 and 2016, the increase in electric car stock in Japan was less than that in Norway. 1


① A-C-B ② B-A-C ③ B-C-A ④ C-A-B ⑤ C-B-A

 


9. 2062-26

 

The impala is one of the most graceful four‒legged animals.


(A) The breeding season occurs at the end of the wet season around May. Females give birth in an isolated spot away from the herd. The average life span of an impala is between 13 and 15 years in the wild. 2

(B) Impalas have the ability to adapt to different environments of the savannas. Both male and female impalas are similar in color, with white bellies and black‒tipped ears. Male impalas have long and pointed horns which can measure 90 centimeters in length. 0

(C) Female impalas have no horns. Impalas feed upon grass, fruits, and leaves from trees. When conditions are harsh in the dry season, they come together to search for food in mixed herds which can number as many as 100-200 individuals. 1


① A-C-B ② B-A-C ③ B-C-A ④ C-A-B ⑤ C-B-A

 


10. 2062-29

 

Every farmer knows that the hard part is getting the field prepared.


(A) But to really understand what he accomplished requires looking beyond the man. Instead of treating him as the manifestation of everything great, we should appreciate his role in allowing America to show that it can be great. 2

(B) We need to give more credit to the community in science, politics, business, and daily life. Martin Luther King Jr. was a great man. Perhaps his greatest strength was his ability to inspire people to work together to achieve, against all odds, revolutionary changes in society's perception of race and in the fairness of the law. 1

(C) Inserting seeds and watching them grow is easy. In the case of science and industry, the community prepares the field, yet society tends to give all the credit to the individual who happens to plant a successful seed. Planting a seed does not necessarily require overwhelming intelligence; creating an environment that allows seeds to prosper does. 0


① A-C-B ② B-A-C ③ B-C-A ④ C-A-B ⑤ C-B-A

 


11. 2062-30

 

Sudden success or winnings can be very dangerous.


(A) We do not take into account the role that luck plays in such sudden gains. We try again and again to recapture that high from winning so much money or attention. We acquire feelings of superiority. 1

(B) We become especially resistant to anyone who tries to warn us— they don't understand, we tell ourselves. Because this cannot be sustained, we experience an inevitable fall, which is all the more painful, leading to the depression part of the cycle. Although gamblers are the most prone to this, it equally applies to business people during bubbles and to people who gain sudden attention from the public. 2

(C) Neurologically, chemicals are released in the brain that give a powerful burst of excitement and energy, leading to the desire to repeat this experience. It can be the start of any kind of addiction or manic behavior. Also, when gains come quickly we tend to lose sight of the basic wisdom that true success, to really last, must come through hard work. 0


① A-C-B ② B-A-C ③ B-C-A ④ C-A-B ⑤ C-B-A

 


12. 2062-31

 

When is the right time for the predator to consume the fruit?


(A) That makes for a win‒win for predator and prey. The animal obtains more calories, and because it keeps eating more and more fruit and therefore more seeds, the plant has a better chance of distributing more of its babies. 2

(B) The plant uses the color of the fruit to signal to predators that it is ripe, which means that the seed's hull has hardened — and therefore the sugar content is at its height. Incredibly, the plant has chosen to manufacture fructose, instead of glucose, as the sugar in the fruit. 0

(C) Glucose raises insulin levels in primates and humans, which initially raises levels of leptin, a hunger‒blocking hormone — but fructose does not. As a result, the predator never receives the normal message that it is full. 1


① A-C-B ② B-A-C ③ B-C-A ④ C-A-B ⑤ C-B-A

 


13. 2062-32

 

We are often faced with high‒level decisions, where we are unable to predict the results of those decisions.


(A) Therefore, I used this process by enrolling in a low‒cost mini course with the same instructor. This helped me understand his methodology, style, and content; and I was able to test it with a lower investment, and less time and effort before committing fully to the expensive program. 2

(B) In many situations, it's wise to dip your toe in the water rather than dive in headfirst. Recently, I was about to enroll in an expensive coaching program. But I was not fully convinced of how the outcome would be. 1

(C) In such situations, most people end up quitting the option altogether, because the stakes are high and results are very unpredictable. But there is a solution for this. You should use the process of testing the option on a smaller scale. 0


① A-C-B ② B-A-C ③ B-C-A ④ C-A-B ⑤ C-B-A

 


14. 2062-33

 

Sociologists have proven that people bring their own views and values to the culture they encounter; books, TV programs, movies, and music may affect everyone, but they affect different people in different ways.


(A) This demonstrates why it's a mistake to assume that a certain cultural product will have the same effect on everyone. 2

(B) In a study, Neil Vidmar and Milton Rokeach showed episodes of the sitcom All in the Family to viewers with a range of different views on race. The show centers on a character named Archie Bunker, an intolerant bigot who often gets into fights with his more progressive family members. 0

(C) Vidmar and Rokeach found that viewers who didn't share Archie Bunker's views thought the show was very funny in the way it made fun of Archie's absurd racism ― in fact, this was the producers' intention. On the other hand, though, viewers who were themselves bigots thought Archie Bunker was the hero of the show and that the producers meant to make fun of his foolish family! 1


① A-C-B ② B-A-C ③ B-C-A ④ C-A-B ⑤ C-B-A

 


15. 2062-34

 

The availability heuristic refers to a common mistake that our brains make by assuming that the instances or examples that come to mind easily are also the most important or prevalent.


(A) Managers give more weight to performance during the three months prior to the evaluation than to the previous nine months of the evaluation period because the recent instances dominate their memories. The availability heuristic is influenced by the ease of recall or retrievability of information of some event. Ease of recall suggests that if something is more easily recalled in your memory, you think that it will occur with a high probability. 2

(B) According to Harvard professor, Max Bazerman, managers conducting performance appraisals often fall victim to the availability heuristic. The recency of events highly influences a supervisor's opinion during performance appraisals. 1

(C) It shows that we make our decisions based on the recency of events. We often misjudge the frequency and magnitude of the events that have happened recently because of the limitations of our memories. 0


① A-C-B ② B-A-C ③ B-C-A ④ C-A-B ⑤ C-B-A

 


16. 2062-35

 

Marketing management is concerned not only with finding and increasing demand but also with changing or even reducing it.


(A) For example, Uluru (Ayers Rock) might have too many tourists wanting to climb it, and Daintree National Park in North Queensland can become overcrowded in the tourist season. Power companies sometimes have trouble meeting demand during peak usage periods. 0

(B) Thus, marketing management seeks to affect the level, timing, and nature of demand in a way that helps the organisation achieve its objectives.2

(C) In these and other cases of excess demand, the needed marketing task, called demarketing, is to reduce demand temporarily or permanently. The aim of demarketing is not to completely destroy demand, but only to reduce or shift it to another time, or even another product. 1


① A-C-B ② B-A-C ③ B-C-A ④ C-A-B ⑤ C-B-A

 


17. 2062-36

 

The invention of the mechanical clock was influenced by monks who lived in monasteries that were the examples of order and routine.


(A) They ate at meal time, rather than when they were hungry, and went to bed when it was time, rather than when they were sleepy. Even periodicals and fashions became "yearly." The world had become orderly. 2

(B) The discovery of the pendulum in the seventeenth century led to the widespread use of clocks and enormous public clocks. Eventually, keeping time turned into serving time. People started to follow the mechanical time of clocks rather than their natural body time. 1

(C) They had to keep accurate time so that monastery bells could be rung at regular intervals to announce the seven hours of the day reserved for prayer. Early clocks were nothing more than a weight tied to a rope wrapped around a revolving drum. Time was determined by watching the length of the weighted rope. 0


① A-C-B ② B-A-C ③ B-C-A ④ C-A-B ⑤ C-B-A

 


18. 2062-37

 

Since we know we can't completely eliminate our biases, we need to try to limit the harmful impacts they can have on the objectivity and rationality of our decisions and judgments.


(A) Then we can choose an appropriate de‒biasing strategy to combat it. After we have implemented a strategy, we should check in again to see if it worked in the way we had hoped. 1

(B) If it did, we can move on and make an objective and informed decision. If it didn't, we can try the same strategy again or implement a new one until we are ready to make a rational judgment. 2

(C) It is important that we are aware when one of our cognitive biases is activated and make a conscious choice to overcome that bias. We need to be aware of the impact the bias has on our decision making process and our life. 0


① A-C-B ② B-A-C ③ B-C-A ④ C-A-B ⑤ C-B-A

 


19. 2062-38

 

It is important to remember that computers can only carry out instructions that humans give them.


(A) A computer cannot make independent decisions, however, or formulate steps for solving problems, unless programmed to do so by humans. Even with sophisticated artificial intelligence, which enables the computer to learn and then implement what it learns, the initial programming must be done by humans. Thus, a human‒ computer combination allows the results of human thought to be translated into efficient processing of large amounts of data. 2

(B) Synergy occurs when combined resources produce output that exceeds the sum of the outputs of the same resources employed separately. A computer works quickly and accurately; humans work relatively slowly and make mistakes. 1

(C) Computers can process data accurately at far greater speeds than people can, yet they are limited in many respects ―most importantly, they lack common sense. However, combining the strengths of these machines with human strengths creates synergy. 0


① A-C-B ② B-A-C ③ B-C-A ④ C-A-B ⑤ C-B-A

 


20. 2062-39

 

For hundreds of thousands of years our hunter‒gatherer ancestors could survive only by constantly communicating with one another through nonverbal cues.


(A) We are not trained, however, to pay attention to people's nonverbal cues. By sheer habit, we fixate on the words people say, while also thinking about what we'll say next. What this means is that we are using only a small percentage of the potential social skills we all possess. 2

(B) With these counterforces battling inside us, we cannot completely control what we communicate. Our real feelings continually leak out in the form of gestures, tones of voice, facial expressions, and posture. 1

(C) Developed over so much time, before the invention of language, that is how the human face became so expressive, and gestures so elaborate. We have a continual desire to communicate our feelings and yet at the same time the need to conceal them for proper social functioning. 0


① A-C-B ② B-A-C ③ B-C-A ④ C-A-B ⑤ C-B-A

 


21. 2062-40

 

Why do we help?


(A) Others believe that we help because we have been socialized to do so, through norms that prescribe how we ought to behave. Through socialization, we learn the reciprocity norm: the expectation that we should return help, not harm, to those who have helped us. In our relations with others of similar status, the reciprocity norm compels us to give (in favors, gifts, or social invitations) about as much as we receive. 2

(B) Social psychologists call it social exchange theory. If you are considering whether to donate blood, you may weigh the costs of doing so (time, discomfort, and anxiety) against the benefits (reduced guilt, social approval, and good feelings). If the rewards exceed the costs, you will help. 1

(C) One widely held view is that self‒interest underlies all human interactions, that our constant goal is to maximize rewards and minimize costs. Accountants call it cost‒benefit analysis. Philosophers call it utilitarianism. 0


① A-C-B ② B-A-C ③ B-C-A ④ C-A-B ⑤ C-B-A

 


22. 2062-4142

 

An organization imported new machinery with the capacity to produce quality products at a lesser price.


(A) A manager was responsible for large quantities in a relatively short span of time. He started with the full utilization of the new machinery. He operated it 24/7 at maximum capacity. He paid the least attention to downtime, recovery breaks or the general maintenance of the machinery. 0

(B) The new manager had to put significant time and effort into repair and maintenance of the machines, which resulted in lower production and thus a loss of profits. The earlier manager had only taken care of the goal of production and ignored the machinery although he had short‒term good results. But ultimately not giving attention to recovery and maintenance resulted in long‒term negative consequences. 2

(C) As the machinery was new, it continued to produce results and, therefore, the organization's profitability soared and the manager was appreciated for his performance. Now after some time, this manager was promoted and transferred to a different location. A new manager came in his place to be in charge of running the manufacturing location. But this manager realized that with heavy utilization and without any downtime for maintenance, a lot of the parts of the machinery were significantly worn and needed to be replaced or repaired. 1


① A-C-B ② B-A-C ③ B-C-A ④ C-A-B ⑤ C-B-A

 


23. 2062-4345

 

Maria Sutton was a social worker in a place where the average income was very low.


(A) To her surprise, staff members began to open their purses. The story of Alice's gift had spread beyond Maria's office, and Maria was able to raise $300―plenty for a Christmas gift for Karen and her son. On Christmas Eve, Maria and Alice visited Karen's house with Christmas gifts. When Karen opened the door, Maria and Alice wished the astonished woman a merry Christmas. Then Alice began to unload the gifts from the car, handing them to Karen one by one. Karen laughed in disbelief, and said she hoped she would one day be able to do something similar for someone else in need. On her way home, Maria said to Alice, "God multiplied your gift." 2

(B) Many of Maria's clients had lost their jobs when the coal industry in a nearby town collapsed. Every Christmas season, knowing how much children loved presents at Christmas, Maria tried to arrange a special visit from Santa Claus for one family. Alice, the seven‒year‒old daughter of Maria, was very enthusiastic about helping with her mother's Christmas event. This year's lucky family was a 25‒year‒old mother named Karen and her 3‒year‒old son, who she was raising by herself. However, things went wrong. Two weeks before Christmas Day, a representative from a local organization called Maria to say that the aid she had requested for Karen had fallen through. No Santa Claus. No presents. 0

(C) Maria saw the cheer disappear from Alice's face at the news. After hearing this, she ran to her room. When Alice returned, her face was set with determination. She counted out the coins from her piggy bank: $4. "Mom," she told Maria, "I know it's not much. But maybe this will buy a present for the kid." Maria gave her daughter a lovely hug. The next day, Maria told her coworkers about her daughter's latest project. 1


① A-C-B ② B-A-C ③ B-C-A ④ C-A-B ⑤ C-B-A

 


[ANSWER]
1. ② 2. ④ 3. ① 4. ④ 5. ② 6. ④ 7. ③ 8. ① 9. ③ 10. ⑤


11. ④ 12. ③ 13. ⑤ 14. ③ 15. ⑤ 16. ① 17. ⑤ 18. ④ 19. ⑤ 20. ⑤


21. ⑤ 22. ① 23. ③ 

728x90
반응형

728x90
반응형

20sw | Since 2005 임희재 | 블루티쳐학원 | 01033383436 | 200710 22:53:52

 

순서배열

 

1. 20수완 2-1

 

Adolescents' understanding of a task and the work necessary to complete it successfully influence their motivation.


(A) If a teacher assigns a chapter to read for homework without letting the students know that they are expected to discuss the major developments in the chapter the next day, then students do not understand the "real" assignment, nor do they know how to complete it successfully. 0

(B) For example, in assigning a textbook chapter for reading, the teacher should be clear about why the reading is assigned and what students are expected to do as a result of reading it. Provide guidance by giving examples of strategies that students can use in reading the chapter and relate that to successful participation in the discussion to enhance motivation for performing the reading activity. 2

(C) Goals and expectations for reading and writing assignments should be clear and specific. 1


① A-C-B ② B-A-C ③ B-C-A ④ C-A-B ⑤ C-B-A

 


2. 20수완 2-01

 

The understanding problem is simply that humans are not really set up to hear logic.


(A) We understand events in terms of other events we have already understood. When a decision-making heuristic, or rule of thumb, is presented to us without a context, we cannot decide the validity of the rule we have heard, nor do we know where to store this rule in our memories. Thus, the rule we are given is both difficult to evaluate and difficult to remember, making it virtually useless. 1

(B) People who fail to couch what they have to say in memorable stories will have their rules fall on deaf ears despite their best intentions, and despite the best intentions of their listeners. A good teacher is not one who explains things correctly, but one who couches his explanations in a memorable (i.e., an interesting) format. 2

(C) People, however, like to hear stories. The reason that people like to hear stories, however, is not transparent to them. People need a context to help them relate what they have heard to what they already know. 0


① A-C-B ② B-A-C ③ B-C-A ④ C-A-B ⑤ C-B-A

 


3. 20수완 2-02

 

Directness and honesty are qualities that our society values highly.


(A) Until we have good reason to know and trust the people behind the instant message pop-up or the chat room screen, we should remain anonymous and it is not dishonest to do so. 2

(B) We expect people to be who they say they are and tell us the truth about themselves. Before the advent of computers, anyone who used an assumed name was thought to be hiding something disreputable. 0

(C) Inexperienced computer users may continue to feel this way about online contacts. Because so many of the cues that we use to evaluate people are missing in cyberspace, computer users need to understand that virtual meeting places are different from face-to face contacts. 1


① A-C-B ② B-A-C ③ B-C-A ④ C-A-B ⑤ C-B-A

 


4. 20수완 2-03

 

To appreciate the long-term benefits of industrialization one does not have to accept its cruelties.


(A) Progress consists of unbundling the features of a social process as much as we can to maximize the human benefits while minimizing the harms. 2

(B) One can imagine an alternative history of the Industrial Revolution in which modern sensibilities applied earlier and the factories operated without children and with better working conditions for the adults. Today there are doubtless factories in the developing world that could offer as many jobs and still tum a profit while treating their workers more humanely. 0

(C) Pressure from trade negotiators and consumer protests has measurably improved working conditions in many places, and it is a natural progression as countries get richer and more integrated into the global community. Progress consists not in accepting every change as part of an indivisible package - as if we had to make a yes-or-no decision on whether the Industrial Revolution, or globalization, is a good thing or bad thing, exactly as each has unfolded in every detail. 1


① A-C-B ② B-A-C ③ B-C-A ④ C-A-B ⑤ C-B-A

 


5. 20수완 2-04

 

As for the advice to say what you said, the key is the expression "in other words."


(A) A summary should repeat enough of the key words to allow the reader to connect it back to the earlier passages that spelled out the points in detail. But those words should be fitted into new sentences that work together as a coherent passage of prose in its own right. 1

(B) There's no sense in copying a sentence from every paragraph and pasting them together at the end. That just forces the reader to figure out the point of those sentences all over again, and it is tantamount to a confession that the author isn't presenting ideas (which can always be clothed in different language) but just shuffling words around the page. 0

(C) The summary should be self-contained, almost as if the material being summarized had never existed. 2


① A-C-B ② B-A-C ③ B-C-A ④ C-A-B ⑤ C-B-A

 


6. 20수완 3-1

 

Because mobile phones are highly personal items (ie, they go with us everywhere and often are in constant contact with our bodies), many critics of wireless advertising (as well as advertisers themselves) are concerned that unwanted messages represent an invasion of privacy.


(A) In addition to privacy invasion, others are skeptical about wireless advertising's future on the grounds that advertising is antithetical to the reasons that people own mobile phones in the first place. 1

(B) The argument, in other words, is that people own mobile phones for reasons of enhancing time utilization and increasing work-related productivity while away from the workplace or home, and the last thing they want while using these devices is to receive unwanted, interrupting advertising messages. 2

(C) Feeling invaded, recipients of undesired advertisements may immediately delete the intruding item and hold negative feelings toward the offending advertiser. 0


① A-C-B ② B-A-C ③ B-C-A ④ C-A-B ⑤ C-B-A

 


7. 20수완 3-01

 

Marine debris affects animals through ingesting it or getting entangled in it; it is estimated that up to 100,000 marine mammals, including endangered species, are killed each year by marine debris.


(A) Large amounts of plastic debris have been found in the habitat of endangered Hawaiian monk seals, including in areas that serve as nurseries. Entanglement in plastic debris has led to injury and deaths in endangered Steller sea lions, with packing bands the most common entangling material. 1

(B) Very serious effects happen when marine animals become entangled in debris such as fishing line and six-pack rings. Birds get fishing line entangled around their legs, which get injured and may be lost. 0

(C) Hatchling sea turtles run down the beach to the ocean, a critical phase in their life cycle. Debris can be a major impediment if they get entangled in fishing nets or trapped in containers such as plastic cups and open canisters. Marine debris is an aspect of habitat quality for nesting sites and may help explain declines in turtle nest numbers on certain beaches. 2


① A-C-B ② B-A-C ③ B-C-A ④ C-A-B ⑤ C-B-A

 


8. 20수완 3-02

 

Regulations covering scientific experiments on human subjects are stringent.


(A) Human anatomy and physiology vary, in small but significant ways, according to gender, age, lifestyle, and other factors. Experimental results derived from a single subject are, therefore, of limited value. There is no way to know whether the subject's responses are typical or atypical of the response of humans as a group. 2

(B) Nonetheless, experimenting on oneself remains deeply problematic. One obvious drawback is the danger involved. Knowing that it exists does nothing to reduce it. A less obvious drawback is the limited range of data that the experiment can generate. 1

(C) Subjects must give their informed, written consent, and experimenters must submit their proposed experiments to rigorous scrutiny by overseeing bodies. Scientists who experiment on themselves can, functionally if not legally, avoid the restrictions associated with experimenting on other people. They can also sidestep most of the ethical issues involved. Nobody, presumably, is more aware of an experiment's potential hazards than the scientist who devised it. 0


① A-C-B ② B-A-C ③ B-C-A ④ C-A-B ⑤ C-B-A

 


9. 20수완 3-03

 

In many schools, the art program traditionally has been viewed as a particularly favorable setting for educating students with special needs.


(A) Children with hearing impairment can visually observe a demonstration of color mixing with paint and try the process with immediately verifiable results, and children with motor disabilities can work with finger paints or with large brushes for painting. 2

(B) In art classes, children are able to interact with such materials as paint or clay in direct response to their senses of sight, sound, smell, and touch. The materials of art are sensory, concrete, and manipulable in direct ways that are unique within the school curriculum. 0

(C) All the senses can be brought into interaction, providing opportunities to adapt art making activities for students who have some sensory or mobility impairment. For example, children with visual impairment can form expressive objects with clay. 1


① A-C-B ② B-A-C ③ B-C-A ④ C-A-B ⑤ C-B-A

 


10. 20수완 3-04

 

Medical imaging techniques are constantly being refined, and this effort has been aided by various space technologies over the years.


(A) Digital image processing techniques developed at NASA's Jet Propulsion Laboratory to allow for computer enhancement of lunar pictures from the Apollo missions have since led to improved MRI and CT imaging. Techniques in astronomy have also refined imaging. 0

(B) Moreover, the procedure is ten times cheaper than a surgical biopsy, and greatly reduces the pain, scarring, radiation exposure and time associated with surgical biopsies. 2

(C) The very same infrared sensors used to remotely observe the temperature of stars and planets are now being used to help surgeons map brain tumors. Charge-Coupled Device chip technologies stemming from the Hubble Telescope have greatly furthered breast cancer detection techniques, allowing breast tissue to be imaged more clearly and efficiently, thus increasing resolution so as to be able to distinguish between malignant and benign tumors without resorting to surgical biopsy. 1


① A-C-B ② B-A-C ③ B-C-A ④ C-A-B ⑤ C-B-A

 


11. 20수완 4-1

 

As the health of the social body began to change, hospitals became the primary institutions to care for the sick and monitor the passage from life to death.


(A) The dramatic rise in the number of hospitals across the country and their increasing control over the health of the nation contributed to the separation of death from everyday life. As doctors achieved professional dominance in the practice of medicine in the early decades of the twentieth century, hospitals emerged as the principal site for the diagnosis and treatment of patients. 0

(B) An 1873 survey counted 178 hospitals, about 50 of which were institutions for the mentally ill. A 1923 tabulation listed 6,830, or an increase of about 3,800 percent. While this change was most dramatic in the urban landscape, it also occurred gradually in rural areas. It should be no surprise that one consequence was an increase in the number of deaths away from home, the traditional place for end of life scenarios. 2

(C) In the words of one social historian, Whereas doctors came to patients in the 1870s, by the 1920s, patients increasingly came to doctors. Over this time span, the American hospital changed in size and clientele. 1


① A-C-B ② B-A-C ③ B-C-A ④ C-A-B ⑤ C-B-A

 


12. 20수완 4-01

 

The way in which people shape landscapes often reflects the dominant culture.


(A) In a highly commercialized setting such as the United States, it is not surprising that many landscapes are seen as commodities. In other words, they are valued because of their market potential. 0

(B) The landscape itself, including the people and their sense of self, takes on the form of a commodity. Over time, the landscape identity can evolve into a sort of "logo" that can be used to sell the stories of the landscape. Thus, California's "Wine Country," Florida's "Sun Coast," or South Dakota's "Badlands" shape how both outsiders and residents perceive a place, and these labels build a set of expectations associated with the culture of those who live there. 2

(C) Residents develop an identity in part based on how the landscape can generate income for the community. This process involves more than the conversion of the natural elements into commodities. 1


① A-C-B ② B-A-C ③ B-C-A ④ C-A-B ⑤ C-B-A

 


13. 20수완 4-02

 

There are multiple views of the learner in theories of cognition, just as there are multiple theories of the nature of art.


(A) Some portray the learner as a lone individual trying to make sense of a work of art or, for that matter, the world. Others picture learners as living within a social or cultural context from which knowledge derives its meaning. 0

(B) But if works of art are thought of as autonomous structures whose meaning is set by the artist, where there is one "objective" or "right" interpretation, independent of its social context, then it might make more sense to pair this conception of art with the view of the learner as a lone individual. 2

(C) Such different views of the learner have consequences for teaching the arts. At the same time, if works of art are to be understood in terms of their social and cultural origins and purposes, then it would make sense to integrate the knowledge of the artwork into those subjects, such as the social studies or history, where knowledge of the culture and society is collaterally provided. 1


① A-C-B ② B-A-C ③ B-C-A ④ C-A-B ⑤ C-B-A

 


14. 20수완 4-03

 

You may resist the notion that music, which you think should be free to express emotion, is best thought of as rule-governed behavior.


(A) But rules govern all meaningful human cultural behavior in just this way. The point is not that musical performance is predetermined by rules, but that it proceeds according to them. 0

(B) Nonetheless, just as meaningful conversations can express emotion, so meaningful music can express it as well, though not, of course, in exactly the same way. Further, if a listener does not understand the rules, he or she can understand neither the intention of the composer or musician nor the music's structure. 2

(C) In this view, music is like a game or a conversation. Without rules we could not have a game, and without agreement about what words are, what they mean, and how they are used, we could not hold a meaningful conversation. 1


① A-C-B ② B-A-C ③ B-C-A ④ C-A-B ⑤ C-B-A

 


15. 20수완 4-04

 

Since the 19th century, economics has borne the dishonorable name of "the dismal science."


(A) But Carlyle's negative characterization of economics in fact appears in an article that he wrote about slavery in the West Indies, not in any of his writings about Malthus. And, as it turns out, Carlyle delivered his insult to economics simply because the free market economists of his time did not support his proslavery views. So, in the end, economics earned its less than auspicious nickname for being on what most would agree was the right side of history. 2

(B) The cheerless nickname has been attributed by some to the Scottish historian Thomas Carlyle, who reportedly coined the term when discussing economist Thomas Malthus's prediction that one day population growth would outstrip food production and cause widespread famine. It's true that Carlyle wrote about Malthus on occasion. 1

(C) Some people think economics is called "the dismal science" because it's a dry and difficult subject. Others think it's because economics tackles depressing topics such as poverty, crime, war, taxes, inflation, and economic collapse. 0


① A-C-B ② B-A-C ③ B-C-A ④ C-A-B ⑤ C-B-A

 


16. 20수완 8-1

 

The abundant supply of commercial fertilizers has made possible the production of large crops on land that was once considered "worn out".


(A) This applies particularly to phosphorus, much of which is chemically fixed in the soil; the same is true to a lesser extent of nitrogen and potassium. Heavy fertilization that results in large yields also commonly slowly increases the soil organic matter content if the soil was very low in it initially, and necessarily the nitrogen content of the soil. 1

(B) It has also led to improved practices with regard to drainage, erosion control, and many other practices because fertilizers make it profitable to spend money on soils to put them in the best possible condition for high yields. Not all of the added fertilizer is removed by the first one or two crops but some of it remains fixed in the soil in a slowly available form. 0

(C) The result then is that fertilizers tend to increase soil fertility, or at least soil productivity. 2


① A-C-B ② B-A-C ③ B-C-A ④ C-A-B ⑤ C-B-A

 


17. 20수완 8-01

 

Communal living became a necessity because the community and the division of labour through which all individuals subordinate themselves to the group, ensured the continued existence of the species.


(A) Consider the difficulties of childbirth and the extraordinary care necessary for keeping a child alive during its infancy! This prolonged care and attention could only be exercised where division of labour existed. 1

(B) Think of the number of illnesses and disorders that human flesh is heir to, particularly in infancy, and you have some conception of the inordinate amount of care each human life demands, and some understanding of the need for communal living. The community is the best guarantor of the continued existence of human beings! 2

(C) Only division of labour (which is another way of saying civilization) is capable of ensuring that the tools of survival are available to humankind. Only after they had learned about the division of labour did humans learn how to assert themselves. 0


① A-C-B ② B-A-C ③ B-C-A ④ C-A-B ⑤ C-B-A

 


18. 20수완 8-02

 

Remember the Great Blackout of 2003?


(A) But some do. For example, back in February 2000, cyberpranksters launched the first major attack of the Internet Age. Their targets included the biggest of the big online operators. The weapon in this case was a "denial of service" attack where attackers bombard a target's servers with thousands of hits. 2

(B) A glitch in the US electrical grid knocked out power in a huge swath stretching from New York City to the Midwest. Similar incidents struck Italy and Norway in 2003. Technical breakdowns that happen on a grand scale also happen on a small scale to individual businesses. Everyone knows what it's like when the company server goes down. 0

(C) You cannot send or receive e-mail. Access to your databases evaporates. Customers cannot place orders on your Web site. Short interruptions, of course, rarely constitute a crisis. 1


① A-C-B ② B-A-C ③ B-C-A ④ C-A-B ⑤ C-B-A

 


19. 20수완 8-03

 

In 1843, German economist Wilhelm Roscher explicitly raised the issue of the need to use the historical method in economics, the essential element of which should be a comparative approach.


(A) Roscher argued that economic behaviours are dependent on their historical and social context, causes that in their studies should be used not only in an economic but also a historical and sociological approach. 0

(B) This explains why so much effort of the representatives of the so-called older historical school, notably Bruno Hildebrand and Karl Knies, focused on describing the stages of the historical development of society. There is often talk of the methodological assumption common to most representatives of this school of thought, shared by their English counterparts, which was a denial of the existence of universally valid economic laws in conjunction with the emphasis on the importance of individual facts. 2

(C) The first task of a researcher is therefore to immerse oneself in history to get as wide a knowledge of economic facts as possible, and on that basis determine the relationship between the economy and society. 1


① A-C-B ② B-A-C ③ B-C-A ④ C-A-B ⑤ C-B-A

 


20. 20수완 8-04

 

A guarantee of authenticity through a form of certification is important to many tourist consumers.


(A) The same study also noted that domestic tourists from New Zealand, as well as visitors from Australia and Asia, considered verification of genuineness on labels very important in their purchasing decisions, while visitors from Europe and North America were less concerned with the issue. 2

(B) Documentation of legitimacy may be provided in the form of official paperwork and certificates, photographs, artisans' signatures, and dates. In their 1993 paper, Littrell and her colleagues suggest that it is typically tourists with a need for status who often judge authenticity by external markers such as these. 0

(C) According to nearly half of the visitors in one New Zealand study, authenticity was important in their decision to buy an item of clothing. When asked if they would be more likely to buy clothing from New Zealand if design authenticity were included on the label, 46% said that they would. 1


① A-C-B ② B-A-C ③ B-C-A ④ C-A-B ⑤ C-B-A

 


21. 20수완 9-1

 

A paradoxical effect of better communications has been social fragmentation and isolation.


(A) Today there are many new ways in which people are able to lead rich and fulfilling social lives. And yet the social fabric that connected people at the local, geographical level has largely disappeared. And the trend towards social fragmentation just keeps on increasing. Many technological changes were introduced in the name of efficiency and time-saving for the consumer.1

(B) It is more convenient if you can do your banking online from home. But the unforeseen side effect is to chip away at our exposure to social contact. Perhaps the most serious consequence is that convenient technologies are fragmenting the social networks that reinforce important moral, ethical and social values. 2

(C) Social contact is a fundamental human need. We are social animals. Technological innovation has contributed relentlessly to the isolation of people from one another. As the increasing frequency of people dying alone reveals, all is not well in modern society. 0


① A-C-B ② B-A-C ③ B-C-A ④ C-A-B ⑤ C-B-A

 


22. 20수완 9-01

 

While humor is one of the important coping mechanisms of Native Americans, it should be used only if the client invites it, meaning that the client trusts the counselor enough to connect on that level.


(A) Indian humor serves the purpose of reaffirming and enhancing the sense of connectedness as part of family, clan, and tribe. To the extent that it can serve that purpose in the counseling relationship, it is all the better. 2

(B) What, in one situation, can be humor between two people, in another can be interpreted as ridicule or wearing a mask. Counselors, therefore, have to be sensitive to using humor in a way that doesn't reinforce various means of oppression that the client has endured probably for all of her or his life. 0

(C) However, in the positive direction, humor provides the opportunity to connect with the client on her or his ground and share a powerful trust. In sum, although counselors working with Native American clients should exercise caution when using humor, they definitely should not overlook it as a powerful therapeutic technique. 1


① A-C-B ② B-A-C ③ B-C-A ④ C-A-B ⑤ C-B-A

 


23. 20수완 9-02

 

For millions of years man and his closest ancestors obtained food (and medicinal substances) from nature in a ready form.


(A) The sharp fall in rural populations that accompanied the growth of cities and the development of so-called public catering necessitated the production and storage of enormous quantities of food products. This coincided with the emergence of a second way of deriving material benefits - 'production'. But side by side with production, in the sense of the creation of completely new objects for consumption, goes the process of purification (distillation, refinement, etc.). 1

(B) We have called this process 'anti-gathering'. It has led to finely ground flour products, polished rice and other grain removed from the husk, refined oil, and refined sugar. The choice of vegetables and wild-growing plants has shrunk; pure spirits such as vodka have replaced natural wines. 2

(C) There was nothing to prevent the whole diversity of biologically active substances from entering his body. Cooking food was an individual domestic affair. 'Gathering' was the first way man received material benefits. 0


① A-C-B ② B-A-C ③ B-C-A ④ C-A-B ⑤ C-B-A

 


24. 20수완 9-03

 

In the offline world, bystander helping is influenced by the number of other people available to provide help.


(A) The combination of visible needs for help and unknown numbers of potential helpers may make the felt need to offer help more salient. Until one person actually offers help, every potential helper may assume that he or she is the only one who could help. 1

(B) Physical invisibility also reduces the barriers to offering help for people whose age, gender, race, or other visible attributes lead people to discount their contributions in the offline world, regardless of their actual usefulness. Help provided is not spumed on the basis of irrelevant physical or social attributes; it can be judged based solely on its quality. 2

(C) If people see that others are available to help, their own motivation to help is diminished. In the online world, it is hard to know how many potential helpers are available. 0


① A-C-B ② B-A-C ③ B-C-A ④ C-A-B ⑤ C-B-A

 


25. 20수완 9-04

 

Fear of strangers (xenophobia) has had a long history.


(A) They do not share our history, our beliefs or ideology, our religion or our work ethic so they are unwelcome. They do not have to be afforded equal consideration in the application of our values or laws. Racism and prejudice are passed along from generation to generation in a process known as cultural transmission. 1

(B) Foreigners, an out-group, were seen as threatening simply by being different. Immigration policies have continued to reflect this concern that foreigners use up our resources, take our jobs, and become a burden or threat to society. People from other lands and other groups gain a negative stereotype. 0

(C) Primary groups reinforce prejudices and stereotypes during socialization. The media disproportionately report negatives about minorities (out-groups), institutions advance discriminatory practices, and governments institute programs and policies that deny equal opportunity. As these become entrenched, they are difficult to challenge. 2


① A-C-B ② B-A-C ③ B-C-A ④ C-A-B ⑤ C-B-A

 


26. 20수완 10-1

 

The history of virtue ethics goes back to Aristotle in The Nicomachean Ethics.


(A) This leads to circularity, to a life of purposelessness. Take the following example. I go to work to earn money. I earn money to enable me to purchase food. I purchase food so that I can eat. 1

(B) I eat in order to go to work. I work to earn money. According to Aristotle, the purpose of all our acts is that they are directed to some ultimate good. We should seek to know the good so that we are able to direct our actions to that end. Without trying to know the good we avoid the fundamental purpose for human life. 2

(C) As in the rest of his philosophy, the notion of purpose (goal, ends) plays an integral role in Aristotle's ethical theory. Every act, he says, is performed for some purpose. He defined this purpose as 'the good'. In other words, we do the things we do because they have a worthwhile purpose. Otherwise, the acts we perform would depend on some other act to give them meaning and so forth. 0


① A-C-B ② B-A-C ③ B-C-A ④ C-A-B ⑤ C-B-A

 


27. 20수완 10-01

 

If the hypothesis under consideration is a simple generalization, it may be sufficient to test it by looking for more examples, seeing whether or not the generalization holds for them.


(A) If the generalization has any reasonable body of supporting data, the finding of new facts which do not fit usually leads to the refinement or elaboration of the original hypothesis rather than its complete rejection. 1

(B) On the other hand, if the original basis for the hypothesis was slender, the unfavorable instances may so outweigh the favorable ones as to make it reasonable to believe that the earlier agreement was a matter of pure chance. Also, a new hypothesis may be developed which fits the original data and the new data as well. 2

(C) Under these circumstances unfavorable examples which violate the generalization may or may not lead to its rejection. 0


① A-C-B ② B-A-C ③ B-C-A ④ C-A-B ⑤ C-B-A

 


28. 20수완 10-02

 

Knowledge — the output of human innovation — is unique among all resources.


(A) It's not a physical resource. It's an information resource. Where all physical resources are depleted by use, and are divided by sharing, knowledge is different. 0

(B) That means the world isn't zero-sum. One person or nation's gain doesn't have to be another's loss. By creating new ideas, we can enrich all of us on the planet, while impoverishing none. Knowledge plays by different rules than physical resources, rules that make it inherently abundant. 2

(C) A wheel may break or wear out, but the idea of the wheel will keep on working. A wheel can only be used in one place and one time, but the design for a wheel can be shared with an infinite number of people, all of whom can benefit from it. Ideas aren't zero-sum. 1


① A-C-B ② B-A-C ③ B-C-A ④ C-A-B ⑤ C-B-A

 


29. 20수완 10-03

 

To at least "compare notes" with other philosophically interested people adds something crucial to the practice of philosophy itself and not just because it's important to make your ideas more widely known.


(A) Rather, it helps you hone your ideas to greater quality. Part of this is making sure that your ideas and reasoning process are as clearly spelled out as possible; ensuring that they're clear to someone else helps make them clearer to yourself. 0

(B) This gives both participants in a philosophical exchange an indispensable opportunity to hone their ideas and reasoning skills. Thus, the social component of philosophy is an essential part of the doing of philosophy itself. 2

(C) This is how a philosopher ensures that he or she is really making sense and using valid reasoning processes. If you are challenged by someone who seems to disagree, this forces you to be as clear as possible about what principles you are starting with and exactly how you are reasoning to a given conclusion. 1


① A-C-B ② B-A-C ③ B-C-A ④ C-A-B ⑤ C-B-A

 


30. 20수완 10-04

 

In a formalized manner the social structure of the indigenous peoples of the Pacific Northwest Coast is based on intricate forms of clan and family lineage regulated by both birth and wealth.


(A) In this way the ownership and display of ritual clothing was literally the visual fabric of the social order. 2

(B) Rank and privilege were proclaimed by song, speech, and costume in tribal cultures where relative status in the community was of prime importance. Whole costumes from headgear to leggings were decorated with a readable heraldic system based on familiar animals associated with clan symbolism, and personal, often inherited, spiritual relationships. 0

(C) Costumes were worn at the social and religious events that expressed their highly differentiated and formally structured society. As a new rank was achieved or a fresh honor bestowed upon an individual, the concurrent rise in prestige and status was often marked by that person's right to wear another highly specialized symbolic garment. 1


① A-C-B ② B-A-C ③ B-C-A ④ C-A-B ⑤ C-B-A

 


31. 20수완 11-1

 

Unexpected events typically lead us to think in more complex ways.


(A) Subjects in one study read about a student who had done either well or poorly in high school and then learned about the student's college grades. For some of the subjects, their expectations were confirmed. For example, the good student in high school received good grades in college. 0

(B) Subjects who learned of the unexpected outcome considered many more causal attributions (eg, "perhaps he did much better than expected because he finally learned how to study") than subjects who simply had their expectation confirmed. This study demonstrates that unexpected events increase our search for explanations. 2

(C) For others, their expectations were violated. For example, the poor student in high school did unexpectedly well in college. Subjects then retold the story into a tape recorder as if they were relaying it to a friend. 1


① A-C-B ② B-A-C ③ B-C-A ④ C-A-B ⑤ C-B-A

 


32. 20수완 11-01

 

There is an old television commercial from the seventies that shows a Native American walking along a polluted river.


(A) Every action has a reaction. You may not realize it today, but someone will in the future. 2

(B) If we pull up our stakes and move the family to Florida, they will grow up as Southerners and not as New Englanders. They will live in a world devoid of snow and cold and be reliant on fans and air conditioners for their comfort. If you work eighteen hours a day and your children are raised by babysitters and other child care providers, do not be surprised at the people they grow up to be. 1

(C) The garbage floats to the top and onto the river banks as a tear flows down his cheek. There are no words spoken by the man but it is clear that he is ashamed and appalled at what we have done to the beautiful land that was once his ancestors'. Every day, we fail to realize that our actions have an impact on the future. 0


① A-C-B ② B-A-C ③ B-C-A ④ C-A-B ⑤ C-B-A

 


33. 20수완 11-02

 

In a series of experiments by Jacques Mehler and his colleagues, infants as young as four days old were able to distinguish their native language from a different language, while they were unable to distinguish utterances in two foreign languages.


(A) The babies were more aroused by utterances in the native language, as indicated by the faster rate at which they sucked on their pacifiers. On the basis of several studies indicating that some sound from speech reaches infants in utero, although reduced in frequency range and intensity, Mehler and his colleagues tested very young infants with highly filtered versions of recordings in the native language and one that was nonnative. 0

(B) The experimenters conclude that prosody is sufficient for infants to discriminate the two languages. 2

(C) The infants were able to discriminate preferentially in favor of their native language. This suggests that prosodic cues play an important role in the infants' responses, since those were the only cues available on the filtered tapes. 1


① A-C-B ② B-A-C ③ B-C-A ④ C-A-B ⑤ C-B-A

 


34. 20수완 11-03

 

We sometimes solve number problems almost without realizing it.


(A) As long as you do not run out of copies before completing this process, you will know that you have a sufficient number to go around. You have then solved this problem without resorting to arithmetic and without explicit counting. 1

(B) There are numbers at work for us here all the same and they allow precise comparison of one collection with another, even though the members that make up the collections could have entirely different characters, as is the case here, where one set is a collection of people, while the other consists of pieces of paper. What numbers allow us to do is to compare the relative size of one set with another. 2

(C) For example, suppose you are conducting a meeting and you want to ensure that everyone there has a copy of the agenda. You can deal with this by labelling each copy of the handout in turn with the initials of each of those present. 0


① A-C-B ② B-A-C ③ B-C-A ④ C-A-B ⑤ C-B-A

 


35. 20수완 11-04

 

For more than two decades, school reform has been driven by an agenda that appears to be uninformed by even the most basic research into what we now know about the functioning of the brain or the healthy development of the child.


(A) Educational leaders and policy makers aren't asking, "What do children need for healthy brain development?" "How do they learn best?" 0

(B) Most reform has been focused on what to cram into children's heads — and testing them ad nauseam to see what sticks — rather than on developing their brains. What this means is that we do more but accomplish less. Educational reforms fail because they hinge on policies that lower the sense of control of students, teachers, and administrators alike, predictably leading to greater stress, lower student engagement, and ever more teacher dissatisfaction and burnout. 2

(C) or "When's the optimal time to teach him or her to read or do algebra?" Rather, they seem to be asking, "What do we need this child to be able to do in order to meet our school, local, or national standards?" 1


① A-C-B ② B-A-C ③ B-C-A ④ C-A-B ⑤ C-B-A

 


36. 20수완 12-1

 

What Mark Twain achieves, with great skill and at tremendous risk, all the way through his book Huckleberry Finn, is an invisible but immense ironic distance between his point of view and Huck's.


(A) Huck might grow up into that kind of man, given a chance. But Huck at this point is an ignorant, prejudiced kid who doesn't know right from wrong (though once, when it really matters, he guesses right). In the tension between that kid's voice and Mark Twain's silence lies much of the power of the book. We have to understand — as soon as we're old enough to read this way — that what the book really says lies in that silence. 2

(B) Huck tells the story. Every word of it is in his voice, from his point of view. Mark is silent. 0

(C) Mark's point of view, particularly as regards slavery and the character Jim, is never stated. It is discernible only in the story itself and the characters — Jim's character, above all. Jim is the only real adult in the book, a kind, warm, strong, patient man, with a delicate and powerful sense of morality. 1


① A-C-B ② B-A-C ③ B-C-A ④ C-A-B ⑤ C-B-A

 


37. 20수완 12-01

 

Think of life as a charity foot race.


(A) Winning can be enjoying the scenery, the nice people along the way, or even the time for reflection during the relaxed race. Remember the old rat race analogy — it might be more important not to be a rat than to win the rat race. After all, if you win the rat race, you will still be a rat. 2

(B) The race, like many things in life, will have a similar outcome for most of the participants —they will all reach the finish line. The only difference then, is the process of running the race. What then is the difference between the person, eyes focused straight ahead, who rushes from the starting line to the finish line in a wild dash hope of winning and the person who looks to the right and left and still arrives at the finish line? 0

(C) Not that there's anything wrong with winning races. However, only one person can win by reaching the finish line first, while everyone can win from the standpoint of the process. The key is to develop a personal definition of winning. 1


① A-C-B ② B-A-C ③ B-C-A ④ C-A-B ⑤ C-B-A

 


38. 20수완 12-02

 

For many people, the words "Cajun" and "Creole" lead to visions of gumbo, red beans and rice, crawfish, and just about anything that's been "blackened, Cajun-style."


(A) While these culinary traditions are distinctive and delicious, they have overshadowed the many other unique cultural contributions made by Louisiana's Cajun and Creole communities, and are often considered without reference to the social and historical contexts that produced them. 0

(B) As poet Sheryl St. Germain writes in "Cajun," she fears "the word's been stolen" by retail commodifiers of a culture emptied of its content and its history, reduced — quite literally - to an object of consumption. 2

(C) When "Cajun" and "Creole" are reduced to adjectives on menus and food labels, it becomes easy to lose sight of the people those adjectives first described. 1


① A-C-B ② B-A-C ③ B-C-A ④ C-A-B ⑤ C-B-A

 


39. 20수완 12-03

 

As Larry Cuban, former president of the American Educational Research Association, points out, schools have spent billions of dollars over the years on technologies that have, in fact, changed very little of how we think about an "education" in the developed world.


(A) In fact, the system almost unwittingly marginalizes digital technologies in schools. We relegate them to labs or libraries, or if we place them in students' hands, they're used only for discrete, narrow purposes like reading textbooks, creating documents, or taking assessments. 1

(B) More often than not, we strip the agency and freedoms that digital tools give to learners and creators outside of school when they bring those same tools into the building. The system of schooling that most of us are products of is based on a series of structures and efficiencies that do not work well with the messier, less linear, more self-organized ways we can learn, create, and connect on the Internet. 0

(C) Few would argue that in schools today, we see technology primarily as an institutional teaching tool, not a personal learning tool. 2


① A-C-B ② B-A-C ③ B-C-A ④ C-A-B ⑤ C-B-A

 


40. 20수완 12-04

 

It is pretty well known that creativity functions best with periods of incubation, but it can be really hard to integrate this nothing time into your work practice.


(A) But sleep has been shown to be a very significant part of the creative process which utilizes intuition and insight and which leads you down a much more successful path to problem solving. Therefore, this proves that not only is nothing actually something, but it is in fact crucial. 2

(B) I've always felt a bit guilty about my regular half-hour afternoon naps, partly because I'm so conscious that I'm not doing anything and it often feels more like I'm escaping. 1

(C) However, it is vital unconscious time when our minds are able to rework elements or problems that have arisen through conscious activity and which, more often than not, result in new and fresh ideas. 0


① A-C-B ② B-A-C ③ B-C-A ④ C-A-B ⑤ C-B-A

 


41. 20수완 13-1

 

Licenses in copyright work similarly to other licenses that we deal with in everyday life; they give a person or entity permission to do certain acts within certain limits.


(A) When granting a copyright license, the scope of the license may also be bounded by time or by the type of licensee to whom it is granted. It may be restricted by the types of use the licensee may make of the work or by the parts of the bundle of rights granted. 2

(B) These limits are the scope of the license and define which acts are permissible. 0

(C) For example, a person may have a license to practice law in one state but not in another state, and that license to practice law does not also give the licensee a driver's license. 1


① A-C-B ② B-A-C ③ B-C-A ④ C-A-B ⑤ C-B-A

 


42. 20수완 13-01

 

The rising challenge today for library workers is to help users not only find information on the Internet, but also become skillful evaluators of its usefulness and reliability from the ocean of resources available.


(A) Staff members in special libraries have a responsibility to sift through and evaluate data for their colleagues. For public librarians and staff, helping users find and evaluate information is central to their mission. 1

(B) In all cases, the library worker's role as an evaluator is just as important as that of a facilitator. For these reasons, libraries remain alive and well, because the Internet complements libraries, but does no replace them. 2

(C) Critical thinking skills are also essential. In school and academic libraries, this responsibility is shared with classroom teachers. 0


① A-C-B ② B-A-C ③ B-C-A ④ C-A-B ⑤ C-B-A

 


43. 20수완 13-02

 

Mathematics is one of the most profound creations of the human mind.


(A) How many of the greatest minds in history, from Pythagoras to Galileo to Gauss to Einstein, have held that "God is a mathematician"? 1

(B) For thousands of years, the content of mathematical theories seemed to tell us something profound about the nature of the natural world — something that could not be expressed in any way other than the mathematical. 0

(C) This attitude reveals a reverence for mathematics that is occasioned by the sense that nature has a secret code that reveals her hidden order. The immediate evidence from the natural world may seem to be chaotic and without any inner regularity, but mathematics reveals that under the surface the world of nature has an unexpected simplicity — an extraordinary beauty and order. 2


① A-C-B ② B-A-C ③ B-C-A ④ C-A-B ⑤ C-B-A

 


44. 20수완 13-03

 

Sociologist Harry Edwards argues that both sport and religion have a body of formally stated beliefs that are accepted on faith by a great number of adherents.


(A) Belief systems are centered on a creed that followers abide by in varying degrees. 0

(B) The philosopher William James coined the phrase "the will to believe" to express the view that sometimes believing that an event will occur can actually help make it happen. This can be applied to both athletes and fans, who feel that their strong dedication to winning might actually tip the scale and make victory occur. 2

(C) Fans are told to have "faith" in their team, especially in times of turmoil, much in the same manner that religious people are told to have faith in times where they doubt their religion. 1


① A-C-B ② B-A-C ③ B-C-A ④ C-A-B ⑤ C-B-A

 


45. 20수완 13-04

 

The advent of technological innovations in sound production has had radical (some might argue liberating) consequences for film scoring and recording.


(A) It is theoretically possible now, in many parts of the world, for a composer to virtually create and produce an entire score, thus eliminating the need for teams of assistants, arrangers, and copyists to realize the score, and live musicians on acoustic instruments to perform it. 0

(B) In Bollywood, for instance, it has become so commonplace for scores to be digitally produced on a synthesizer that the very nomenclature has begun to change, with the term "programmer" replacing "music director." Such changes to film scoring have been so dramatic and the economic consequences so profound that Gregory Booth, in a recent book on the Mumbai film industry, labels the preprogramming years "Old Bollywood" and postprogramming "New Bollywood." 2

(C) In many film industries it is becoming increasingly necessary for composers to have computer expertise. 1


① A-C-B ② B-A-C ③ B-C-A ④ C-A-B ⑤ C-B-A

 


46. 20수완 14-1

 

One point of difference between the consumption of water and electricity is that water can be reused multiple times while electricity cannot.


(A) Withdrawn water, on the other hand, can be returned to its original water source. The argument can be made that all water demand eventually returns as precipitation via the hydrologic cycle and therefore is not "consumed". 1

(B) As a result, water can be classified as "consumed" or simply "withdrawn". In the former, water is removed from its source and lost through either evaporation (in the case of power plant cooling or flood irrigation), or transpiration (in the growing of biocrops). 0

(C) However, evaporation and precipitation are both spatially and temporally uneven. Water that is accessible, especially in arid and semi-arid regions, satisfies the immediate needs of water users, whereas future precipitation may not occur in the same location or at the desired timing. 2


① A-C-B ② B-A-C ③ B-C-A ④ C-A-B ⑤ C-B-A

 


47. 20수완 14-01

 

Today, Luddite is a disparaging term used to refer to a person who is opposed to or cautiously critical of technology.


(A) But it's important to remember that the original Luddites were not, in fact, opposed to technology per se. It was not the machines themselves that the Luddites feared and reacted against. Rather, they understood that technology is meant to serve humans, not the other way around. 0

(B) Suddenly they were answerable not to themselves but to a factory owner; they had to give up autonomy, or starve. They saw what the machines meant to their livelihood, to their lives, to their families, and to their communities. And they didn't like what they saw. 2

(C) Luddites were not protesting the technology itself; they were objecting to the new economic realities brought about by the machines. In former times, craftsmen had been able to work at their own pace and set their own prices for their goods. But with the dawn of industrialization and mass production, craftsmen fell on hard times and were increasingly forced to work for the hated factories. 1


① A-C-B ② B-A-C ③ B-C-A ④ C-A-B ⑤ C-B-A

 


48. 20수완 14-02

 

The free market will tend to oversupply goods with negative externalities, and undersupply goods with positive externalities.


(A) Public goods, on the other hand, are subject to the free-rider problem. Why should I contribute to supply street lights if I will get the benefit whether or not I contribute? But if everyone thinks like this — and the market encourages this type of reasoning — no lighting will be provided. 1

(B) It is easy to see why. Creating a negative externality is often a way of dumping your costs on another: literally. If it is cheaper to use a noisy production process than a quiet one, other people are inadvertently 'subsidizing' my use of the noisy process by bearing the cost of being disturbed by the noise. 0

(C) It is normally assumed that the solution to these problems is to make the state the supplier of public goods, taxing citizens to pay for them. Similarly the state can make pollution illegal, returning the costs to the polluter. 2


① A-C-B ② B-A-C ③ B-C-A ④ C-A-B ⑤ C-B-A

 


49. 20수완 14-03

 

The September 11, 2001, terrorist attacks left the economy reeling from a lack of spending and confidence.


(A) One industry that was affected the most was the airline industry. This industry suffered huge and insurmountable losses that threatened the survival of the largest airline companies. Without air travel, businesses became slower, less efficient, and less reliable. In this instance, a lack of demand wasn't necessarily indicative of consumers' true feelings toward air travel. 0

(B) The answer depended on time. If the government had stood by and let the economy regulate itself in this situation, the airline industry may have recovered, but in the process, countless more jobs would have been lost. So the government intervened and put together a monetary aid package to keep the airline industry alive. 2

(C) Rather, this lack of demand was created by an extreme disaster. So the question became, what should be done? Should the government have let the forces of supply and demand take over and eventually revamp the airline industry? Or should the government have taken a more proactive approach? 1


① A-C-B ② B-A-C ③ B-C-A ④ C-A-B ⑤ C-B-A

 


50. 20수완 14-04

 

While time is being compressed by the frantic pace of modern life, our sense of space has expanded to the point where the concept of place is no longer salient.


(A) While democratic in appearance, in reality these spaces are designed to coordinate economic activity across nodes in the global commodity chain, obedient to the protocols of external control, with every public space given over to marketing the same products to consumers seduced by ahistorical, transregional brands created by the advertising industry. 1

(B) Most public spaces are now commercial spaces standardized to provide comfort zones and facilitate easy access and mobility by anyone who might enter - a famous coffee shop on every corner, with vast swathes of every city resembling an airport terminal. 0

(C) In the process, genuinely local places are stripped of uniqueness and particularity. Each location resembles every other location, and only the occasional extraordinary effort on the part of the locals preserves a genuine sense of place. 2


① A-C-B ② B-A-C ③ B-C-A ④ C-A-B ⑤ C-B-A

 


51. 20수완 15-1

 

Businesses that own enterprises and seek to market their goods and services in more than one nation are known as multinational corporations (MNCs).


(A) Poor nations are particularly vulnerable to MNCs, many of which have yearly budgets greater than those of poor-nation governments. For example, in 2014, each of the world's 20 largest MNCs had gross revenues of more than $161 billion, larger than all but about 50 of the 208 countries tracked by the World Bank. 1

(B) MNCs bring employment opportunities as well as goods and services to people who otherwise would not have them. At the same time, they create major and controversial changes in the natural, economic, social, and political environments. 0

(C) The financial power of these corporations enables them to exert enormous influence on poor nations and makes it extremely difficult for these nations to regulate them. In addition, like all capitalist corporations, the fundamental goal of MNCs is to return wealth to their shareholders, the vast majority of whom live in wealthy nations. Thus, most MNC profits in poor nations contribute to the economy of wealthy nations. 2


① A-C-B ② B-A-C ③ B-C-A ④ C-A-B ⑤ C-B-A

 


52. 20수완 15-01

 

In the popular media, archaeology is mainly identified with spectacular discoveries of artifacts from prehistoric and ancient cultures, such as the tomb of the Egyptian king Tutankhamun.


(A) Their principal task is to infer the nature of past cultures based on the patterns of the artifacts left behind. Archaeologists work like detectives, slowly sifting and interpreting evidence. 1

(B) The context in which things are found, the location of an archaeological site, and the precise position of an artifact within that site are critical to interpretation. In fact, these considerations may be more important than the artifact itself. 2

(C) As a result, people often think of archaeologists primarily as collectors. But contemporary archaeologists are much more interested in understanding and explaining their finds in terms of what those objects say about the behavior that produced them than in creating collections. 0


① A-C-B ② B-A-C ③ B-C-A ④ C-A-B ⑤ C-B-A

 


53. 20수완 15-02

 

Violence is common among many living organisms and probably has roots that stretch back to the time of the first noncloned life forms that were capable of physical interaction.


(A) "Violence" is defined as one or more individuals physically attacking one or more other individuals. The term "violence" is typically used in a negative, antisocial context, especially regarding humans. 1

(B) The term "violence" is often used interchangeably with "aggression" or "agonism," but it differs in its precision, and that difference is worth recognizing. While "aggression" is normally defined as any behavior relating to attack, threat, or defense, and "agonism" includes all of those behaviors plus fleeing, "violence" is more specific. 0

(C) I did a brief survey of the scientific literature over the past 4 years and found no articles that used the term "violence" when referring to animal behavior. Instead, the term was reserved for human behavior and typically for behaviors that are categorized as delinquent or antisocial. 2


① A-C-B ② B-A-C ③ B-C-A ④ C-A-B ⑤ C-B-A

 


54. 20수완 15-03

 

Less than one hundred years ago most rural households ln the United States sustained themselves by farming.


(A) While there was a well-established division of labor along gender and age lines in many farm households, there was not a well-articulated and formalized occupational structure within most rural areas. In this social and economic context, the household, the community, and the economy were tightly bound up with one another. 1

(B) While some agricultural products were sold for money on the open market, others were produced solely for household consumption or for bartering with neighbors. All family members, including husbands, wives, and children, contributed their labor to the economic maintenance and survival of the household. 0

(C) The local economy was not something that could be isolated from society. Rather the economy was embedded in the social relations of the farm household and the rural community. 2


① A-C-B ② B-A-C ③ B-C-A ④ C-A-B ⑤ C-B-A

 


55. 20수완 15-04

 

Democratic peoples — especially Americans — respond strongly to moral narratives that cleanly distinguish between the forces of good and evil.


(A) For example, most economists believe that poorly timed public thrift — austerity — can make a bad economic situation worse. But most people have a hard time understanding why it can be right for a government to spend more than it is taking in — especially if the public deficit is used to finance current consumption. While many parents grasp the rationale of going into debt to finance a college education, they are loath to cosign loans for children's fancy cars and flat-screen TVs. 2

(B) They have a harder time coming to grips with moral complexity and ambiguity. Private and public morality sometimes diverge. 0

(C) The norms of foreign policy and war are not congruent with those of domestic affairs. And the virtues of the private household do not always map neatly onto those of the public household. 1


① A-C-B ② B-A-C ③ B-C-A ④ C-A-B ⑤ C-B-A

 


56. 20수완 16-1

 

When the natural communication systems of primates are examined, no straightforward increase in complexity from monkeys to apes to humans is observed.


(A) Many researchers characterize great ape communication systems as more limited in range than those of monkeys. For example, monkeys, but not other apes, have functionally referential alarm calls, although whether monkey calls are truly referential like human language remains contested. This particular ape-monkey difference makes biological sense. 0

(B) Apes do possess gestures to initiate play, for instance, or when infants signal they wish to be carried — many of these gestures have learned elements. However, apes seemingly do not use their gestures referentially, nor do their gestures exhibit any symbolic or conventionalized features. 2

(C) Great apes are larger and stronger than monkeys, and hence are less vulnerable to predation. Apes almost certainly didn't evolve referential alarm calls because they had comparatively little to be alarmed about. Indeed, there is little that is learned at all in the vocal communication of nonhuman apes. 1


① A-C-B ② B-A-C ③ B-C-A ④ C-A-B ⑤ C-B-A

 


57. 20수완 16-01

 

In one experiment, researchers projected a picture that was completely out of focus onto a screen.


(A) Remarkably, the individuals who were the first to think they knew the identity of the picture (and write it down) were the last to correctly identify it. Why? 1

(B) The slide was then gradually brought into sharper and sharper focus. The participants were instructed to guess what the picture showed (a fire hydrant in one case), and write it down. 0

(C) Because rather than scratch through their initial opinion as the image became clearer and they were increasingly and obviously wrong, they clung to it. People who were among the last to make an identification were more successful — simply because their first choice had a better chance of being right. 2


① A-C-B ② B-A-C ③ B-C-A ④ C-A-B ⑤ C-B-A

 


58. 20수완 16-02

 

Imagine being in an art appreciation class and having to learn the styles of different artists.


(A) Traditional teaching methods would approach the works of each artist separately so as not to confuse the learners. In a fascinating study, participants were tasked with learning the styles of 12 artists based on a viewing of six sample paintings per artist. 0

(B) Now that you have learned Picasso, let's move on to Miro..."). The other half of the learners studied the paintings in a randomized fashion, with any given artist's paintings interwoven among those of the other artists. When all participants were later shown a series of new paintings (paintings by these artists that they had not seen during the learning phase) and were asked to identify the artist, the group that viewed the interwoven learning set was much better at identifying the correct artist. 2

(C) In the experiment, half the learners got the paintings in blocks as they would be presented in the typical classroom. ("Here are six paintings by Picasso. 1


① A-C-B ② B-A-C ③ B-C-A ④ C-A-B ⑤ C-B-A

 


59. 20수완 16-03

 

In addition to appropriating (and sometimes anticipating) the modern medical model for the diagnosis of crime, Arthur Conan Doyle also perfected a literary form for Sherlock Holmes that owed its origins to the medical profession as well — the case history.


(A) Medical diagnosis became based upon a principle of "differential" semiological examination of the patient's body in the form of the medical case history. 2

(B) The new discipline of pathological anatomy as a form of medical diagnosis in the period brought with it a new epistemological imperative for the physician to observe carefully every detail of the patient's environment and physical condition, and to record that information in the form of a case study that would explore the network of possible connections between the seemingly insignificant details. 0

(C) In the case history, the physician would test and retest every conceivable set of relations until the cause of the patient's illness made sense, much as Holmes would do in his explanations of cases to Watson. 1


① A-C-B ② B-A-C ③ B-C-A ④ C-A-B ⑤ C-B-A

 


60. 20수완 16-04

 

Genetic engineering followed by cloning to distribute many identical animals or plants is sometimes seen as a threat to the diversity of nature.


(A) Conversely, the renewed interest in genetics has led to a growing awareness that there are many wild plants and animals with interesting or useful genetic properties that could be used for a variety of as-yet-unknown purposes. This has led in tum to a realization that we should avoid destroying natural ecosystems because they may harbor tomorrow's drugs against cancer, malaria, or obesity. 2

(B) However, humans have been replacing diverse natural habitats with artificial monoculture for millennia. Most natural habitats in the advanced nations have already been replaced with some form of artificial environment based on mass production or repetition. 0

(C) The real threat to biodiversity is surely the need to convert ever more of our planet into production zones to feed the ever-increasing human population. The cloning and transgenic alteration of domestic animals makes little difference to the overall situation. 1


① A-C-B ② B-A-C ③ B-C-A ④ C-A-B ⑤ C-B-A

 


61. 20수완 T1-18

 

We at Bovice Children's Trust work for underprivileged children and sometimes need volunteers to help us in our project.


(A) He ensured that the children's parents understood the program that our organization was working for and thus it helped us a lot in succeeding in enrolling these children for our program. 1

(B) Besides, Chris also engaged in popularizing our organization mission that gave us the financial assistance as well as community participation by many organizations. Hence, I would like to confirm that Chris has put in about fifty hours of community service and a certificate for his service is enclosed along with this letter for your reference. 2

(C) A few months back, Chris came forward and helped us in reaching out to the underprivileged children in the Griffin Street region. 0


① A-C-B ② B-A-C ③ B-C-A ④ C-A-B ⑤ C-B-A

 


62. 20수완 T1-19

 

There were about ten people standing around a long table, dressed in black robes and chanting.


(A) He felt he was outside a window looking in. He tried to speak, but nothing came out of his mouth. He tried to run, but his legs wouldn't move. He wanted to leave, but something seemed to be holding him there. 1

(B) As he slowly started to move back, the circle of people turned and looked at him. They waved their hands for him to come to join them. He didn't want to join them; he was afraid of what they might do to him. Despite his struggle not to join them, he was pulled slowly toward them. He was no longer in control of his own body. 2

(C) David could not make out what they were saying. He only saw their mouths move in slow motion as if he was not there at all, and he felt his hands and knees trembling with fear. Then, he caught a glimpse of a table with a shadow of someone on it. He couldn't see the face. 0


① A-C-B ② B-A-C ③ B-C-A ④ C-A-B ⑤ C-B-A

 


63. 20수완 T1-20

 

If you want a friend, get a dog.


(A) In order to be seen as legitimate, the media must be seen as truthful, accurate, unbiased, and fair. 2

(B) Journalism professors and professionals have shared this humorous, colloquial saying with countless neophytes in classrooms and newsrooms because it points to a serious underpinning. The media generally and reporters in particular do not need to be loved or even to have their motives fully understood in order to carry out their obligations to inform the public. 0

(C) But, according to Stephen Klaidman and Tom Beauchamp in The Virtuous Journalist, it is essential that the public trust the press and see it as credible in its role as watchdog over governments and their agencies. Credibility is an attitude, a belief that citizens hold about whether the news media legitimately have the power to call out elected officials or others in high positions who are not playing by the rules. 1


① A-C-B ② B-A-C ③ B-C-A ④ C-A-B ⑤ C-B-A

 


64. 20수완 T1-21

 

Psychologist and author Jeremy Dean explains in relation to cultivating happiness-boosting habits that 'unfortunately there's rather a large fly in the ointment.


(A) Dean suggests that one way that we can deal with our automatic adaptation to pleasure is by varying our habits rather than repeating them in exactly the same way over and over again. This could mean, for example, making a conscious effort to respond more consciously to the question 'how are you' (rather than saying 'Fine' every time). Introducing conscious variations in some of our habits can be effective in reducing the effects of habituation. 1

(B) That fly is habituation'. Habituation means that we adapt to positive experiences more quickly than negative ones. This means that we lose the pleasure from good habits more quickly than the pain from bad ones. 0

(C) Although Dean explains that this idea stretches the formal definition of a habit which involves the same behaviour or thought in the same situation, for 'happy' habits we need an 'automatic initiation of the behaviour, but then a continuously mindful way of carrying it out. A new type of hybrid habit: a mindful habit'. 2


① A-C-B ② B-A-C ③ B-C-A ④ C-A-B ⑤ C-B-A

 


65. 20수완 T1-22

 

Social workers need to recognize that it is the client who owns the problem and therefore has the chief responsibility to resolve it.


(A) In sharp contrast, social workers seek to establish not an expert- inferior relationship but rather a relationship between equals. The expertise of the social worker does not lie in knowing or recommending what is best for the client; it lies in assisting clients to define their problems, to identify and examine alternatives for resolving the problems, to maximize their capacities and opportunities to make decisions for themselves, and to implement the decisions they make. Many students, when they first enter social work or some other helping profession, mistakenly see their role as that of "savior" or "rescuer." 2

(B) In this respect, social work differs markedly from most other professions. Most professionals, such as physicians and attorneys, advise clients about what they ought to do. 0

(C) Doctors, lawyers, and dentists are viewed as experts. Clients' decision making in such situations is generally limited to the professional's advice. 1


① A-C-B ② B-A-C ③ B-C-A ④ C-A-B ⑤ C-B-A

 


66. 20수완 T1-23

 

While memories in STM (short-term memory) are encoded acoustically, information held in LTM (long-term memory) seems to be different.


(A) When recall was delayed for 20 minutes, however, more errors were made on list items with similar meanings. This latter finding can be explained by the use of a semantic code in LTM; items were confused when they had similar meanings, so were less likely to be recalled accurately. 2

(B) Here, memories are encoded according to their meaning - that is, using a semantic code. Baddeley provided evidence for this in an experiment that required participants to learn lists of words and recall them. 0

(C) The lists contained some items that were acoustically similar (such as mad, man, cad, can) and others that were semantically similar (such as big, long, broad, high). When asked for immediate recall, participants' errors were affected by the similar-sounding words, reflecting the use of an acoustic code in STM. 1


① A-C-B ② B-A-C ③ B-C-A ④ C-A-B ⑤ C-B-A

 


67. 20수완 T1-24

 

One key to the problem of the shopping information gap is to realize that we need a better way of presenting and organizing information.


(A) Such a method is to organize information into different levels, where different amounts of consumer demand for information are presented at each level. Any shopper, at any point in the shopping experience, has a certain level of interest in products. 0

(B) If a consumer has a high level of interest then more information should be available to him. Conversely, if a consumer is indifferent to a product, then he should not be overwhelmed with information. 2

(C) Sometimes this level of interest is intense, where the consumer just cannot get enough, and sometimes this level is one of indifference, where the consumer is just browsing. The best way to convey information to consumers is to look at them as shopping on a number of possible levels of interest and to direct information to them according to their level. 1


① A-C-B ② B-A-C ③ B-C-A ④ C-A-B ⑤ C-B-A

 


68. 20수완 T1-25

 

The graph above shows the percentage of the US population with total diabetes (diagnosed and undiagnosed) and the percentage with undiagnosed diabetes both as a whole and by ethnic/racial subgroup in 2011 – 2012.


(A) Non-Hispanic Asians showed the highest proportion of undiagnosed diabetes among all of the ethnic/racial subgroups. All Hispanics showed the highest percentage of total diabetes, with 10 percent undiagnosed diabetes. 1

(B) The percentage of all Hispanics with undiagnosed diabetes was higher than that of overall Americans with undiagnosed diabetes. 2

(C) Non-Hispanic whites had the lowest percentage of total diabetes among all ethnic/racial subgroups. More than 1 in 5 non-Hispanic blacks had total diabetes, and non-Hispanic blacks had a lower proportion of undiagnosed diabetes than non-Hispanic Asians. 0


① A-C-B ② B-A-C ③ B-C-A ④ C-A-B ⑤ C-B-A

 


69. 20수완 T1-26

 

Caspar David Friedrich, a 19th-century German Romantic landscape painter, began with topographical drawings in pencil and sepia wash and did not take up oil painting until 1807.


(A) Friedrich had a severe stroke in 1835 and returned to his small sepias. He was virtually forgotten at the time of his death and his immediate influence was confined to members of his circle in Dresden, notably Georg Friedrich Kersting, who sometimes painted the figures in Friedrich's work. 1

(B) It was only at the end of the 19th century, with the rise of Symbolism, that Friedrich's greatness began to be recognized. Most of his work is still in Germany. 2

(C) His choice of subjects often broke new ground and he discovered aspects of nature so far unseen: an infinite stretch of sea or mountains, snow-covered or fog-bound plains seen in the strange light of sunrise, dusk, or moonlight. He seldom used obvious religious imagery, but his landscapes convey a sense of haunting spirituality. 0


① A-C-B ② B-A-C ③ B-C-A ④ C-A-B ⑤ C-B-A

 


70. 20수완 T1-29

 

Modern science emerged out of traditions that are clearly Western.


(A) It is important to recognize that thinking scientifically is not natural or automatic. Indeed learning to perceive the world in ways consistent with this dichotomous, observation-based perspective is something we must learn. For some of us, this may seem to be very natural. 1

(B) In actuality it is an extension of the cultural traditions within which we were raised. Classifying is an example of a very particular way of thinking that teachers must introduce to students to familiarize them with the culture of science. 2

(C) It seems that the process of sorting into either/or categories can be traced to ancient Greek thought. The process of organizing objects according to whether they have or do not have a particular property, with no intermediate category, is sometimes called Aristotelian. This very formal approach to classifying, unlike the version of classifying we might use in everyday living, makes no allowance for fuzziness. 0


① A-C-B ② B-A-C ③ B-C-A ④ C-A-B ⑤ C-B-A

 


71. 20수완 T1-30

 

Each of us views reality through the thoughts we have accumulated in our individual preserved state of awareness.


(A) Pay attention the next time you disagree with somebody. Notice how your mind instantly produces thoughts that validate your point of view. For example, as your friend defends a political candidate who you dislike, notice the rush of opposing thoughts that enter your mind. 1

(B) As this happens, consider for a moment where these thoughts came from and why you produced them at this particular moment. Consider their purpose. You did attract them and they do have a purpose. Their purpose is to reinforce your beliefs. 2

(C) When you encounter a new thought, you call upon this state to derive meaning, agree, disagree, judge, or remain neutral. Every thought you encounter or conjure up you filter through this state. You can observe this scrutiny as you produce thoughts that stimulate your feelings. 0


① A-C-B ② B-A-C ③ B-C-A ④ C-A-B ⑤ C-B-A

 


72. 20수완 T1-31

 

As an academic discipline, architecture is outside of the humanities — so in that sense there's no question that it is not one of the humanities.


(A) What I can say positively as an outsider is that architecture is a discipline seeking self-definition, and for that self-definition it looks outside of itself, to see what others say about it. 2

(B) Not institutionally at least, even if there is something of a family resemblance. Architecture students aren't oriented to thinking, reading, and writing in quite the same way as are students within the humanities. 0

(C) What's interesting about architecture is that it has always been unsure as to where to position itself and its own identity as a discipline: it is itself internally divided about whether it is a science, a technological discipline, or a mode of art or aesthetic production. This uncertainty regarding its own identity has led it to be quite open to philosophical and critical theory in a way that is unimaginable for other disciplines, like engineering or medicine, for example. 1


① A-C-B ② B-A-C ③ B-C-A ④ C-A-B ⑤ C-B-A

 


73. 20수완 T1-32

 

Important work regarding preservatives indicated that making organ meats look familiar (through their cuts, shapes, and packaging) influenced perceptions of taste.


(A) This insight was found during research on what made preserved foods most acceptable. At the beginning of World War II, there was a need for canned meats that tasted like fresh meat, for powdered milk that was reconstituted to taste like fresh whole milk, and for preserved bread that tasted like fresh bread. 0

(B) Because of this work, initial efforts introduced some organ meats as filler in ground beef and sausages. In both ground meat and sausage forms, replacing existing meat with organ meats was accepted because they did not cause the meat to look different than expected. 2

(C) The government pushed food companies to preserve foods to resemble fresh foods. Because they looked and tasted fresh, people believed they must be safe and that preservatives were not harmful. 1


① A-C-B ② B-A-C ③ B-C-A ④ C-A-B ⑤ C-B-A

 


74. 20수완 T1-33

 

Most people exhibit physical symptoms when they lie.


(A) This technology works nicely for most people. Most of us have brains that want to tell the truth, want to be trusted, and we feel guilty when we lie or when we think we will be caught. Our bodies respond to our thoughts and feelings. 1

(B) There is even new brain imaging lie detector equipment and companies springing up, such as No Lie MRI, to swear in court whether or not you are telling the truth. Not only does your body react to lies, your brain does as well. Whenever most people lie, their brain becomes overall much more active than when telling the truth. It really does take more out of you to lie than to tell the truth. 2

(C) Their bodies respond to the stress they feel when they think they will be found out. Immediately, their hearts beat faster as adrenaline bathes their system, their breathing becomes more shallow in response to feelings of panic, their hands become colder as blood flow becomes constricted, and their hands sweat and their muscles become tense. Examiners can see this pattern on the lie detector equipment and the liar is caught. 0


① A-C-B ② B-A-C ③ B-C-A ④ C-A-B ⑤ C-B-A

 


75. 20수완 T1-34

 

Social domain theory views emotions and moral judgments as reciprocal processes that cannot be disentangled.


(A) Rather, the assumption is that affective experiences are an important component of moral judgment and that the latter involves a complex integration of thoughts, feelings, and experiences. To borrow from Kant's famous saying, moral reasoning without emotion is empty; emotions without reasoning are blind. 1

(B) This view differs from emotivist or intuitionist approaches to morality, which are principally based on research with adults and give priority to emotional and implicit processes while avoiding reasoning as largely post hoc rationalizations. From the social domain perspective, this treatment of emotions and reasoning as distinct, opposing influences represents a false dichotomy. 0

(C) Children's affective experiences influence their understanding, encoding, and memory of moral violations and are part of a complex evaluative process. Information obtained from observing the affective consequences of acts for others, as well as past or immediate emotional responses to moral situations, may constitute the foundation on which moral understanding is constructed. 2


① A-C-B ② B-A-C ③ B-C-A ④ C-A-B ⑤ C-B-A

 


76. 20수완 T1-35

 

Creativity researchers now believe that creativity cannot always be defined as a property of individuals; creativity can also be a property of groups.


(A) In the same way, organization theorists and socioculturalists argue that problem solving in work groups, and learning in classrooms and informal settings, often occurs in social interactions characterized by group creativity. 1

(B) These researchers are still exploring the complex relationship between group creativity and the creativity of individual members of the group. These are different types of creativity; the interactional processes of group creativity that we can observe among improvising actors are not likely to look anything like the cognitive processes going on within any single actor's head. 2

(C) For example, the performance that is generated by an improvisational theater ensemble is the creative product of the entire ensemble; there is no way to attribute the performance to any single member of the group. 0


① A-C-B ② B-A-C ③ B-C-A ④ C-A-B ⑤ C-B-A

 


77. 20수완 T1-36

 

Socrates provided a theater of presence in Athens for the young people who gathered to watch him show that their elders — even the ones with the most brilliant reputations for wisdom — could not answer the vital questions he put to them, questions like "What is justice?"


(A) It was also dramatic, more tightly focused on the false claims to wisdom that people actually made in the public places of Athens, false claims that affected directly the young men who clustered around Socrates as his audience. And the elders who were refuted, did they gain wisdom? 1

(B) In most cases, they plainly did not. They were deep inside the space of theater, unable to see themselves, and all they knew at the end was that they had been humiliated by a fiendishly clever man. So they went away not wiser but more angry at Socrates and his cleverness. 2

(C) And so these watchers were drawn into philosophy as they learned something about human limitations. They could have picked this lesson up from comedy, but Socratic theater was more immediate, more transformative of watchers into thinkers. 0


① A-C-B ② B-A-C ③ B-C-A ④ C-A-B ⑤ C-B-A

 


78. 20수완 T1-37

 

Wildlife management in North America is complicated because most land is owned by individuals who can restrict public access or manage it as they see fit, so long as their actions do not adversely impact other people.


(A) When English colonies were established in America, colonial governments administered on the crown's behalf, and they owned the wildlife. When the colonies gained their independence following the American Revolution and became states, they continued to own wildlife but now on behalf of "the people" who had replaced the crown as the ultimate authority on which government was based. 1

(B) Wildlife, however, is not owned by individuals or landowners but by society. This is the result of English common law, which stated that wildlife was owned by the crown. 0

(C) Consequently, in North America, we have this interesting phenomenon in which society owns and manages the wildlife, but private individuals own and manage the land and habitat on which wildlife depend. Clearly a partnership is required between landowners and government if the wildlife resource is going to be managed wisely. 2


① A-C-B ② B-A-C ③ B-C-A ④ C-A-B ⑤ C-B-A

 


79. 20수완 T1-38

 

The concept of 'producer responsibility' reflects a fundamental legal principle for the allocation of responsibilities: the person acting must ensure that each action is performed in a way that does not cause damage or nuisance to other legally protected interests.


(A) It is generally accepted that these preventive responsibilities apply to manufacturing processes and include the waste generated by the manufacturing process. It seems also accepted that this principle of allocation of responsibilities also applies to manufactured goods. 1

(B) Products placed on the market must be safe and secure in general. For certain products, such as pesticides, genetically modified organisms and vehicles, the placing of these products on the market is subject to a prior licensing scheme. 2

(C) In relation to the manufacturing of goods, this concept requires operators to prevent pollution, hazards, the risk of damaging public health and damage to neighboring properties — or at least to minimize these negative externalities to a certain level defined by legislation and/or a specific permit. Hence, the preventative costs must be paid by the producer. 0


① A-C-B ② B-A-C ③ B-C-A ④ C-A-B ⑤ C-B-A

 


80. 20수완 T1-39

 

Up until 2 million years ago, the relative brain size of our hominid ancestors was the same as that of the great apes today.


(A) As our head started to increase in size to accommodate our expanding brains, this put pressure on hominid mothers to deliver their babies before their heads got too big. However, this is not a problem for our nearest non-human cousins, the chimpanzee. 1

(B) In terms of movement, chimps do not naturally walk upright and so did not develop a narrow pelvis. Their birth canals are large enough to give a relatively easier birth to their babies, which is why chimpanzees waddle when they do try to walk upright. They usually deliver by themselves in less than 30 minutes, whereas human delivery takes considerably longer and is most often assisted by other adults. 2

(C) However, something happened in our evolution to change the course of the development of our brain, which grew significantly larger. Human brain size increased to be 3-4 times larger than the brain of our ancestral apes. 0


① A-C-B ② B-A-C ③ B-C-A ④ C-A-B ⑤ C-B-A

 


81. 20수완 T1-40

 

Historians have observed that the peak age for scientific productivity was about 25 years of age in the year 1500, but by 1960 it was 37.


(A) There's been no increase in the productivity of innovators beyond middle age to make up for this shortened career, and as a result, there's been a decline in innovative output per researcher over the century. 2

(B) His data showed that the peak age increased because early age innovation is declining, and that's a result of the increased educational demands required to acquire the knowledge necessary to contribute to an innovation. At the beginning of the 20th century, great minds began their work at age 23; at the end, at age 31. 1

(C) The increasing complexity of scientific domains seems to have caused this increase; this complexity makes the ideation and elaboration rates decline, and this results in a later career peak. Educational psychologist B. F. Jones studied 700 Nobel Prize winners and technological inventors in the 20th century and found that over the course of the century, the greatest achievements occurred at later and later ages; the mean age at great achievement rose by about six years over the century, and in 2000, the peak age was 36 to 40. 0


① A-C-B ② B-A-C ③ B-C-A ④ C-A-B ⑤ C-B-A

 


82. 20수완 T1-4142

 

Environmental problems associated with agricultural production for export in developing countries can be difficult to tackle using conventional regulations because producers are typically so numerous and dispersed, while regulatory agencies are commonly inadequately funded and staffed.


(A) The authors found that organic certification did improve coffee growers' environmental performance. Specifically, they found that certification significantly reduced the use of pesticides, chemical fertilizers, and herbicides, and increased the use of organic fertilizers. In general, their results suggest that organic certification has a stronger causal effect on preventing negative practices than on encouraging positive ones. 2

(B) One study examined this question for certified organic coffee grown in Turrialba, Costa Rica, an agricultural region in the country's central valley, about 40 miles east of San Jose, the capital city. This is an interesting case because Costa Rican farmers face significant pressure from the noncertified market to lower their costs, a strategy that can have severe environmental consequences. In contrast, organic production typically not only involves higher labor costs, but the conversion from chemically based production can also reduce yields. In addition, the costs of initial certification and subsequent annual monitoring and reporting are significant. 1

(C) In principle, eco-certification of production could circumvent these problems by providing a means for the socially conscious consumer to identify environmentally superior products, thereby providing a basis for paying a price premium for them. These premiums, in turn, would create financial incentives for producers to meet the certification standards. Do socially conscious buyers care enough to actually pay a price premium that is high enough to motivate changes in the way the products are produced? Apparently, for Costa Rican coffee at least, they do. 0


① A-C-B ② B-A-C ③ B-C-A ④ C-A-B ⑤ C-B-A

 


83. 20수완 T1-4345

 

At the time, nobody in France would have accepted a woman as an apprentice, so I decided to move to Brussels for a three-year course.


(A) After returning to our Niedermorschwihr shop from Paris, I started baking and making pastry on a daily basis. Until then, my father had kept that only for weekends and celebrations. He was very pleased to accept this help because it gave him more time for his own passion to be a cook. As a baker, he had always worked like a cook, by experimenting, testing, tasting and correcting. Today, I take a similar approach with my jams. When you are creative, you cannot simply follow recipes. 1

(B) When I got back in 1978, my father had plans to expand his shop, and wanted me to work with him. However, I still had dreams of seeing the world. In the end, we found a compromise: since boys had to go away for a year for their military service, I, too, would be allowed to travel for a year. I knew that no laboratory in Paris would ever accept a woman, so I decided to enter an artistic baking competition — which I won. After this, Monsieur Peltier, who had the best store in Paris, invited me to join him for a year. He gave me a very thorough and precise training in baking and confectionery, and shared with me his avant-garde spirit. 0

(C) It is important to do what you like to do, the way you like to do it. However, I started feeling lonely in this tiny 360-inhabitant village which was too remote to attract anybody. I discussed the idea of opening a store in town with my father, on a busy street. He said, "If this were the condition for success, then how could you explain the success of Marc Haeberlin and his restaurant being awarded three Michelin stars in the tiny Illhaeusern in Alsace?" He told me to be patient, to play the long game, and to do what I love. Deep down, I knew he was right, and decided to stay and fully embrace pastry making at Niedermorschwihr. 2


① A-C-B ② B-A-C ③ B-C-A ④ C-A-B ⑤ C-B-A

 


84. 20수완 T2-18

 

Ms. Martineau and I wish to thank you for the pleasure of allowing us to interview you J last Thursday.


(A) We were delighted to learn about your superb academic record, and we also appreciated your attentiveness in listening to the description of our company's operations. However, we had many well-qualified applicants who were interested in the advertised position of human resources assistant. 0

(B) We hired a female graduate who had most of the qualities we sought. Although we realize that the job market is difficult at this time, you have our heartfelt wishes for good luck in finding what you are looking for. 2

(C) As you may have guessed, we were particularly eager to find a minority individual who could help us fill out our employment equity goals. Although you did not fit one of our goal areas, we enjoyed talking with you. 1


① A-C-B ② B-A-C ③ B-C-A ④ C-A-B ⑤ C-B-A

 


85. 20수완 T2-19

 

I watched with curiosity as a strange couple walked through the house taking measurements.


(A) It was a small one, but very poisonous. Our servant, Niti, was called in to dispose of it, but he was also at a loss for what to do. 2

(B) When I was at the threshold of the living room, Mother, seeming quite anxious, shouted, "Stay away, Pieter, stay away!" I soon learned what all the fuss was about. In their enthusiasm, the decorators had moved a few cartons of books that were stored in a corner and managed to disturb a snake, which slithered out and frightened them. 1

(C) The process took about two hours, so I lost interest and hurried back to my own project outside. I had nearly forgotten about our visitors when I heard a sudden scream. I raced back into the house. 0


① A-C-B ② B-A-C ③ B-C-A ④ C-A-B ⑤ C-B-A

 


86. 20수완 T2-20

 

Overconfidence about their social media skills prevents most social media managers from getting the necessary training.


(A) Be skeptical of the self-proclaimed social media mavens, rock stars, or experts who have no practical experience running, maintaining, and building online communities around causes or nonprofit brands on social networking sites. 2

(B) Take Webinars, attend boot camps, or go to conferences that focus on the how to of social media. The most valuable advice should be practical and should come from someone who actually manages online communities and works with these tools every single day. 1

(C) Every second of every day, nonprofits on various kinds of social media are making obvious mistakes that are counterproductive to their presence on these sites. Please get training! 0


① A-C-B ② B-A-C ③ B-C-A ④ C-A-B ⑤ C-B-A

 


87. 20수완 T2-21

 

After his failure in Edinburgh, Darwin went to the University of Cambridge to study for a basic arts degree, with the idea that he would become a clergyman.


(A) They inspired him to become a naturalist. John Henslow took him plant collecting in the Cambridge countryside. Adam Sedgwick went with him to Wales to study the local rocks and fossils. 1

(B) He passed his exams. Just. But Cambridge turned out to be vitally important because of the friendships he forged with the professors of botany and geology. 0

(C) After this tour with Sedgwick, Darwin had graduated from the university and was at a loose end, not sure what to do next. He was saved by an unusual offer: would he like to become the 'gentleman naturalist' on a surveying voyage aboard the ship HMS Beagle, led by Captain Robert Fitzroy of the Royal Navy? His father said no, but his uncle convinced him that it was actually a great idea for his son. The voyage on the Beagle was the making of Charles Darwin. 2


① A-C-B ② B-A-C ③ B-C-A ④ C-A-B ⑤ C-B-A

 


88. 20수완 T2-22

 

To attract the best people and succeed as a business, the authentic organization of the future will need to foster environments where creativity and innovation are at a premium, employees feel engaged and committed, and leadership pipelines are carefully cultivated for future success.


(A) Case in point: Back in the 1980s, the business division of a US publisher had one of the most widely diverse workforces. One senior editor had been part of a Washington think tank and was an expert on Asian culture; another held a PhD in American history; another had worked as a speechwriter for a US president and was an environmental activist. 1

(B) In our research, workplaces with those qualities look for an unusual kind of diversity, hiring people for differences that are more than skin deep. Differences in thought processes, frames of references and skills, among other things. 0

(C) There was also an associate editor who had interned at the New Yorker magazine and another who had a background in foreign affairs. Only two of those editors held MBAs — and this was the business division! 2


① A-C-B ② B-A-C ③ B-C-A ④ C-A-B ⑤ C-B-A

 


89. 20수완 T2-23

 

The pursuit of flavor is one path to a good life, a truth recently discovered by Americans that demands a new account of the meaning of food and its consumption.


(A) "Pursue pleasure only in moderation," say countless sages throughout history; "it's only transitory," according to the timeless. "It is inimical to spirituality," the bodiless would have us believe; "it will not lead to happiness," the ubiquitous self-help books tell us. 1

(B) Any discussion of food and its place in our lives must begin with the role of pleasure. Yet our attitude toward pleasure is ambivalent: "Love People, Not Pleasure," blares the New York Times; "There Is More to Life Than Being Happy," proclaims the Atlantic Monthly. 0

(C) We spend much time and many resources pursuing pleasure but then condemn it with a fervor usually reserved for death and taxes. 2


① A-C-B ② B-A-C ③ B-C-A ④ C-A-B ⑤ C-B-A

 


90. 20수완 T2-24

 

Mobile crowd sensing for smart cities can support efficient, safe and green mobility in urban environments.


(A) Environmental sensors may also report air or noise pollution levels. This enables tourists in unfamiliar places to make even better decisions than well informed locals might take. 2

(B) Here crowd sourcing can inform people about whether to seek alternative routes, when best to arrive at attractions or restaurants, how to avoid unpleasant surprises when traveling, where to park, and which public transport solution would be best. 1

(C) Given the ubiquity of mobile devices carried by people worldwide, social mobile crowd sensing through the IoT can allow tourists to know about popular events in a destination, provide interactive feedback with other tourists at different locations, reveal the best places to be at a certain time, local weather forecasts, and expected travel times throughout the day. 0


① A-C-B ② B-A-C ③ B-C-A ④ C-A-B ⑤ C-B-A

 


91. 20수완 T2-25

 

The graph above shows pet ownership in the US, with the total number of pets owned and the number of households that owned each type of pet in 2016.


(A) Regarding the total number of pets, freshwater fish were the most owned pets, whereas cats were the second most owned pets, at 139.3 million and 94.2 million pets respectively. The number of households that owned dogs was larger than the combined number of households that owned cats and households that owned freshwater fish. 0

(B) Lastly, among the six types of pets, horses were the least owned. 2

(C) Although the number of households with saltwater fish as pets was the smallest, as many as 18.8 million saltwater fish were owned. Households that owned birds ranked fourth among pet-owning households, with 7.9 million households. 1


① A-C-B ② B-A-C ③ B-C-A ④ C-A-B ⑤ C-B-A

 


92. 20수완 T2-26

 

The fame of French director Andre Cayatte reached its peak in the 1950s.


(A) As he had no valid papers, he was forced to work for Continental Films, the German production company in Paris, for which he made his first films from 1942 onwards. This was to cause him a certain amount of trouble after the liberation of Paris. 1

(B) Cayatte, a journalist, lawyer and novelist, became involved in cinema at the end of the 1930s by writing screenplays. He was taken prisoner in 1940, but managed to escape. 0

(C) After the Second World War, he gained the public's and the critics' attention through his committed films on issues of public concern, pleas against the death penalty, satires directed at the failings of the justice system, well-argued attacks against the weaknesses of family upbringing, and so on. He was strongly criticised by the film-makers of the New Wave, who accused him of concentrating too much on expounding an argument to the detriment of the aesthetic considerations of direction. He gradually withdrew from film-making in the 1970s. 2


① A-C-B ② B-A-C ③ B-C-A ④ C-A-B ⑤ C-B-A

 


93. 20수완 T2-29

 

In some ways, food media takes readers and viewers into the back of the house where gourmet cooking takes place.


(A) This is not to say such depictions are accurate, entertaining as they might be. As Andrew Chan notes, contemporary shows "create a gap that separates the viewer from the reality of actual cookery." 0

(B) As such, the media has made visible some of the hidden work that goes into producing our eating experiences but often in ways that are more fantasy than reality. 2

(C) In fact, Kathleen Collins explains that, "Television cooking shows, and the media in general, are largely responsible for glamorizing what will always be, in actuality, toiling, sweaty labor." She also concedes, however, that such media coverage — and the public's seemingly insatiable appetite for programming related to food and eating — "has brought deserved attention to a previously uncelebrated class of laborers and artisans." 1


① A-C-B ② B-A-C ③ B-C-A ④ C-A-B ⑤ C-B-A

 


94. 20수완 T2-30

 

DNA reproduces and replaces itself continually.


(A) A third-generation copy is slightly less sharp than the original, the fourth-generation copy even less so, and so forth. Eventually, the process breaks down; it is no longer possible to make a legible copy, and the message is lost. When this happens to DNA, the result is the death of a cell. 2

(B) Under optimal conditions, this reproduction is flawless and every copy is an exact replica of the one it replaces. This is normally true in children. As we age, however, we are increasingly vulnerable to factors that damage our DNA during replication. 0

(C) The body's process of DNA replication is similar to that of a copy machine making photocopies of an original, with each subsequent copy made from the previous photocopy. If the copy machine is in perfect condition, the first copy will hardly be distinguishable from the original. Gradually, though, even copies made on a good-quality machine will lose definition. 1


① A-C-B ② B-A-C ③ B-C-A ④ C-A-B ⑤ C-B-A

 


95. 20수완 T2-31

 

Sometimes, of course, we might have good reasons, conceptually and critically, to isolate the content of a film from its form.


(A) But limited access to some specific details, and the demands of cinematic storytelling, compelled the filmmakers to take liberties with the original content when giving the movie its ultimate form. Zero Dark Thirty invents and combines characters, rearranges and condenses action, and speculates on events. 1

(B) The subject of Kathryn Bigelow's Zero Dark Thirty is the tracking and eventual killing of the terrorist Osama Bin Laden by United States CIA agents and US Navy Seals. That content is historical fact. 0

(C) Many critiques of Zero Dark Thirty, noting the significance and sensitivity of the content, questioned the film's completeness, accuracy, and reliability. Yet the movie could be considered a formal success; it received Academy Award nominations for Best Picture, Actress, Original Screenplay, Film Editing, and Sound Editing. By focusing solely on content, we may risk overlooking the aspects that make movies interesting as individual works of art. 2


① A-C-B ② B-A-C ③ B-C-A ④ C-A-B ⑤ C-B-A

 


96. 20수완 T2-32

 

Even before Socrates the Greeks had come to a critical differentiation between natural and human phenomena.


(A) The crucial point of this distinction was that while nature, physis, provided the fixed context to which human beings could do nothing else but adjust their lives, human-made things and institutions, nomoi (plural of nomos), could, at least in principle, be changed by human activity. 2

(B) As early as the seventh or eighth century BC — as evidenced by the Iliad and the Odyssey —it had become customary to distinguish those objects in the surrounding world that appeared to remain always and everywhere the same — plants, animals, the seasons, the stars, and so forth — from others that seemed to vary from time to time and place to place, such as languages, customs, laws, political institutions, and so on. 0

(C) The Greeks came to refer to the former sorts of things collectively as physis, or what we would roughly translate as nature, and to the latter as nomos, or, approximately, custom or perhaps even culture. 1


① A-C-B ② B-A-C ③ B-C-A ④ C-A-B ⑤ C-B-A

 


97. 20수완 T2-33

 

The transition to a network society has, by all accounts, turned research on youth and media on its head — leading to new opportunities and new challenges.


(A) With the rapid and often short-lived developments in the media landscape, such research is far more complicated than it used to be. 2

(B) Perhaps most obvious is the fact that our object of study has become a moving target. Many of the media and communication technologies that we investigate today are continually changing — often while we try to understand the phenomenon in question. 0

(C) This is especially problematic for the social sciences, because social scientists often need sufficient time to answer questions with appropriate methodological rigor. To establish causal relationships between media use and longer-term outcomes, youth must be followed over several years, and their media use needs to be measured repeatedly. 1


① A-C-B ② B-A-C ③ B-C-A ④ C-A-B ⑤ C-B-A

 


98. 20수완 T2-34

 

Some species have evolved not only a theory of mind but also, distinctly, a theory of relationships — which is evolutionarily advantageous, because recognizing relationships between other individuals helps predict their social behavior.


(A) The most basic type of such knowledge is when one animal knows the relative dominance rank of two other animals, not just its own rank with respect to the others. This important ability is widespread, seen in hyenas, lions, horses, dolphins, and, of course, primates, but also in fish and birds. 0

(B) All three animals understand what it means for two of them to have a special bond. 2

(C) Capuchin monkeys in conflict preferentially seek out allies that they know to be higher ranked than their opponents, and they also seek out allies that they know have closer relationships with themselves than with their opponents. If two chimpanzees have a fight and a bystander offers consolation to the loser, this can reconcile the two combatants, but only if the bystander has a friendship with the aggressor. 1


① A-C-B ② B-A-C ③ B-C-A ④ C-A-B ⑤ C-B-A

 


99. 20수완 T2-35

 

Social systems exist to mitigate risk: that is why human beings live in communities in the first place.


(A) A society in which people consider themselves to be one big family is more likely to show resilience in a disaster or prolonged stress than one in which each individual models his or her behavior on self-reliant individualism, all other things being equal. This is the fundamental idea behind insurance, extended family networks of assistance, and disaster assistance. 1

(B) Therefore this class of problem often depends importantly on the level of resilience a society demonstrates under stress. 2

(C) People living in groups can protect themselves better from enemies or predators, share resources, and help one another beyond the immediate kinship group. Through technology, trade, and political organization, organized communities under stress can access help that will offset the local impact of ecological problems and diffuse the impact over a much larger area or population. 0


① A-C-B ② B-A-C ③ B-C-A ④ C-A-B ⑤ C-B-A

 


100. 20수완 T2-36

 

Daniel Merton Wegner was the first to analyze transactive memory, especially as it occurs in couples.


(A) Wegner was especially interested in the use of people as memory aids. He speculated that a transactive memory system may develop in many groups to ensure that important information is remembered. 1

(B) He noted that people often supplement their own memories, which are limited and can be unreliable, with various external aids. These include objects (eg, address or appointment books) and other people (eg, friends or coworkers). 0

(C) This system combines the knowledge possessed by individual group members with a shared awareness of who knows what. So when group members need information, but cannot remember it themselves or doubt that their memories are accurate, they can tum to each other for help. A transactive memory system can thus provide a group's members with more and better information than any of them could remember alone. 2


① A-C-B ② B-A-C ③ B-C-A ④ C-A-B ⑤ C-B-A

 


101. 20수완 T2-37

 

The manner in which people communicate has some obvious relationship to the way in which their society is organized and governed.


(A) In ancient oral societies, people were ruled by unpredictable gods who worked in mysterious ways. When human beings learned how to cast their messages and conversations into written language, they were able to begin to organize their lives in more rational systems. 0

(B) This posed a severe challenge to those among them who were priests and holy men and women; obviously these leaders regarded writing as subversive, destructive, and sinful. It didn't take them long to realize that if they could control the writing, they could regain their power as emissaries of the gods. 2

(C) They could record history and discover patterns in the weather, their food sources, and the ways of their enemies. They could take more control of their lives, solve some of nature's mysteries, and become less dependent on the whims of their gods. 1


① A-C-B ② B-A-C ③ B-C-A ④ C-A-B ⑤ C-B-A

 


102. 20수완 T2-38

 

The hypothalamus is the part of the brain that receives messages from nerves about the internal condition of the body and the external environment.


(A) For example, cells may resist the signal of the hormone insulin, causing the pancreas to overwork, secreting ever more insulin. An excess of insulin causes a series of effects on other hormones, and some become elevated while others become depressed. Many hormone imbalances can be corrected nutritionally if they are detected early enough, but some may require medication. 2

(B) It responds to this information by sending signals to the appropriate glands to release hormones. Once hormones are released into the bloodstream they travel to particular target cells to deliver their chemical messages and cause a specific biological response. 0

(C) Ideally, our hormones exist in a delicate balance and deliver their messages to the cells accurately and reliably. But hormone imbalances become increasingly common with advancing age. 1


① A-C-B ② B-A-C ③ B-C-A ④ C-A-B ⑤ C-B-A

 


103. 20수완 T2-39

 

The primary perceptual faculty in human beings, as in all primates, is vision.


(A) Until the 1970s, Zeki points out, neurological models of perception were heavily influenced by the mistaken philosophic view, probably traceable to Kant, that "sensing" reality and "understanding" (grasping) it are fundamentally disparate phenomena. Now, however, through sophisticated techniques for studying both normal subjects and patients who have suffered various impairments of brain function, we know that normal perception entails simultaneous "seeing" and "understanding." 1

(B) Specialized areas of the brain not only detect visible attributes such as color, form, and motion but also "identify" and integrate them into a unified, coherent "picture." The integration of visual information that results constitutes both sensory perception and recognition of the visual world. 2

(C) Much of the improved understanding of perceptual processes has therefore derived from the neurobiology of visual perception. As Semir Zeki, a principal researcher in the field, has observed, the study of vision is a "profoundly philosophical enterprise," for it constitutes an inquiry into "how the brain acquires knowledge of the external world, which is no simple matter." 0


① A-C-B ② B-A-C ③ B-C-A ④ C-A-B ⑤ C-B-A

 


104. 20수완 T2-40

 

Choosing similar friends can have a rationale.


(A) However, the carrying capacity of a given environment places a limit on this strategy. If resources are very limited, the individuals who live in a particular place cannot all do the exact same thing (for example, if there are few trees, people cannot all live in tree houses, or if mangoes are in short supply, people cannot all live solely on a diet of mangoes). 1

(B) Assessing the survivability of an environment can be risky (if an environment turns out to be deadly, for instance, it might be too late by the time you found out), so humans have evolved the desire to associate with similar individuals as a way to perform this function efficiently. This is especially useful to a species that lives in so many different sorts of environments. 0

(C) A rational strategy would therefore sometimes be to avoid similar members of one's species. 2


① A-C-B ② B-A-C ③ B-C-A ④ C-A-B ⑤ C-B-A

 


105. 20수완 T2-4142

 

Talents, unlike strengths, are largely innate.


(A) For example, it is possible to train oneself to be diligent, patient, and kind. Strengths can be acquired by almost any individual as long as there is time, effort and commitment. Strengths are therefore usually far more voluntary; there are many choices when it comes to strengths. You can decide whether you want to have a particular strength, whether you want to use it and develop a strength even further, whether to keep building it and to some extent whether to acquire it in the beginning. Also, strengths cannot be squandered. 2

(B) Of course they involve some choices, but the choices are in the context of choosing whether to use and develop the talent rather than about possessing that talent in the beginning. Talents are abilities that are brought to the surface when we begin to use them when pursuing various activities. Hence the importance of encouraging young children to engage in a variety of experiences so that they have opportunities to discover their talents. For example, some very young children naturally have good hand-eye coordination which makes them very good at being able to do things such as hitting a ball. 0

(C) This talent can be enhanced to the point of exceptional ability. Malcolm Gladwell suggests, however, that this requires ten thousand hours of practice before the age of 18 years. Strengths, on the other hand, are personality traits that can be acquired through active intentions such as practising, reading about them or learning from others. Just because we were not born with certain personality traits doesn't mean we can't develop them. 1


① A-C-B ② B-A-C ③ B-C-A ④ C-A-B ⑤ C-B-A

 


106. 20수완 T2-4345

 

President Carter used the power of emotions during the historic peace negotiations between Israel and Egypt.


(A) This groundwork did not just "happen." It took honest work. Carter and Begin began to establish rapport at their first meeting more than a year prior to the negotiation. They met at the White House, where Carter invited the Prime Minister for an open, private discussion about the Mideast conflict. Months later, Carter and his wife invited Begin and his wife to a private dinner, where they talked about their personal lives. Later, during the Camp David negotiation, Carter demonstrated that he was looking out for each party's welfare. For example, before Begin met with Sadat for the first time at Camp David, Carter alerted Begin that Sadat would present an aggressive proposal; he cautioned Begin not to overreact. Carter did not want the negotiation to fail, nor did Begin or Sadat. Positive emotions between Carter and each leader helped to move the negotiation forward. 2

(B) He invited Israel's Prime Minister, Menachim Begin, and Egypt's President, Anwar Sadat, to Camp David. His goal was to help the two leaders negotiate a peace agreement. After thirteen long days, the negotiation process was breaking down. The Israelis saw little prospect for reaching agreement. By this time, Carter had invested a lot of time and energy in the peace process. He could easily have expressed frustration, perhaps approaching Begin with a warning to accept his latest proposal "or else." But an adversarial approach might have caused Begin to abandon the negotiation process completely. Instead, Carter made a gesture that had a significant emotional impact. Begin had asked for autographed pictures of Carter, Sadat, and himself to give to his grandchildren. 0

(C) Carter personalized each picture with the name of a Begin grandchild and handed Begin the photographs. Begin saw his granddaughter's name on the top photograph and spoke her name aloud. His lips trembled. He shuffled through the photographs and said each grandchild's name. He and Carter talked quietly about grandchildren and about war. This was a turning point in the negotiation. Later that day, Begin, Sadat, and Carter signed the Camp David Accord. The open discussion between Carter and Begin could not have happened if there were a poor relationship between them. The groundwork of positive emotions allowed nonthreatening conversation about serious differences. 1


① A-C-B ② B-A-C ③ B-C-A ④ C-A-B ⑤ C-B-A

 


107. 20수완 T3-18

 

Mr Liam Baker, wants to return a pair of shoes he bought from this showroom a week ago.


(A) Mr Newman and I tried to convince him about our policies and explained worn shoes are not refundable, but it was all in vain. Finally, to prevent him from creating a scene, we promised that we would let him know our decision after consulting the head office. 1

(B) He claims that there is a manufacturing defect in the design. In fact, he is accusing us of selling seconds from this shop. 0

(C) I have also been successful in convincing him to take the shoes back with him. But he will certainly want an answer from us within a couple of days. Please advise us as to what course of action we should take. 2


① A-C-B ② B-A-C ③ B-C-A ④ C-A-B ⑤ C-B-A

 


108. 20수완 T3-19

 

Jake lifted his head high and tried to control his breathing.


(A) She said it with such conviction that Jake's heart felt as if it would explode. Jake then repeated the same vows. He placed the ring upon Leah's finger. Someone cleared their throat. "You may now kiss your bride." 2

(B) Leah smiled at him, and his heart beat double time. He still had a hard time believing that Leah loved him and would soon be his wife. He only half listened to the wedding vows. Leah's voice was soft when she vowed to "love and to cherish, until death do us part." 1

(C) He would start his new life with Leah. His heart pounded in his chest with joy. Jake watched as a vision of beauty floated down the stairs. He'd never seen anyone look as lovely in his whole life. 0


① A-C-B ② B-A-C ③ B-C-A ④ C-A-B ⑤ C-B-A

 


109. 20수완 T3-20

 

Most of the eating people do while standing up is impulse eating, like nibbling on something you see when opening the fridge to get something else out, grabbing a chocolate as you walk past an open box sitting on your colleague's desk or taking little bites as you prepare food or clean up after meals.


(A) But here's the thing: all the little nibbles here and there do count and will have consequences. 1

(B) It's important that you make every bite count. Not only will eating sitting down make you feel more physically satisfied, it will also make you feel more psychologically satisfied if you see a full meal or snack spread out in front of you rather than grabbing bites here or there while on the move. 2

(C) There's a real tendency to think that little nibbles here or there while going about your business don't actually count or won't have any consequences. 0


① A-C-B ② B-A-C ③ B-C-A ④ C-A-B ⑤ C-B-A

 


110. 20수완 T3-21

 

The objective of battle, to "throw" the enemy and to make him defenseless, may temporarily blind commanders and even strategists to the larger purpose of war.


(A) And one side's will has to be transmitted to the adversary at some point during the confrontation (it does not have to be publicly communicated). A violent act and its larger political intention must also be attributed to one side at some point during the confrontation. History does not know of acts of war without eventual attribution. 2

(B) This insight was famously captured by Clausewitz's most famous phrase, "War is a mere continuation of politics by other means." To be political, a political entity or a representative of a political entity, whatever its constitutional form, has to have an intention, a will. That intention has to be articulated. 1

(C) War is never an isolated act, nor is it ever only one decision. In the real world, war's larger purpose is always a political purpose. It transcends the use of force. 0


① A-C-B ② B-A-C ③ B-C-A ④ C-A-B ⑤ C-B-A

 


111. 20수완 T3-22

 

In the field of education, the sequence of human ages is reflected in the relationship between teacher and student.


(A) What needs to be remembered, however, is that classics is not simply a synonym for oldness or for wisdom construed in the past tense. Treated in this manner, the classics would be a pastime reserved only for teachers and older people, which would vitiate the need for continuity of transmission. 2

(B) A short label for these ripened fruits of learning is the "classics," and in this sense, every major cultural tradition on earth can boast a storehouse of classical texts and insights. 1

(C) In past centuries, this relationship was a close and intimate bond of apprenticeship (paradigmatically captured in the Indian formula of guru-shishyaparampara) in which the teacher transmitted to the student not only information but also the continuity of a tradition of learning, the fruits of the slow labor of intellectual and moral seasoning (far removed from clever dexterity). 0


① A-C-B ② B-A-C ③ B-C-A ④ C-A-B ⑤ C-B-A

 


112. 20수완 T3-23

 

The words used in survey questions should not trigger biases, unless doing so is the researcher's conscious intent.


(A) Take the difference between welfare and assistance for the poor. On average, surveys have found that public support for more assistance for the poor is about 39 percentage points higher than for welfare. Most people favor helping the poor. 1

(B) Biased words and phrases tend to produce misleading answers. Some polls ask obviously loaded questions, such as "Isn't it time for Americans to stand up for morality and stop the shameless degradation of the airwaves?" Especially when describing abstract ideas (eg, freedom, justice, fairness), your choice of words can dramatically affect how respondents answer. 0

(C) Most people oppose welfare. The "truly needy" gain our sympathy, but "loafers and bums" do not. 2


① A-C-B ② B-A-C ③ B-C-A ④ C-A-B ⑤ C-B-A

 


113. 20수완 T3-24

 

Competitive activities can be more than just performance showcases where the best is recognized and the rest are overlooked.


(A) Performance feedback requires that the program go beyond the "win, place, or show" level of feedback. Information about performance can be very helpful, not only to the participant who does not win or place but also to those who do. 2

(B) The provision of that type of feedback can be interpreted as shifting the emphasis to demonstrating superior performance but not necessarily excellence. The best competitions promote excellence, not just winning or "beating" others. The emphasis on superiority is what we typically see as fostering a detrimental effect of competition. 1

(C) The provision of timely, constructive feedback to participants on performance is an asset that some competitions and contests offer. In a sense, all competitions give feedback. For many, this is restricted to information about whether the participant is an award- or prizewinner. 0


① A-C-B ② B-A-C ③ B-C-A ④ C-A-B ⑤ C-B-A

 


114. 20수완 T3-25

 

The two pie charts above show past world energy consumption for 2011 and projected world energy consumption for 2040.


(A) "Biomass" accounted for 4 percent of world energy consumption for 2011, ranking fifth. By 2040, the consumption of "Other Renewables" and "Nuclear" is expected to increase by 1 percentage point, respectively. 1

(B) In 2011, the combined share of "Petroleum", "Natural Gas", and "Coal" accounted for more than three quarters of world energy consumption, and the share of each fuel is expected to decrease except for "Natural Gas" in 2040. "Petroleum" is expected to remain the most consumed fuel, even with its share projected to fall from 3 7 percent in 2011 to 31 percent in 2040. 0

(C) "Hydropower" was the second least consumed energy source at 3 percent in 2011 and is expected to account for the same share in 2040. 2


① A-C-B ② B-A-C ③ B-C-A ④ C-A-B ⑤ C-B-A

 


115. 20수완 T3-26

 

The eagle dance is a dramatization of the relationship believed to exist between man and the eagle and deific powers.


(A) On the head, is a close fitting headdress covered with feathers; the eyes are indicated, and at the front, is a long, curved beak —in all, a very good representation of an eagle's head. Over the shoulders and attached to the arms, are great feathered wings, and a feathered tail is attached to the belt in the back. This dance is a favorite with the public and is frequently performed at public exhibitions. 2

(B) Although the costume may vary from pueblo to pueblo, the basic features are the same. Each dancer's body is painted realistically; he wears a kilt, usually decorated with an undulating snake design. 1

(C) Two young men are costumed as eagles, one a male and the other female; in the course of the dance, they imitate almost every movement of these great birds. One sees them in the act of soaring, hovering over the fields, circling, perching on high places, and resting on the ground. 0


① A-C-B ② B-A-C ③ B-C-A ④ C-A-B ⑤ C-B-A

 


116. 20수완 T3-27

 

Free Coffee Event This is an Awesome Gift of Coffee for Coffee Lovers by Kaffa Kaldi.


(A) But you can only redeem one coffee from each outlet per week. Are there minimum age requirements to enter the event? - There is no minimum age to redeem a free coffee. But we do not encourage children below the age of 16 to drink coffee. 2

(B) FAQs : How can this event last for a full year? - The sponsors for this agree to extend this event for the whole of 2020, so you can enjoy a free coffee on a weekly basis. So, how many free coffees can I redeem per week? - You can redeem a free coffee from any of the outlets on the list. 1

(C) Where can I get a free coffee? Sign up today and we will send you information on where and how to get your free coffee. There is no purchase required (though some very good and reasonably priced food is being offered for all participants of this program). Simply show your vouchers to redeem your free coffee! 0


① A-C-B ② B-A-C ③ B-C-A ④ C-A-B ⑤ C-B-A

 


117. 20수완 T3-28

 

Fun Run and Walk Saturday, October 10, 2020 Red South Beach, San Diego, CA Join Barnes Community Clinic (BCC) for our first Fun Run and Walk event!


(A) Register by September 30 to ensure your bib has your name on it. • Event Details - 3 Mile Run: 9 a.m. - 1 Mile Walk: 9:30 a.m. - Barnes Community Festival: 11 a.m. The festival is held in downtown Barnes, featuring sidewalk sales and family entertainment from a wide variety of local vendors. 2

(B) • Register Early and Save - Online registration: $20 until September 15, $25 until September 30 - Race day registration: $30 (opens 7 a.m.) • Amenities The first 500 participants who register online get a free T-shirt. Plus, everyone gets a beautiful custom bib. 1

(C) This is a fun noncompetitive event, and open to everyone. The best part is that 100% of the registration fees go back directly to BCC, helping to give patients the best medical care possible. 0


① A-C-B ② B-A-C ③ B-C-A ④ C-A-B ⑤ C-B-A

 


118. 20수완 T3-29

 

In graphic form and function, the newspaper is coming to resemble a computer screen, as the combination of text, images, and icons turns the newspaper page into a static snapshot of a World Wide Web page.


(A) USA Today in fact makes considerable use of "hypertextual" links back and forth throughout its pages, and these links are sometimes cued by small graphics. The purpose of these icons together with the other pictures and graphics is not merely decorative. 1

(B) Together they help to redefine the function of the newspaper, which is no longer only to transmit verbal information, but also to provide an appropriate visual experience and through that experience dictate an appropriate reaction to the stories being told. 2

(C) In many newspapers the index now consists of summaries gathered in a column running down the left-hand side of the page, and a small picture is often included with the summary. Anyone familiar with multimedia presentations can easily read such a picture as an iconic button, which the user would press in order to receive the rest of the story. 0


① A-C-B ② B-A-C ③ B-C-A ④ C-A-B ⑤ C-B-A

 


119. 20수완 T3-30

 

"What's in a name?


(A) What a work of art is titled, on the other hand, has a significant effect on the aesthetic face it presents and on the qualities we correctly perceive in it. A painting of a rose, by a name other than the one it has, might very well smell different, aesthetically speaking. The painting titled Rose of Summer and an indiscernible painting titled Vermillion Womanhood are physically, but also semantically and aesthetically, distinct objects of art. 2

(B) That which we call a rose, by any other name would smell as sweet." This thought of Shakespeare's points up a difference between roses and, say, paintings. Natural objects, such as roses, are not interpreted. 0

(C) They are not taken as vehicles of meanings and messages. They belong to no tradition, strictly speaking have no style, and are not understood within a framework of culture and convention. Rather, they are sensed and savored relatively directly, without intellectual mediation, and so what they are called, either individually or collectively, has little bearing on our experience of them.1


① A-C-B ② B-A-C ③ B-C-A ④ C-A-B ⑤ C-B-A

 


120. 20수완 T3-31

 

There is a long-standing debate within psychology as to whether emotions per se are stored in memory.


(A) Remembering the circumstances in which an emotion was experienced also may cause people to experience a similar but new emotion in the present, and it is this new emotion that is then reported. As William James put it, "The revivability in memory of the emotions, like that of all the feelings of the lower senses, is very small.... We can produce, not remembrances of the old grief or rapture, but new griefs and raptures, by summoning up a lively thought of their exciting cause." 2

(B) According to this view, when asked to remember emotions, people retrieve not the fleeting emotional experience but a redescription of it based on memory for relevant details concerning the event or based on beliefs about how one is likely to have felt. 1

(C) Some investigators argue that emotion cannot be stored in memory but must be reconstructed based on knowledge concerning the circumstances in which the emotion was experienced. 0


① A-C-B ② B-A-C ③ B-C-A ④ C-A-B ⑤ C-B-A

 


121. 20수완 T3-32

 

The apparent "mess" of the bacterial domain is understandable.


(A) From the examples of the macrobiological world, it is clear that lifestyle or morphology is only of limited use to establish relatedness, and many bacteria look more or less the same under a microscope. So how should we group bacteria, if not by their looks and behavior? 0

(B) For example, shigellosis is a type of severe diarrhea caused by Shigella species, for instance Shigella dysenteriae, which, by objective criteria, are just particular nasty brands of E. coli (the "E." of E. coli stands for the genus Escherichia). There is no scientific reason to grant Shigella bacteria their own genus name, but taxonomists have not renamed Shigella bacteria to be incorporated into the Escherichia genus — yet. 2

(C) In the old days, when research was dedicated to medical microbiology, distinctions were frequently made based on the diseases bacteria could cause. This has led to some inaccurate classifications that we live with even today. 1


① A-C-B ② B-A-C ③ B-C-A ④ C-A-B ⑤ C-B-A

 


122. 20수완 T3-33

 

Many people, perhaps even you, associate philosophy with an image of bearded men in togas discussing deep thoughts.


(A) For example, in history, it is absolutely necessary to know specific facts about events. You can't study history just by reasoning or by thinking about ideas. Likewise, you can't do psychology without having a certain scientific view toward human nature and observing actual patterns in human behavior (this is why psychology is, in fact, a relatively recently developed discipline). 2

(B) There were such people in fact, and our Western intellectual heritage owes a lot to the ancient Greeks (which is why we have this mental picture). But it wasn't just the ancient Greeks who did philosophy. Philosophy is a basic human intellectual activity that has been pursued in every advanced culture since time immemorial. 0

(C) Its universality and timeless appeal should tell you something about it right off the bat. The doing of philosophy does not depend on any particular body of outside knowledge. This distinguishes it from any of the other "social sciences." 1


① A-C-B ② B-A-C ③ B-C-A ④ C-A-B ⑤ C-B-A

 


123. 20수완 T3-34

 

Musical judgments are never made in complete isolation.


(A) The formation of "taste cultures" has always been socially defined. Participation in certain genres of music was historically determined by a person's social position, not by a purely independent aesthetic choice. Indeed, from a sociological perspective, taste is always a social category rather than an aesthetic one; it refers to the way we use cultural judgments as social "currency," to mark our social positions. 0

(B) These factors contribute to a sense of the relativity of any single position. Contemporary musical choices are plural as never before, and the effect of that plurality is inevitably to confirm that, in matters of musical judgment, the individual can be the only authority. 2

(C) This may be less clear today, since contemporary society is characterized by the fragmentation of older taste cultures and the proliferation of new ones. In this context, cultural transactions take place with increasing rapidity — hence the heating up of the cultural economy and its rapid turnover of new products. Not only are taste cultures themselves shifting, but people now tend to move between them with greater ease. 1


① A-C-B ② B-A-C ③ B-C-A ④ C-A-B ⑤ C-B-A

 


124. 20수완 T3-35

 

Mathematics and logic are closely related, and in many branches of science forms of mathematics are available which are suitable for the deduction of the consequences of hypotheses.


(A) Nevertheless a sacrifice is usually made when reliance is placed on mathematics, because the existing forms of mathematics are adequate only for simplified cases. 1

(B) For example, much of organic chemistry has been developed with little assistance from mathematics. As a consequence the methods of argument which have developed in this field are not exact and certain, but they are applicable to a very wide range of problems quite beyond the reach of more formal procedures. 2

(C) When this is so, much more elaborate and far-reaching deductions become possible because of the great power of mathematical notation and methods, which permit deductions to be made that would be overwhelmingly complex if argued in ordinary language. 0


① A-C-B ② B-A-C ③ B-C-A ④ C-A-B ⑤ C-B-A

 


125. 20수완 T3-36

 

As we trap more heat in the atmosphere, weather patterns become more volatile.


(A) That 7 percent isn't uniform, though. The greater moisture capacity of the air means that water can be sucked out of one area and deposited in another. Moisture becomes more concentrated in a few times and places, leading to droughts in one area or one season, followed by torrential rains in another. 1

(B) And indeed, across the United States, while precipitation is up 6 percent, droughts are also up, and the amount of rain that falls in the hardest 1 percent of rainstorms is up 20 percent. There's more rain, concentrated in fewer places, with more left to dry out, and those that do get rain receive more intense rainstorms. 2

(C) A rise in temperature of 1 degree can translate into an increase in the most severe extremes by several degrees. And warmer au can move more moisture around more quickly. For every degree Celsius that the planet warms, the atmosphere can absorb 7 percent more moisture. 0


① A-C-B ② B-A-C ③ B-C-A ④ C-A-B ⑤ C-B-A

 


126. 20수완 T3-37

 

Personal adornment may reflect inner emotional states called mood.


(A) The culturally encouraged interpretation is that the redirection of eye movements required by line contrasts is symbolic of a dynamic inner state within the wearer. Thus, at least for Americans, contrasting line and color in costume can express exuberant mood to others and also reinforce the same mood in the wearer. 2

(B) It may also reinforce, disguise, or create mood. An individual caught up in a certain mood may wish to externalize it so it can be conveyed to and shared with others. Perhaps an individual feels light-hearted and energetic. 0

(C) In America, a culturally recognized way to create this effect is to choose costume with colors and linear arrangements that show contrast rather than sameness. Typically, color contrast can be attained by using a number of different colors together, by using bright colors that contrast with a somber background, and by using sharp differences in lightness and darkness of color. Linear contrast occurs if lines suddenly change direction or intersect when proceeding in different directions. 1


① A-C-B ② B-A-C ③ B-C-A ④ C-A-B ⑤ C-B-A

 


127. 20수완 T3-38

 

The most striking characteristic of state public health law — and the one that underlies many of its defects — is its overall antiquity.


(A) When many of these statutes were written, public health (eg, epidemiology and biostatistics) and behavioral (eg, client-centered counseling) sciences were in their infancy. Modem prevention and treatment methods did not exist. 2

(B) These laws often do not reflect contemporary scientific understandings of injury and disease (eg, surveillance, prevention, and response) or legal norms for protection of individual rights. Rather, public health laws use scientific and legal standards that prevailed when they were enacted. Society faces different sorts of risks today and deploys different methods of assessment and intervention. 1

(C) Certainly, some statutes are relatively recent in origin. However, much of public health law was framed in the late nineteenth and early to mid-twentieth centuries and contains elements that are 40 to 100 years old. Old public health statutes are often outmoded in ways that directly reduce their effectiveness and conformity with modern standards. 0


① A-C-B ② B-A-C ③ B-C-A ④ C-A-B ⑤ C-B-A

 


128. 20수완 T3-39

 

The lesson of history is that civilizations have never learned to live in harmony with their environment.


(A) Instead, by depleting natural resources, they eventually reached a point where they could no longer maintain their civil and military infrastructure adequately. At that point, they became vulnerable to invaders who wiped them out. This pattern seems to have been the case for the Mayans, for Angkor and for Sumeria. 2

(B) There is abundant historical evidence to show that disregard for the environment leads to human disaster. The current furore over climate change is reminiscent of events within many doomed societies. 0

(C) Archaeological research shows that many, perhaps most, ancient civilizations destroyed themselves by degrading their environment. In most cases this did not mean that people all suddenly died of hunger or were consumed by storms. 1


① A-C-B ② B-A-C ③ B-C-A ④ C-A-B ⑤ C-B-A

 


129. 20수완 T3-40

 

In the few studies indicating somewhat favorable outcomes for aggressive humor, we note that mildly aggressive humor, when enacted as teasing or ridicule, may serve as a means of delivering certain types of messages.


(A) Participants viewed one of three videos (ie other-ridicule, self-ridicule, or no-target humor) and then performed different tasks to assess fear of failure, conformity, and creativity. Compared to participants who viewed the self-ridicule or no-target humor video, those in the other-ridicule condition were more conforming in completing their tasks and indicated a greater fear of failure. 2

(B) For example, research by Janes and Olsen examined young adults' perceptions of peer teasing and ridicule, which they labeled "jeer pressure." 0

(C) The researchers studied whether observing peers being ridiculed and teased in a video was related to college students' fear of failure, creativity, and conformity. 1


① A-C-B ② B-A-C ③ B-C-A ④ C-A-B ⑤ C-B-A

 


130. 20수완 T3-4142

 

Rather than offering suggestions to your students, offer them questions instead.


(A) By responding with questions you avoid being prescriptive. Asking a student questions about his writing, as would an honest reader attempting to understand the student's point of view, allows you to indicate places where his logic breaks down, to inquire about missing data or ideas, or to register confusion. Asking questions doesn't fix the broken logic, fill in the missing data, or clear up the writing that's confused you. It's up to the student to do these things as he adjusts his writing in response to your questions. 0

(B) We should read as though we are coming to the topic afresh, with no more knowledge than our students present us in their work. We should allow ourselves to become confused when a student's writing wanders and to lose our way when that writing is unclear. Challenge the student to understand why it is you are confused and to confront that confusion by adjusting her explanation. After all, it's her task to explain her meaning to you, and should you lose yourself in her arguments it's up to her to help you find your way out. 2

(C) Instead of showing the student how to fix his work, you've challenged him to find a way to fix it himself. Admittedly, it can be hard for us to respond as the honest readers for whom our students intend to write. We're considerably more knowledgeable about our disciplines than our students are, so it's easy for us to pave over the holes in our students ' logic without them having to do it for us. Ideally we should resist the urge to read their writing this way. 1


① A-C-B ② B-A-C ③ B-C-A ④ C-A-B ⑤ C-B-A

 


131. 20수완 T3-4345

 

Around the turn of the 20th century, when X-rays and ultraviolet light were still brand-new discoveries, the French academician Rene-Prosper Blondlot announced that he too had discovered a new kind of radiation.


(A) Then the lights came back on. In the front row, Wood sat wordlessly, holding up the aluminum prism for everyone to see. He had secretly removed the prism in the middle of Blondlot's demonstration to show that N-rays were an illusion. Without the prism, the machine couldn't work. After all, N-rays turned out to be fictive. The story of Blondlot is a story of self-deception among scientists. All scientists can fall into various forms of self-deception during research. To prevent self-deception, scientists need a strong commitment to carefulness, skepticism, and rigor. 2

(B) He visited Blondlot's laboratory. Blondlot attempted to demonstrate the existence of N-rays and make clear to Wood some of the difficulties involved. Blondlot's main apparatus was an N-ray spectroscope he had constructed with lenses and a prism all made of aluminum instead of glass. Blondlot would turn a dial to rotate the prism while his assistant read off the intensity of the N-ray beam focused on a screen. Time and again Blondlot rotated the prism and the assistant read off wavelengths of the transmitted beam. The emission spectrum seemed to be completely reproducible. As a final confirmation, and to make the measurements even more precise, Blondlot repeated turning the dial with the lights turned off, again with identical results. 1

(C) He called the new type of radiation N-rays, after Nancy, the name of the university where he worked. Within three years hundreds of papers had been written about N-rays, and he was awarded the prestigious Prix Leconte by the French Academy of Sciences. However, the problem was that almost no one outside Blondlot's laboratory could detect the rays at all. The problem, Blondlot explained, was that N-rays were sensitive to a thousand influences of every kind. Inexperienced workers could hardly hope to produce or detect them. Nature magazine was skeptical of his and sent American physicist Robert W. Wood of Johns Hopkins University to investigate his discovery. Wood suspected that N-rays were an illusion. 0


① A-C-B ② B-A-C ③ B-C-A ④ C-A-B ⑤ C-B-A

 


132. 20수완 T4-18

 

We are residents of the City of Racine and we support the Kenosha-Racine-Milwaukee (KRM) commuter rail.


(A) We welcome the KRM because it would make it easy for us to commute from Racine to Chicago and then to O ' Hare Airport, without getting in our car. It would also allow us to get to the Milwaukee Airport as well. 0

(B) Commuter rail will play an important part in contributing to economic development. We also have to be very sensitive to the environmental impact of all the automobiles and congestion in the region. Commuter rail, again, will have a positive impact on the environment. 2

(C) We are somewhat regular flyers and leaving our car at the airport can be inconvenient, time consuming and expensive. It would seem quite clear that the Chicago-Milwaukee Corridor represents the economic future for the region. 1


① A-C-B ② B-A-C ③ B-C-A ④ C-A-B ⑤ C-B-A

 


133. 20수완 T4-19

 

Ingrid traveled with her friend to Qinghai Province, to her hometown.


(A) These animals must not have learned to be afraid of humans. They watched them curiously, and it was not until they were very close that the animals trotted away; some even galloped along as if keeping them company. The sky was so blue that it seemed to have absorbed all the oceans in the world. 1

(B) Her uncle, a truck driver, drove them around. For a long time, their truck cruised on the desert-like land, no people or houses in sight. They saw many deer and wild goats, as well as coyotes. 0

(C) They arrived at Qinghai Lake. The huge prairie around it extended all the way to the horizon. She and her friend ran, jumped, sang, danced, rolled on the grass, and shouted at the lake. 2


① A-C-B ② B-A-C ③ B-C-A ④ C-A-B ⑤ C-B-A

 


134. 20수완 T4-20

 

One of the first questions I' m asked by adults who are interested in teaching philosophy to children is: "How will I know what material is age-appropriate for the kids I'm working with?"


(A) I believe that any topic can be discussed once you determine the general maturity level of your group and shift gears as needed. 2

(B) Interestingly, I've found there is no guarantee that one approach or presentation will work for a particular age group, and that it's best to trust your intuition about whether or how to approach a topic with your class or child. I have met second-graders who discussed death with ease, and seventh-graders who were uncomfortable talking about friendship. 0

(C) It's important not to assume immediately that a topic is appropriate or inappropriate on the basis of a child's age. Children's maturity levels, interests, backgrounds, and verbal abilities vary. 1


① A-C-B ② B-A-C ③ B-C-A ④ C-A-B ⑤ C-B-A

 


135. 20수완 T4-21

 

In his bright blue postman's uniform with gold buttons, Vincent's new friend, Joseph Roulin, sat stiffly posing for his portrait.


(A) Vincent thought him more interesting than anyone he'd met in Arles. When he finished the portrait, the Roulins invited him to stay for supper. Roulin, after a bottle of wine, expounded on his socialist politics and offered the younger artist advice about life. 1

(B) Vincent thought the postman looked like Socrates, with his large head, ruddy cheeks, and long salt-and-pepper beard. He painted quickly, as Roulin could hardly contain himself. Roulin's wife had just delivered a baby girl, and he was "proud as a peacock and aglow with satisfaction." 0

(C) For Vincent, who long ago had lost faith in his own father, Roulin, "so wise and so trustful," became a father figure. He painted eight versions of the postman, as well as portraits of Mrs. Roulin and their two sons. Despite his poverty, Roulin refused to be paid, so Vincent ended up buying him food and many drinks at the local cafe. 2


① A-C-B ② B-A-C ③ B-C-A ④ C-A-B ⑤ C-B-A

 


136. 20수완 T4-22

 

Wherever we find creativity, we almost always find it was the result of a person who willingly went to work on a real problem.


(A) Thomas Edison once remarked that "Everything comes to him who hustles." Work. Don't worry. 0

(B) When we read the words of people like Giacomo Puccini, that great operatic composer, who once remarked, "The music of this opera Madame Butterfly was dictated to me by God. I was merely instrumental in putting it on paper and communicating it to the public," what are we to think? Obviously, he and others feel as though they are merely the instrument through which creative energies are flowing. While it might have felt this way to Puccini, it is also evident that Puccini underestimated his own abilities.2

(C) That was Edison's advice. And he proved its usefulness by his own example. But despite Edison's experience and that of countless others who continue to make breakthroughs, there remains considerable mystery about how creative ideas actually come to people. 1


① A-C-B ② B-A-C ③ B-C-A ④ C-A-B ⑤ C-B-A

 


137. 20수완 T4-23

 

Placing organic products into the global market has a number of implications.


(A) Entry into global markets may offer grounds on which to challenge national subsidies for conventional agriculture, but retaliatory challenges against organic farming are likely. A further concern is that global markets are uncertain and often volatile, which has the effect of reducing the security of farming enterprises and can be added to the economic incentives for larger-scale enterprises. 2

(B) Global markets are characterized by the strong role played by corporations in transport, handling, distribution, marketing and sales. Entering into the same markets as conventional agricultural products is likely to result in organic produce being subject to the same economic conditions that have shaped conventional agriculture and made sustainable practices unattractive. 0

(C) Organic producers competing in existing global markets will face economic incentives likely to erode the principles of organic farming. An emerging issue of potentially great concern is challenges brought against nations whose trading preferences run counter to such groups as the World Trade Organization. 1


① A-C-B ② B-A-C ③ B-C-A ④ C-A-B ⑤ C-B-A

 


138. 20수완 T4-24

 

Speaking of the place of rational thinking in the life of primitive humanity, rationality itself meant more than mechanical, conscious figuring; it also expressed itself in different ways.


(A) Conscious, rational thinking can be an exceptionally sluggish and dangerous process in emergency situations, where split-second decisions often make the difference between survival and annihilation. 0

(B) It's not clear, however, whether the capacity for instant but unconscious thinking and split-second decision-making (eg, quickly running from a predator or thoughtlessly striking a debilitating or lethal blow to an enemy) is a new human response mechanism, an old instinct that has been carried over from humanity's animal ancestry, or a creative combination of the two. Whatever it is, it certainly realizes an important goal of rational figuring, to the extent that it is typically an orderly response that increases the chances for survival. 2

(C) It is for this reason that the capacity for rapid and dramatic decision-making had to evolve, which, oddly enough, was accomplished through bypassing conscious, rational thinking altogether. 1


① A-C-B ② B-A-C ③ B-C-A ④ C-A-B ⑤ C-B-A

 


139. 20수완 T4-26

 

Little Sable Point Light stands 107 feet tall in Silver Lake State Park.


(A) Although no longer manned, the tower still sends its light out into the night. The lighthouse is open to the public on weekends during the summer.2

(B) It is a simple lighthouse that reflects the tall cylindrical structure so common to Michigan lighthouses. It is one of the oldest brick lighthouses on Lake Michigan. Its red brick tower is topped by a black cast-iron parapet with arc-shaped supports. 0

(C) In 1874 it was considered one of the loneliest stations for a keeper because there was no village nearby. But, for anyone seeking solitude and space in the middle of unsurpassed beauty it was an ideal assignment. The keeper's house was demolished during the 1950s. 1


① A-C-B ② B-A-C ③ B-C-A ④ C-A-B ⑤ C-B-A

 


140. 20수완 T4-29

 

The artist is often unable to tell beforehand how his painting is going to turn out, because it develops as he paints it.


(A) But the medium has limitations of its own. There are feelings that can or cannot be translated effectively into a particular medium. In considering these limitations, the artist must realize how he changes the material and also in what ways the material changes his expression and his conceptions. 2

(B) The artist realizes what the layman has experienced but cannot express. The artist frees original fantasies, aims, desires, and emotions, and expresses them through his medium. 1

(C) Even with a particular compositional or subject matter in mind the design of a painting changes as one form is added to complement another and as these forms create new and unanticipated relationships when seen in a whole composition. A painting grows like a living organism in the artist's mind and feelings as he interacts with his work at different stages in its development. 0


① A-C-B ② B-A-C ③ B-C-A ④ C-A-B ⑤ C-B-A

 


141. 20수완 T4-30

 

Policies aimed at providing mobility for low-income carless people might effectively be focused on the census tracts that have the largest percentages of households with these characteristics.


(A) Even when an area-targeted policy can be implemented, it provides services to many households who live in the targeted tracts but do have a car or are not in poverty, and it would miss the many carless households that do not live in the target census tracts. 1

(B) Also, there are numerous individuals (rather than households) who are carless for much of the day — people, for example, who remain at home while someone else takes the household's one car to work. The census tract maps are little help in locating these people. 2

(C) You can see that such policies would be far easier to implement in the city, where target tracts are clustered together, than in the suburbs, where they are widely dispersed. 0


① A-C-B ② B-A-C ③ B-C-A ④ C-A-B ⑤ C-B-A

 


142. 20수완 T4-31

 

Probably the easiest way to tum your passion into your job is to do it gradually.


(A) If possible, don't quit your day job before launching your business. If you want to podcast, start with a monthly show or a very short weekly show and see how it goes. 0

(B) In many cases, if your idea is a good one, you'll eventually become so busy or successful in your part-time endeavor that it will be clear when you should quit your day job and become a full -time independent business owner. 2

(C) If you want to open a yogurt shop, take a part-time job in someone else's yogurt shop and learn everything you can about how to run the business. If you're passionate enough, it won't even feel like work. 1


① A-C-B ② B-A-C ③ B-C-A ④ C-A-B ⑤ C-B-A

 


143. 20수완 T4-32

 

When a class of 30 college students read a not very- good biography about Maria Tallchief, an Osage ballerina who captured the attention of the dancing world in the early 20th century, all the students, except for five women, pronounced the book "mediocre."


(A) The others, without ballet backgrounds, did not find enough to interest them in the shallow way the author presented Tallchief's story. 2

(B) When they read about Maria Tallchief, they were reading their own stories. For them, the book served as a link to a meaningful personal experience. 1

(C) That enthusiastic handful loved the book and couldn't understand why the others were not impressed by this story that had meant so much to them. During the short discussion, the fact surfaced that all of the five young women had taken and loved ballet as children. 0


① A-C-B ② B-A-C ③ B-C-A ④ C-A-B ⑤ C-B-A

 


144. 20수완 T4-33

 

Many have claimed that, with billions of likely Earth-like planets, civilizations like ours must be common in our galaxy.


(A) More fundamentally, complex biological beings did not evolve to traverse the vastness of interstellar space; if interstellar travelers exist they'll be robots capable of "sleeping" over many thousands of years. Remember that stars are separated in distances measured by "light years" and, with light speed at 186,000 miles (300,000 km) in a single second, interstellar travel by living things remains a fantasy. 1

(B) Setting astrobiology ("the science without a subject") aside we'll confine ourselves to the planet on which we live. 2

(C) However, the more we learn, the more unlikely that appears. SETI — the Search for Extraterrestrial Intelligence program 一has been scanning the firmament for radio signals over more than forty years, and they have failed to intercept a single coherent message. 0


① A-C-B ② B-A-C ③ B-C-A ④ C-A-B ⑤ C-B-A

 


145. 20수완 T4-34

 

When someone gives a reason for his action, he makes reference to an end (or goal), plus a belief that the action will somehow advance the end.


(A) Humans assign variable meanings to what they perceive, dependent on perception, belief, emotion, and long-term goals. Knight notes that human consciousness does not fit neatly into the natural science paradigm. "Science can find no place for it[consciousness], and no role for it to perform in the causal sequence. 1

(B) The very invocation of a reason for action suggests that the person himself is a cause, directing himself toward an end. Economist Frank Knight addresses the problem of reasons for scientific accounts of choice as part of the more general problem of the place of human consciousness in scientific explanation. The challenge of human consciousness arises from the fact that humans react to their interpretation of reality, not to reality directly. 0

(C) It is epiphenomenal." The nature of social science, and its relationship to natural science, depends crucially on whether or not we take seriously the reasons that people give as causes of their actions. 2


① A-C-B ② B-A-C ③ B-C-A ④ C-A-B ⑤ C-B-A

 


146. 20수완 T4-35

 

It is time to put agriculture and food on the political agendas of local communities.


(A) This integration means that local agriculture and food businesses need the same access to economic development resources — such as grants, tax incentives, and loans — as nonfarm-related businesses. 1

(B) Additionally, communities should ensure that agricultural constituencies are represented on community boards, task forces, and governing bodies. Likewise, local agriculture and food systems activities should be addressed and integrated into any comprehensive planning processes. 2

(C) Locally organized agriculture and food enterprises must be fully integrated into a community's general planning and economic development efforts. 0


① A-C-B ② B-A-C ③ B-C-A ④ C-A-B ⑤ C-B-A

 


147. 20수완 T4-36

 

Increased daytime temperature of both pavement and near-surface air, reduced nighttime cooling, and associated higher air pollution levels can affect human health by contributing to general discomfort, respiratory difficulties, heat cramps and exhaustion, nonfatal heat stroke, and heat-related mortality.


(A) Excessive heat events, or abrupt and dramatic temperature increases, are particularly dangerous and contribute to premature deaths, potentially resulting in above-average rates of mortality. The heat-related mortality may tend to increase under the future potential climate change, under which more extreme surface weather conditions and heat events will probably be showing up. 2

(B) Sensitive populations, such as children, older adults, and those with existing health conditions, are at particular risk from these events. 1

(C) Heat islands can also exacerbate the impact of heat waves, which are periods of abnormally hot, and often humid, weather. 0


① A-C-B ② B-A-C ③ B-C-A ④ C-A-B ⑤ C-B-A

 


148. 20수완 T4-37

 

It is important to recognize the nature of mathematics and the very radical abstraction that it involves.


(A) Escaping from the earlier era of crude superstition and magic, they saw themselves emerging into a world where reason and evidence would triumph. But reason, in its purest form, is seen in logic and mathematics, and it was therefore natural to expect that the world would be, in principle, comprehensible in terms of 'laws of nature' which, with mathematical precision, would determine the movement of all things. 2

(B) That it should be possible for an abstract formula to correspond to nature was a fundamental assumption made by those involved in the emerging sciences. Beneath it lay the deeper assumption that the world is a predictable and ordered place. 1

(C) Galileo, Descartes, Huygens and Newton all produced formulae. In other words, they were seeking to create a mathematical and abstract way of summing up physical phenomena, using mathematics to express patterns seen in nature. 0


① A-C-B ② B-A-C ③ B-C-A ④ C-A-B ⑤ C-B-A

 


149. 20수완 T4-38

 

An interesting observation that is pertinent to the idea of an instinctive urge is displacement activity.


(A) But on other occasions the behavior is not distinguishable from normal grooming activity. Certain species of fish, such as the stickleback, also exhibit such out-of-context displacement activity. When at the boundary between its own territory and that of another stickleback, where both attack and escape behaviors are elicited, inappropriate nest-building behavior is often displayed. 2

(B) Displacement activity occurs when two incompatible response tendencies are simultaneously aroused. For example, a bird might be faced with a rival that elicits both attack and flight. 0

(C) In this situation, behaviors are displayed that appear to be irrelevant to the situation, such as grooming. At times the activity differs from normal grooming behavior in that it seems hurried and is discontinued before it is completed. 1


① A-C-B ② B-A-C ③ B-C-A ④ C-A-B ⑤ C-B-A

 


150. 20수완 T4-39

 

As the number of information security incidents rose, a community of security practitioners and academics attempted to categorize the types of threats.


(A) This is an incredibly problematic term especially when it is employed outside the computer and information security community. We all know a military attack or a terrorist attack is an event that carries with it the likely loss of life and destruction of property, but what the policy crowd didn't have a firm grasp upon was what sort of damage a cyber attack could do. 2

(B) Into these categories, supplemented by a few others that often are subject to debate, almost any security can be placed. In computer and information security, any "single unauthorized access attempt, or unauthorized use attempt, regardless of success" carries the label of "attack," a word with many applications in the English language, from verbal dispute to acts of war. 1

(C) While the US military had considerable experience in how it identified, classified, protected, transmitted, processed, and destroyed information, outside of government such knowledge was scarcer. Study of security produced a characterization of three categories in which information or information systems could be compromised: confidentiality, integrity, and availability. 0


① A-C-B ② B-A-C ③ B-C-A ④ C-A-B ⑤ C-B-A

 


151. 20수완 T4-40

 

Researchers have now integrated findings that used methods such as positron emission tomography (PET) imaging studies of dopamine's role in human drug addiction and longitudinal observations of the eating behaviors of overweight and obese individuals to show that increased body weight can increase behaviors that are common markers of addiction.


(A) By hiring food and flavor chemists to design products that elicit this primal "wanting," food companies can bypass the rational processes that protect people from harming themselves. 2

(B) Dopamine rewards the brain, thus signaling people to continue the behaviors that release it. One characteristic of addiction is that it is experienced as wanting, not simply liking, the addictive substance. 0

(C) A biopsychological research group in the United Kingdom has developed experimental methods to distinguish between explicit liking and implicit wanting. Their studies found that individuals with a tendency towards binge eating reported "liking" most food types, but "wanting" high-fat sweet foods. 1


① A-C-B ② B-A-C ③ B-C-A ④ C-A-B ⑤ C-B-A

 


152. 20수완 T4-4142

 

There is a distinction between the social and physical environments and the different types of selection pressures that they represent; this is because the physical environment changes a lot less over evolutionary time than the social.


(A) It is much better if behavioural responses are flexible. Many human brain adaptations have evolved in response to aspects of the social environment, including cheating, deception, and cooperation. 2

(B) For example, the qualities of gravity feel no different to you and me than they did to the first land-dwelling vertebrates. This stability means that there are certain properties of tetrapod bone structure, including in us four-limbed humans, that are set and have not changed. Tetrapods have a good solution to the problem of bearing weight under gravity, and any mutations are likely to result in a lower survival and reproductive outcome (a lower fitness). 0

(C) The social environment — made up primarily of other humans, but also more broadly other animals — is a completely different matter. In this environment, humans have to react to situations as they occur. Here, doing the same thing every time a situation occurs — say, always dodging left when being charged by a bull, or always sharing your meat when asked to 一 would be a distinct disadvantage, because the other animals or humans would evolve a response to outwit you. 1


① A-C-B ② B-A-C ③ B-C-A ④ C-A-B ⑤ C-B-A

 


153. 20수완 T4-4345

 

'Granddad,' William shouted, still dazed.


(A) William, not wanting to be left on his own, had followed his grandfather up to his room, and entered in time to see and hear him utter those unusual words. To his astonishment and delight, he saw the shiny box turn red, as if it had suddenly become very hot, and then the lid opened and revealed the most strikingly beautiful ring imaginable. 'Granddad' shouted William excitedly. 'Is that the ring of ever-changing colours you are always telling me about? It's so — so cool,' he said, not wanting to use a girly word like beautiful, although he thought it was. He came closer to get a better look at it, and to his amazement it began changing colours. First it went green, then blue, then pink and then through every colour in the spectrum. 1

(B) 'What's wrong?' His grandfather didn't answer — he probably never heard him, his mind wrestling with his worst fears. Granddad rushed into his room, flung back the door to his cupboard and searched through a pile of old shoeboxes. Hidden inside one of them was a smaller box, which he removed carefully. He unhooked the little catch at the front and lifted the lid. Inside was an even smaller box made of what looked like stainless steel. He lifted the box out, took it over to his desk and said, 'Ism Sprism.' 0

(C) Just when William thought there couldn't possibly be any more colours, new ones he had never seen before started appearing. 'Those are magic colours,' said his grandfather. They watched, fascinated. For a moment, his grandfather even forgot the reason for rushing to his room. 'Then you really are a wizard after all' shouted William, shattering the calm around them. A shiver of amusement ran down his spine. Before his grandfather could answer, a light exuded from the ring which was so deep and vibrant that William thought he would drown in it. 2


① A-C-B ② B-A-C ③ B-C-A ④ C-A-B ⑤ C-B-A

 


154. 20수완 T5-18

 

I reside in the downtown area of Louisville, close to the proposed parking garage location.


(A) In addition, the proposed size is out of character for downtown in general. Even a three-story parking garage would be the largest building in this area, damaging the historic character of the neighborhood and looming over the skyline. 1

(B) As an active resident who spends time downtown on most Friday and Saturday nights, the only times there seems to be a true parking problem are during the Summer Concerts in July. Building a four-story parking garage for those eight nights seems to be a bit extreme. 0

(C) I think there are other projects that I'd prefer the city to prioritize before undertaking a new parking garage. 2


① A-C-B ② B-A-C ③ B-C-A ④ C-A-B ⑤ C-B-A

 


155. 20수완 T5-19

 

When I reached the intersection, Sheldon was not there.


(A) Could I be in the wrong place? Perspiration began to form on my forehead and I wiped away the dampness on my upper lip. I forced myself to control my fear, and tried to think of a plan. If I called Sheldon there would be no answer, as he was waiting for me on a street corner somewhere nearby. It might be a good idea to drive closer to his office. 1

(B) Fortunately, I had his business card in my wallet, because I simply could not remember his address. Then, I spotted him standing on the corner. Concerned, he had walked down the block. Thank goodness! 2

(C) Where was he? Cars behind me began to honk. I had to drive on. There was no place to park. Again I passed the intersection and no Sheldon. 0


① A-C-B ② B-A-C ③ B-C-A ④ C-A-B ⑤ C-B-A

 


156. 20수완 T5-20

 

Collection reevaluation, most commonly known as weeding, is the process of removing books and other materials from the library collection when they are no longer useful.


(A) Instead, it should be motivated by a desire to maintain an up-to-date, inviting, and useful collection. Just as weeds can overtake a garden and hide the beauty of flowers, materials left in the library collection which are no longer useful can make it more difficult to find the ones which are. 2

(B) It is an essential part of collection development. A good weeding policy and procedure is as important as good selection procedures in building and maintaining an attractive and useful collection. 0

(C) Because it is a task many librarians don't enjoy, weeding is often put off until space is needed. However, overcrowded shelves should not be the first or only motivation for weeding. 1


① A-C-B ② B-A-C ③ B-C-A ④ C-A-B ⑤ C-B-A

 


157. 20수완 T5-21

 

Ideas about uncertainty are governed by the way society perceives the relationship between the present and the future.


(A) When, as today, the future is regarded as a dangerous territory, uncertainty is framed in a negative light. In such a setting, change itself is perceived as threatening. A potent undercurrent of apprehension towards change — whether technological, social or political — permeates the day-to-day affairs of the contemporary Western world. 0

(B) In this way, the dangers of the future acquire an immediate and intimate quality. They demand that we ring the alarm bells while implying that there is very little that can be done to avoid the dangers that lie ahead. 2

(C) Uncertainty was at times regarded as an opportunity — that it now tends to be cast in a negative light is symptomatic of a mood of fatalism towards the challenges faced by society. This fatalistic attitude is summed up by the often-repeated catch-phrase — 'The question is "not if, but when?"' Warnings of catastrophic climate events, deadly flu epidemics or mass casualty terrorism usually conclude with this defeatist refrain, which implicitly and sometimes explicitly calls into question humanity's capacity to avoid the destructive consequences of the threats it faces. 1


① A-C-B ② B-A-C ③ B-C-A ④ C-A-B ⑤ C-B-A

 


158. 20수완 T5-22

 

At a recent workshop for mental health professionals, Bonnie Zucker, the author of two excellent books on childhood anxiety, gave a presentation on the treatment of anxiety.


(A) Dr. Zucker then said, "Meditation is so powerful that I ask all of you who don't yet meditate to learn meditation — and then call me in a year to tell me how it's changed your life." We couldn't agree more. 1

(B) She asked the three hundred professionals in the audience if they meditated on a regular basis. A handful raised their hands. 0

(C) Practicing meditation is increasingly important as changes in the world lead to higher levels of anger and fear, and as advances in technology quicken the pace of life, giving us little time to simply "be" with ourselves. Although kids and teens rarely beg their parents to find them a meditation teacher, research indicates that when children and adolescents establish a practice regularly, meditation benefits them in the same ways as it does adults. 2


① A-C-B ② B-A-C ③ B-C-A ④ C-A-B ⑤ C-B-A

 


159. 20수완 T5-23

 

In times of crisis, the media react to society's need for surveillance and information by devoting massive time and energy to coverage of the crisis.


(A) The media highlight the wisdom of leaders and the bravery of rescue workers or soldiers to reassure society that "we are all in this together" and that everything possible is being done for survival. So, although the media may be unable to fulfill surveillance and correlation needs, they are able to offer assurance and tension reduction. 2

(B) All too often, though, it is difficult to gather information. Yet, it would be dysfunctional for media coverage to cease until information can be collected and verified. 0

(C) In order to reduce tension in society, media devote a good deal of coverage to media content intended to comfort their audience. Solidarity building is functional for society in times of crisis. 1


① A-C-B ② B-A-C ③ B-C-A ④ C-A-B ⑤ C-B-A

 


160. 20수완 T5-24

 

Affect can be expressed through facial expressions and through modulations in the tone and prosody of the voice.


(A) What is striking is the finding that the input from the body — including signals from the muscles, bones, and viscera (such as the heart and the intestines) — is more highly integrated in the right hemisphere than in the left. In other words, the whole body is represented in an integrated way in the right hemisphere. 1

(B) Even the regulation of the body's autonomic nervous system is primarily mediated by right-brain mechanisms. The right hemisphere therefore appears to play a major role in mediating regulatory emotional processes, as well as in permitting the expression of emotional states and the conscious awareness of emotional experience. 2

(C) These nonverbal aspects of language communication, in both their expression and perception, appear to be mediated predominantly by the right hemisphere. The body's posture and movement can also blend with the voice and facial expression in sending affective signals that are readily perceived by other people. 0


① A-C-B ② B-A-C ③ B-C-A ④ C-A-B ⑤ C-B-A

 


161. 20수완 T5-25

 

The above graph shows the online market shares of eight European countries in 2014 and 2015.


(A) In 2015, the Netherlands' online market share was more than twice as large as Spain's. 2

(B) The online market shares of the eight European countries all increased from 2014 to 2015. Both in 2014 and in 2015, the country with the largest online market share was the United Kingdom, followed by Germany. 0

(C) Sweden took third place in the online market share in 2014, but it fell to fourth place behind France in 2015. Spain's online market share was 0.5 percentage point larger in 2015 than it was in 2014. 1


① A-C-B ② B-A-C ③ B-C-A ④ C-A-B ⑤ C-B-A

 


162. 20수완 T5-29

 

At the root of the task of epistemology is the challenge posed by the simple fact that appearances don't always correspond with reality.


(A) Sherlock Holmes seemed to have an acute recognition of this insight, perhaps accounting for what can be called his aversion to the obvious. Rarely was he content with what may have seemed clear-cut and obvious to others. He was interested in what accounted for all the facts, not just those most at the surface. 1

(B) An obvious explanation in "The Beryl Coronet" case was that the son was the guilty perpetrator, appearing to have been caught red-handed. Holmes, searching for an explanation for all the observations he had made and facts in need of an account, remained skeptical. Healthy skepticism about appearances tends to be a salient feature of any credible epistemologist. 2

(C) They can be deceiving. Sometimes what seems simple is deceptively complex, and sometimes what appears complicated admits of a simple explanation. The fact that we can be deceived or deluded complicates the epistemic task of finding the truth. 0


① A-C-B ② B-A-C ③ B-C-A ④ C-A-B ⑤ C-B-A

 


163. 20수완 T5-30

 

For some teens, screen use has contributed to an underdeveloped identity.


(A) They may be devoting so much time to gaming and staying connected with "friends" that they don't have time or desire to broaden their interests or learn new skills, which would grow and solidify their identity. Also, because technology makes many things easier, they may be lacking the perseverance, diligence, and teachability that are often essential for adding to their skill sets and character development. 0

(B) For example, they may post often about their musical interests and never mention that they volunteer at an animal shelter. As time goes by and no friends acknowledge or support the teen's interest in animal rescue, the teen may begin to devalue that interest and let it wane. 2

(C) Posting on social networks can limit identity development because the tendency is for posts to acknowledge only certain aspects of their lives. That means friends and family can only provide comments or ask questions about those elements — the ones the teens choose to show. 1


① A-C-B ② B-A-C ③ B-C-A ④ C-A-B ⑤ C-B-A

 


164. 20수완 T5-31

 

Although both papyrus and parchment were used in various places throughout the ancient world for centuries, the eventual displacement of papyrus as the standard writing medium was fueled by a rivalry.


(A) As Alexandria was accumulating volumes and copies of documents, so was the library in Pergamum. Another important center of scholarship, the Greek kingdom of Pergarnum was located in present-day Turkey and was founded in the second century B.C. 0

(B) By the waning of the Western Roman Empire in 476 A.D., the parchment codex had become the predominant form of the book, replacing the papyrus roll. 2

(C) Not to be outdone by a competitor, Egypt placed an embargo on the export of the native papyrus, thus preventing Pergamum from obtaining the needed writing medium. Cutting off the supply of papyrus only led the scholars in Pergamum to improve parchment and to rely on it as their main writing material. 1


① A-C-B ② B-A-C ③ B-C-A ④ C-A-B ⑤ C-B-A

 


165. 20수완 T5-32

 

Technology now allows for addition as well as subtraction.


(A) We can not only imagine that the full assembly is present, but also prove it. 1

(B) At an annual gathering of my college friends in Vermont's Green Mountains, those unable to make it up for the weekend are photoshopped in by one friend; though absent, they are still sitting only slightly awkwardly on the steps or by the porch rail, reaffirming the inclusive spirit of our extended family. 0

(C) In one such photo taken several years ago, a man who happened to be in New York City that weekend appeared on the Vermont porch tossing a basketball to some hoop outside the frame. This magical realism was consistent with the way our long-standing friendships are integrated — seamlessly, unpredictably, and mysteriously — into geographically remote lives and experiences. 2


① A-C-B ② B-A-C ③ B-C-A ④ C-A-B ⑤ C-B-A

 


166. 20수완 T5-33

 

Most early primate studies assumed that primates were highly territorial and that groups would fight to defend their territories.


(A) Or just running away. Severe violence and aggression between groups is rare and seldom results in death. 2

(B) This is not to say that if one spends enough time watching primates she won't see two groups corning together over a contested area and putting on a big show for each other — lots of hooting and hollering and maybe even some fighting. These conflicts can result in serious injury or death but rarely do. Just as within groups, between-group conflicts are often resolved via negotiations or avoidance. 1

(C) We now know that most primates are not what we would call "territorial," because areas they use overlap with areas that other groups of the same species use. But there are conflicts over space, and in most cases groups of the same species tend to avoid being in the same place at the same time (though not always). Researchers have argued that this is a way to minimize the risk of conflict and violence between groups. 0


① A-C-B ② B-A-C ③ B-C-A ④ C-A-B ⑤ C-B-A

 


167. 20수완 T5-34

 

Of all approaches to health, the biomedical is unquestionably the most influential.


(A) There are obvious merits to understanding these physiological influences, not least to treat infectious diseases, which were the main cause of ill health and death until early in the 20th century. However, as Engel recognised, ill health is not reducible to disease processes alone, and if it were, then there should be much greater consistency in how people experience and respond to disease and its treatment than is actually observed. 1

(B) This model understands health primarily through the lens of disease, and it attributes the cause of ill health to some breakdown in normal biological and physiological functioning. In so doing, it gives a clear direction in how best to manage health — and this is to focus on repairing or treating the source of breakdown in the body. 0

(C) It is also the case that the health landscape has changed dramatically to one in which chronic conditions (eg, diabetes, depression, arthritis) have become the prevailing cause of ill health. For these conditions, there is generally no simple biomedical fix that can be administered to restore health.2


① A-C-B ② B-A-C ③ B-C-A ④ C-A-B ⑤ C-B-A

 


168. 20수완 T5-35

 

Most people are socialized to believe in the reciprocity principle — the rule that one should pay back in kind what one receives from others.


(A) Charities frequently make use of this principle. 0

(B) The belief that people should reciprocate others' kindness is a powerful norm; thus, people often feel obliged to reciprocate by making a donation in return for the gift. According to Cialdini, the reciprocity norm is so powerful that it often works even when the gift is uninvited, the gift comes from someone you dislike, or the gift results in an uneven exchange. 2

(C) Groups seeking donations for the disabled, the homeless, and so forth routinely send "free" address labels, key rings, and other small gifts with their pleas for donations. 1


① A-C-B ② B-A-C ③ B-C-A ④ C-A-B ⑤ C-B-A

 


169. 20수완 T5-36

 

Color defines our world and our emotions.


(A) Upon entering a room, we first see the color or colors used in the interior design and then discern the furnishings and artifacts contained within the space. An artwork, be it fine or commercial, is aesthetically pleasing to the viewer when its color usage allows the viewer to see the content of the piece (both color and imagery) together. When this is accomplished, a work's message is conveyed immediately, without a "second look" on the part of the viewer. 2

(B) At first glance we do not see the different species of trees present in a summer woodland, but rather see the preponderance of green. The artist, architect, and designer, however, are generally concerned with having color and imagery perceived simultaneously. 1

(C) It is usually seen before imagery. Our eyes are attracted to color to such an extent that the color of an object is perceived before the details imparted by its shapes and lines. 0


① A-C-B ② B-A-C ③ B-C-A ④ C-A-B ⑤ C-B-A

 


170. 20수완 T5-37

 

How the bandwagon effect occurs is demonstrated by the history of measurements of the speed of light.


(A) If a result fit what they expected, they kept it. If a result didn't fit, they threw it out. They weren't being intentionally dishonest, just influenced by the conventional wisdom. The pattern only changed when someone had the courage to report what was actually measured instead of what was expected. 2

(B) Then, from 1900 to 1950, the opposite happened — all the experiments found speeds that were too low! This kind of error, where results are always on one side of the real value, is called "bias." It probably happened because over time, experimenters subconsciously adjusted their results to match what they expected to find. 1

(C) Because this speed is the basis of the theory of relativity, it's one of the most frequently and carefully measured quantities in science. As far as we know, the speed hasn't changed over time. However, from 1870 to 1900, all the experiments found speeds that were too high. 0


① A-C-B ② B-A-C ③ B-C-A ④ C-A-B ⑤ C-B-A

 


171. 20수완 T5-38

 

Loess is a homogeneous, fine yellow soil that has been deposited across 1 million square miles (2 million sq km) of land that covers several areas of the world: Asia, Europe, and North America.


(A) It ranges in thickness from area to area and can be as thick as 10 feet (3 m) in some locations. Loess originated from glacial processes. 0

(B) As the area dried out, winds carried loess across the land. Its texture was so fine that it was carried great distances. This spread the deposits across wide areas and left rich, easily recognizable, homogeneous soil. 2

(C) As the massive weight of the glacial ice moved across the Earth's surface, the ice ground along the rock slowly and abraded and pulverized it into a powderlike substance. Later, as the climate warmed and the ice melted, running water washed the flourlike deposits from under the glaciers and into streams along the edges of the ice. 1


① A-C-B ② B-A-C ③ B-C-A ④ C-A-B ⑤ C-B-A

 


172. 20수완 T5-39

 

In one experiment, participants received verbal information about a product that they believed to be either familiar (a standard computer mouse) or unfamiliar (a trackball mouse).


(A) Consequently, visualizers evaluated the unfamiliar product less favorably than the familiar one. However, presenting a picture of the unfamiliar product increased visualizers' evaluation of it to a level similar to that of the familiar one. 1

(B) In contrast, individuals with a disposition to process information verbally (verbalizers) based their evaluations on the semantic implications of the product-attribute descriptions, which were the same regardless of the product's ostensible familiarity. Therefore, they evaluated the familiar and unfamiliar products similarly even in the absence of a picture. 2

(C) Individuals with a disposition to process information visually (visualizers) presumably had a previously formed mental image of the familiar product that they could use in organizing the verbal attribute descriptions. When the mouse was unfamiliar and a picture of it was unavailable, this was not the case, and a visual image of the product was difficult to construct on the basis of the attributes alone. 0


① A-C-B ② B-A-C ③ B-C-A ④ C-A-B ⑤ C-B-A

 


173. 20수완 T5-40

 

Richerson and Boyd argue that people could learn cooperative behavior through "conformist transmission," which means copying whatever behaviors are most common within a population.


(A) Herbert Simon has made a similar argument about our disposition to learn from others, which he calls "docility." This is a highly advantageous trait, yet it allows other people to occasionally manipulate us into learning altruistic behaviors which do not benefit us. 2

(B) Given that humans rely so much on socially learned behaviors, this bias toward conformist transmission is probably adaptive on average, even if it occasionally causes us to learn maladaptive behaviors. Thus, it is possible that our tendency to copy prosocial behavior could be a maladaptive byproduct of our adaptive tendency to copy common behaviors. 1

(C) Copying common behaviors is normally an adaptive way of learning how to do things because others may have already figured out the best way of doing things. However, it can also cause people to make systematic errors when they also copy behaviors that tum out to not be optimal. 0


① A-C-B ② B-A-C ③ B-C-A ④ C-A-B ⑤ C-B-A

 


174. 20수완 T5-4142

 

The last, most needed, most paradoxical attribute of democratic leadership is the willingness to forgo power when attaining and maintaining it requires morally unacceptable compromise.


(A) On the other hand, principled aspirants cannot hope to win power by bluntly saying exactly what they believe. For example, while his desire to support Britain's struggle against Nazi Germany was completely justified, Franklin Roosevelt might well have lost his 1940 re-election campaign if he had been completely candid about it. So he equivocated. When Wendell Willkie, the Republican presidential nominee, claimed that a vote for Roosevelt meant war in 1941, Roosevelt countered with a flat promise to the contrary — "Your boys are not going to be sent into any foreign wars" — deliberately omitting the Democratic platform's qualifying phrase, "except in case of attack." 1

(B) Democratic politics at its best is the use of publicly authorized power to advance the common good. Would-be leaders, then, can fail in two ways. They may be unable to obtain public support for their agenda, or they may win support by advocating only what the people want to hear. While modem survey research has raised the assessment of public beliefs to a high art, the temptation to pander to them is a recurrent weakness of democratic politics. 0

(C) When one of his speechwriters asked about the omission, he replied, "Of course we'll fight if we're attacked. If someone attacks us, then it isn't a foreign war, is it?" This mental reservation allowed Roosevelt to pretend that he wasn't trying to mislead the people, which of course he was. 2


① A-C-B ② B-A-C ③ B-C-A ④ C-A-B ⑤ C-B-A

 


175. 20수완 T5-4345

 

One Friday afternoon in July, an editor at Australian Football League (AFL) media called David, an intern reporter, and asked if he was interested in covering the game at the Melbourne Cricket Ground (MCG) that night (Collingwood versus Adelaide).


(A) He was still sitting in the MCG press box at 1 a.m. with a few other reporters, but it was all worth it in the end. David's experience at AFL media has, above all things, given him the best indication yet of what it takes to be a journalist at that level. Watching the experienced guys go about their work and being able to pick their brains at various times has been a really positive experience. David hopes his time there has given him a solid grounding for future work in sports journalism, an area he is most passionate about. 2

(B) One of the reporters suddenly quit her job and he was asked to take her spot. Naturally, he jumped at the chance and was assigned to sit in the press box and provide coverage of the game alongside one of the other AFL media reporters. Sitting with prominent football journalists in Melbourne was very exciting to David. As the night progressed he realised how ruthless match reporting could be. He was expected to file his report on or within five minutes of the final siren, which proved difficult given the match's relatively close result. 0

(C) After the game he went down to the losing team's rooms (Adelaide) and approached their media manager for a player interview. Before the game he had seen on Twitter that Adelaide player Jarryd Lyons — who kicked four goals on the night — had played school cricket with new Australian sensation Ashton Agar. Ashton Agar had just scored a brilliant 98 on debut in England the night before and David wanted to speak with Jarryd Lyons for a potential story. His request was instantly granted and he spoke to him, firstly about the game and eventually about playing with Agar at De La Salle College. He also attended Adelaide coach Brenton Sanderson's press conference and wrote a report on that. 1


① A-C-B ② B-A-C ③ B-C-A ④ C-A-B ⑤ C-B-A

 


[ANSWER]
1. ① 2. ④ 3. ③ 4. ③ 5. ② 6. ④ 7. ② 8. ⑤ 9. ③ 10. ①


11. ① 12. ① 13. ① 14. ① 15. ⑤ 16. ② 17. ④ 18. ③ 19. ① 20. ③


21. ④ 22. ③ 23. ④ 24. ④ 25. ② 26. ④ 27. ④ 28. ① 29. ① 30. ③


31. ① 32. ⑤ 33. ① 34. ④ 35. ① 36. ③ 37. ③ 38. ① 39. ② 40. ⑤


41. ③ 42. ④ 43. ② 44. ① 45. ① 46. ② 47. ① 48. ② 49. ① 50. ②


51. ② 52. ④ 53. ② 54. ② 55. ③ 56. ① 57. ② 58. ① 59. ③ 60. ③


61. ④ 62. ④ 63. ③ 64. ② 65. ③ 66. ③ 67. ① 68. ④ 69. ④ 70. ④


71. ④ 72. ③ 73. ① 74. ④ 75. ② 76. ④ 77. ④ 78. ② 79. ④ 80. ④


81. ⑤ 82. ⑤ 83. ② 84. ① 85. ⑤ 86. ⑤ 87. ② 88. ② 89. ② 90. ⑤


91. ① 92. ② 93. ① 94. ③ 95. ② 96. ③ 97. ③ 98. ① 99. ④ 100. ②


101. ① 102. ③ 103. ④ 104. ② 105. ③ 106. ③ 107. ② 108. ⑤ 109. ④ 110. ⑤


111. ⑤ 112. ② 113. ⑤ 114. ② 115. ⑤ 116. ⑤ 117. ⑤ 118. ④ 119. ③ 120. ⑤


121. ① 122. ③ 123. ① 124. ④ 125. ④ 126. ③ 127. ⑤ 128. ③ 129. ③ 130. ①


131. ⑤ 132. ① 133. ② 134. ③ 135. ② 136. ① 137. ③ 138. ① 139. ③ 140. ⑤


141. ④ 142. ① 143. ⑤ 144. ④ 145. ② 146. ④ 147. ⑤ 148. ⑤ 149. ③ 150. ⑤


151. ③ 152. ③ 153. ② 154. ② 155. ④ 156. ③ 157. ① 158. ② 159. ③ 160. ④


161. ③ 162. ④ 163. ① 164. ① 165. ② 166. ⑤ 167. ② 168. ① 169. ⑤ 170. ⑤


171. ① 172. ④ 173. ⑤ 174. ② 175. ③ 

728x90
반응형

728x90
반응형

20ss2 | Since 2005 임희재 | 블루티쳐학원 | 01033383436 | 200710 22:53:52

 

순서배열

 

1. 20영독 1-1

 

It will never be possible to establish the origins of human music with any certainty; however, it seems probable that music developed from the prosodic exchanges between mother and infant which foster the bond between them.


(A) It has continued to be used as an accompaniment to collective activities; as an adjunct to social ceremonies and public occasions. 2

(B) Today, we are so accustomed to considering the response of the individual to music that we are liable to forget that, for most of its history, music has been predominantly a group activity. Music began by serving communal purposes, of which religious ritual and warfare are two examples.1

(C) From this, it became a form of communication between adult human beings. As the capacity for speech and conceptual thought developed, music became less important as a way of conveying information, but retained its significance as a way of communicating feelings and cementing bonds between individuals, especially in group situations. 0


① A-C-B ② B-A-C ③ B-C-A ④ C-A-B ⑤ C-B-A

 


2. 20영독 1-2

 

As much as we want to believe that students learn everything we try to teach, we must admit that we aren't perfect as teachers.


(A) But for teachers who are able to accept their potential fallibility, the disappointment about the results translates into clearly identified areas for improvement. As odd as this may sound, when you suspect that you have the potential to enhance your teaching, assessment results can help clear up any confusion about where you might begin to make changes. 2

(B) In fact, recognizing our fallibility is said to be one characteristic that separates great teachers of diverse students from those teachers who are just adequate. Recognizing that you can make mistakes as a teacher and constantly strive to recover from those failings is significant. 0

(C) If you use preconcept and postconcept mapping exercises with your students and you discover that some of the gaps appearing at the beginning persisted until the end, then you have a couple of ways of responding. One thought that too many teachers invoke is that the kids were lazy and didn't care. 1


① A-C-B ② B-A-C ③ B-C-A ④ C-A-B ⑤ C-B-A

 


3. 20영독 1-3

 

Vocabulary knowledge extends beyond the local meaning of a sentence or paragraph; it provides insight into how the text works more broadly.


(A) Their, not the. One word subtly shifts the meaning to acknowledge the existence of more than one belief system. Seemingly small word choices can profoundly affect the analysis and interpretation of a reading. 2

(B) The craft and structure of a text is understood in part by being mindful of the choices the writer makes in word selection. Students are challenged to view the text as a whole in order to understand the perspectives of the writer, as in a historical document, or the concepts related in a scientific article or technical reading. 0

(C) Writers establish mood and tone by the artful selection of words and phrases and link them to rhetorical purposes, especially in history. For instance, consider the use of the word their in the first sentence of the Declaration of Independence: "We hold these truths to be self-evident, that all men are created equal, that they are endowed, by their Creator with certain inalienable Rights, that among these are Life, Liberty, and the pursuit of Happiness." 1


① A-C-B ② B-A-C ③ B-C-A ④ C-A-B ⑤ C-B-A

 


4. 20영독 1-4

 

Ethics itself is not primarily concerned with the description of the moral systems of different societies.


(A) Words such as good and bad just mean, it is claimed, "approved in my society" or "disapproved in my society," and so to search for an objective, or rationally justifiable, ethics is to search for what is, in fact, an illusion. 2

(B) That task, which remains on the level of description, is one for anthropology or sociology. In contrast, ethics deals with the justification of moral principles (or with the impossibility of such a justification). 0

(C) Nevertheless, ethics must take note of the variations in moral systems, because it has often been claimed that this variety shows that morality is simply a matter of what is customary and that it thus is always relative to particular societies. According to this view, no moral principle can be valid except in the societies in which it is held. 1


① A-C-B ② B-A-C ③ B-C-A ④ C-A-B ⑤ C-B-A

 


5. 20영독 1-5

 

The destructiveness of idealized social media presentation is twofold.


(A) In one study, some college students had to pretend to understand made-up words (e.g., besionary) to complete a test, while others did not. Afterward, both groups were praised for their performance. 1

(B) The praise raised the self-esteem of those who did not have to fake understanding of nonwords, but lowered the self-esteem of the students who pretended to understand them. In other words, when people portray their life as better than it actually is on social media, all of the "likes" they get for their status updates and pictures may actually make them feel worse about themselves. 2

(C) When people read the glowing status updates of others, they are likely to feel worse about their own flawed lives, especially if they are the type of people who tend to judge their self-worth in comparison to others. In addition, when people put forth a version of themselves that is not genuine, they can actually feel bad about themselves. 0


① A-C-B ② B-A-C ③ B-C-A ④ C-A-B ⑤ C-B-A

 


6. 20영독 1-6

 

A quality of the human brain is known as induction, how something positive generates a contrasting negative image in our mind.


(A) This is most obvious in our visual system. When we see some color — red or black, for instance — it tends to intensify our perception of the opposite color around us, in this case, green or white. As we look at the red object, we often can see a green halo forming around it. In general, the mind operates by contrasts. 0

(B) Every no sparks a corresponding yes. We cannot control this vacillation in the mind between contrasts. This predisposes us to think about and then desire exactly what we do not have. 2

(C) We are able to formulate concepts about something by becoming aware of its opposite. The brain is continually dredging up these contrasts. What this means is that whenever we see or imagine something, our minds cannot help but see or imagine the opposite. If we are forbidden by our culture to think a particular thought or entertain a particular desire, that taboo instantly brings to mind the very thing we are forbidden. 1


① A-C-B ② B-A-C ③ B-C-A ④ C-A-B ⑤ C-B-A

 


7. 20영독 1-7

 

The bottlenose dolphin is second only to humans in the ratio of brain size to body size, and dolphins apparently outdo humans in some cooperative games.


(A) Dolphins face similar dilemmas. When eating from a school of fish, dolphins encircle the fish and take turns eating, one dolphin at a time. There is an incentive for the circling dolphins to cheat by eating while on duty. 1

(B) However, if a significant number of dolphins followed that incentive, the fish would disperse and the benefits from coordination would be lost. In reality, the trustworthiness of on-duty dolphins prevails to benefit all of dolphin society. 2

(C) The discussions of cartels and the prisoner's dilemma convey the value and difficulty of cooperation among players when there is an incentive to cheat. Individual cartel members undermine cooperative strategies by selling more than they should, hoping nobody will notice. Firms that can't cooperate on pricing or environmental strategies end up taking actions with inferior outcomes. 0


① A-C-B ② B-A-C ③ B-C-A ④ C-A-B ⑤ C-B-A

 


8. 20영독 1-8

 

As my colleague Richard Gregory has argued, illusions like the missing-square pattern reveal that the mind is not lazy.


(A) You can't stop yourself because your mind has evolved to organize and see structure. The ease with which we see faces in particular has led to the idea that we are inclined to see supernatural characters at the drop of a hat. Each year some bagel, muffin, burnt toast, potato chip, or even ultrasound of a fetus showing the face of some deity is paraded as evidence for divine miracles. 2

(B) Some beans would instantly cluster together into groups as you simply looked at the array. Have you ever watched the clouds on a summer's day turn into faces and animals? 1

(C) Our minds are actively trying to make sense of the world by thinking of the best explanation. For example, if someone took a handful of coffee beans and scattered them across a table in front of you, you would immediately see patterns. 0


① A-C-B ② B-A-C ③ B-C-A ④ C-A-B ⑤ C-B-A

 


9. 20영독 1-9

 

As the ancient skeptics taught, contentment is possible without the need to cling to comforting beliefs.


(A) As proof, we have the example of David Hume, who lived an entirely admirable life without any belief in the supernatural. His personal example shows that nobody need feel gloomy because life has no ultimate purpose, or because conventional conceptions of moral responsibility are built on foundations of sand. 0

(B) In discarding the metaphysical baggage with which the human race bolstered its youthful sense of self-importance, Hume taught us that we throw away nothing but a set of intellectual chains. 2

(C) So what if our fine feelings and intellectual achievements are just the stretching and turning of so many springs or wheels, or our value systems are mirrored by those of chimpanzees and baboons. Our feelings are no less fine, and our values no less precious because the stories we have traditionally told ourselves about why we hold them turn out to be fables. 1


① A-C-B ② B-A-C ③ B-C-A ④ C-A-B ⑤ C-B-A

 


10. 20영독 1-10

 

The history of science and human invention is full of examples of important advances resulting from synthesizing previously fragmented ideas.


(A) When he spoke to the membrane, the other end of the stick would trace a record of his voice sounds on a piece of smoked glass. In 1874 a Scotsman from Canada, working in Cambridge, Massachusetts, put these scattered and diverse elements into one instrument. The instrument was the telephone and the man was Alexander Graham Bell. 2

(B) One such process began in 1820 when a Dane, H. C. Oersted, discovered that a wire carrying an electric current was surrounded by a magnetic field. In 1825 an Englishman, W. Sturgeon, wound a live wire around an iron bar and created an electromagnet. 0

(C) In 1859 a German pianist and scientist, H. von Helmholtz, discovered he could make piano strings vibrate by singing to them. Later a Frenchman, L. Scott, attached a thin stick to a membrane. 1


① A-C-B ② B-A-C ③ B-C-A ④ C-A-B ⑤ C-B-A

 


11. 20영독 1-11

 

There have been vigorous arguments among biologists about whether complicated goal-directed behaviour among higher mammals is reliable evidence for their consciousness.


(A) At the other end of the animal kingdom octopuses and squid have entirely different brain anatomies from ourselves and our common ancestor probably had no brains at all. Nevertheless they are capable of learning and memorizing facts for months. If they are to be included in the realm of conscious beings, this indicates that consciousness does not depend upon a particular type of brain anatomy. 2

(B) Indeed the admission of consciousness into animal research is quite a recent phenomenon. Injury-avoidance behaviour is often based on reflexes, and it is not completely obvious that the inner sensation of pain must be attached to it. 0

(C) Even in our own case pain is often felt only after the limb has been moved away. Again, many birds build sophisticated nests entirely instinctively, and may or may not be conscious of what they are doing. 1


① A-C-B ② B-A-C ③ B-C-A ④ C-A-B ⑤ C-B-A

 


12. 20영독 1-12

 

The cultural area is where the sky is truly the limit in regard to music.


(A) There are countless examples. Music is a wonderful avenue to introduce cultures from around the world. 0

(B) It is important to include an equitable balance of multicultural materials in the classroom to ensure that each child's heritage is represented and all children are exposed to the heritage of other cultures. Today the holistic classroom method draws and builds upon what the child already knows, engaging the child ― since one learns best when passion and interaction are at play ― and also addressing the needs of the whole child. Learning begins with the "whole," progresses through to analysis of the parts, and finishes full circle with the "whole" picture. 2

(C) The many recordings depicting the traditional music of each culture are readily available for teachers to add to their collection. Also, musical recordings of the various instruments from countries around the world provide children with an auditory and visual representation of culture. 1


① A-C-B ② B-A-C ③ B-C-A ④ C-A-B ⑤ C-B-A

 


13. 20영독 2-1

 

Your comfort zone is like an invisible barrier around you, inside which if you stay, you feel comfortable.


(A) If you step outside your comfort zone, and do something you are fearful or nervous about doing, then your comfort zone expands and your confidence increases. Try something new to expand your comfort zone and increase your confidence. Trying something new reduces your limitations and you'll live your life with fewer barriers. 2

(B) However, your comfort zone is not fixed. If you constantly stay within your comfort zone it shrinks, filling you with fear of what is outside it, and then your confidence reduces. 1

(C) Your comfort zone and your confidence are linked together. Step outside it and you feel uncomfortable and fearful. 0


① A-C-B ② B-A-C ③ B-C-A ④ C-A-B ⑤ C-B-A

 


14. 20영독 2-2

 

Given our unique life-scripted beliefs about how things should be: our expectations ― differences in preferences, attitudes, and beliefs are inevitable, and not all of them need to be resolved.


(A) Many, in fact, add the spice to relationships. But sometimes you cannot just agree to disagree. Some issues impact each of you and perhaps others (your children or coworkers) in ways that require a clear, unambiguous resolution. 0

(B) For example, you can't practically visit your mother in Florida and your father in Connecticut on Thanksgiving Day. So coping with conflicts as we traverse the ups and downs of daily life is not just a useful tool; it is absolutely necessary for the kinds of successful relationships and outcomes we most desire. 2

(C) You and your partner may need to decide where you will live and whether to rent or purchase a home. A decision must be made, or you may find yourselves living in the backseat of your car. Often you can't have things both ways, so a choice must be made. 1


① A-C-B ② B-A-C ③ B-C-A ④ C-A-B ⑤ C-B-A

 


15. 20영독 2-3

 

The personal computer has done more to alter work methods and procedures than any other innovation of the past several decades.


(A) Unfortunately, the computers ― and tablets and smart phones and other electronic devices ― have also opened wide a door to a variety of time-wasting personal uses including games and nonbusiness e-mail (personal correspondence, jokes, inspirational messages, anecdotes, etc.). 1

(B) Computers have replaced typewriters and other office machines almost completely, and they have dramatically changed the way many jobs are performed. 0

(C) It is not unreasonable to conclude that much of the efficiency gained through the use of such devices is cancelled out by their misuse. The personal computer may well be the most useful and versatile tool ever to come into common organizational use, but by many who spend hours at keyboard and screen the computer is treated more as a toy than a tool. 2


① A-C-B ② B-A-C ③ B-C-A ④ C-A-B ⑤ C-B-A

 


16. 20영독 2-4

 

In the circumstances of entertainment and aesthetic engagement, overt manifestations of the perception-action cycle are often blocked or transformed.


(A) The interruption or suspension of the perception-action cycle that characterizes some forms of aesthetic engagement is, of course, culturally specific; it is at its most extreme in some of the "high" art forms of the West and in circumstances in which formal ceremony and aesthetics interact. 2

(B) Watching films and television, looking at paintings or sculptures in a gallery, and listening to music in a concert hall deliberately place perceivers in a relationship with the objects of perception that prevents them from acting upon or exploring those objects in an unhindered fashion. 0

(C) Many of the reactions that people have to these special circumstances (reaching out to touch a sculpture; foot- and finger-tapping in response to music) are a residue of the more usual relationship between perception and action, as are the specific conventions that regulate these reactions ("Please do not touch" signs at exhibitions, socially enforced silence and immobility at concerts, applause at regulated moments). 1


① A-C-B ② B-A-C ③ B-C-A ④ C-A-B ⑤ C-B-A

 


17. 20영독 2-5

 

People from more individualistic cultural contexts tend to be motivated to maintain self-focused agency or control as these serve as the basis of one's self-worth.


(A) However, people from more interdependent cultural contexts tend to be less focused on issues of individual success and agency and more motivated towards group goals and harmony. Research has shown that East Asians prefer to receive, but not seek, more social support rather than seek personal control in certain cases. 1

(B) With this form of agency comes the belief that individual successes depend primarily on one's own abilities and actions, and thus, whether by influencing the environment or trying to accept one's circumstances, the use of control ultimately centers on the individual. The independent self may be more driven to cope by appealing to a sense of agency or control. 0

(C) Therefore, people who hold a more interdependent self-construal may prefer to cope in a way that promotes harmony in relationships. 2


① A-C-B ② B-A-C ③ B-C-A ④ C-A-B ⑤ C-B-A

 


18. 20영독 2-6

 

Severe depression is not something people can pull themselves out of any more than they can pull themselves out of congestive heart failure, kidney disease, or gallstones.


(A) In my experience, once older adults understand that depression is a disease of the brain, and not something they have control over, they become more open to considering treatment. It's not that they can't handle their problems any longer; rather, their brain has let them down. I often say to my patients, "It's not you; it's your brain." 2

(B) When patients with congestive heart failure develop difficulty breathing, they are usually grateful for treatment that relieves their distress. They rarely believe they can handle such illnesses themselves because they have no sense of being in control over the workings of their heart. 0

(C) We also do not sense our brains at work, but we feel in control of our minds. This sense of being in control of our minds allows those with depression to believe they can pull themselves out of the severe depression. 1


① A-C-B ② B-A-C ③ B-C-A ④ C-A-B ⑤ C-B-A

 


19. 20영독 2-7

 

In the process of selling your property, you may hear the phrase "real property" and "personal property."


(A) If a refrigerator is somehow permanently attached to the home (such as a built-in model), it is real property and stays. When selling a property, it is assumed that you are selling all real property. Ripping things like banisters, fireplaces, etc. 1

(B) Real property is fixed and attached; personal property is usually mobile and unattached. Where this is likely to come up is in regard to items within your property. Most refrigerators that can roll out, be unplugged, and taken with you, are considered personal property. 0

(C) off their moorings and taking them with you is not only boorish behavior, it would most likely be a violation of your sales contract. Even if it is possible to remove them, the buyer is assuming all real property to be his. Granted, anything is negotiable, but if I was a buyer and I allowed you to do such a thing at all (which I most likely wouldn't), I would demand significant financial consideration off the previously negotiated sales price, so much so that you would most likely say, "Forget it." As the buyer, I don't need you trashing the property as you leave. 2


① A-C-B ② B-A-C ③ B-C-A ④ C-A-B ⑤ C-B-A

 


20. 20영독 2-8

 

Much prosocial behavior is stimulated by others, such as when someone acts more properly because other people are watching.


(A) Participants in a study by Kay L. Satow sat alone in a room and followed tape-recorded instructions. Half believed that they were being observed via a one-way mirror (public condition), whereas others believed that no one was watching (private condition). At the end of the study, the tape-recorded instructions invited the participant to make a donation by leaving some change in the jar on the table. 1

(B) Dogs will stay off the furniture and out of the trash when their owners are present, but they casually break those rules when alone. Humans may have more of a conscience, but they also still respond to the presence or absence of others. Public circumstances generally promote prosocial behavior. 0

(C) The results showed that donations were seven times higher in the public condition than in the private condition. Apparently, one important reason for generous helping is to make (or sustain) a good impression on the people who are watching. 2


① A-C-B ② B-A-C ③ B-C-A ④ C-A-B ⑤ C-B-A

 


21. 20영독 2-9

 

Say you're driving down the interstate at sixty-five miles an hour with three friends from out of town, and you suddenly say to them, "Hey, there's that amazing Pink House!"


(A) What happens? Probably there's a lot of sudden head swiveling, and someone's elbow ends up in someone else's ribs, and maybe one of your friends gets a glimpse, but probably nobody really gets a chance to see it (and somebody might not believe you if she didn't see it for herself!). 0

(B) Writers need to advise their readers in a similar way. That advice doesn't always need to be in a thesis statement or a topic sentence, but it does need to happen regularly so that readers don't miss something crucial. 2

(C) What if you had said instead, "Hey, coming up on the right here in about two miles, there's an amazing huge neon Pink House: watch for it"? They'd be ready, they'd know where to look and what to look for, and they'd see what you wanted them to see. 1


① A-C-B ② B-A-C ③ B-C-A ④ C-A-B ⑤ C-B-A

 


22. 20영독 2-10

 

So far as diet is concerned, I belong to no school; I have learned something from each one, and what I have learned from a trial of them all is to be shy of extreme statements and of hard and fast rules.


(A) We cannot live without asserting our right to subject the lower forms of life to our purposes; we kill innumerable germs when we swallow a glass of grape juice, or for that matter a glass of plain water. 1

(B) I shall be much surprised if the advance of science does not some day prove to us that there are basic forms of consciousness in all vegetable life; so we shall justify the argument of Mr. Dooley, who said, in reviewing "The Jungle," that he could not see how it was any less a crime to cut off a young tomato in its prime or to murder a whole cradleful of baby peas in the pod! 2

(C) To my vegetarian friends who argue that it is morally wrong to take sentient life, I answer that they cannot go for a walk in the country without committing that offense, for they walk on innumerable bugs and worms. 0


① A-C-B ② B-A-C ③ B-C-A ④ C-A-B ⑤ C-B-A

 


23. 20영독 2-11

 

Indeed, one of the most problematic aspects of global warming from the point of view of social policy stems from the fact that the phenomenon has so far manifested itself very unevenly around the world.


(A) Some places have had little warming in the past century, and some have even experienced cooling. "For extensive regions of the Earth, the warming of the past 80 years has deviated strongly from the global average," notes climate expert Ken Hare. 0

(B) "If you're considering political action, you have to remember that you're asking a considerable number of people in the world to take on faith that this is a truly global effect," he said. 2

(C) "This fact raises major difficulties for political action: in many countries, future temperatures will differ strongly from the global norm and global warming will seem like a fiction to local politicians." He points out, for example, that the lack of a strong warming trend in the United States accounted in part for the reluctance of the U.S. government to support the 1992 international convention on climate change. 1


① A-C-B ② B-A-C ③ B-C-A ④ C-A-B ⑤ C-B-A

 


24. 20영독 2-12

 

In the fifth century BCE, the Greek philosopher Protagoras pronounced, "Man is the measure of all things."


(A) Such an assumption makes us overlook a lot. Abilities said to "make us human" 一 empathy, communication, grief, toolmaking, and so on — all exist to varying degrees among other minds sharing the world with us. 1

(B) In other words, we feel entitled to ask the world, "What good are you?" We assume that we are the world's standard, that all things should be compared to us. 0

(C) Animals with backbones (fishes, amphibians, reptiles, birds, and mammals) all share the same basic skeleton, organs, nervous systems, hormones, and behaviors. Just as different models of automobiles each have an engine, drive train, four wheels, doors, and seats, we differ mainly in terms of our outside contours and a few internal tweaks. But like naive car buyers, most people see only animals' varied exteriors. 2


① A-C-B ② B-A-C ③ B-C-A ④ C-A-B ⑤ C-B-A

 


25. 20영독 3-1

 

If you find it difficult to stay wise-minded when your teen is rude, it's no surprise.


(A) Teens want to discharge the garbage of their day onto someone who will take it and love them anyway, sticking with them through thick and thin. If it becomes evident that everything you say is "wrong" (even though you know you're right), you can stop the merry-go-round whenever you like by simply withdrawing. Don't walk out with an angry refrain like "Well, I was just trying to be nice, and look at how you treat me!" 1

(B) Instead, say something humble (and accurate), such as, "I can see that you aren't in the mood for chatting. Oh, well, maybe later." Unless their nastiness is persistent, assume that the interaction is more about an opportunity for dumping the garbage than a reflection of your overall relationship. 2

(C) The deck is stacked against you because of several inescapable facts of normal teenage behavior. For one thing, teens often try to pick a fight. That's because, in the chaos and uncertainty of adolescence, parents are a secure base ― sort of like the eye of a storm. 0


① A-C-B ② B-A-C ③ B-C-A ④ C-A-B ⑤ C-B-A

 


26. 20영독 3-2

 

If I say to you, 'Don't think of a white bear', you will find it difficult not to think of a white bear.


(A) This dual-process system involves, first, an intentional operating process, which consciously attempts. to locate thoughts unrelated to the suppressed ones. Second, and simultaneously, an unconscious monitoring process tests whether the operating system is functioning effectively. 1

(B) If the monitoring system encounters thoughts inconsistent with the intended ones, it prompts the intentional operating process to ensure that these are replaced by appropriate thoughts. However, it is argued, the intentional operating system can fail due to increased cognitive load caused by fatigue, stress and emotional factors, and so the monitoring process filters the inappropriate thoughts into consciousness, making them highly accessible. 2

(C) In this way, thought suppression can actually increase the thoughts one wishes to suppress instead of calming them. One common example of this is that people on a diet who try not to think about food often begin to think much more about food. This ironic effect seems to be caused by the interplay of two related cognitive processes. 0


① A-C-B ② B-A-C ③ B-C-A ④ C-A-B ⑤ C-B-A

 


27. 20영독 3-3

 

A trait can be said to be adaptive if it is maintained in a population by selection.


(A) We can put the matter more precisely by saying that another trait is nonadaptive, or "abnormal," if it reduces the fitness of individuals that consistently manifest it under environmental circumstances that are usual for the species. In other words, deviant responses in abnormal environments may not be nonadaptive 一 they may simply reflect flexibility in a response that is quite adaptive in the environments ordinarily encountered by the species. 0

(B) In Americans of African descent, it is nonadaptive, for the simple reason that its bearers are no longer confronted by malaria. 2

(C) A trait can be switched from an adaptive to a nonadaptive status by a simple change in the environment. For example, the sickle-cell trait of human beings, determined by the heterozygous state of a single gene, is adaptive under living conditions in Africa, where it confers some degree of resistance to falciparum malaria. 1


① A-C-B ② B-A-C ③ B-C-A ④ C-A-B ⑤ C-B-A

 


28. 20영독 3-4

 

The philosopher Nelson Goodman argued that we should replace the question "What is art?"


(A) We could get the same information from a set of numbers. But if this same line is part of a drawing (say, the outline of a mountain), all of the line's physical properties are suddenly important and part of what the artist wants us to attend to — its color, texture, edges, thickness, among other things. And we cannot translate this experience into a set of numbers. 2

(B) For example, an object functioning as art is relatively replete (full), meaning that more of its physical properties are part of its meaning and should be attended to than when that same object is not functioning as a work of art. Goodman asks us to consider a zigzag line. Told that the line is a stock market graph, all we attend to are the peaks and dips. 1

(C) with the question "When is 띠t?" The same object can function as a work of art or not, depending on how the object is viewed. When an object functions as art, it exhibits certain "symptoms" of the aesthetic. 0


① A-C-B ② B-A-C ③ B-C-A ④ C-A-B ⑤ C-B-A

 


29. 20영독 3-5

 

Because of the perceptual frames users of computer software and websites have, they often click buttons or links without looking carefully at them.


(A) Even after unintentionally going backward a few times, they might continue to perceive the buttons in their standard locations. This is why consistent placement of controls is a common user-interface guideline, to ensure that reality matches the user's frame for the situation. 2

(B) For example, if the positions of the "Next" and "Back" buttons on the last page of a multistep dialog box switched, many people would not immediately notice the switch. Their visual system would have been lulled into inattention by the consistent placement of the buttons on the prior several pages. 1

(C) Their perception of the display is based more on what their frame for the situation leads them to expect than on what is actually on the screen. This sometimes confounds software designers, who expect users to see what is on the screen — but that isn't how human vision works. 0


① A-C-B ② B-A-C ③ B-C-A ④ C-A-B ⑤ C-B-A

 


30. 20영독 3-6

 

In 1979, Christopher Connolly cofounded a psychology consultancy in the United Kingdom to help high achievers perform at their best.


(A) They employed what Hogarth called a "circuit breaker." They drew on outside experiences and analogies to interrupt their inclination toward a previous solution that may no longer work. Their skill was in avoiding the same old patterns. 2

(B) They "traveled on an eight-lane highway," he wrote, rather than down a single-lane one-way street. They had range. The successful adapters were excellent at taking knowledge from one pursuit and applying it creatively to another, and at avoiding cognitive entrenchment. 1

(C) Over the years, Connolly became curious about why some professionals floundered outside a narrow expertise, while others were remarkably adept at expanding their careers ― moving from playing in a world-class orchestra, for example, to running one. Thirty years after he started, Connolly returned to school to do a PhD investigating that very question. Connolly's primary finding was that early in their careers, those who later made successful transitions had broader training and kept multiple "career streams" open even as they pursued a primary specialty. 0


① A-C-B ② B-A-C ③ B-C-A ④ C-A-B ⑤ C-B-A

 


31. 20영독 3-7

 

In a recent discussion of human rights in social work and human services practices, a researcher argues that the risk of strongly held primary values is that they can easily become an inflexible form of universalism, in which a single view of what it is to be human can become imposed by those with power (whether political, economic, professional, academic or cultural).


(A) This requires that practitioners rethink their understanding of community, in which there is a balance between what unites people and the many differences between them. 2

(B) The answer, for the researcher, is to seek a 'shared humanity', in which all members of a community are able to play active roles in the construction of what humanity means, and allows for these definitions to differ and to overlap without having to be identical. 1

(C) This can lead to an ironic situation in which human rights become associated with totalitarian ways of imposing particular ideals, through asserting that what it is to be human has to take one particular form. 0


① A-C-B ② B-A-C ③ B-C-A ④ C-A-B ⑤ C-B-A

 


32. 20영독 3-8

 

When biologists consider complex human activities such as the arts, they tend to assume that their compelling qualities are derivations of basic drives.


(A) If any given activity can be seen to aid survival or facilitate adaptation to the environment, or to be derived from behaviour which does so, it 'makes sense' in biological terms. For example, the art of painting may originate from the human need to comprehend the external world through vision; an achievement which makes it possible to act upon the environment or influence it in ways which promote survival. 0

(B) It enables the draughtsman to study an object in its absence, to experiment with various images of it, and thus, at least in fantasy, to exert power over it. 2

(C) The Paleolithic artists who drew and painted animals on the walls of their caves were using their artistic skills for practical reasons. Drawing is a form of abstraction which may be compared with the formation of verbal concepts. 1


① A-C-B ② B-A-C ③ B-C-A ④ C-A-B ⑤ C-B-A

 


33. 20영독 3-9

 

Self-awareness, or reflective thought, is the main attribute distinguishing humans from animals.


(A) In consequence, it is denied access to a whole domain of reality in which mankind can move freely. Systems of physics, philosophy, mathematics, and astronomy, for example, have all been constructed because of man's unique ability to reflect inwardly. 2

(B) It is the consciousness that enables us to contemplate ourselves. Reflection is the power to turn one's consciousness upon oneself, to know oneself and, especially, to know that one knows. 0

(C) Humans are the only creation in the universe who can be the object of their own reflection and, because of that, another world is born: an inner world, a reality in which no lower animal can ever participate. Incapable of contemplating itself, or of being aware of itself as the conscious subject, not even a higher type of animal, such as a dog or cat that knows who its master is and where its food is, can know that it knows. 1


① A-C-B ② B-A-C ③ B-C-A ④ C-A-B ⑤ C-B-A

 


34. 20영독 3-10

 

I understand it is not likely you are going to tell an interviewer about all of your job search activities or provide a status report, no, clearly it's none of their business.


(A) There is no need to, and I suggest you should not, share the name or details of the other company; just making them aware of your status is enough. I would, however, caution you that if it isn't true, don't fake it. 2

(B) However, there is nothing wrong with being honest to a limited degree, if you are reaching a critical stage with another company with whom you are also interviewing. Reasonably speaking, most of us are pursuing more than one job at a time. 0

(C) It's not a mistake to say to a hiring official, "I appreciate the opportunity for this interview, I am interested in this job and your company, but I think it is fair to tell you I am also talking to some other companies, and one of them has invited me to a final interview." Yes, this can be considered a take-away close, but it is simply the truth. 1


① A-C-B ② B-A-C ③ B-C-A ④ C-A-B ⑤ C-B-A

 


35. 20영독 3-11

 

The fact that emotions are unlearned, automated, and set by the genome always raises the specter of genetic determinism.


(A) There are things that you fear that I do not, and vice versa; things you love and I do not, and vice versa; and many, many things that we both fear and love. In other words, emotional responses are considerably customized relative to the causative stimulus. In this regard, we are quite alike but not entirely. 2

(B) Is there nothing personal and educable about one's emotions? The answer is that there is plenty. 0

(C) The essential mechanism of the emotions in a normal brain is indeed quite similar across individuals, and a good thing too because it provides humanity, in diverse cultures, with a common ground of fundamental preferences on the matters of pain and pleasure. But while the mechanisms are distinctly similar, the circumstances in which certain stimuli have become emotionally competent for you are unlikely to be the same as for me. 1


① A-C-B ② B-A-C ③ B-C-A ④ C-A-B ⑤ C-B-A

 


36. 20영독 3-12

 

One of the most widespread, sadly mistaken, environmental myths is that living "close to nature" out in the country or in a leafy suburb is the best "green" lifestyle.


(A) The pattern of life in the country and most suburbs involves long hours in the automobile each week, burning fuel and spewing exhaust to get to work, buy groceries, and take kids to school and activities. City dwellers, on the other hand, have the option of walking or taking transit to work, shops, and school. 1

(B) The larger yards and houses found outside cities also extract an environmental toll in terms of energy use, water use, and land use. It's clear that the future of the Earth depends on more people gathering together in compact communities. 2

(C) Cities, on the other hand, are often blamed as a major cause of ecological destruction — artificial, crowded places that suck up precious resources. Yet, when you look at the facts, nothing could be farther from the truth. 0


① A-C-B ② B-A-C ③ B-C-A ④ C-A-B ⑤ C-B-A

 


37. 20영독 4-1

 

Opera is conventional.


(A) We understand how each genre works, and we know that not every murder has six suspects who can be gathered in one room in the last chapter by a brilliant detective. Yet we gladly accept the unreality of the situation because of the pleasure it provides us. 2

(B) We like conventions, provided that we understand, accept, and desire them. Conventions are simply the result of participants ' agreeing on the rules, of simplifying a complex world so that we can concentrate on what interests us. We are accustomed, for example, to detective novels, television situation comedies, and western movies. 1

(C) Nobody sings all the time in the real world. Nobody has an orchestra that begins to play whenever he feels emotional. Conventions are of course necessary in the theater, and even more so in opera. 0


① A-C-B ② B-A-C ③ B-C-A ④ C-A-B ⑤ C-B-A

 


38. 20영독 4-2

 

One great danger of intellectual property lies in the threat to liberty.


(A) But we all have an interest in seeing public research programmes into diseases and health being carried out. We want, for example, public researchers to continue working on the genes for breast and ovarian cancer and helping to develop cheaper, more effective clinical tests. 1

(B) When a group of scientists stop working on a protein molecule because there are too many intellectual property rights that surround the use of the molecule, a basic freedom, the freedom to research, has been interfered with. The liberty cost of intellectual property rights may seem remote because most of us do not carry out research on proteins. 0

(C) We do not want them obstructed by announcements like the following: 'This important patent solidifies Myriad's dominant proprietary position on the BRCA1 and BRCA2 genes' (the genes linked to breast and ovarian cancer). Companies are entitled to protect their treatments for disease but not, through use of their patents, to prevent others from access to genes which are linked to the origins of disease. 2


① A-C-B ② B-A-C ③ B-C-A ④ C-A-B ⑤ C-B-A

 


39. 20영독 4-3

 

Too much choice is overwhelming for many people and results in consumers who are less satisfied with the shopping experience, which ultimately hurts retail profitability.


(A) Consider, for example, a consumer who wants a product to relieve her cold symptoms. First, she has to decide where to shop for such a product. 0

(B) Even within a particular brand, she can choose products that vary in when they should be taken, how they can be taken, and what symptoms they treat. Ironically, all this is enough to make a healthy person ill. 2

(C) Over-the-counter pharmaceuticals are now commonly available in a variety of locations ranging from hotel gift shops and convenience stores to drug and grocery stores. Once she has chosen a store and is standing in front of the shelf, the consumer faces a dizzying array of products from a variety of brands with a broad spectrum of ingredients. 1


① A-C-B ② B-A-C ③ B-C-A ④ C-A-B ⑤ C-B-A

 


40. 20영독 4-4

 

In today's business environment, firms may face competition from companies located in their own home market as well as from those based halfway around the world.


(A) It should be noted that whether or not a firm elects_ to operate internationally, it is still vulnerable to changes taking place in the global marketplace. 2

(B) One need look no further than the sovereign debt crisis in Europe that began to unfold in 2009 or the sub-prime mortgage crisis in the USA, to appreciate the impact of such events on economic growth, consumer spending and prosperity. Economic and political events taking place around the world may have a profound effect on a company's prospects for survival and growth. 1

(C) Also, customer trends which take root in one country may quickly spread to other parts of the world, creating either new marketing opportunities or potential threats to a firm's established products and business models. In addition, political and economic crises in one region may have important implications for consumer and business confidence around the world. 0


① A-C-B ② B-A-C ③ B-C-A ④ C-A-B ⑤ C-B-A

 


41. 20영독 4-5

 

Clarity is often a difficult thing for a leader to obtain.


(A) And the merits of a leader's most important decisions, by their nature, typically are not clear-cut. Instead, those decisions involve "a process of assigning weights to competing interests, and then determining, based upon some criterion, which one predominates. The result is one of judgment, of shades of gray; like saying that Beethoven is a better composer than Brahms." 2

(B) Concerns of the present tend to loom larger than potentially greater concerns that lie farther away. Some decisions by their nature present great complexity, whose many variables must align a certain way for the leader to succeed. 0

(C) Compounding the difficulty is what ergonomists call information overload, where a leader is overrun with inputs — via e-mails, meetings, and phone calls — that only distract and clutter his thinking. Alternatively, the leader's information might be only fragmentary, which might cause her to fill in the gaps with assumptions — sometimes without recognizing them as such. 1


① A-C-B ② B-A-C ③ B-C-A ④ C-A-B ⑤ C-B-A

 


42. 20영독 4-6

 

Many women find their inner critic speaks up most loudly around their most deeply felt dreams for their lives and work, because they feel particularly vulnerable about them.


(A) As long as you don't venture forth out of that zone, the inner critic can leave you alone ― like a guard taking a nap. Yet when you approach the edge of your comfort zone, test old beliefs, contemplate change, or stretch into playing bigger, you wake the sleeping guard. 1

(B) They experience the most panicky, overwhelming self-doubt when they are moving toward what they truly long to do. The inner critic is like a guard at the edge of your comfort zone. 0

(C) The inner critic recites its lines in an attempt to get you to go back into the familiar zone of the status quo. Many women find that the more strongly the inner critic shows up, the louder and meaner and more hysterical its voice, the closer they are to a breakthrough or the more likely they are to be on the edge of taking a very important step. In this sense, when you hear a major inner critic attack, it likely means you are playing bigger. 2


① A-C-B ② B-A-C ③ B-C-A ④ C-A-B ⑤ C-B-A

 


43. 20영독 4-7

 

Researchers of the Earth's system have been focused, appropriately, on developing a better understanding of the vast and interconnected processes that create our environment, and they have made a great deal of progress since the publication of A Sand County Almanac, a 1949 non-fiction book by Aldo Leopold.


(A) Although there are many problems left to solve, knowledge about planetary life-support systems has progressed far more rapidly than society's willingness to use this knowledge. The biggest challenge facing humanity is that our political, social, and economic systems are shortsighted. 0

(B) This is what "thinking like a mountain" should come to mean in the Anthropocene. If we succeed in transforming our culture, residents of the later Anthropocene will look back on the early twenty-first century as a time of human enlightenment, when people learned to truly think like mountains by anticipating their long-lasting and complex effects on the world. 2

(C) Long-term planning typically considers years or decades, but the global environmental processes we are now influencing play out over centuries, millennia, or more. We need to instill a sense of geologic time into our culture and our planning, to incorporate truly long-term thinking into social and political decision making. 1


① A-C-B ② B-A-C ③ B-C-A ④ C-A-B ⑤ C-B-A

 


44. 20영독 4-8

 

From the early twentieth century through the beginning of the 1970s, the sociological analysis of cultural objects took one of two competing paths, which interestingly shared a core assumption.


(A) The products of mediated culture, whether books, songs, or fashion, were thought to be expressive symbols that changed in lockstep with evolutions in society. For example, in 1919 the anthropologist Alfred Kroeber argued that the hemlines of women's dresses were prescribed through "civilizational determinism"; they were a window into macro-level cultural values and belief systems. 0

(B) While these "nothing-but" arguments quibbled on the direction of the association between culture and the economy, they both assumed that hemline lengths in women's fashion were reflections of outsized societal forces. 2

(C) In turn, by the mid-l920s the economist George Taylor argued that instead the hemlines of dresses go up with rises and go down with declines in the stock market. For Taylor, hemlines were determined by macro-level economic, not cultural, shifts. 1


① A-C-B ② B-A-C ③ B-C-A ④ C-A-B ⑤ C-B-A

 


45. 20영독 4-9

 

It is clear that even a single initial encounter with a word can potentially leave a memory trace of its use.


(A) But then no memory trace of the word would be left upon this second encounter. This situation could be repeated ad infinitum without any memory trace of the meaning being retained. 1

(B) Why is this clear? Consider the counterfactual: if no memory trace could exist after a single exposure, then the second time the word was encountered would be exactly the same as the first time. 0

(C) If this were the case, we would be utterly unable to learn any words. Therefore, it must be possible for an initial memory trace to exist in order for it to be strengthened upon subsequent exposure. Fortunately, we know that human brains have a vast capacity for implicit memory, even though memories may not readily be brought to consciousness (they are not always easy to recall or make explicit). 2


① A-C-B ② B-A-C ③ B-C-A ④ C-A-B ⑤ C-B-A

 


46. 20영독 4-10

 

Consider for a moment a fish.


(A) The thought that I am trying to convey to you is that, once you discover who you are and operate in that realm, you will always come out successful. But if you follow the guile of other things that may seem attractive and leave the place of your power, you will never win. Life is about winning, not necessarily about winning against others but winning at being you, and the way to win is to figure out who you are and do it. 2

(B) Fish belong in the water, and when it is in the place where it belongs, it dominates all other things that do not belong there that may try to compete with it. Man is no match for the fish as long as it remains in the water, so in order for us to have any power over the fish, we have to capture it by using tools and many forms of trickery to get it out of the place of its dominance. 0

(C) We understand that man versus fish in the water, fish wins. But if we can succeed in taking it out of the water, the fish will lose every time. 1


① A-C-B ② B-A-C ③ B-C-A ④ C-A-B ⑤ C-B-A

 


47. 20영독 4-11

 

For a while, people thought that 10,000 hours of practice was what it took to become an expert at something.


(A) They know what time of day works best for them, when to push on through tiredness or confusion, and when to take a break. A pianist knows that sometimes it helps to play a piece at half speed, to get the fingering exactly right, and sometimes it is worth trying to play it at double speed, mistakes and all, to get a better feel for the flow and cadence of the piece. A footballer is able to suggest to the coach a new way of practicing an attacking maneuver. 2

(B) But now we know that this figure is a gross oversimplification, because the quality of practice matters even more than the quantity. Expert practicers get better faster. 0

(C) They have learned to pick out the difficult parts of what they are trying to do, and work especially hard on those. They make good use of recordings and videos of their own performance. 1


① A-C-B ② B-A-C ③ B-C-A ④ C-A-B ⑤ C-B-A

 


48. 20영독 4-12

 

Today companies frequently require of their employees a different level and quality of engagement with the company.


(A) As Michael Hardt and Antonio Negri describe it, we have transitioned from a society in which there are factories to a factory society in which the entire social performs as a factory. 2

(B) In earlier periods, employees were often treated like machines, but their private lives, consisting in their leisure time, passions, and beliefs, remained largely unaffected. Nowadays, employees frequently contribute more than physical labor; they are required to innovate, make decisions, and work effectively as a team. 0

(C) As a result, they no longer leave work when they go home but instead continue at some level nonstop. The fact that workers are being asked to contribute collectively to the production of goods and services has begun to reweave the fabric of the social, from one based in the distinction between public and private spaces to one in which networks of associations and the advantages they may offer to move ahead now function as the organizing force in most daily interactions. 1


① A-C-B ② B-A-C ③ B-C-A ④ C-A-B ⑤ C-B-A

 


49. 20영독 5-1

 

If you have become much less active, spending a lot of time alone focused on feeling depressed, think about activities that engage your attention and that require a moderate level of concentration and effort.


(A) But you would not want to choose exceedingly complex, demanding tasks, such as studying for a math exam, because any difficulties you have doing such a task could reinforce negative, self-critical thinking. When you are feeling depressed, a mentally demanding task may become overwhelming, and then you will start ruminating about failing the task (e.g., "Depression is going to ruin me, because I can't even concentrate on this simple math"). 1

(B) Driving, for example, is probably not ideal, because it is such an automatic behavioral sequence for most people that it siphons off only a small amount of attention. Something like strenuous aerobic exercise may be much more effective, because it captures more attention. 0

(C) Thus moderately engaging activities are probably the best distracters for rumination. Take some time to discover the positive activities that are most effective in reducing or eliminating your bouts of rumination. 2


① A-C-B ② B-A-C ③ B-C-A ④ C-A-B ⑤ C-B-A

 


50. 20영독 5-2

 

Body water is involved in several functions critical to performance.


(A) An abundant supply of body water, first to transport muscle-generated heat and then to produce the sweat needed for evaporative cooling, is the best insurance against the complications of heat cramps, heat exhaustion, and life-threatening heat stroke. 2

(B) Of most importance to the exercising athlete is the fact that a large amount of heat generated by exercising muscles is transported by water in the blood to the skin, where water is essential for the production of sweat. Body heat is dissipated most efficiently through the evaporation of sweat on exposed skin surfaces. 1

(C) The body's chemical processes that provide the energy for muscle work occur in water. All of the transport functions of oxygen, nutrients, and body wastes are carried on in body water. 0


① A-C-B ② B-A-C ③ B-C-A ④ C-A-B ⑤ C-B-A

 


51. 20영독 5-3

 

Sport sends some messages that support socioeconomic inequities.


(A) For example, winning is the most prevalent organizing theme in newspaper stories and telecasts of sporting events. Winning is usually attributed to self-discipline, talent, and hard work. If an athlete or a team doesn't win, then we assume that the player or the team was lazy or lacked talent and so didn't deserve to win. 0

(B) The point here is not that merit is a bad idea. The problem is that this logic often leads us to overlook the societal barriers (e.g., poor nutrition, neighborhood gang violence, poor access to libraries and computers, dysfunctional families, lack of child care) that prevent poor people from developing themselves to the fullest and becoming valuable members of society. 2

(C) Such beliefs underscore the American conception of merit ― we often link hard work and talent to financial success. The flip side is that if someone fails financially, it must be because she or he isn't talented or didn't work hard. This reasoning allows us to hold the belief that the rich and poor both deserve whatever money they have. 1


① A-C-B ② B-A-C ③ B-C-A ④ C-A-B ⑤ C-B-A

 


52. 20영독 5-4

 

According to Wikipedia, of the countries that have adopted color television, twenty-nine had done so by 1969.


(A) The addition of color must have been seen as a powerful boost if used wisely. Though, no doubt, early color commercials were likely simply reshot black-and-white spots, creative directors at agencies from New York to London to Paris and beyond must have seen this new technology as an advantage in promoting brands. 1

(B) The vast majority of these were in Europe and North America. The rise in the use of television in the 1950s opened up the creativity of advertising within a more emotional and powerful medium. 0

(C) In many ways, this transition from black and white to color must have been similar to the challenges facing actors when sound was introduced to movies. For advertising agencies in the 1960s, an entire world of new possibilities and requirements put them back to square one: they could either understand how to use color effectively or face losing clients. 2


① A-C-B ② B-A-C ③ B-C-A ④ C-A-B ⑤ C-B-A

 


53. 20영독 5-56

 

In the business world, large bureaucratic organizations are sometimes unable to compete against smaller, innovative firms, particularly in industries that are changing quickly.


(A) This situation occurs partly because innovative firms tend to have flatter and more democratic organizational structures. Compare the flat network structure in smaller, innovative firms with the traditional bureaucratic structure in large bureaucratic organizations. Note that the network structure has fewer levels than the traditional bureaucratic structure. Moreover, in the network structure, lines of communication link all units. 0

(B) They began eliminating middle-management positions. They allowed worker participation in a variety of tasks related to their main functions and delegated authority to autonomous teams of a dozen or so workers that were allowed to make many decisions themselves. They formed "quality circles" of workers to monitor and correct defects in products and services. Consequently, product quality, worker morale, and profitability improved. Today, these ideas have spread well beyond the Swedish and Japanese automobile industries and are evident in many large North American companies, both in the manufacturing and in the service sectors. 2

(C) In the traditional bureaucratic structure, information flows only upward. Much evidence suggests that flatter bureaucracies with decentralized decision making and multiple lines of communication produce more satisfied workers, happier clients, and bigger profits. Some of this evidence comes from Sweden and Japan. Beginning in the early 1970s, Volvo and Toyota were at the forefront of bureaucratic innovation in these countries. 1


① A-C-B ② B-A-C ③ B-C-A ④ C-A-B ⑤ C-B-A

 


54. 20영독 5-7

 

The lesson of ecology is that, as species of the planet, we are all connected in a web of life.


(A) A Buddhist parable brings to life this rather stark and scientific lesson from ecology. During his meditation, a devotee fantasizes that he is eating a leg of lamb, an act proscribed by Buddhism where strict adherence to vegetarianism is required. 0

(B) A more prosaic way of reaching the same sense of connection is to think about a time when you might have hit an animal or bird when driving your car. The sense of shock and horror that you have destroyed something so precious is the same, no matter how insignificant the animal appears. 2

(C) His spiritual master suggests that when this fantasy comes to him he draws a cross on the leg of lamb. The devotee follows the advice and, on returning to self-consciousness, is amazed to find the cross on his own arm. 1


① A-C-B ② B-A-C ③ B-C-A ④ C-A-B ⑤ C-B-A

 


55. 20영독 5-8

 

The distinctions ― between mind and body, and war and peace — appear to have lost credibility altogether, with the result that we now experience conflict intruding into everyday life.


(A) Meanwhile, new forms of violence have emerged, in which states are attacked by non-state groups, interstate conflicts are fought using nonmilitary means (such as cyberwarfare), and the distinction between policing and military intervention becomes blurred. 1

(B) Since the 1990s, rapid advances in neuroscience have elevated the brain over the mind as the main way by which we understand ourselves, demonstrating the importance of emotion and physiology to all decision making. 0

(C) As society has been flooded by digital technology, it has grown harder to specify what belongs to the mind and what to the body, what is peaceful dialogue and what is conflict. In the obscure space between mind and body, between war and peace, lie nervous states: individuals and governments living in a state of constant and heightened alertness, relying increasingly on feeling rather than fact. 2


① A-C-B ② B-A-C ③ B-C-A ④ C-A-B ⑤ C-B-A

 


56. 20영독 5-9

 

The obvious problems being caused by economic growth have not been ignored by academics: they were noticed by some in the economics profession, who then attempted to incorporate these concerns into their discipline.


(A) Environmental economists were keen to bring these negative impacts back within the discipline. However, they still approached the subject in a scientific and measurement-based way, for example, using shadow pricing to measure how much people were concerned about noise pollution or the loss of habitat. 1

(B) This led to the development of environmental economics, and also the related study of natural-resource economics. Conventional economics considers environmental impact to be an 'externality', something outside its concern. 0

(C) In other words, the way in which economics traditionally marginalizes or ignores something that cannot be priced was still adhered to, but the response was to attempt to evaluate in some way aspects of life which economics had ignored. Green economists would consider this to be a category error; in other words, they believe it is important to accept that some aspects of life have social or spiritual worth that simply cannot be measured. 2


① A-C-B ② B-A-C ③ B-C-A ④ C-A-B ⑤ C-B-A

 


57. 20영독 5-10

 

Until fairly recently, human beings lived in kin bands of usually no more than twenty people, loosely associated into tribes of perhaps a few hundred.


(A) Spend some time alone with a person or small group in silence, and observe whether, after just a few days or even hours, you feel more intimately connected with them than if you'd been talking. The empathy and intuitive understanding of others that develops in such circumstances is amazing. 2

(B) Open to nature and each other, they knew each other more intimately than we can imagine today. Speech may have been superfluous, as it often is between lovers, or between mother and baby. 0

(C) When we know someone that well, we know without asking what they are thinking and feeling. All the more in prelinguistic times, when our empathetic faculties were yet unclouded by the mediatory apparatus of language. 1


① A-C-B ② B-A-C ③ B-C-A ④ C-A-B ⑤ C-B-A

 


58. 20영독 5-1112

 

When students in a civilian college are found to be cheating on an examination, it does not make a story in the national media — not even headlines in the local papers and probably not a story in the college newspaper.


(A) In the junior class, 184 students were formally accused of cheating, and 152 of those were expelled. Similar cheating "scandals" at the Naval Academy in Annapolis and at the Air Force Academy in Colorado Springs have also been given the most serious attention. Why is cheating by an officer candidate taken more seriously than cheating by a civilian student at the same educational level? The question almost answers itself. Civilian schools have honor codes, but moral education is usually not a conscious educational goal. 1

(B) The students may have a hearing before a student/faculty disciplinary board, and a penalty may be imposed if the verdict is that the students are guilty. The penalty may be a failure in the course or a brief suspension from the institution; often it is less severe than either of these. The West Point scandal of 1976 made front-page news across the country. Military students were cheating, which violated the honor code. That event, a most serious matter, was followed by student dismissals and lengthy editorial comment. 0

(C) The Military Academy at West Point has a well-known honor code requiring that "a Cadet will not lie, cheat or steal, nor tolerate anyone who does." The experience of living by such a code, we hope, will help produce officers who can be trusted to avoid moral individualism. They will have consciously practiced the reflex of honesty, of consistently doing what they promised to do, regardless of temptation. The thought and temptations of individualism are always in the mind, but we assume that people can be found who consciously adopt another moral style. 2


① A-C-B ② B-A-C ③ B-C-A ④ C-A-B ⑤ C-B-A

 


59. 20영독 6-1

 

In the 1990s the Internet became the newest entrant in the baby advice field.


(A) Depending on her location and the post office this entire process might take several weeks. In 2000 she could access the Internet from her home computer at 2:00 a.m. if necessary and find an answer within minutes. Hospitals and pediatricians also embraced the Internet and Web sites written by and directed by them were set up across the country. 2

(B) The Internet meant that the speed of information available to mothers had compressed from weeks and months in the early 1900s to near instantaneous by 2000. In the 1910s a mother with a baby care question that was unanswerable in her immediate surroundings could write to the Children's Bureau and wait for a reply. 1

(C) Major manufacturers of baby equipment as well as the neighbor down the street set up Web sites to help parents care for their babies. Chat rooms brought mothers from all around the country together online to discuss, question, and support each other. 0


① A-C-B ② B-A-C ③ B-C-A ④ C-A-B ⑤ C-B-A

 


60. 20영독 6-2

 

We presumably play many games because they are exciting, but will playing a particular game result in a positive or a negative mood?


(A) The higher the stakes, in terms of time investment, public acknowledgment, and personal importance, the higher are the potential losses and rewards. We make very rough estimates of this gamble, factoring in the likelihood of failure along with the time investment required, the audience for our performance, and our personal investment in performing well. 1

(B) We are probably also not very good at doing the calculation — optimists may be unable to believe that failure is a possibility, for example. 2

(C) This is where the fundamental unpredictability of games comes in, since failure will likely result in a worse mood than success will. To play a game is to take an emotional gamble. 0


① A-C-B ② B-A-C ③ B-C-A ④ C-A-B ⑤ C-B-A

 


61. 20영독 6-3

 

Just as other living creatures thrive or perish depending on how well they adapt to the environment, so too do humans, although the consequences are not usually so dramatic.


(A) They might induce lower levels of stress (and its related health risks) by shielding individuals from certain jobs that place undue tension on work-life balance. Those who do adapt to the demands of higher education might enter high-stress careers that call for adaptive techniques of their own, such as meditation or effective time-management. 2

(B) For example, students often fail to adapt their attention and work habits to the demands of educational institutions. In the short term, their grades suffer, but the long-term difficulties associated with this maladaptation are no less palpable. 0

(C) They might have trouble adjusting their behaviour once they enter the workforce, which ultimately might deprive them of career advancement and financial remuneration. On the other hand, more relaxed work habits can be adaptable in other ways. 1


① A-C-B ② B-A-C ③ B-C-A ④ C-A-B ⑤ C-B-A

 


62. 20영독 6-4

 

Some years ago in the United States, a woman named Linda Kenney nearly died when, prior to an operation, anesthesia was administered to her improperly.


(A) The Kenneys were deeply moved by the doctor's letter, and even more impressed when he came to their home and begged forgiveness. The couple ended up not pursuing a malpractice suit, and instead, in conjunction with the doctor, started a group to help both doctors and families deal with the trauma of medical and surgical errors. 2

(B) Her husband, understandably, wanted to sue the doctor and the hospital. Then the anesthesiologist wrote the couple a note expressing his regrets and grief. He told her, "Whenever you want to speak to me, I will make myself available. 0

(C) Here is my phone number." The Kenneys later learned that the doctor did not inform the hospital or its lawyers of what he was doing. He knew that they would probably have forbidden him to contact the patient, because acknowledging his responsibility so frankly would put him and the hospital at great risk if the patient sued. 1


① A-C-B ② B-A-C ③ B-C-A ④ C-A-B ⑤ C-B-A

 


63. 20영독 6-56

 

Because the phrase is so well known, some readers may be surprised to know that Darwin never wrote "survival of the fittest."


(A) A second and related effect was to justify genocide and colonization (the dash to carve up Africa culminated in the Berlin Conference of 1884, when Europeans sat around tables and drew lines on maps to delimit "their" possessions, and the "taming" of the American West took place at the same time), and to undermine any suggestion of social welfare for the poor in Europe. That people were not as well off as those (whites) with the most resources was taken as evidence that they were not as "fit," nor deserving. This Social Darwinism helped define as "natural" the hierarchy of races that had been constructed and classified through the discourses of science. 2

(B) In the process, it detoured from Darwin's ideas; Organisms do not consciously "struggle" in this way 一 they do not know if a mutation has taken place that may require millennia to become widespread in their species. They are simply born, live, and die, and pass on whatever was in their genes. This idea of competition grew out of and had more application to the burgeoning industrialization and class divisions that were occurring in Britain than it did to Darwin's idea of natural selection. 1

(C) This term was coined by British philosopher Herbert Spencer (who, incidentally, also first used the term "evolution") in applying Darwin's ideas to topics he himself did not address. The "survival of the fittest" concept had several effects. First, it naturalized competition as part of a "struggle for existence." 0


① A-C-B ② B-A-C ③ B-C-A ④ C-A-B ⑤ C-B-A

 


64. 20영독 6-7

 

Now, back to the essential issue at hand — that of human intellect being dependent upon the application of logical principles.


(A) Even though mathematics is inherently a highly logical discipline, any person who performs brilliantly in language, or any such endeavor, is showing high intelligence, because any such effective use of language, and such reasoning powers, must involve much logical thinking. And, in general, the greater capacity one has for effective application of logical principles (whether in language, mathematics, artistic creations, or whatever), the greater is one's power of intelligence. 1

(B) There is much evidence that the quality that we usually refer to as intelligence (in humans), is closely related to the individual's capacity for logical thought. For example, a great mathematician must be highly intelligent, because his/her successful learning and manipulating of mathematical rules is impossible, apart from much capacity for logical thinking. 0

(C) Therefore, it can be seen that intelligence and logic are closely intertwined. 2


① A-C-B ② B-A-C ③ B-C-A ④ C-A-B ⑤ C-B-A

 


65. 20영독 6-8

 

Millions of people around the world suffer a lack of necessary nutrients because of limitations on what will grow in their region.


(A) Millions of people suffer from this problem worldwide because they live in areas where vegetables containing the vitamin will not grow. Since rice will grow in these regions, golden rice could provide at least some of this critical vitamin to the local population. 2

(B) One area that scientists are exploring is genetically modifying vegetables to increase their nutritional value. An example of this is "golden rice." 0

(C) This variety of rice is genetically engineered to produce vitamin A — the vitamin that gives carrots their orange color. A vitamin A deficiency can have serious effects, including blindness and even death. 1


① A-C-B ② B-A-C ③ B-C-A ④ C-A-B ⑤ C-B-A

 


66. 20영독 6-9

 

The growing emphasis on 'work readiness' is the subject of much debate.


(A) However, the demand for work-ready graduates, who are familiar with organizational practices in the workplace, is increasing. Employers value work experience believing that exposure to the workplace while studying provides students with the opportunity to acquire valuable insights into how the workplace operates and what is expected of them in different workplace settings. Employers report that work experience improves graduates' soft skills, increases confidence and helps relate their studies to employment, making them more rounded and with more realistic expectations of work. 1

(B) The opportunity to make contacts and create networks for future employment is another advantage. In other words, work experience improves work readiness. 2

(C) Some believe that work and education are qualitatively different social sites. While education provides skills and knowledge useful both in the short and long term, it can only provide broad or generic training for work. Specific training for a particular job can only be undertaken after study. 0


① A-C-B ② B-A-C ③ B-C-A ④ C-A-B ⑤ C-B-A

 


67. 20영독 6-10

 

A baby who can't hear would have a stronger need to remain in visual contact with her parents as she begins to roam.


(A) A baby who can't see would benefit from hearing lots of encouraging words and vocalizations, as well as touches and smells. In fact, we often recommend interesting games that enable babies who can't see to locate people and objects by touch and sounds, as well as smells. 1

(B) In that way, they can create a sensory road map of their home even though they can't see. The important sense of space and spatial relationships that we all need to feel secure and to navigate can be formed from many of our senses, and not just our vision. 2

(C) Mom and Dad could make a concerted effort to make their friendly waves or approving smiles especially vivid to their toddler, even at a distance. They could even make a point of coming over to her from time to time and offering a quick hug or peck on the cheek to reassure their toddler that her explorations won't isolate her from them. 0


① A-C-B ② B-A-C ③ B-C-A ④ C-A-B ⑤ C-B-A

 


68. 20영독 6-1112

 

Ideation in its many forms is an area today where humans have a comparative advantage over machines.


(A) Scientists come up with new hypotheses. Journalists sniff out a good story. Chefs add a new dish to the menu. Engineers on a factory floor figure out why a machine is no longer working properly. Many of these activities are supported and accelerated by computers, but none are driven by them. Picasso's quote — Computers are useless. 0

(B) Ideation, creativity, and innovation are often described as 'thinking outside the box,' and this characterization indicates another large and reasonably sustainable advantage of human over digital labor. Computers and robots remain lousy at doing anything outside the frame of their programming. Watson, for example, is an amazing Jeopardy! player, but would be defeated by a child at Wheel of Fortune, The Price is Right, or any other TV game show unless it was substantially reprogrammed by its human creators. Watson is not going to get there on its own. 2

(C) They can only give you answers. — is just about half right. Computers are not useless, but they're still machines for generating answers, not posing interesting new questions. That ability still seems to be uniquely human, and still highly valuable. We predict that people who are good at idea creation will continue to have a comparative advantage over digital labor for some time to come, and will find themselves in demand. In other words, we believe that employers now and for some time to come will, when looking for talent, follow the advice attributed to the Enlightenment sage Voltaire: "Judge a man by his questions, not his answers." 1


① A-C-B ② B-A-C ③ B-C-A ④ C-A-B ⑤ C-B-A

 


69. 20영독 7-1

 

Whether or not they allow some contemporary technology to be squeezed in, the reformers fundamentally believe that they can bring back "what once worked."


(A) It is tragic because so much of what we do currently teach, and what so many want to preserve, is now unimportant because the context for education has changed so radically. 1

(B) That belief has tragic ramifications for our students today. 0

(C) In the current environment, every field and job — from factory work to retail to healthcare to hospitality to garbage collection ― is in the process of being transformed dramatically, and often unrecognizably, by technology and other forces. And while most reformers recognize that society is going through dramatic changes (even though few truly "get" their extent, speed, and implications), they too often — and paradoxically 一do not see the need for education to change fundamentally to cope with themselves. 2


① A-C-B ② B-A-C ③ B-C-A ④ C-A-B ⑤ C-B-A

 


70. 20영독 7-2

 

Heller and Eisenberg claimed that gene patenting may cause a problem of the "tragedy of the anti-commons" for biomedical research, because a gene patent can be broad enough to cover any commercial use of the gene and the gene product.


(A) This overuse is generally referred to as a "tragedy of the commons," and privatization is often used to solve this problem. 1

(B) When people hold a resource in common, they tend to overuse it because they lack any incentive to conserve the resource. 0

(C) However, when a scarce resource is overprivatized the result can be a "tragedy of the anti-commons," which will result in the under-use of a resource because too many people are excluded from using the resource. Under-use in human gene patents was said to be pervasive, because the high licensing fee limits any further research, especially when most diseases are polygenic, meaning that multiple genes are involved in the manifestation of a disease and several pieces of genetic material are needed to develop a product. 2


① A-C-B ② B-A-C ③ B-C-A ④ C-A-B ⑤ C-B-A

 


71. 20영독 7-3

 

The thing about maintaining a lawn is that the very action of cutting encourages grasses to multiply.


(A) If you cut these plants off at the ground, you chop off the meristem, and they have to start the process of producing a shoot all over again. This regrowth can only happen at considerable energy cost to the plant. If you keep doing this, eventually the plant will give up the ghost and die. 1

(B) On the other hand, grasses keep their meristems tucked away at the base of the plant. If you chop off the leaves of a grass plant it merrily continues to grow from the bottom, unconcerned by the decapitation it receives. Grasses have evolved this system in response to being repeatedly eaten by herbivores. By regularly mowing a lawn, the only plants that can survive are grasses. 2

(C) Trees, for example, do very poorly if you repeatedly and regularly cut them down just above the ground. In fact, many plants cannot cope with being regularly felled. The bit of a plant that does the growing is called a meristem, and in plants such as tulips, begonias and carnations, the meristems are at the tips of the growing shoots. 0


① A-C-B ② B-A-C ③ B-C-A ④ C-A-B ⑤ C-B-A

 


72. 20영독 7-4

 

The effect of one's actions on collective consequences, and one's participation in those consequences regardless of one's actions, is relevant because there is a dollar value associated with these actions.


(A) There is a push and pull of cost and relative savings that is difficult for consumers to predict, as it is affected so greatly by their collective actions. It brings to mind economist John Maynard Keynes' description of stock-market investing: a beauty contest in which the winner is not any of the contestants, but the judge whose scores come closest to the average scores of the judges collectively. 2

(B) Furthermore, that dollar value can be complicated. When the cost of gasoline is high enough, the extra initial cost of a more fuel-efficient vehicle ― or one that does not use gasoline at all ― can seem less expensive, even if one does not quite believe the car will "pay for itself" through its fuel savings.0

(C) However, if enough people were to buy those vehicles ― in sufficient numbers to reduce the demand for gasoline — the cost of gasoline would come down. Then again, so would the cost of the vehicles, thanks to economies of scale. 1


① A-C-B ② B-A-C ③ B-C-A ④ C-A-B ⑤ C-B-A

 


73. 20영독 7-56

 

The metaphor of barking dogs has been used by historians of religions to describe various uses of comparison, but like all good myths, it bears retelling in each new context and can always be used in new ways.


(A) Comparison makes it possible for us literally to cross-examine cultures, by using a myth from one culture to reveal to us what is not in a telling from another culture, to find out the things not "dreamt of in your philosophy"(as Hamlet said to Horatio). Moreover, we can use comparative work to test theories about our own culture, by noting where our own dogs have not barked. Comparison defamiliarizes what we take for granted. We can only see the inflection of a particular telling when we see other variants. 2

(B) The fact that the dog did not bark when someone entered the house at night was evidence, in this case evidence that the criminal was someone familiar to the dog. Dogs bark at difference ― in this case, someone different from those with whom the dog was familiar. We cannot hear the sound of one hand clapping; we cannot hear sameness. But through the comparative method we can see the blinkers that each culture constructs for its retellings of myths. 1

(C) Sherlock Holmes once solved a mystery, the case of Silver Blaze, a racehorse, by using a vital clue of omission. When Inspector Gregory asked Holmes whether he had noted any point to which he would draw the inspector's attention, Holmes replied, "To the curious incident of the dog in the night-time." "The dog did nothing in the night-time," objected the puzzled inspector, the essential straight man for the Socratic sage. "That was the curious incident," remarked Sherlock Holmes. 0


① A-C-B ② B-A-C ③ B-C-A ④ C-A-B ⑤ C-B-A

 


74. 20영독 7-7

 

Most employees want to do a good job, make a difference, and be valued for their efforts.


(A) The closer the recognition to the occurrence, the greater will be the motivation, creating a positive environment that fosters improved performance. Really listening to what employees are saying lets employees feel they are contributing, giving them a sense of worth. 2

(B) Recognizing accomplishments in a timely manner by a simple, sincere "Thank You" can go a long way in improving performance. The level of motivation in part will be determined by the time lapse between the occurrence and awarding the recognition. 1

(C) Regardless of the situation, the company must take time to determine what employees need. Employee performance is often tied more to personal factors like being appreciated for doing a good job than to money. 0


① A-C-B ② B-A-C ③ B-C-A ④ C-A-B ⑤ C-B-A

 


75. 20영독 7-8

 

Internationalization is a critical issue in higher education today.


(A) Students who take courses with international content are believed to be better equipped to effectively communicate in global contexts. Therefore, employers are looking for candidates who not only have appropriate degrees for the job but also foreign language skills and intercultural competence. 1

(B) Furthermore, many colleges and universities are admitting more and more international students, and this makes it necessary to train fellow students as well as faculty and staff about intercultural awareness and multiculturalism. 2

(C) Many colleges and universities are transforming their curriculum by integrating international perspectives and providing professional development to faculty and staff so that they can effectively participate in today's increasingly global environment. A major reason for internationalization in higher education is that college graduates are expected to become global citizens to be able to successfully communicate in a diverse workplace. 0


① A-C-B ② B-A-C ③ B-C-A ④ C-A-B ⑤ C-B-A

 


76. 20영독 7-9

 

Even people who do not live near the ocean can have an impact on marine communities and ecosystems.


(A) The decomposition of their bodies removes more oxygen from the water, making a bad situation even worse. The result of this excessive decomposition is an area of ocean water that is oxygen depleted. Because so little marine life can survive in such an area, it is referred to as a dead zone. 2

(B) Burning of fossil fuels, for instance, increases the amount of greenhouse gases in the atmosphere, leading to climate change such as global warming, rising sea levels, and increased acidity of the ocean. Another way in which inland populations can affect the marine environment is by contributing nutrients to the ocean. Nitrogen is a major nutrient that supports the growth of algae in aquatic ecosystems. 0

(C) When nitrogen-containing chemicals from terrestrial sources reach the ocean they support an enormous increase in the growth of algae. When the algae die, the decomposition of their remains robs the water of oxygen. Marine organisms that can swim away, such as fishes, migrate to better water while those that cannot, such as clams and worms, die from lack of oxygen. 1


① A-C-B ② B-A-C ③ B-C-A ④ C-A-B ⑤ C-B-A

 


77. 20영독 7-10

 

The use of critical thinking has been identified as particularly important in the digital age as relatively quick access to a wide range of information means that the user needs the ability to critically evaluate the validity and relative value of information accessed.


(A) In the past, the library, a book, or an expert (e.g., a teacher) were the student's source of knowledge, and the value or validity was unlikely to be questioned. When the Internet was originally introduced to school-based learning programmes, a number of educators were reluctant to use it as a teaching resource as the information may not contain correct facts. 0

(B) This type of critical thinking expands the scientific orientation of critical thinking using reasoning to evaluate credibility. 2

(C) This view reflected the limited information available at the time through the Internet, the lack of social media where large numbers of people and experts were developing knowledge, and a positivist orientation to schooling (purpose is to learn truths). The abundance of information at the touch of a digital technology means that learners need to be able to critically evaluate its relevance, validity and significance. 1


① A-C-B ② B-A-C ③ B-C-A ④ C-A-B ⑤ C-B-A

 


78. 20영독 7-1112

 

Predicting how inventions and technological innovations will be used and how they will ultimately affect society is often very difficult.


(A) Given enough experiences of this kind, one gets the idea that every new technology has not only known and expected benefits and costs but also unknown and unforeseen benefits and costs. New technologies sometimes even produce consequences exactly the opposite of what they were intended to produce, what the author Edward Tenner calls "revenge effects." Powerful new technologies alter the social context in which they arise; they change the structure of our interests and values; they change the ways in which we think and work, and they may even change the nature of the communities in which we live. 2

(B) Thomas Edison apparently believed that the phonograph would be mainly used for recording people's last wills and testaments and would undoubtedly be amazed by today's tapes, CDs, and MP3 players, all of which are descended from his invention for recording sound. And who, until recently, would have thought that chlorofluorocarbons, which have been used for decades as refrigerants, would be eating away the ozone layer in the upper atmosphere? 1

(C) The history of technology is full of stories of inventors and innovators who had no idea of how their inventions and innovations would ultimately be used or the far-reaching effects that they would have on society. Johannes Gutenberg, inventor of the printing press and movable metal type, was a devout Catholic who would have been horrified to know that his invention enabled the Bible to be widely printed and so helped stimulate the Protestant Reformation. 0


① A-C-B ② B-A-C ③ B-C-A ④ C-A-B ⑤ C-B-A

 


79. 20영독 8-1

 

From its very beginnings, tango showed its changeable profile: first it was simply music played on piano in houses of dubious reputation.


(A) In Buenos Aires, you can breathe tango at every comer, and there are countless tango shows, many including dinner, first-class orchestras, musicals and also 'milonga' dances where you are taught the ABCs of this passionate rhythm. 2

(B) Tango is such a wide-ranging rhythm that it can only be compared with jazz, insofar as its richness and ability to adapt to changing times are concerned. But because tango is danced and most forms of popular music are not, it invariably ended up in the concert halls instead of on the streets. 1

(C) Later it was joined by the guitar, the flute and the violin as it started to be accepted in the more prestigious ballrooms. The great change in tango was brought about by the arrival of the 'bandoneon' squeeze box from Germany, an instrument which was to become emblematic of tango and played by great musicians such as Astor Piazzolla. 0


① A-C-B ② B-A-C ③ B-C-A ④ C-A-B ⑤ C-B-A

 


80. 20영독 8-2

 

Land transportation systems have become a crucial component of modernity.


(A) By speeding up communications and the transport of goods and people, they have generated a revolution in contemporary economic and social relations. However, incorporating new technology has not come about without cost: environmental contamination, urban stress and deteriorating air quality are directly linked to modern land transport systems. 0

(B) Equally significant are the rising costs in health services and the added burden on public finances. 2

(C) Above all, transportation is increasingly associated with the rise in road accidents and premature deaths, as well as physical and psychological handicaps. Losses are not limited to reduced worker productivity and trauma affecting a victim's private life. 1


① A-C-B ② B-A-C ③ B-C-A ④ C-A-B ⑤ C-B-A

 


81. 20영독 8-3

 

The Westernized "developed" economies are, by a very large margin, the largest markets for prescription medicines.


(A) This situation is now changing because of economic, political and social factors, including the rise of "venture philanthropy" and new pricing models. Perhaps most significantly, rapidly growing economies (China, India and Brazil, for example) are sustaining a large number of people with Western lifestyles and the diseases to match. 1

(B) This may be one reason for an increased willingness on the part of multinational pharmaceutical companies to invest heavily in research and development in these countries and to offer generous pricing models for drugs that treat infectious diseases such as malaria. 2

(C) It is, therefore, inevitable that any coverage of the biopharmaceutical industry will assume that its research and development activities are directed almost exclusively at these affluent nations. The problem for millions of people in the developing world is that treatments for tropical diseases such as malaria are not economical to develop and that medicines for "Western" diseases are too expensive. 0


① A-C-B ② B-A-C ③ B-C-A ④ C-A-B ⑤ C-B-A

 


82. 20영독 8-4

 

The relevance of generalized knowledge that is applicable to particular individual phenomena is especially important in the applied areas of psychology — where the layperson's and scientist's perspectives cross paths.


(A) Successful application of the basic knowledge of psychology in particular concrete situations ― be those situations examples of individual or group psychotherapy, of consultation in a business firm, or of dealing with a troubled adolescent — can be consistent only if the basic scientific basis of these applications is adequate to the reality. 0

(B) Such occasional success, however, would be based on the particular combination of circumstances in the case of a concrete application, and need not follow from any adequate scientific understanding of the phenomenon. 2

(C) Certainly it is possible to achieve occasional practical success on the basis of inadequate scientific knowledge — as with the many people who believe in, and try to confirm, predictions made on the basis of horoscopes. 1


① A-C-B ② B-A-C ③ B-C-A ④ C-A-B ⑤ C-B-A

 


83. 20영독 8-56

 

One feature of progress and separation from nature is the growing incidence of physical inactivity.


(A) Maybe it is too late for some of us. We have forgotten, and will never reconnect enough. The real challenge is to get to today's young children, connect them with nature and its mysteries early, and prevent the extinction of ecological literacy that will dog us to our graves. 2

(B) Hunter-gatherers and farmers expend energy to catch and grow their food; the rest of us rely on cars and are gradually losing the ability to walk. Physical inactivity (and junk food) is killing us, and our kids. It also reduces the chance of accidental or designed connection with nature. We know that the natural environment positively affects our mental states. 0

(C) Is it any surprise to learn that mental ill-health is on the increase just as environments and biodiversity come under serious threat, just as we seem to stop going there? The World Health Organization predicts that depression and mental ill-health will be the greatest source of ill-health worldwide by 2020. Yet green places are good places ― from the small patches in cities to the wide open wildernesses, and there are many ways to engage in green exercise, from gardening to forest schools to country walks. All these are good for health, but for many adults are no more than temporary remedial measures, as we dash back to the rat race. 1


① A-C-B ② B-A-C ③ B-C-A ④ C-A-B ⑤ C-B-A

 


84. 20영독 8-7

 

At the heart of learning is good, old-fashioned trial and error.


(A) Sometimes we know what we want to achieve, and our experiments are refined as we approach the goal. And sometimes we are just playing with material (as artists do), ideas (scientists), or bodily movements (choreographers) to see what happens. 2

(B) In many areas of learning, including baseball, writing, and math, we tinker our way toward understanding and competence. Once we have the glimmerings of an idea about how to proceed, we give it a go, observe the effects and the success, adjust our action, and have another go. 0

(C) We can rarely figure everything out in advance so well that our first attempt is a surefire success. Watch an engineer sketching a bridge, an athlete adjusting her run-up, a teacher or an executive polishing their PowerPoint presentations, a child learning to dive, a cook tinkering with a recipe, and you will see the power of this kind of rehearsing, practicing, and drafting. 1


① A-C-B ② B-A-C ③ B-C-A ④ C-A-B ⑤ C-B-A

 


85. 20영독 8-8

 

Living butterflies hold their wings in positions that differ from those of mounted museum specimens.


(A) Thus, the shape of the hindwings, especially the length versus the width, may appear very different in the field than on museum specimens. In addition, relatively fresh individuals often have distinctive sheens that are useful for identification; these sheens are lost upon aging and after death, as are some markings on the butterfly's body and especially its eye color. 2

(B) For example, an obvious difference is that live grass-skippers spread their hindwings flat but open their forewings only partially, appearing very different from completely flat-spread museum specimens. 0

(C) A less often noted difference is that when landed with their wings closed, living skippers fold under the trailing edge of their hindwings, hiding about one-fifth of the wings. 1


① A-C-B ② B-A-C ③ B-C-A ④ C-A-B ⑤ C-B-A

 


86. 20영독 8-9

 

In terms of parenting, limited funds may restrict parents' ability, for example, to pay for the best private schools or to satisfy their children's demands for the latest gaming console.


(A) Some parents need to work long hours, cutting down the time they can spend with their children. In some instances, time constraints can be extreme: some parents migrate without their families in pursuit of work, enduring separation from their children for years. 1

(B) Yet constraints need not be exclusively of a financial nature. For many parents, the most significant constraints are time and capabilities. 0

(C) Limits to parents' knowledge and abilities are equally important. Some parents may have the time and resources to care for their children, but fail to provide them with an appropriate diet because they are unaware of the nutritional properties of different types of food. Others underestimate the importance of education as a means of getting on in society and do not put effort into motivating their children to do well in school. 2


① A-C-B ② B-A-C ③ B-C-A ④ C-A-B ⑤ C-B-A

 


87. 20영독 8-10

 

From today's perspective, it is difficult to imagine the depth of the Great Depression, and the desperation and deprivation it created among people from all walks of life and social conditions.


(A) Not only that: uncles and cousins who had gone to faraway places, such as Argentina or Australia, were in even worse conditions. There were no jobs, no relief, and nowhere to go. 2

(B) Complete industries disappeared, the ranks of the unemployed swelled to unthinkable levels, families lost their life savings and had no one to turn to. Homes and farms were repossessed by the thousands. 0

(C) Soup kitchens could not serve enough meals to those going hungry, banks collapsed in rapid succession, and children stopped going to school. Complete families thought about emigrating, only to find out that the Depression was a worldwide phenomenon and that relatives who had stayed behind in the old world were suffering as much as they were. 1


① A-C-B ② B-A-C ③ B-C-A ④ C-A-B ⑤ C-B-A

 


88. 20영독 8-1112

 

Intelligence is a 'relative' or normative construct.


(A) An intelligence test designed for 18-year-olds in 1930 would be expected to yield very different performance norms if administered today, yet an IQ score for 18-year-olds in 1930 on a then-current test has the same normative meaning as an IQ score for an 18-year-old today on a current test. The IQ score only tells us the individual's standing with respect to other members of the norming sample. 2

(B) The principal disadvantage to this approach is that it renders comparisons across norming groups somewhat problematic. For example, it is arguably nonsensical to say that a large sample of today's 18-year-olds is more or less 'intelligent' than a large sample of 18-year-olds in 1930. The average 18-year-old today has very different knowledge and skills from the 18-year-old in 1930, in areas of math, science, arts and literature, and so on. 1

(C) One of Alfred Binet's seminal contributions to the assessment of intelligence was to introduce the idea that we can best index intelligence, especially during childhood when rapid cognitive development occurs, as the individual's performance in comparison to a reference group (e.g., all six-year-old children). It is almost universally accepted that one can only quantify an individual's intelligence by referring to the reference or norming group. The principal advantage to this approach is that an individual's intelligence is indexed in a way that it has the same meaning, even though norming groups may change from one decade to the next (e.g., in terms of the core knowledge and skills that are within the capabilities of the larger reference group). 0


① A-C-B ② B-A-C ③ B-C-A ④ C-A-B ⑤ C-B-A

 


89. 20영독 9-1

 

Scientists hope to someday establish beyond a doubt that aging and all the nefarious things that go with it can be indefinitely postponed simply by reducing the amount of food and calories we consume.


(A) This means that a junk food junkie who is blessed with a high metabolic rate that keeps her from gaining weight may still be at a higher risk for developing a memory problem. If we consider the logic that explains how caloric restriction exerts its beneficial effects on the body and mind, this makes a lot of sense. 1

(B) Take note that in the prevention of Alzheimer's disease, maintaining an ideal weight may not be enough. Studies have shown that the risk of Alzheimer's disease is more closely linked to caloric intake than to weight or body mass index (BMI). 0

(C) The amount of age-accelerating oxygen free radicals generated from our diet is related to the amount of calories we consume, not to our weight. Thus a person with a high metabolic rate who consumes greater calories may actually be producing more harmful forms of oxygen than someone with a slower metabolic rate. 2


① A-C-B ② B-A-C ③ B-C-A ④ C-A-B ⑤ C-B-A

 


90. 20영독 9-2

 

Economies are organized in different ways to answer the question of what is to be produced.


(A) That same group decides on the number and size of school buildings, refrigerators, shoes, and so on. Other countries, including the United States, much of Europe, and increasingly, Asia and elsewhere have largely adopted a democratic and participatory decision-making process where literally millions of individual producers and consumers of goods and services determine what goods, and how many of them, will be produced. A country that uses such a decentralized decision-making process is often said to have a market economy. 2

(B) Sometimes this highly centralized economic system is referred to as a command economy. Under this type of regime, decisions about how many tractors or automobiles to produce are largely determined by a government official or committee associated with the central planning organization. 1

(C) The dispute over the best way to answer this question has inflamed passions for centuries. Should a central planning board make the decisions, as in North Korea and Cuba? 0


① A-C-B ② B-A-C ③ B-C-A ④ C-A-B ⑤ C-B-A

 


91. 20영독 9-3

 

Research with hμman runners challenged conventional wisdom and found that the ground-reaction forces (GRFs) at the foot and the shock transmitted up the leg and through the body after impact with the ground varied little as runners moved from extremely compliant to extremely hard running surfaces.


(A) As a result, impact forces passing through the legs are strikingly similar over a wide range of running surface types. Contrary to popular belief, running on concrete is not more damaging to the legs than running on soft sand. 2

(B) As a result, researchers gradually began to believe that runners are subconsciously able to adjust leg stiffness prior to foot strike based on their perceptions of the hardness or stiffness of the surface on which they are running. 0

(C) This view suggests that runners create soft legs that soak up impact forces when they are running on very hard surfaces and stiff legs when they are moving along on yielding terrain. 1


① A-C-B ② B-A-C ③ B-C-A ④ C-A-B ⑤ C-B-A

 


92. 20영독 9-4

 

The MNM philosophy ― Make New Mistakes ― recognizes that mistakes are opportunities to learn.


(A) Making no mistakes means that we continue to execute a familiar model or formula; making no mistakes means a lack of creativity and new strategies. Conversely, making new mistakes means that different activities and directions are being attempted. We should make mistakes when we try new things; if we don't do so, then we aren't being ambitious enough. 1

(B) The key is to learn from our new mistakes so that we grow as a result of them. If we want our teachers to learn from their experiences 一to actively engage in anticipating, hypothesis testing, reflecting, and analyzing ― they need to know that learning is messy and that it's all right to feel comfortable when they make a mistake. 2

(C) Of course, making the same old mistakes over and over isn't very smart. We should learn from our mistakes so that we don't repeat them. Yet making no mistakes isn't very smart either. 0


① A-C-B ② B-A-C ③ B-C-A ④ C-A-B ⑤ C-B-A

 


93. 20영독 9-57

 

One beautiful spring day, a farmer was plowing his ground to plant carrots, when a bear wandered by.


(A) But the next year, the farmer didn't plant carrots. Instead, he planted wheat. He figured since he was doing all the work, he should get the best end of the deal. When it was time to harvest the wheat, the bear showed up again. The farmer gave the bear all the roots and then loaded the wheat in his wagon. When the bear got home, he couldn't think of anything to do with the roots. He was furious! He went to the farmer's house and warned him, "You've shown how smart you are, Farmer. But if you're really smart, from now on you'll stay out of my woods!" To this day, the farmer is always a little nervous when he goes into the woods. And with good reason, for the bear still hasn't forgiven the farmer for tricking him. 2

(B) The bear was quite pleased with the huge bundle of carrot leaves he took home. But he was not pleased with their bitter taste. He returned to the farmer and demanded to taste one of the roots. The bear ate a carrot and said, "These are sweet and delicious. You've tricked me, Farmer. You had better not go in the woods again!" "I'm sorry, Bear. I didn't mean to trick you. Next year you can have all the roots, and I'll take what grows above the ground. It's only fair." 1

(C) The bear was just about to grab him, but the farmer begged, "Don't hurt me, Bear. Why don't we farm together? I'll do all the work for both of us. You can have everything that grows above the ground, and I'll take the roots." "That sounds fair," said the bear. "But you'd better not try to trick me, or you won't be safe in the woods anymore!" The carrots the farmer planted grew to be quite large. At last the day came to harvest them. When they were all dug up, the farmer said to the bear, "Now let's divide them evenly. Just as I promised, you get all the tops, and I get the roots." 0


① A-C-B ② B-A-C ③ B-C-A ④ C-A-B ⑤ C-B-A

 


94. 20영독 9-8

 

Socially anxious people usually feel friendly towards others and certainly have their fair share of the positive characteristics that other people appreciate.


(A) The anxiety interferes with their expression, and the ability to display them may have gone rusty from lack of use. Indeed, socially anxious people may have altogether lost belief in their likeable qualities together with their self-confidence. 1

(B) One of the rewards of learning to overcome social anxiety is that it enables you to express aspects of yourself that may previously have been stifled, and allows you to enjoy, rather than to fear, being yourself. 2

(C) They may have a sense of fun, be energetic and generous, kind and understanding, serious, amusing, quiet or lively, and they spontaneously behave in these ways when they feel at ease. But feeling at ease in company is so hard for them, and makes them so anxious, that these qualities are often hidden from view. 0


① A-C-B ② B-A-C ③ B-C-A ④ C-A-B ⑤ C-B-A

 


95. 20영독 9-9

 

Exotic pets pose a risk to human health and safety, particularly because some infectious diseases they carry are transmittable to humans.


(A) But the traded animals themselves arguably bear the risks of the exotic pet trade most profoundly. Pre-purchase mortality rates within the trade are as high as 70 percent for reptiles and some birds, or 80 percent for wild-caught marine fish, with similar mortality rates persisting within the first year after purchase. Experts argue it is difficult if not impossible to provide adequate care for exotic pets. 2

(B) Ecological risks are also significant. Species loss due to the exotic pet trade can be so dramatic that experts have coined the term "empty forest syndrome" to describe some of these exporting zones. 0

(C) In importing regions, too, exotic pets can escape or be illegally released into non-native environments, where they may become invasive. The most famous case of this is the breeding population of Burmese pythons now established in the Florida Everglades. 1


① A-C-B ② B-A-C ③ B-C-A ④ C-A-B ⑤ C-B-A

 


96. 20영독 9-10

 

What was arguably the all-time greatest example of selection bias resulted in the embarrassing 1948 Chicago Tribune headline "Dewey defeats Truman."


(A) In reality, Harry Truman trounced his opponent. All the major political polls at the time had predicted Thomas Dewey would be elected president. 0

(B) First, they stopped polling too far in advance of the election, and Truman was especially successful at energizing people in the final days before the election. Second, the telephone polls conducted tended to favor Dewey because in 1948, telephones were generally limited to wealthier households, and Dewey was mainly popular among elite voters. The selection bias that resulted in the infamous Chicago Tribune headline was accidental, but it shows the danger and potential power — for a stakeholder wanting to influence hearts and minds by encouraging others to hop on the bandwagon — of selection bias. 2

(C) The Chicago Tribune went to press before the election results were in, its editors confident that the polls would be correct. The statisticians were wrong for two reasons. 1


① A-C-B ② B-A-C ③ B-C-A ④ C-A-B ⑤ C-B-A

 


97. 20영독 9-11

 

Despite advances in desktop mapping software, which would empower reporters as mapmakers, news publishers tend to treat maps like photographs and other images ― as illustrations developed by specialists working in an art department outside the news room.


(A) The resulting maps are often team efforts involving reporters, editors, and graphics specialists. 2

(B) Some newspapers have a separate graphics department that is responsible for maps and other information graphics or a "graphics editor" who mediates between the news desk and the art department. Elsewhere the integration of illustrations and art relies on informal alliances — inviting the art director to the daily editorial meeting is a common concession. 0

(C) Because reporters and editors are rarely trained in graphic design and mapmaking, this division of labor is likely to persist. Even so, some newspapers committed to investigative reporting acquired a geographic information system and made the software available to reporters covering crime, elections, or the environment. 1


① A-C-B ② B-A-C ③ B-C-A ④ C-A-B ⑤ C-B-A

 


98. 20영독 9-1214

 

At the 2008 Washington State Class 4A Track and Field Championship, Nicole Cochran had just finished the 3,200-meter race.


(A) Then the third-place finisher gave her medal to the second-place runner, and so on down the line. Finally, a girl named Lyndy Davis from Monroe High School gave her eighth-place medal away. That meant she wouldn't be receiving one at all. "It gave me chills," said Cochran. "It shows how much respect distance runners have for each other." Cochran competed in two more events, including the 800-meter race. She finished in eighth place. Afterward, she found Lyndy Davis and gave her the eighth-place medal. "After what she had done, I didn't want her to go home from the meet in her senior year without a medal," said Cochran. Then, 10 days after the competition, officials decided the original ruling had been wrong. Cochran was formally named the 3,200-meter champ. 2

(B) The defending champion thought she had won. After all, Cochran ― a senior at Bellarmine Prep ― seemed to have finished first by more than three seconds. But then her coach was called to the officials' tent. The officials said that Cochran had stepped outside of her lane during one of the turns. Cochran knew she hadn't done it, but the ruling was final. She was going to be disqualified. Almost everybody, including the other runners, believed the judge had made a mistake. Still, the title was awarded to the runner-up, Andrea Nelson from Shadle Park High. Nelson wasn't happy about it. In fact, she was upset. She had been running in the lane next to Cochran's, and she knew Cochran had run a clean race. 0

(C) "That's not how I wanted to win the state championship," said Nelson. "It wasn't fair. She deserved it. She totally crushed everybody." So as the eight top finishers took their places on the podium to receive their medals, Nelson made a decision. She stepped off the podium, walked over to Cochran, and placed the first-place medal around her neck. "It's your medal," she said. Cochran was moved by the gesture. She was pretty astonished. The other runners were inspired, too. The second-place finisher gave her medal to Nelson. 1


① A-C-B ② B-A-C ③ B-C-A ④ C-A-B ⑤ C-B-A

 


99. 20영독 10-1

 

One of the great risks of writing is that even the simplest of choices regarding wording or punctuation can sometimes prejudice your audience against you in ways that may seem unfair.


(A) Suppose you have written a position paper trying to convince your city council of the need to hire security personnel for the library, and half of the council members 一 the people you wish to convince — remember their eighth-grade grammar teacher's warning about splitting infinitives. How will they respond when you tell them, in your introduction, that librarians are compelled "to always accompany" visitors to the rare book room because of the threat of vandalism? 1

(B) How much of their attention have you suddenly lost because of their automatic recollection of what is now a nonrule? It is possible, in other words, to write correctly and still offend your readers' notions of your language competence. 2

(C) For example, look again at the old grammar rule forbidding the splitting of infinitives. After decades of telling students to never split an infinitive (something just done in this sentence), most composition experts now concede that a split infinitive is not a grammar crime. 0


① A-C-B ② B-A-C ③ B-C-A ④ C-A-B ⑤ C-B-A

 


100. 20영독 10-2

 

While we dislike failing in our regular endeavors, games are an entirely different thing, a safe space in which failure is okay, neither painful nor the least unpleasant.


(A) To prevent other people from achieving their goals is usually hostile behavior that may end friendships, but we regularly prevent other players from achieving their goals when playing friendly games. Games, in this view, are something different from the regular world, a frame in which failure is not the least distressing. 1

(B) The phrase "It's just a game" suggests that this would be the case. And we do often take what happens in a game to have a different meaning from what is outside a game. 0

(C) Yet this is clearly not the whole truth: we are often upset when we fail, we put in considerable effort to avoid failure while playing a game, and we will even show anger toward those who foiled our clever in-game plans. In other words, we often argue that in-game failure is something harmless and neutral, but we repeatedly fail to act accordingly. 2


① A-C-B ② B-A-C ③ B-C-A ④ C-A-B ⑤ C-B-A

 


101. 20영독 10-3

 

Emotions can easily intrude upon the most simple messages.


(A) Mine or yours? Most likely you'd feel like socking the person who sent it. There are certainly people who can push us over the edge of civilized decorum. The question is how to respond to them. 1

(B) In this case, perhaps it's best not to respond at all. The writer is clearly upset and resentful, perhaps even insecure about something you may have said or suggested. If you receive an unsettling message such as this, do not respond immediately. No matter how justified or outraged you feel, your emotions will get the better of your ability to express your thoughts and, ironically enough, you may end up appearing the aggressor. 2

(C) Some people can send us letters and e-mail messages that are clearly hostile or nasty and tempt us to respond in kind. At times maybe we should. How would you feel if you received this message? Whose job do you think you can do better? 0


① A-C-B ② B-A-C ③ B-C-A ④ C-A-B ⑤ C-B-A

 


102. 20영독 10-4

 

Within my family, obtaining a university degree was never presented as a choice.


(A) I was exceptionally lucky because my parents always cultivated in my sister and me deep admiration for academic and professional achievement. As I grew up, my parents would repeat again and again that education was an investment that would always yield returns. They convinced us that knowledge was the one thing in life nobody could take away from you. 0

(B) In these circumstances, the promise of a better life depended on my parents' ability to exercise their professions in the United States and on the education my sister and I could obtain. There seemed to be no American Dream without a college degree. 2

(C) Money, properties, even loved ones could disappear. But not knowledge. This thinking had acquired greater meaning in our new context as recent immigrants facing significant scarcity. 1


① A-C-B ② B-A-C ③ B-C-A ④ C-A-B ⑤ C-B-A

 


103. 20영독 10-57

 

While she was going to Elanor Hales's place, Anika kept thinking about the baby elephant.


(A) Can I see him?" Mrs. Hales laughed and said, "Yes, he is. He's still weak, but he has a good chance of surviving." A sense of comfort filled Anika. "I've named him Kioko. I will show you around," Mrs. Hales said. Mrs. Hales took Anika to where they were taking care of elephants. There were two other small elephants. All of them were having a mud bath out front. Kioko was there. An animal caretaker was rubbing cool, muddy water behind Kioko's ear. Kioko leaned against her and touched her with his trunk. Anika wanted to go pet Kioko, but Mrs. Hales said no. She said Kioko needed to feel peacefully secure. It wouldn't be good for strangers to pet him yet. 1

(B) 'Would he still be alive?' she thought. As soon as the car stopped at Elanor Hales's place, Anika burst out of the car. She was in a hurry to see if the baby elephant was still alive. Then she saw an older woman who was standing with her arm around an eland, a large African antelope. She was talking to some people and patting the eland. She looked over at Anika and then walked over. She was barefoot. The eland followed her. "You must be Anika," she said. "I'm Elanor Hales." Her voice was very English, clipped, and no-nonsense. She had kind eyes. Anika blurted, "Is he still alive? Is the baby elephant still alive? 0

(C) Mrs. Hales explained that baby elephants die unless they feel safe and get lots of attention. The tiny ones used to die, even with lots of attention. After having tea in the veranda, it was time for Anika to leave. She looked at Elanor Hales and said "Mrs. Hales, is there any chance I could work with you?" Mrs. Hales raised her eyebrows. "What do you mean by work?" she asked. "Anything there is to do. I could learn immense amounts from you," Anika said. "Well," Mrs. Hales said in a dry voice, "what good would you be to me? I have workers already who know the animals." Anika shrugged and raised her hands. Mrs. Hales laughed, "Write to me. I'll consider it." She shook Anika's hand and said goodbye. 2


① A-C-B ② B-A-C ③ B-C-A ④ C-A-B ⑤ C-B-A

 


104. 20영독 10-8

 

Avoid the myth that writing is easier at the last minute.


(A) It's a popular, but dangerous myth. Last-minute deadlines are more likely to create stress that can paralyze your thinking and ability to write. You may feel "energized" by the stress, but the stress also undermines your ability to make logical connections and correct choices while writing. Inevitably, last-minute writing results in embarrassing mistakes, omissions, and a lack of clarity. 0

(B) Always read what you've written out loud. Reading out loud will reveal errors and omissions that you didn't notice the previous day. Reading out loud helps you locate run-on sentences, awkward phrases, and unnecessary ideas. 2

(C) Finish a day ahead of time, and review your work the next day. Never post, publish, or submit a project immediately after you finish writing. Instead, put it aside for an hour, or — even better — overnight. Then, carefully review what you've written. 1


① A-C-B ② B-A-C ③ B-C-A ④ C-A-B ⑤ C-B-A

 


105. 20영독 10-9

 

Composers compose music.


(A) They write down a series of dots and lines on a page; then performers come along with their instruments and voices, look at the dots and lines on the page, and make sounds from them. It's all very mysterious. Or is it? 0

(B) There's no musical sound meaning 'sausage' or 'dirty laundry', for instance. On the other hand, a musical sentence, or phrase, can sound happy, sad, thoughtful, nostalgic and eager — all at the same time! Words would get exhausted if they tried to express as many meanings as that. 2

(C) After all, these words you're reading are just another series of dots and lines; you know what they mean, so you can look at them and make sounds (and sense) from them. So maybe music is really just another language, with its own meaning; but there IS something more magical about music than about any other language. The range of sounds is far, far huger than that of any spoken language; and because they aren't tied to any specific meaning, the sounds can express much more. 1


① A-C-B ② B-A-C ③ B-C-A ④ C-A-B ⑤ C-B-A

 


106. 20영독 10-10

 

In judging that a particular explanation is the best one, you need to compare it with other possible explanations; and the more alternatives you are able to imagine, the better your judgement is likely to be.


(A) Similarly, when you make a moral judgement about someone's behaviour, not only do you need to look at what they actually did, but you also need to imagine what they could have done. 1

(B) In science, a chemist working with the same data as their colleagues may reject an 'obvious' explanation of the phenomena because they have the intellectual ability to imagine a range of different explanations and the judgement to be able to choose between them. 0

(C) If someone does something bad, your judgement is likely to be harsher if you think there were better choices available to them, and more lenient if you think they really had no choice. 2


① A-C-B ② B-A-C ③ B-C-A ④ C-A-B ⑤ C-B-A

 


107. 20영독 10-11

 

Solving the productivity problem is a double-edged sword.


(A) As soon as the business sector raises productivity and salaries start to rise in absolute terms, wage disparities between poorly trained and highly trained workers are likely to become more pronounced. While the rising tide of higher salaries will lift more people above the poverty line, the income differences among different sectors of society are likely to grow. 0

(B) In an economy dominated by innovation and mass customization, the highly skilled and the highly trained are likely to prosper. 2

(C) As companies strive to become more productive as well as more innovative in differentiating their products from their competitors, they will increasingly either spin off low-paying, low-value jobs to Third World countries or eliminate them altogether through automation. The remaining high-value, high-paying jobs are thus likely to require an increasingly well-educated labor force. 1


① A-C-B ② B-A-C ③ B-C-A ④ C-A-B ⑤ C-B-A

 


108. 20영독 10-1214

 

Once upon a time there was a woman named June.


(A) He thought the best way to teach me self-reliance was to never encourage or praise me. He wanted me to be tough and independent." "Two or three times a week, we played catch. Sometimes we would play catch with a baseball, at other times with a football. Either way, the goal was always the same. I was to catch the ball ten times straight. I would catch that ball eight or nine times, but always on the tenth, he would do anything to make me miss. He would throw it on the ground or over my head, but always so I had little chance of catching it." Michael paused for a long moment and then finished, "He never let me catch the tenth ball ― never! No matter how hard I tried, he always set me up to fail. And I guess that's why I have to get away from my father's business; I want to catch the tenth ball." 2

(B) He had witnessed this frustration in other parents. Knowing June's disappointment was genuine, he agreed to talk with Michael. Michael arrived early for their appointment. Rather than being reluctant to talk, Michael jumped right into the reasons for his decision. Michael explained, "There was a time when I would have loved nothing more than to run my father's business. As a boy, I idealized my dad. I wanted to please him. I wanted to hear him say he was proud of me. But you need to understand the relationship. My father was a driven man who came up the hard way. He was determined to teach me self-reliance, but his method was demoralizing. 1

(C) June was the widow of a successful entrepreneur. Over a period of twenty-six years her late husband, Walter, had built a family-owned corner drugstore into a chain of fifty-eight stores with annual sales in excess of 326 million dollars. June and Walter were the parents of a single child, Michael. As Michael grew toward adulthood, his mother assumed that Michael would follow in his father's footsteps. As president and CEO, Michael would fulfill his father's vision of one hundred stores with annual sales of over four hundred million dollars. This, June believed, was the only course her son's life could take. June was to be disappointed. When Michael completed undergraduate school, he announced he would not be entering the family business. Upset and worried, June sought the advice of an old family friend. The old friend, who happened to be a retired high school principal, listened patiently as June wandered through various stages of grief — denial, anger, depression, and back to anger. June's pain was not new to the former principal. 0


① A-C-B ② B-A-C ③ B-C-A ④ C-A-B ⑤ C-B-A

 


109. 20영독 11-1

 

To many people, having a goal is synonymous with commitment, and commitment to a goal — in turn — is nearly synonymous with success.


(A) Quitting, on the other hand, is reserved for the morally and physically weak. As you might guess, we challenge the notion that giving up (an indisputable psychological discomfort, by the way) is so awful. Blind devotion to goals has led to, among other things, "gold fever," most often associated with the California Gold Rush, when miners expended enormous physical, emotional, and financial capital in their fruitless pursuit of riches. 1

(B) Legendary boxer Muhammad Ali once remarked, "I hated every minute of training but I said, 'Don't quit. Suffer now and live the rest of your life as a champion."' And there you have it ― the clear sentiment that doubling down on goals is more likely to lead to success. 0

(C) In fact, researcher Eva Pomerantz of the University of Illinois argues that heavy investment in a goal can erode a person's psychological quality of life by creating a spike in their anxiety. This is especially true when people push themselves by focusing on the potential negative impact of not achieving their goals. 2


① A-C-B ② B-A-C ③ B-C-A ④ C-A-B ⑤ C-B-A

 


110. 20영독 11-2

 

Our insatiable appetite for seafood, coupled with the brutal efficiency of our industrial fishing technologies, has wreaked havoc.


(A) But above all, it has been a combination of government weakness, industrial greed and a scientific community lacking the courage to sound the alarm that has resulted in one of the greatest ecological tragedies of our time. Decision-makers have routinely ignored the warning signs. 0

(B) For example, in November 2008, the inappropriately named International Commission for the Conservation of Atlantic Tuna (ICCAT) set a catch quota for bluefin tuna that is nearly 50 per cent higher than its own scientists advise. Citing concern for jobs, livelihoods and consumer interest, politicians have brought fish stocks to the brink of collapse, and by their failure, they threaten the very people in whose interests they claim to be acting. 2

(C) The reason is that they have been frightened of upsetting the 'fishing lobby'. As a result, they have set hopelessly unrealistic quotas, and have gone out of their way to appease industrial fishing companies. 1


① A-C-B ② B-A-C ③ B-C-A ④ C-A-B ⑤ C-B-A

 


111. 20영독 11-3

 

We live in times when speed of reaction often takes precedence over slower and more cautious assessments.


(A) As we become more attuned to 'real time' events and media, we inevitably end up placing more trust in sensation and emotion than in evidence. Knowledge becomes more valued for its speed and impact than for its cold objectivity, and emotive falsehood often travels faster than fact. 0

(B) News, financial markets, friendships and work engage us in a constant flow of information, making it harder to stand back and construct a more reliable portrait of any of them. The threat lurking in this is that otherwise peaceful situations can come to feel dangerous, until eventually they really are. 2

(C) In situations of physical danger, where time is of the essence, rapid reaction makes sense. But the influence of 'real time' data now extends well beyond matters of security. 1


① A-C-B ② B-A-C ③ B-C-A ④ C-A-B ⑤ C-B-A

 


112. 20영독 11-4

 

Quite often, a party seeking to show statistical significance combines data from different sources to create larger numbers, and hence greater significance for a given disparity.


(A) Conversely, a party seeking to avoid finding significance disaggregates data insofar as possible. In a discrimination suit brought by female faculty members of a medical school, plaintiffs aggregated faculty data over several years, while the school based its statistics on separate departments and separate years. 0

(B) When the figures were broken down by department, however, it appeared that in most departments the women's acceptance rate was higher than the men's. The reason for the reversal was that women applied in greater numbers to departments with lower acceptance rates than to the departments to which men predominantly applied. The departments were therefore variables that confounded the association between sex and admission. 2

(C) The argument for disaggregation is that pooled data may be quite misleading. A well-known study showed that at the University of California at Berkeley female applicants for graduate admissions were accepted at a lower rate than male applicants. 1


① A-C-B ② B-A-C ③ B-C-A ④ C-A-B ⑤ C-B-A

 


113. 20영독 11-57

 

Professor Povzner taught a course at the Military Academy for Engineers.


(A) The student said, 'This is so interesting, about medieval Russian mathematics. Could you tell us, please, where we could get more information about it — what the reference books would be? I would like to learn more.' Having no time to think, the professor immediately answered: 'Well, that's impossible! All the archives were burned during the Tatar invasion!' When the class was over, the general got up slowly from his seat. 1

(B) He walked into a class one day, ready to start his lecture with a routine spiel about Russian primacy in mathematics, and then settle down to a serious session of really teaching mathematics. But to his alarm, the minute he got up in front of the class he saw that among the audience was a general, the chief of the Academy. Povzner pulled up short and decided that he had better devote the whole lecture to the subject of early Russian genius in mathematics. Luckily, he was a very talented man, good at thinking on his feet, so on the spur of the moment he invented a wonderful lecture on Russian mathematics in the twelfth century. He engaged in flights of fancy for the entire hour, stopping only five minutes before the end to ask, as was customary, 'Are there any questions?' Povzner saw that one of the students had raised his hand. 0

(C) Then he came up to the lecturer and said, 'So, Professor... All the archives were burned?' Only then did poor Povzner realize what he had said. The unspoken question hung in the air: If all the evidence of Russian primacy in this science was burned, how in the world did the professor himself know the history of pre-invasion mathematics? He was ready to panic when, unexpectedly, the general smiled at him sympathetically, turned around, and left. This high-ranking commander was a clever ancent person; otherwise Professor Povzner would have been in deep trouble. 2


① A-C-B ② B-A-C ③ B-C-A ④ C-A-B ⑤ C-B-A

 


114. 20영독 11-8

 

When romantic partners lie to each other they do so relatively often by concealing information.


(A) There are several reasons as to why liars prefer concealments. First of all, they are difficult to detect. Once information is provided, lie detectors can verify the accuracy of this information by searching for further evidence that supports or contradicts it. 0

(B) Another problem with telling an outright lie or exaggerating is that liars need to remember the details they provided in case the topic of the lie comes up on subsequent occasions. However, they don't need to remember anything if they don't provide information (concealment). 2

(C) In the case of concealments, however, no information is given. Moreover, concealing information is relatively easy. When telling an outright lie or when exaggerating, a liar should invent a story that sounds plausible, whereas nothing needs to be invented when concealing information. 1


① A-C-B ② B-A-C ③ B-C-A ④ C-A-B ⑤ C-B-A

 


115. 20영독 11-9

 

Control of the crime scene is obviously important.


(A) The recording of the names of those who enter and leave the defined scene (or come into contact with it) and at what time maintains the integrity of the scene management process. The scene or investigation also extends to persons removed from the scene and those who may be potentially connected with it. The availability of trained crime scene investigators to examine such scenes may not always be adequate. 1

(B) This can be done by establishing the boundary of the scene (be it a location, item, or person) and protecting it. The establishment of a cordon at a major crime scene, marking it with incident tape and protecting it with police officers, is the common practice. The same principles apply if the examination is that of a person (a suspect, victim, or witness), recovered vehicle, or any other item. 0

(C) It is a long-established principle that all areas such as scene, victim, vehicle, and suspect should be dealt with by separate scene investigators. But at some stage, most often in the laboratory, items will be examined and compared by the same scientist, where systems are also required to ensure that there is no contamination. 2


① A-C-B ② B-A-C ③ B-C-A ④ C-A-B ⑤ C-B-A

 


116. 20영독 11-10

 

In a study, 77 undergraduate students were interviewed.


(A) During these interviews, they were presented with various events (e.g., falling on their head, getting a painful wound, or being sent to a hospital emergency room). They were told that, according to their parents, these events had occurred in their childhood. 0

(B) Guided imagery instructions were given to the participants to help them generate images for the false event (e.g., "Visualize what it might have been like and the memory will probably come back to you"). Results indicated that 26% of students "recovered" a complete memory for the false event, and another 30% recalled aspects of the false experience. 2

(C) The interviewer gave further details about the events supposedly given by the parents. Unknown to the interviewees, the events were invented by the researchers and had never happened to the participants according to their parents. 1


① A-C-B ② B-A-C ③ B-C-A ④ C-A-B ⑤ C-B-A

 


117. 20영독 11-11

 

Noise is often thought to affect performance, as anyone who has tried to concentrate in a noisy environment can attest.


(A) Unfortunately, most of the evidence on task performance under noisy conditions comes from laboratory experiments. Based on the research conducted so far, we cannot assess whether the findings of lab research on noise will generalize to less controlled conditions in real work settings. 2

(B) Performance deficits are particularly obvious for difficult or demanding tasks. Simple or routine tasks, on the other hand, are typically not affected by noise, and sometimes noise increases performance on simple tasks. This enhancement effect probably occurs because the noise acts as a stressor, raising the person's arousal level and therefore overcoming the boredom associated with the task. 1

(C) High intensity noise clearly is related to a generalized stress response. However, the effects of noise on performance are far from clear-cut. In general, task performance is only impaired at very high noise intensities. 0


① A-C-B ② B-A-C ③ B-C-A ④ C-A-B ⑤ C-B-A

 


118. 20영독 11-1214

 

I recently had the privilege of listening to Robert Cooper, author of Executive EQ, address an auditorium of 900 people.


(A) He described the decline in his grandfather's health and how after each major heart attack his grandfather would call Robert to his side, burning to share his latest near-death insight. Robert had us leaning forward in our seats, as he recounted his grandfather's words "I've been thinking about what is most important in life, and I've concluded that the most important thing in life is...." We wanted to share the insights of this great man. By the fourth time he had us laughing at the old man's revisions and Robert's adolescent fear that he was going to be tested on remembering what the last heart attack's "most important thing in life" was. As we continued to smile, he told us about his grandfather's last revision: "My grandfather said to me, 'Give the world the best you have and the best will come back to you. I have asked myself — what if every day I had refused to accept yesterday's definition of my best? So much would have come back to me ... to your father ... to you. But now it won't, because I didn't. 1

(B) The story he told in the first ten minutes of his speech demonstrated his authenticity. He chose to tell us "who he was" by telling a story about his grandfather, who died when Robert was sixteen years old. His father's father had four major heart attacks before he eventually died from the fifth. During that time, he had taken great care to assist in Robert's development as a young man. He invested long talks and personal time with him. We could see the love Robert felt for his grandfather when he used words to help us see this man as he saw him back then. He said, "If you could measure intelligence in the quality of intensity in a man's eyes, he surely must have been a genius." 0

(C) It is too late for me. But it's not too late for you.'" I held my breath along with everyone there at the power of a man's regret at the end of his life. "It is too late for me." Our common humanity means that we, too, will die. Every person in that audience had a flicker of awareness toward our own deaths and potential regrets. He didn't pull any punches with this story, but Robert glowed with the intensity of total authenticity and his integrity gave him the right to tell such a powerful story. 2


① A-C-B ② B-A-C ③ B-C-A ④ C-A-B ⑤ C-B-A

 


119. 20영독 12-1

 

Clutter beats us up psychologically and physically.


(A) It clouds our minds, making it difficult to access what truly matters in our lives. According to Ab Jackson, a fellow organizer, "Are you a person that buys things that you don't need, with money that you don't have, to impress people that you don't even like?" Sound familiar? 0

(B) Why spend it performing tasks that don't match your prewired tendencies? How can we learn to do this with less stress and more grace; even, dare I say, a sense of satisfaction and pleasure? Here is where your own organizing skills can greatly assist you in feeling intact, leading a life that is proactive rather than reactive, purposeful, and ultimately satisfying. 2

(C) How reassuring would it be to get organized in a way that not only helps you cope with the basic demands of life but also gives you more energy and joy? How about a way that actually resonates with what is easy for your unique brain? We only have so much life force, so much chi, pran, or energy.1


① A-C-B ② B-A-C ③ B-C-A ④ C-A-B ⑤ C-B-A

 


120. 20영독 12-2

 

TV shows were more popular in the seventies than they are now not because they were better, but because we had fewer alternatives to compete for our screen attention.


(A) The economics of the broadcast era required hit shows — big buckets — to catch huge audiences. The economics of the broadband era are reversed. Serving the same stream to millions of people at the same time is hugely expensive and wasteful for a distribution network optimized for point-to-point communications. 2

(B) But it can't do the opposite — bring a million shows to one person each. Yet that is exactly what the Internet does so well. 1

(C) What we thought was the rising tide of common culture actually turned out to be less about the triumph of Hollywood talent and more to do with the sheepherding effect of broadcast distribution. The great thing about broadcast is that it can bring one show to millions of people with unmatchable efficiency. 0


① A-C-B ② B-A-C ③ B-C-A ④ C-A-B ⑤ C-B-A

 


121. 20영독 12-3

 

In one study Barbara Weston and I attempted to determine whether or not 2- and 3-yearold babies would show evidence of familiarity to the perfume the mother wore during regular feedings.


(A) There were two control groups: a test with an odourless cotton swab, and a test with another perfume, 'Cachet', which was not worn by any of the mothers in the experimental group. The results for 15 infants, 8 girls and 7 boys, showed 80 per cent response on trials with the perfume versus virtually zero with the odourless control. However, the infants responded about equally to mothers' perfume and the control perfume, 'Cachet'. 2

(B) After a few such exposures, we tested the babies in the laboratory. The perfume was presented to the infant on a cotton swab and the babies' sucking, respiration, general activity and heart rate were recorded on a polygraph. 1

(C) She would wear her own perfume and the same one each time. The perfumes included 'L'Air du Temps', 'Jontu' and 'Maximi'. 0


① A-C-B ② B-A-C ③ B-C-A ④ C-A-B ⑤ C-B-A

 


122. 20영독 12-4

 

In addition to efforts to develop natural and more sustainable adhesive materials for commercial use, recent years have seen the emergence of a wide variety of "green adhesives."


(A) Low-temperature hot melt glues, for example, require less energy to melt and apply, and new cardboard and foil-based packaging has also been developed to reduce the landfill waste from plastic tube applicators. 2

(B) Efforts to produce and market these have been associated not only with the use of more sustainable raw materials but also with minimizing the environmental impacts of adhesives, particularly with regard to reducing harmful compounds and solvents contained in the adhesives. A number of governments have introduced regulations on the chemical emissions produced when using adhesives. 0

(C) These regulations have attempted to place limits on the amount of volatile organic compounds contained in adhesive products, as these compounds are thought to release hazardous air pollutants posing both health and environmental risks. Other products are designed to save energy and reduce waste. 1


① A-C-B ② B-A-C ③ B-C-A ④ C-A-B ⑤ C-B-A

 


123. 20영독 12-57

 

Joe has one specific memory of when he was 17 years old.


(A) He had been working summers and vacations for nearly five years at his father's appliance store. Joe was out on a service call one afternoon to fix a customer's washing machine. Some members of the high school yearbook staff came into the store to get a picture of him for a story they were doing on how the senior class spent summer vacation. His dad directed them to the customer's home (it's a small-town thing). So, much to Joe's surprise, two of the prettiest girls in the senior class showed up to take his picture. At the time Joe was so self-conscious. At 17, the presence of pretty girls made him incredibly nervous. Also, he looked dirty and completely unprepared for this unannounced visit. Making it all the worse was the utilitarian way the two classmates handled the task. 0

(B) Joe remember thinking, "Really! 'Cheer up, it gets worse?' That has got to be the worst motivational speech I have ever heard in my life." Years later, he reminded his dad, now softened with age, of that conversation. He offered more nuance this time. First, he said, the phrase is true. Life is hard, but no matter how difficult today is, there will be a worse one ahead. "Cheer up, it gets worse," is not negative, he said. It is a reminder not to wallow in your troubles today because there will be a day in the future you will want to trade for today. Truly, an amazing piece of advice. 2

(C) They briefly announced the purpose of their visit, snapped two photos, and left with no more than three sentences of interaction. The manner in which they showed up, executed their chore, and exited all in the span of five minutes with not even a pleasantry was bruising to a teenage boy's ego. After completing the service call, Joe returned to the store. It was clear, even to his emotionally restricted father, that he was bummed. "What happened to you?" His father asked him. Reluctantly, Joe shared his encounter with the two girls. After listening to his story, he said, "Cheer up, it gets worse." And with that, Dad returned to the task at hand. 1


① A-C-B ② B-A-C ③ B-C-A ④ C-A-B ⑤ C-B-A

 


124. 20영독 12-8

 

Imagine I tell you that Maddy is bad.


(A) But there is more detail nevertheless, perhaps a stronger connotation of the sort of person Maddy is. In addition, and again assuming typical linguistic conventions, you should also get a sense that I am disapproving of Maddy, or saying that you should disapprove of her, or similar, assuming that we are still discussing her moral character. 2

(B) In contrast, if I say that Maddy is wicked, then you get more of a sense of her typical actions and attitudes to others. The word 'wicked' is more specific than 'bad'. I have still not exactly pinpointed Maddy's character since wickedness takes many forms. 1

(C) Perhaps you infer from my intonation, or the context in which we are talking, that I mean morally bad. Additionally, you will probably infer that I am disapproving of Maddy, or saying that I think you should disapprove of her, or similar, given typical linguistic conventions and assuming I am sincere. However, you might not get a more detailed sense of the particular sorts of way in which Maddy is bad, her typical character traits, and the like, since people can be bad in many ways. 0


① A-C-B ② B-A-C ③ B-C-A ④ C-A-B ⑤ C-B-A

 


125. 20영독 12-9

 

When electromagnetic wavelengths start to get too long to bend retinal (a biological chemical in the retina of the eye), we call them infrared because they are just below the visible color red.


(A) Those wavelengths shorter than visible light carry more energy. The shorter wavelength radiations (think x-rays) carry enough energy that they go beyond simply bending molecules; they can actually break them. And it is the breaking of biological molecules that results in radiation's adverse biological effects. 1

(B) In contrast, those radiations with longer wavelengths carry much less energy than light (think radio waves), not even enough to bend retinal or other biological molecules. If these low energy radiations have biological effects, their mechanism is more obscure and beyond what we currently understand about biology. 2

(C) When the wavelengths are a little too short to be seen we call them ultraviolet because they are just beyond the visible color violet. Visible light is squeezed in between the invisible infrared and ultraviolet wavelengths. If we consider visible light to be the dividing line within the universe of invisible electromagnetic waves, what can we say about wavelengths on either side of the visible? 0


① A-C-B ② B-A-C ③ B-C-A ④ C-A-B ⑤ C-B-A

 


126. 20영독 12-10

 

Pain has always been philosophically and politically problematic.


(A) Whereas physical injuries and diseases can be observed by others, pain has an apparently private quality, which potentially cuts the sufferer off from others. By its nature, it can be difficult to adequately communicate, a quality that led the cultural theorist Elaine Scarry to describe intense pain as "world-destroying." 0

(B) This generates its own political strains, as some sufferers are inevitably viewed as more credible than others, while some are assumed to exaggerate their pains. The politics of pain involves differing views of who deserves compassion and how much, a matter that generates its own distinctive political positions. For example, American conservatives have historically taken the harsher view that those in pain are less deserving of sympathy or pain relief. 2

(C) The sufferer feels alone with their pain, and depends on the capacity of others to empathize and to believe them. As Scarry puts it, "To have pain is to have certainty; to hear about pain is to have doubt." 1


① A-C-B ② B-A-C ③ B-C-A ④ C-A-B ⑤ C-B-A

 


127. 20영독 12-11

 

In the classic model of the Sumerian economy, the temple functioned as an administrative authority governing commodity production, collection, and redistribution.


(A) For that matter, it is not clear how widespread literacy was at its beginnings. The use of identifiable symbols and pictograms on the early tablets is consistent with administrators needing a lexicon that was mutually intelligible by literate and nonliterate parties. 1

(B) As cuneiform script became more abstract, literacy must have become increasingly important to ensure one understood what he or she had agreed to. 2

(C) The discovery of administrative tablets from the temple complexes at Uruk suggests that token use and consequently writing evolved as a tool of centralized economic governance. Given the lack of archaeological evidence from Uruk-period domestic sites, it is not clear whether individuals also used the system for personal agreements. 0


① A-C-B ② B-A-C ③ B-C-A ④ C-A-B ⑤ C-B-A

 


128. 20영독 12-1214

 

Although Joyce had little patience with elderly people, she applied for the activity director position in a nursing facility.


(A) She was a broad-shouldered woman with large hands and feet, in a near prone position. She spent her days in a blue chair. Her hair, sparse and iron-gray, had twin cowlicks that caused it to stick out in all directions. Worse yet, Miss Lilly never spoke. Joyce had seen her one relative, a niece, several times. Each visit was the same. Standing a few feet in front of the blue chair, her niece would say, "Your check came, your bill is paid." Never a personal word, a hug, or any sign of affection. Months passed, and Miss Lilly seemed to shrink lower and lower down in her chair. Joyce discovered she was not eating well and gave up her lunch hour to feed Miss Lilly. Seeing how much Miss Lilly enjoyed Jell-0 and pudding, Joyce brought her extra. Joyce talked to her constantly ― about the weather, current events, anything that she could think of. 1

(B) One day, to her amazement, Miss Lilly spoke. "Bend down," Miss Lilly said. Quickly, Joyce knelt at her side. "Put your arms around me and pretend you love me," Miss Lilly whispered. "Me love Miss Lilly?" Joyce had never thought about it. Joyce gathered Miss Lilly into her arms and felt her heart bursting with love. There have been many Miss Lillys in Joyce's life since then and she knows there will be others. They are the ones who need more than kindness and care; they need a little piece of her heart. She loves each day of work, sharing with the residents with her life, her joys, and her sorrows. They share with her their past, their fear of the future, their families, and most of all, their love. Because of Miss Lilly, Joyce never feels the same about the older generation, about nursing facilities, or even about life. 2

(C) When the ringing of the telephone woke her, it was 8:05 am. The woman on the other end sounded cheerful. "I have your application for activity director," she said. "We are about to open a new unit. How soon can you be here for an interview?" Trying her best to sound awake, Joyce said, "One hour. I can come in one hour." From that day on, Joyce's life changed. Each waking moment, her thoughts are on the residents of the nursing facility. The residents fill her thoughts and her heart. Her first love was Miss Lilly, a lonely woman with only one living relative. Miss Lilly was not a pretty sight. 0


① A-C-B ② B-A-C ③ B-C-A ④ C-A-B ⑤ C-B-A

 


129. 20영독 M3-01

 

Since the Book Tree Library was founded in 2001 as a place for research and study, we have made efforts to meet the users' various needs.


(A) Funds are needed to employ more administrative staff. For 20 years, our dream has been to build the library of everything and make it available to everyone. Please help the Book Tree Library. 1

(B) We are about to modernize our services. We strive to develop electronic library resources, as well as provide remote services to our users. We need to buy more paperback books to sustain a library you can trust. 0

(C) If everyone donates $5, we can end this fundraising campaign successfully. With your donation we can make our dream come true. If you find our library useful, please help us. 2


① A-C-B ② B-A-C ③ B-C-A ④ C-A-B ⑤ C-B-A

 


130. 20영독 M3-02

 

It was July 1985.


(A) Billy gazed up at the enormous building with its nine floors and its thousand windows. It was the big day, a watershed in his life, and he couldn't get his head round it. He'd retired! At last! The enormity of the event began to sink in, and a shiver of joy ran down his spine. 0

(B) No more trying to persuade uncooperative colleagues to take one of his improvement courses; no more having to bow and scrape to bosses who paid lip service to the need for his job. No more having to join the morning rush hour to get to work on time. No more being ruled by the demands of tight timetables and having to jump whenever some superior gave the command. Farewell to all that! Now he knew how a prisoner felt on the day of his release when he heard the gate finally clang behind him. 2

(C) He'd left! Actually left! Finally got away from the William Pitt College of Technology. What a nightmare of a job it'd been. Now he was free. 1


① A-C-B ② B-A-C ③ B-C-A ④ C-A-B ⑤ C-B-A

 


131. 20영독 M3-03

 

If you're a young athlete, you, your parents, and your coaches want you to experience success now because you and they believe that early success is highly predictive of later success in your sport.


(A) For example, out of the thousands of young baseball players who have competed in the Little League World Series throughout the years, fewer than 50 went on to major-league careers. In fact, phenoms are a statistical rarity, and those can't-miss kids often do miss later in their athletic careers. 1

(B) More often than not, it is the athletes who keep at it through setbacks, plateaus, and failures who ultimately "make it." Your efforts early on as you strive for your sports goals should be devoted to preparing yourself for success in the future, when it matters most, not achieving quick and immediate success. 2

(C) Our athletic culture is obsessed with the "phenom" and the "can't-miss kid," who show earlier dominance in a sport. Yet, although there have been phenoms who went on to great success later in their careers, this perception is as much fantasy as reality. 0


① A-C-B ② B-A-C ③ B-C-A ④ C-A-B ⑤ C-B-A

 


132. 20영독 M3-04

 

Why did evolution decide to ban muscle activity during REM sleep?


(A) It wouldn't take long before you quickly left the gene pool. The brain paralyzes the body so the mind can dream safely. 2

(B) Because by eliminating muscle activity you are prevented from acting out your dream experience. During REM sleep, there is a nonstop barrage of motor commands swirling around the brain, and they underlie the movement-rich experience of dreams. 0

(C) Wise, then, of Mother Nature to have tailored a physiological straitjacket that forbids these fictional movements from becoming reality, especially considering that you've stopped consciously perceiving your surroundings. You can well imagine the disastrous outcome of falsely enacting a dream fight, or a frantic sprint from an approaching dream foe, while your eyes are closed and you have no comprehension of the world around you. 1


① A-C-B ② B-A-C ③ B-C-A ④ C-A-B ⑤ C-B-A

 


133. 20영독 M3-05

 

When you and your daughter hear the word volunteering, what's the first image that comes to your mind?


(A) That's a long way from eating raw bugs in Africa! She found something that she loved doing and gives her time helping others. 2

(B) Donating your outgrown clothes to the Salvation Army? Maybe you think volunteering means you have to go to Africa and live in a mud hut and eat raw bugs while teaching Sunday School to starving children. Actually, those are ways to volunteer, yet the world of volunteering is much broader than the stereotypical ideas most people have. 0

(C) There are hundreds of ways to volunteer, from collecting newspapers for the Humane Society to playing the guitar at an assisted living center. There's a volunteer possibility just right for your daughter. I have a friend who loves gymnastics, so she volunteers at a local gym and teaches kids to somersault and do cartwheels. 1


① A-C-B ② B-A-C ③ B-C-A ④ C-A-B ⑤ C-B-A

 


134. 20영독 M3-06

 

How often do we think about the air we breathe, the water we drink, or the soil our agribusiness conglomerates plant our vegetables in?


(A) Only when people can see — and smell — the air they breathe and cough when they inhale does air become a visible resource. Water, the universal solvent, causes no concern (and very little thought) until shortages occur, or until it is so foul that nothing can live in it or drink it. Only when we lack water or the quality is poor do we think of water as a resource to "worry" about. 1

(B) Is soil a resource or is it "dirt?" Unless you farm, or plant a garden, soil is only "dirt." Whether you pay any heed to the soil/dirt debate depends on what you use soil for ― and on how hungry you are. 2

(C) Not often enough. The typical attitude toward natural resources is often deliberate ignorance. Only when someone must wait in line for hours to fill the car gas tank does gasoline become a concern. 0


① A-C-B ② B-A-C ③ B-C-A ④ C-A-B ⑤ C-B-A

 


135. 20영독 M3-07

 

Twenty to thirty years is a long time in the annals of information technology ― long enough to allow us to discern a fundamental rift between the inner workings of yesterday's and today's computational tools.


(A) In a metaphorical sense, computers are now developing their own science ― a new kind of science. Thus, just as the digital revolution of the 1990s (new machines, same old science) generated a new way of making, today's computational revolution (same machines, but a brand-new science) is generating a new way of thinking. 2

(B) At the beginning, in the 1990s, we used our brand-new digital machines to implement the old science we knew — in a sense, we carried all the science we had over to the new computational platforms we were then just discovering. 0

(C) Now, to the contrary, we are learning that computers can work better and faster when we let them follow a different, nonhuman, postscientific method; and we increasingly find it easier to let computers solve problems in their own way — even when we do not understand what they do or how they do it. 1


① A-C-B ② B-A-C ③ B-C-A ④ C-A-B ⑤ C-B-A

 


136. 20영독 M3-09

 

Walter Mischel was born in Vienna in 1930, in a house that was a short walk away from where Sigmund Freud lived.


(A) He suggested that the early link with Freud led him to begin his career as an advocate of Freud and psychoanalysis. However, he found that the psychoanalytic approach was of little help in his work with inner-city aggressive youngsters. 1

(B) His family moved to New York when he was 10 years old to escape from the Nazis. He studied psychology but qualified as a social worker. 0

(C) This led him to undertake a PhD in psychology at Ohio State University, where he worked with George Kelly and Julian Rotter. After graduation he worked at Harvard University and then Stanford University before moving to Columbia University in 1984. While at Harvard he worked on a project assessing performance for the Peace Corps and found that global trait measures of personality were not good predictors of performance. 2


① A-C-B ② B-A-C ③ B-C-A ④ C-A-B ⑤ C-B-A

 


137. 20영독 M3-12

 

In no way is the use of the word "concept" intended to suggest that perceiving is an intellectual operation.


(A) It seems now that the same mechanisms operate on both the perceptual and the intellectual level, so that terms like concept, judgment, logic, abstraction, conclusion, computation, are needed in describing the work of the senses. 2

(B) The processes in question must be thought of as occurring within the visual sector of the nervous system. But the term concept is intended to suggest a striking similarity between the elementary activities of the senses and the higher ones of thinking or reasoning. 0

(C) So great is this similarity that many psychologists attributed the achievements of the senses to secret aid supposedly rendered them by the intellect. Those psychologists spoke of unconscious conclusions or computations because they assumed that perception itself could do no more than mechanically register the impingements of the outer world. 1


① A-C-B ② B-A-C ③ B-C-A ④ C-A-B ⑤ C-B-A

 


138. 20영독 M3-13

 

One alternative to self-disclosure is to keep your thoughts and feelings to yourself.


(A) Social scientists have found that people often make distinctions between "lies of omission" and "lies of commission" — and that saying nothing (omission) is usually judged less harshly than telling an outright lie (commission). One study showed that in the workplace, holding back information is often seen as a better alternative than lying or engaging in intentional deception. 2

(B) Telling the whole truth may be honest, but it can jeopardize you, the other person, and your relationship. Most thoughtful communicators would keep quiet rather than give unwanted opinions like "You look awful" or "You talk too much." 1

(C) You can get a sense of how much you rely on silence instead of disclosing by keeping a record of when you do and don't express your opinions. You're likely to find that withholding thoughts and feelings is a common approach for you. 0


① A-C-B ② B-A-C ③ B-C-A ④ C-A-B ⑤ C-B-A

 


139. 20영독 M3-14

 

Sociologist Erving Goffman notes "that when an individual appears before others he will have many motives for trying to control the impression they receive of the situation."


(A) In words, gestures, and small signs, we leave a trail of cues that are meant to guide the responses of our audiences. No moment in the routine events of the day is too small to be completely without persuasion. 2

(B) He referred to such strategies as impression management. Since we perform many of these roles simultaneously, we are constantly faced with the imperatives of making our actions and attitudes acceptable to others. Every role we play carries a number of possible strategies for influencing others. 1

(C) We want to be liked and to have our ideas accepted. We want others to show regard for our feelings and for the values that serve as the anchors for our actions. Goffman reminds us that children, teachers, parents, close friends, employees, employers, spouses, lovers, and coworkers all have strategies for projecting their interests to those with whom they come in contact. 0


① A-C-B ② B-A-C ③ B-C-A ④ C-A-B ⑤ C-B-A

 


140. 20영독 M3-15

 

Entrepreneurs succeed by providing customers with businesses and services they value.


(A) Business people draw on their knowledge acquired from experience to make more accurate assessments of areas of possible demand. Part of this involves getting into the heads of the consumers to see how they perceive products in relation to their needs. In so doing, you need to be aware of changes in lifestyles of consumers and their product needs. 1

(B) This requires knowledge of what people value and how to provide those goods and services. It is hard to succeed without that knowledge. The person who observes a change in consumption patterns will not necessarily realize its importance unless he or she is familiar with the product or industry. 0

(C) With this knowledge, you can create a product that connects with the changing lifestyles of the potential market. Firms that put together new combinations of technologies and build products that fit into buyers' thought systems should have greater potential for survival than those that do not. 2


① A-C-B ② B-A-C ③ B-C-A ④ C-A-B ⑤ C-B-A

 


141. 20영독 M3-16

 

The mystery of why we are attracted to sad music is a particularly fascinating paradox that has puzzled philosophers for centuries, with very little empirical research on the subject until the last decade.


(A) Research supports this idea, with findings that people do mostly prefer to listen to up-tempo music in major keys, music which is usually perceived as happy. Counterintuitively, however, in the case of music or other aesthetic experiences, the evidence suggests that we also willingly seek out experiences of sadness, even seeming to enjoy them. 1

(B) As David Hume says, "they are pleased as they are afflicted, and never so happy as when they employ tears, sobs and cries to give vent to their sorrow." 2

(C) 'Negative' emotions such as sadness are generally held to involve avoidance behaviours according to most models of emotion, impelling us to escape from situations or people that make us feel sad, thus protecting us from potential danger. We could expect, therefore, that people would usually display a preference for listening to happy music. 0


① A-C-B ② B-A-C ③ B-C-A ④ C-A-B ⑤ C-B-A

 


142. 20영독 M3-17

 

The form of street art changes to meet the conditions of the political system.


(A) In an authoritarian system, political matters take precedence over social, cultural, and economic questions. The opposition's push is to resolve political questions first; their themes record a regime's sins. In a competitive system, once the transition questions are resolved, the street art begins to reflect an array of pressing problems — political, social, economic, or cultural in nature. These are recorded in posters, graffiti, wallpaintings, and murals. 2

(B) In open, pluralistic societies, collectives competing for political space often utilize all forms to record their historical memory. However, in noncompetitive, authoritarian systems where government dominates public space, graffiti becomes the primary medium; posters, wallpaintings, and murals are more risky. Until the twilight of the Augusto Pinochet regime, the painting of murals ceased because of the high political risks. 0

(C) Leaflets supplemented the underground graffiti. At times they were displayed as wallposters. Themes also adapt to reflect pressing national problems. 1


① A-C-B ② B-A-C ③ B-C-A ④ C-A-B ⑤ C-B-A

 


143. 20영독 M3-18

 

The American Revolution ranks among the most written about episodes in history.


(A) It achieved independence and forged a great nation. But historians and readers have mostly approached it as an isolated American drama, the decisive formative episode in the history of the nation-state. 0

(B) Like the French Revolution, these were all profoundly affected by, and impacted on, America in ways rarely examined and discussed in broad context. 2

(C) That it also exerted an immense social, cultural, and ideological impact on the rest of the world that proved fundamental to the shaping of democratic modernity has attracted little attention since the mid-nineteenth century until very recently. The American Revolution, preceding the great French Revolution of 1789-99, was the first and one of the most momentous upheavals of a whole series of revolutionary events gripping the Atlantic world during the three-quarters of a century from 1775 to 1848-49. 1


① A-C-B ② B-A-C ③ B-C-A ④ C-A-B ⑤ C-B-A

 


144. 20영독 M3-19

 

A very few animal species have rudimentary culture.


(A) A local troop of Japanese macaques have learned, from the example an innovative female in their midst provided, how to clean sweet potatoes by washing them in water. Equally impressive, members of at least one chimpanzee troop use bush stems stripped of leaves to fish for termite soldiers, the suicidally aggressive insect fighters that bite and hold on to any invader of their nest. Members of a second group of chimpanzees have learned from one another how to swim and dive or otherwise move through water. 0

(B) Linguists define it as the highest form of communication, an endless combination of words translatable into symbols, and arbitrarily chosen to confer meaning. They are used to label any conceivable entity, process, or one or more attributes that define entity and process. 2

(C) These are among the very rare examples of true cultures ― behavior invented by individuals and groups and passed on by the social learning of others. But no animal species, at least none out of the more than one million known, has a language. What then is language ― what exactly? 1


① A-C-B ② B-A-C ③ B-C-A ④ C-A-B ⑤ C-B-A

 


145. 20영독 M3-20

 

Fear of radiation is problematic considering the trend in radiation exposures.


(A) Since 1980, the background radiation exposure level for Americans has doubled, and is likely to continue to climb. Similar patterns are occurring in all of the developed and developing countries. This increase in background radiation is almost entirely due to the expanding use of radiation procedures in medicine. 0

(B) While some people are getting no medical radiation exposure at all, others are receiving substantial doses. Under such circumstances, the "average" background radiation level means little to the individual. People need to be aware of their personal radiation exposures and weigh the risks and benefits before agreeing to subject themselves to medical radiation procedures. 2

(C) The benefits of diagnostic radiology in identifying disease and monitoring treatment progress have been significant. However, radiation has also been overused in many circumstances, conveying little or no benefits to patients while still subjecting them to increased risks. Furthermore, medical radiation is not distributed evenly across the population. 1


① A-C-B ② B-A-C ③ B-C-A ④ C-A-B ⑤ C-B-A

 


146. 20영독 M3-21

 

Unlike conventional marketing activities, like advertising and promotions, that are planned and scripted, sports events are inherently unpredictable.


(A) Fans, athletes, teams, and companies do not know outcomes. Despite even the most formidable track records of success, one cannot know for certain whether past sport performances will continue or whether expectations will be turned upside down. 0

(B) Yet sports fans follow sports partly because outcomes are not guaranteed. Fans have an emotional attachment to their favorite teams and athletes, irrespective (mostly) of their recent performances. If sports were scripted then they would lose credibility, spontaneity would be lost, and they would be no different than a conventional company-directed ad campaign. 2

(C) This very unpredictability separates sports from almost all other corporate marketing activities. Indeed, many business managers find this prospect of uncertainty distinctly uncomfortable and consequently shy away from using sports as a marketing platform. 1


① A-C-B ② B-A-C ③ B-C-A ④ C-A-B ⑤ C-B-A

 


147. 20영독 M3-22

 

Formal education has had a major and positive impact on society, but it is also true that not all students meet their learning aspirations.


(A) This subset of problems is nevertheless fundamental to education and, in general, includes the difficulties that many students have in effectively learning and understanding new ideas and concepts, correcting misconceptions, achieving proficiency in math and reading, and thinking critically. Even in the best of circumstances, many students will still struggle, and many of the efforts of cognitive and educational psychologists are aimed at helping students more effectively learn and teachers more effectively teach. 2

(B) Many children and adults struggle to learn and many are left behind. The problems that undermine their efforts to succeed (and instructors' efforts to help them) arise from numerous sources. 0

(C) A short list includes poor nutrition, poor physical or mental health, a lack of motivation, boredom, social and interpersonal problems at school or at home, ineffective approaches to learning, learning disabilities, and poor access to educational resources. Successfully solving these problems will require many solutions and only a subset of them are targeted by cognitive psychologists. 1


① A-C-B ② B-A-C ③ B-C-A ④ C-A-B ⑤ C-B-A

 


148. 20영독 M3-23

 

Whereas nineteenth-century dietary reformers worried that we'd stopped baking our own bread, today's food evangelists worry that we've stopped cooking altogether.


(A) It's true that families eat out more than in the past. And women spend less time cooking than they did a few generations ago. 0

(B) The difference is that these women previously worked inside the home, as domestic laborers, rather than in restaurants. At the peak, almost two million domestic workers were employed in American households. Anthropologist Amy Trubek notes that idealized visions of home cooking persistently neglect "the many generations of paid cooks who first worked in homes and then in commercial settings to make these meals possible." 2

(C) But oversimplified comparisons of today's families with those of previous generations fail to acknowledge the fact that Americans have long depended on the labor of others to get dinner on the table. Poor white women and women of color prepared many people's meals a century ago, just as they do today. 1


① A-C-B ② B-A-C ③ B-C-A ④ C-A-B ⑤ C-B-A

 


149. 20영독 M3-2425

 

In an experiment, more than one hundred volunteers were shown two photographs, each of a woman's face.


(A) That is, immediately after deciding that woman A was more attractive, a double-card ploy was used to confront subjects with the picture of woman B and they had to explain why they chose her (the two women depicted on the photos were quite distinct). Remarkably, most of the time the subjects were fooled. Only in fewer than 25% of trials were participants aware that their original choice was not honored, that they had been fooled. Most of the time, they ignored the discrepancy between their original conscious decision and what they were told they had decided. 1

(B) And even more remarkably, they proceeded to justify this choice even though it contradicted what they actually did a few seconds earlier: "She's radiant. I would rather have approached her than the other one. I like her earrings," even though the original choice looked solemn and had no earrings. What choice blindness reveals is that people often have no idea why they choose the way they do. But their urge to explain their actions is such that this does not prevent them from making up a story on the spot, confabulating without knowing it. 2

(C) After looking at both pictures for a few seconds, they had to choose the one that looked most attractive to them. Immediately after three such choices, subjects were shown again the face they had just chosen and were asked to explain their choice. They readily complied. On three other trials, the experimentalist, in a sleight of hand, exchanged the picture of the chosen woman with the opposite image. 0


① A-C-B ② B-A-C ③ B-C-A ④ C-A-B ⑤ C-B-A

 


150. 20영독 M3-2628

 

It was one of the great moments in Australian sport.


(A) And it's you, Betty Cuthbert.' Betty's knees tremble with emotion. And yet, though hands reach out to support her, she waves them away, stands tall for the national anthem, then makes her way, still upright and unaided, 100 metres or more back into the bowels of the stadium ― using all those qualities of pride, resilience and strength that had made her such a champion all those years ago. Just one step inside the tunnel, however, the instant she is out of public view, she collapses into the arms of officials. There is a flood of tears, of pain from the multiple sclerosis and sheer emotion. It takes a while, but after she sobs out the story of what Raelene had said to her on the dais, everyone within earshot is crying too. 2

(B) But not for this occasion! No, to give Raelene her medal, Betty is determined to do it on her own two feet, and now, painfully, slowly, makes her way forward unaided right to the dais in the centre of the stadium. As the crowd roars even more, she reaches up with the gold medal to put it around the neck of the crowd's heroine, while the big screen flashes the very words the announcer is intoning to the stadium, 'RAELENE — OUR GOLDEN GIRL.' But now, as Raelene leans further forward, it brings her mouth close to Betty's ear, enabling her to whisper, 'Don't believe it. There's only one golden girl. 1

(C) You see, when Raelene Boyle won gold in her last race, at the Brisbane Commonwealth Games in 1982 — roaring home in the 400 metres ― the stadium roared even as the nation came to a standstill. Raelene, a beloved figure, who had been denied gold in the previous two Olympics at the hands of East Germans, had at least and at last the perfect finish to her career. And who has been organised to present the medal at such an emotion-charged, proud, national moment? Why, none other than Betty Cuthbert! Betty, a legend of her own time, had three Olympic gold medals to her credit from a quarter-century earlier, before she contracted multiple sclerosis — which is why in 1982 she was mostly wheelchair-bound. 0


① A-C-B ② B-A-C ③ B-C-A ④ C-A-B ⑤ C-B-A

 


[ANSWER]
1. ⑤ 2. ③ 3. ③ 4. ③ 5. ④ 6. ① 7. ④ 8. ⑤ 9. ① 10. ③


11. ③ 12. ① 13. ⑤ 14. ① 15. ② 16. ③ 17. ② 18. ③ 19. ② 20. ②


21. ① 22. ④ 23. ① 24. ② 25. ④ 26. ④ 27. ① 28. ⑤ 29. ⑤ 30. ⑤


31. ⑤ 32. ① 33. ③ 34. ③ 35. ③ 36. ④ 37. ⑤ 38. ② 39. ① 40. ⑤


41. ③ 42. ② 43. ① 44. ① 45. ② 46. ③ 47. ③ 48. ③ 49. ② 50. ⑤


51. ① 52. ② 53. ① 54. ① 55. ② 56. ② 57. ③ 58. ② 59. ⑤ 60. ④


61. ③ 62. ③ 63. ⑤ 64. ② 65. ③ 66. ④ 67. ④ 68. ① 69. ② 70. ②


71. ④ 72. ③ 73. ⑤ 74. ⑤ 75. ④ 76. ③ 77. ① 78. ⑤ 79. ⑤ 80. ①


81. ④ 82. ① 83. ③ 84. ③ 85. ③ 86. ② 87. ③ 88. ⑤ 89. ② 90. ⑤


91. ③ 92. ④ 93. ⑤ 94. ④ 95. ③ 96. ① 97. ③ 98. ③ 99. ④ 100. ②


101. ④ 102. ① 103. ② 104. ① 105. ① 106. ② 107. ① 108. ⑤ 109. ② 110. ①


111. ① 112. ① 113. ② 114. ① 115. ② 116. ① 117. ⑤ 118. ② 119. ① 120. ⑤


121. ⑤ 122. ③ 123. ① 124. ⑤ 125. ④ 126. ① 127. ④ 128. ④ 129. ② 130. ①


131. ④ 132. ③ 133. ③ 134. ④ 135. ③ 136. ② 137. ③ 138. ⑤ 139. ⑤ 140. ②


141. ④ 142. ③ 143. ① 144. ① 145. ① 146. ① 147. ③ 148. ① 149. ④ 150. ⑤


 

728x90
반응형

728x90
반응형

20ss | Since 2005 임희재 | 블루티쳐학원 | 01033383436 | 200710 22:53:52

 

순서배열

 

1. 20수특 1-1

 

My name is Susan and I work in the accounting department.


(A) It is obvious to both me and the rest of our colleagues that she has a natural talent for what she does. Her clients love her and so do we. She is always quick to help with whatever we need no matter what it is. 1

(B) I happened to hear that you are looking to promote someone within our company to the position of manager of the accounting department, and I felt a need to write you this letter. I think Victoria Kimball, my department colleague, is an ideal candidate for the position. She is always on time, she never leaves early, and she does excellent work while she is here. 0

(C) She is a fantastic employee and deserves the position. You would not regret choosing her. As you are thinking about who might best fit into the position, I hope you will consider Victoria. 2


① A-C-B ② B-A-C ③ B-C-A ④ C-A-B ⑤ C-B-A

 


2. 20수특 1-2

 

You took a month-long leave of absence starting from November to December.


(A) As it was a month-long leave, it was obviously a part of your duties to hand over your work to your team members. Hence, it was very disappointing to discover that you went on the leave without giving them any information about the projects that you were handling. 0

(B) As you know, any work loss because of a failure to hand over work is a serious problem. This letter is the final warning regarding your irresponsible behavior. Failure to fulfill your duties in the future will not be tolerated and will result in us taking more severe actions. 2

(C) In turn, your carelessness led to unfortunate delays in the projects. We are sure that you are aware that during one's absence from work, the company cannot afford to put any work on hold. 1


① A-C-B ② B-A-C ③ B-C-A ④ C-A-B ⑤ C-B-A

 


3. 20수특 1-3

 

My name is David Lee and I'm a third-year chemistry major at Arizona State University.


(A) My résumé is attached in case you are interested. I look forward to hearing from you. 2

(B) While completing both my introductory and upper-level coursework, I've developed a passion for science and am extremely interested in pursuing independent research as an undergraduate. Personally, I am especially interested in nanotechnology. Recently I read your 2017 papers on the potential applications of graphene and became fascinated by your work. 0

(C) In particular, I found it amazing that graphene can be utilized to make flexible, transparent solar cells that can turn virtually any surface into a source of electrical power. If possible, I would be honored to work on a long-term project in your lab. Would you be interested in including me as part of your research team? 1


① A-C-B ② B-A-C ③ B-C-A ④ C-A-B ⑤ C-B-A

 


4. 20수특 1-4

 

We would like to express our deepest gratitude for considering AGL as your insurance provider.


(A) We hope that you will still consider taking out a policy with us when it becomes available. We appreciate your patience and understanding on this matter. 2

(B) Currently we are working on bringing that policy back. Therefore, we are keeping your application on hold for future reference and further processing when your preferred policy becomes available again. If you would prefer not to wait for approval and would like us to remove your application from our system, please let us know. 1

(C) You have made a wise choice, and your business is very important to us. Unfortunately, however, we are afraid to inform you that your insurance policy application cannot be approved at the moment. The type of policy you applied for isn't available right now. 0


① A-C-B ② B-A-C ③ B-C-A ④ C-A-B ⑤ C-B-A

 


5. 20수특 2-1

 

I was once out in the foothills of the Absaroka Mountains near my home in southern Montana when I saw a front of windy and snowy weather coming toward me.


(A) All I could do for some period of time was to crouch down and wait. 2

(B) Because the bare hillside was steep and already snow covered, the going was slow, so I didn't make it off the slippery slope before the blizzard hit. The wind hit with such force that I couldn't stand upright, and there was so much falling and blowing snow that I couldn't see much either. 1

(C) Because of the open nature of that part of the upper Yellowstone River Valley, I was able to see the storm coming from a long distance away, but I was so far out on an exposed hillside that I wasn't able to make it to cover before the blizzard hit. The slope where I was hiking was vegetated only with grass and sagebrush, so I started for a north-facing and timbered slope a mile or so away. 0


① A-C-B ② B-A-C ③ B-C-A ④ C-A-B ⑤ C-B-A

 


6. 20수특 2-2

 

My first morning in New York, I put on my girl-writer dress and heels and went to meet my editor, looking forward to a promising day.


(A) I kept touching my forehead, the way you pat your head to make sure your hair is okay. Then I started to cry and told him I had to go right that very second. He told me to phone him the next day. I said I would, although I had no intention of actually doing so. 2

(B) I looked at him quizzically. "I am so, so sorry," he said. "But it still doesn't work." I sat there staring at him as if his face were melting. 1

(C) I figured we would start editing together that very morning, and then he could give me the last of the advance. It would turn out that I had bounced back from this devastating setback and that truth and beauty had once again triumphed. Everyone would be so shocked to hear that this book had almost been thrown away. But my hopes were shattered when my editor said, "I'm sorry." 0


① A-C-B ② B-A-C ③ B-C-A ④ C-A-B ⑤ C-B-A

 


7. 20수특 2-3

 

In those more hopeful and innocent days there was much to enjoy about being newly a queen – not only that I had survived against the odds to inherit.


(A) The grand garment smelt faintly of beeswax and the softest leather. Unable to resist, I buried my features in it and laughed with delight. 1

(B) It was thrilling to know that such luxury was mine. My ladies laughed with me, equally delighted. They could not wear the elegant clothes and jewelry that as queen I wore even a queen with her hair as yet unbrushed, but their circumstances as my attendants were vastly better now that I was queen. 2

(C) I still remember the sensation of the royal white fur collar around my neck. I put my cheek to it and the fur seemed strangely warm against my skin. 0


① A-C-B ② B-A-C ③ B-C-A ④ C-A-B ⑤ C-B-A

 


8. 20수특 2-4

 

About three months ago, my sister was having problems with her daughter, Amy, so my sister and I decided to have Amy move in with my family.


(A) Instead of remaining supportive of her when she would get excited, I minimized her feelings by telling her to grow up. I expected her to understand the cause of her fears. Also, I expected her to magically allow herself to be a part of a loving family. It wasn't that simple for her, and I should not have reacted to Amy's emotions. 1

(B) We thought she could benefit from a chance of environment. Sadly, almost instantly, my relationship with Amy started to fall apart. She tried to verbalize her need for limits and boundaries in the home and for time spent alone together. However, our conversations quickly spiraled out of control. 0

(C) She needed a safe space to calm down, but I didn't give her that and didn't listen to her better. I should have respected her perspective and demonstrated that I was understanding of her fears. I should have allowed her to adjust to sharing space with my family. 2


① A-C-B ② B-A-C ③ B-C-A ④ C-A-B ⑤ C-B-A

 


9. 20수특 3-1

 

You are much more than just a list of your accomplishments.


(A) Or will you push yourself to perform like an all-star, the legendary player you know you can be, and dive for the ball? Your special effort will be recorded in the minds and memories of the others. More importantly, you will have the peace of mind and self-respect that come from knowing that you gave your very best – even if you don't end up making the catch. That's the stuff of legends. 2

(B) It says you give 110%. Every performance in your life becomes a choice you have to make. Will you do just enough to get by, letting the ball drop in front of you for a base hit? 1

(C) It's not only what you do, but how you do it that counts in the real scorebook of your life. Making the spectacular diving catch says more about you than the "out" that is recorded in the scorebook. It says you have game. 0


① A-C-B ② B-A-C ③ B-C-A ④ C-A-B ⑤ C-B-A

 


10. 20수특 3-2

 

Irrational acts don't just sabotage us.


(A) The television mini-series Lonesome Dove was a big hit in the 1980s. Two rugged Texas Rangers shared a life together that eventually led them from Texas to Montana, where they made their fortune from a cattle drive. One partner died and the other promised to personally return the body to Texas; an incredible sacrifice. 1

(B) From rational point of view, the dead partner wouldn't know where he was buried; he was already dead. The surviving partner's friends thought his promise was foolish. They pressured him to ship the boy to Texas by train. No rational argument would ever win this battle, and if it had, the movie would have lost its charm. 2

(C) They can also make us heroes, lovers, and generous helpers. The qualities we admire most in others are their emotional ones, not their intellectual ones. Rarely does intellect alone inspire romantic acts or heroic deeds. 0


① A-C-B ② B-A-C ③ B-C-A ④ C-A-B ⑤ C-B-A

 


11. 20수특 3-3

 

Ideas or theories about human nature have a unique place in the sciences.


(A) Forty years ago, the distinguished anthropologist Clifford Geertz said that human beings are "unfinished animals." What he meant is that it is human nature to have a human nature that is very much the product of the society that surrounds us. That human nature is more created than discovered. 1

(B) We "design" human nature, by designing the institutions within which people live. So we must ask ourselves just what kind of a human nature we want to help design. 2

(C) We don't have to worry that the cosmos will be changed by our theories about the cosmos. The planets really don't care what we think or how we theorize about them. But we do have to worry that human nature will be changed by our theories of human nature. 0


① A-C-B ② B-A-C ③ B-C-A ④ C-A-B ⑤ C-B-A

 


12. 20수특 3-4

 

Near my old office building, the window of a show store advertised the generous offer of a free shoe shine.


(A) I had to even the score, somehow. Since I didn't need shoes, I found myself mindlessly looking at shoe trees, laces, and polish. Finally, I quietly walked out of the store empty-handed and uneasy. Even though I had managed to escape from the store, I was sure many others were not so fortunate. 2

(B) Free was free, he said. I climbed down from the chair feeling distinctly indebted. "How could this guy shine my shoes," I thought, "and expect nothing?" So I did what I suspect most people who take the offer do – I looked around for something to buy. 1

(C) I walked by this tore dozens of times and thought nothing of it. One day, though, with my shoes looking a little scuffed and some time on my hands, I decided to avail myself of this small bounty. After my shine, I offered the shoeshine man a tip. He refused. 0


① A-C-B ② B-A-C ③ B-C-A ④ C-A-B ⑤ C-B-A

 


13. 20수특 4-1

 

People everywhere have their special cultural rites.


(A) Physicians scrub for seven minutes before doing a surgical procedure. While the necessity of the prolonged scrub is open to question with the advent of modern germicides, its traditional role in preparing the surgical team for a delicate procedure is undeniable. In the airline business, the first officer deplanes the aircraft and conducts a walk-around inspection before takeoff. 1

(B) It is just as true in the workplace as it is in the surrounding culture. All professions observe their distinctive ways. What appears to be a superficial set of actions to outsiders knits insiders together and puts them in an appropriate frame of mind to do their work successfully. 0

(C) Very seldom do they discover something wrong. But symbolically it prepares the cockpit crew for their awesome responsibility of getting all the souls aboard safely to their destinations. 2


① A-C-B ② B-A-C ③ B-C-A ④ C-A-B ⑤ C-B-A

 


14. 20수특 4-2

 

From what I have seen in counseling, the besetting sin of the father as performance-focused parent is his misguided belief that dogged disapproval will cause his teenagers to try harder and do better.


(A) And the father contaminates his connection with his teenager by provoking his or her increased resentment and dislike. "Dad is never satisfied, no matter how I do, which is all he really cares about!" As a performance coach, a father would be far better served by praising the good than by only faulting mistakes in the misguided belief that his expressions of dissatisfaction will cause improvement. 2

(B) The paternal criticism is offensive, not appreciated. The teen wants less to do with him and for him, not more. 1

(C) "I'll keep criticizing you until your attitude and motivation improve!" In fact, excessive criticism only hurts the teenager's feelings and discourages him or her from wanting to hear what the father has to say and from doing what the father wants him or her to do. 0


① A-C-B ② B-A-C ③ B-C-A ④ C-A-B ⑤ C-B-A

 


15. 20수특 4-3

 

When a young police officer puts on a uniform for the first time, it almost certainly feels strange and foreign.


(A) Equally, they are powerful statements to ourselves about what to expect of ourselves. This, together with the way other people react to our appearance, powerfully shapes how we feel, think and behave. 2

(B) they help people think themselves into a particular way of behaving, and communicate clearly to other people what function that person is expected to perform. Our dress and appearance are a sort of uniform as well, whether we like it or not. They are very powerful statements to other people about what to expect from us. 1

(C) Yet other people react to that uniform in a range of more or less predictable ways ― just as they do to a priest or to a white-coated doctor. These reactions help to make the police officer feel a part of the uniform and more comfortable with the role that goes with it. This is the point of uniforms:. 0


① A-C-B ② B-A-C ③ B-C-A ④ C-A-B ⑤ C-B-A

 


16. 20수특 4-4

 

While individualism gives strength, it also can create a weakness if not moderated by involvement with others ― family, friends, and society.


(A) Pure individualism may lead to a philosophy of convenience and a lack of participation in or appreciation of the civic and social process. This limits personal growth, mutes gaining leadership skills and traits, and deprives one of the true contexts of life that is the reality model of one's mind. 0

(B) We look at the world as centered upon us and give ourselves undue influence on the reality of events. We need to move into a position of not thinking the world rotates around us but how we fit into the world. This is the concept of context. 2

(C) The biggest problem the world now has is that we do not interact personally much anymore. Our technology removes some of reality and replaces it with perception, relativity, and inherent self-interest often pushing us to convenience. 1


① A-C-B ② B-A-C ③ B-C-A ④ C-A-B ⑤ C-B-A

 


17. 20수특 4-5

 

By taking a conscious interest in your brain and how it works, and by consciously trying some of the techniques that follow, you can acquire a variety of useful mental skills.


(A) A cross section of our society represents the "average" level of thinking skill ― the level one might expect of a large number of people who have never thought very much about thinking. To move beyond the level of average thinking, you must think about thinking. 1

(B) You must pay attention to how your brain works, and you must experiment with new techniques. Once you do that, you will clearly see the value of it. 2

(C) It isn't really very difficult, but it won't happen by accident. The prevalence of negative thinking, fuzzy and illogical thinking, and rigid thinking in our society proves that fact that these higher-level thinking skills do not come naturally. 0


① A-C-B ② B-A-C ③ B-C-A ④ C-A-B ⑤ C-B-A

 


18. 20수특 4-6

 

The term statistical significance is an unfortunate choice of words.


(A) But too many people hear the phrase "statistically significant" and assume it also suggests that the results are significantly important. That may or may not be true. 1

(B) Instead, think of the term as suggesting that you would expect to find the same results 95 out of 100 times if a study is replicated in a similar manner or 90 out of 100 times, depending on what measure of reliability is used. True significance lies in interpreting the data correctly to ensure that it has meaning or importance for the organization you represent. 2

(C) But it's part of our research vocabulary, and it will continue to appear in reports. It refers to the fact that the results discovered, or differences between two sets of data, could reliably be expected to occur again if another study was conducted in a similar manner. 0


① A-C-B ② B-A-C ③ B-C-A ④ C-A-B ⑤ C-B-A

 


19. 20수특 4-7

 

The survival of wilderness ― of places that we do not change, where we allow the existence even of creatures we perceive as dangerous ― is necessary.


(A) And I would argue that we do not need just the great public wildernesses, but millions of small private or semiprivate ones. Every farm should have one; wildernesses can occupy corners of factory grounds and city lots ― places where nature is given a free hand, where no human work is done, where people go only as guests. 1

(B) Our sanity probably requires it. Whether we go to those places or not, we need to know that they exist. 0

(C) These places function, I think, whether we intend them to or not, as sacred groves ― places we respect and leave alone, not because we understand well what goes on there, but because we do not. 2


① A-C-B ② B-A-C ③ B-C-A ④ C-A-B ⑤ C-B-A

 


20. 20수특 4-8

 

Facing your difficulty head-on is the first positive step in the process of fixing it.


(A) Follow the remedies they prescribe. If some project you are working hard to finish on time encounters severe problems, examine the difficulty as a scientist would. What caused the problem? What are the options? Try to discover the best ways of dealing with the realities you face, focus on what will be most beneficial, then act accordingly. 2

(B) Your promotion went to someone else ― now what? Face it directly. There must be a reason why your boss selected the other person. Get your hurt and anger under control and go find out why you didn't get the promotion. 0

(C) Ask your boss what you need to do to improve yourself so you'll be considered more seriously next time. If you have just learned that you have a health problem, face it squarely and intelligently. What is the best treatment? Ask the top specialists for their advice. 1


① A-C-B ② B-A-C ③ B-C-A ④ C-A-B ⑤ C-B-A

 


21. 20수특 5-1

 

Finding an ideal location for a piano is often difficult.


(A) In the order of importance, the location should help preserve the instrument, be acoustically satisfactory, and be aesthetically pleasing. Ideally, a piano should be placed on an inside wall, away from the direct rays of the sun. 0

(B) Instruments that are placed directly beneath water pipes or emergency sprinkler systems should be protected with a water proof cover from possible water damage. Finding the best location for a piano also includes acoustical considerations; usually a piano sounds best in a room without thick wall-to-wall carpeting or heavy, sound-absorbing draperies. 2

(C) Moreover, it should not be placed next to heaters, stoves, air conditioners, or near heat ducts or cold air returns. Drafty locations next to open windows or doors should also be avoided. 1


① A-C-B ② B-A-C ③ B-C-A ④ C-A-B ⑤ C-B-A

 


22. 20수특 5-2

 

Power, considered by some theorists to be the "entrance requirement" for anger, is not necessary for sadness.


(A) Anger is an "approach" emotion, while sadness is a "retreat" emotion. Thinking of a person as sad makes us see them as weaker and more submissive. Anger, not sadness, is associated with controlling one's circumstances, such as competition, independence, and leadership. 0

(B) Like happy people, angry people are more optimistic, feeling that change is possible and that they can influence outcomes. Sad and fearful people tend toward pessimism, feeling powerless to make change. 2

(C) Anger, not sadness, is linked to assertiveness, persistence, and aggressiveness. Anger, not sadness, is a way to actively make change and confront challenges. Anger, not sadness, leads to perceptions of higher status and respect. 1


① A-C-B ② B-A-C ③ B-C-A ④ C-A-B ⑤ C-B-A

 


23. 20수특 5-3

 

Sadly enough, some of us have distorted lessons of happiness that developed in our childhood.


(A) Most believe attaining true happiness is like winning the lottery, and only some of us are lucky enough to win it. Or maybe some of us believe in "works of righteousness" ― a theology that says if you work hard enough at anything, you will receive what you work for. 1

(B) Any one of these theories of happiness is born of the philosophy that happiness is scarce. Looking around our world right now I would have to agree that true happiness is in short supply. But this is because we have bought into a belief system that teaches us that happiness is as scarce as hen's teeth. 2

(C) Our experiences developed as we grew up in different systems, such as our original family, our religious community, and our neighborhood. Many of us believe that only a few of us experience true happiness. 0


① A-C-B ② B-A-C ③ B-C-A ④ C-A-B ⑤ C-B-A

 


24. 20수특 5-4

 

The causes and consequences of war may have more to do with pathology than with politics, more to do with irrational pressures of pride and pain than with rational calculations of advantage and profit.


(A) it also suggests that if there is a root cause of human conflict and of the power drive of nations, it lies not in hopes of economic development, historical forces, or the workings of the balance of power, but in the ordinary hopes and fears of the human mind. 2

(B) What this "proves," if anything, is that computers are more rational than men;. 1

(C) There is a Washington story, perhaps apocryphal, that the military intellectuals in the Pentagon conducted an experiment in which they fed data derived from the events of the summer of 1914 into a computer and that, after weighing and digesting the evidence, the machine assured its users that there was no danger of war. 0


① A-C-B ② B-A-C ③ B-C-A ④ C-A-B ⑤ C-B-A

 


25. 20수특 6-1

 

Aging is an economic challenge because unless retirement ages are drastically increased so that older member of society can continue to contribute to the workforce (an economic imperative that has many economic benefits), the working-age population falls at the same time as the percentage of dependent elders increases.


(A) In addition, fewer people are likely to take entrepreneurial risks because aging workers tend to preserve the assets they need to retire comfortably rather than set up new businesses. 1

(B) As the population ages and there are fewer young adults, purchases of big-ticket items such as homes, furniture, cars and appliances decrease. 0

(C) This is somewhat balanced by people retiring and drawing down their accumulated savings, which in total lowers savings and investment rates. 2


① A-C-B ② B-A-C ③ B-C-A ④ C-A-B ⑤ C-B-A

 


26. 20수특 6-2

 

During the 1890s Richard Henry was caretaker of Resolution Island in remote Fiordland on the west coast of New Zealand's South Island.


(A) A keen naturalist, he noted with concern the impact on native birds of the arrival of recently introduced stoats as they invaded this last corner of New Zealand. In a desperate attempt to protect populations of the flightless kakapo and little spotted kiwi between 1894 and 1900 he translocated hundreds of individuals from the mainland on to Resolution Island. 0

(B) Nevertheless, the technique of isolating species in danger on predator-free islands that may or may not have been occupied by the species in the past became a vital tool to prevent extinctions from predation by introduced predators in New Zealand. 2

(C) Unfortunately, Resolution was too close to the mainland and stoats invaded in 1900. Thus, Henry's efforts were in vain. 1


① A-C-B ② B-A-C ③ B-C-A ④ C-A-B ⑤ C-B-A

 


27. 20수특 6-3

 

Genes give us the foundation of our models.


(A) It is our job as supportive adults to find a constructive purpose. This does not mean that we should view violent behavior as resourceful; rather, we can enlist the core of violent behavior as a positive resource (eg, Violent behavior may exemplify an eagerness to take control, an ability to respond authoritatively, or a refusal to be victimized). As yourself in what context or situation the core of a particular behavior would signify value. 1

(B) Experiences give us individual identities. Behaviors express our individual needs, desires, urges, attitudes, beliefs, and so on. In this way, all behaviors are purposeful. 0

(C) For example, "Your refusal to be victimized will help you grow more tolerant with people as you mature." This comment orients the child toward a more fulfilling future because it validates the child's world view and enlists the core of the behavior as a positive resource. 2


① A-C-B ② B-A-C ③ B-C-A ④ C-A-B ⑤ C-B-A

 


28. 20수특 6-4

 

Managing relationships is a value that represents how someone considers the interests and well-being of other people involved in a person's social world.


(A) Being a host, guest or co-worker also shapes food choice situations where roles and relationships are primary considerations in food choice. 2

(B) When people provide food for others, share food with others or receive food from others, they typically consider the needs, preferences and feelings of those people related to what, how, when and where food is eaten. Personal needs and preferences are often compromised to build, maintain or repair relationships. 0

(C) Food is central to family harmony, and someone who adopts the role of the 'household food manger' is typically very attentive to the preferences, dislikes and patterns of eating of others. For example, newly married couples must negotiate ways to make joint food choices and parent-child relationships contribute to constructing family food decisions. 1


① A-C-B ② B-A-C ③ B-C-A ④ C-A-B ⑤ C-B-A

 


29. 20수특 8-1

 

Kate Seredy was born in 1899, in Budapest, Hungary.


(A) Seredy illustrated two children's books in Hungary before moving to the United States in 1922. She supported herself in her new homeland by illustrating lamp shades and greeting cards. As her knowledge of English increase, she found work illustrating textbooks and children's trade books. 1

(B) Her father, a teacher, helped her develop an appreciation for books. After high school she earned an art teacher's diploma from the Academy of Arts in Budapest and also spent time studying in Italy, France, and Germany. She served as a nurse for two years during World War I, and the pacifist stance she subsequently developed later influenced her writing. 0

(C) Seredy wrote her first book, The Good Master, after an editor suggested she try writing about her childhood in Hungary. Seredy won the Newbery Medal in 1938 for The White Stag, a book based on legends about the founding of Hungary that her father told her as a child. 2


① A-C-B ② B-A-C ③ B-C-A ④ C-A-B ⑤ C-B-A

 


30. 20수특 8-2

 

Eastern cottontails are the most common rabbits in North America.


(A) A pregnant female digs a shallow hole, which is deeper at one end than the other. She lines the nest with grass and fur from her belly. 2

(B) The name "cottontail" is derived from their short, rounded tails, which have white fur on their underside. Easter cottontail rabbits do not dig burrows, although they may shelter in disused ones dug by other animals. Unlike hares, which rely on their speed to outrun predators, cottontails freeze when under threat, blending into their surroundings. 0

(C) If they have to run, they follow zigzag paths, attempting to shake off their pursuers. In warmer parts of their range cottontails breed all year round, but farther north breeding is restricted to summer. Males fight to establish hierarchies, with top males getting their choice of mates. 1


① A-C-B ② B-A-C ③ B-C-A ④ C-A-B ⑤ C-B-A

 


31. 20수특 8-6

 

Born on February 9, 1874, Amy Lowell was the last of the five children of Augustus and Katherine Bigelow Lawrence Lowell, who resided in a mansion on a ten-acre estate in Brookline, Massachusetts.


(A) Although Lowell dabbled in verse from an early age, it was a chance encounter with Leigh Hunt's Imagination and Fancy in her father's collection that inspired her serious interest in poetry. During this same interval she discovered the poetry of John Keats, whose work would profoundly influence her aesthetics. 1

(B) Lowell's lifelong appreciation of Keats's writings, especially their correlation of beauty and human longing, was summed up in her two-volume study of the poet, published in the last year of her life. 2

(C) Privately schooled in Boston until the age of seventeen, Lowell learned little of literature during her formal education. She fostered her literary interests by reading in the extensive library at home and at the Boston Athenaeum. 0


① A-C-B ② B-A-C ③ B-C-A ④ C-A-B ⑤ C-B-A

 


32. 20수특 8-7

 

On March 20, 1883, Jan Matzeliger (1852-1889) patented the first successful shoe-lasting machine.


(A) The next year he settled in nearby Lynn, Massachusetts. There he developed his device while working in a shoe factory. The machine increased productivity as much as fourteen times over hand methods and led to concentration in the industry. 1

(B) Matzeliger continued to work on the machine to improve its quality, and received a patent for a third and improved model on March 20, 1883. He invented a number of other devices, including a mechanism for distributing tacks and nails. 2

(C) Matzeliger was born in Surinam of a Dutch father who was an engineer and a black mother who was Surinamese and probably came from West Africa. He left Surinam in 1871 and became a sailor on an East Indian ship. Matzeliger settled in Philadelphia for a while, holding odd jobs until he moved to Boston in 1876. 0


① A-C-B ② B-A-C ③ B-C-A ④ C-A-B ⑤ C-B-A

 


33. 20수특 9-1

 

Thomas Edison failed ten thousand times until, like a bolt of lightening, the solution to the challenge of the incandescent light bulb hit him, and because of this, he transformed the world.


(A) Rosa Parks had no idea that choosing to resist her conditioning ― the injustice of racial prejudicing ― and risking imprisonment, beating, or even worse, would spark a revolution that would transform history for all men and women, regardless of race. When Mother Teresa chose to leave her position educating the privileged classes of India in order to care for the poor and forgotten, people thought that she was crazy. 0

(B) So, too, will you receive your aha moments of brilliance, unexpectedly and through no effort of your own. 2

(C) At the time, Mother Teresa had no idea of the global impact that this decision would have on the world. Albert Einstein received the solution to his Theory of Relativity while dreaming that he was riding on a beam of light. 1


① A-C-B ② B-A-C ③ B-C-A ④ C-A-B ⑤ C-B-A

 


34. 20수특 9-2

 

The major themes of country music are work, freedom, and alienation, appearing in approximately one-sixth of all Top 20 selections.


(A) Consequently, from the start country music was an eclectic mix, which included ancient British ballads, Americanized versions of these, sacred songs, minstrel tunes, early blues, and songs of many sorts absorbed from the commercial popular music industry over the years. 1

(B) Over the 20th century it has grown from a homegrown and heartfelt music, expressing working-class identity, into a commercial music, produced by others to appeal to a working-class identity, whether or not its listeners are actually working class. The symbolic meaning of country music as the declared favorite music of New England-educated, upper-class President George Bush ― presenting him as a "regular American" ― is unmistakable. 2

(C) Country music is itself a commercial offshoot of the traditional song and instrumental music of the South, formerly carried on in an oral tradition. 0


① A-C-B ② B-A-C ③ B-C-A ④ C-A-B ⑤ C-B-A

 


35. 20수특 9-3

 

We must understand that busyness does not necessarily equal productivity, and in order to accomplish anything in life, we must prioritize.


(A) What we put at the top of our list determines how we spend our time and how much value we get out of each and every day. The truth is, most of us are unconscious about how we truly spend our time, prioritizing things that we say don't really matter to us. 0

(B) We do so much in a day, yet wonder where all of our time went. Once we get honest about how we're actually spending our days, we can begin to prioritize what's most important to us, and start taking back our time. 2

(C) We waste our extra time watching TV, complaining, or sleeping our lives away. We mindlessly scroll through social media and spend hours talking on the phone, and we don't make good use of our time traveling to and from work. 1


① A-C-B ② B-A-C ③ B-C-A ④ C-A-B ⑤ C-B-A

 


36. 20수특 9-4

 

The term genius can be traced back to the Latin word ingenium: a natural-born talent.


(A) It was not until the Renaissance that people began to describe an artistic creative potential or the source of inspiration as genius. The key significance for invention is that the so-called genius develops ideas that no one has had previously and, in the words of Immanuel Kant, that 'genius must be considered the very opposite of a spirit of imitation'. 1

(B) The essence of this talent is seen as original productivity, which employs confident intuition to access new areas of creativity. The person who has genius ― a brilliant creative power ― is also known as a genius. 0

(C) In addition, Kant established that genius 'cannot indicate scientifically how it brings about its product, but rather gives the rule as nature. Hence, where an author owes a product to his genius, he does not himself know how he conceived the ideas, nor is it in his power to invent the like at pleasure, or methodically, and communicate the same to others in such precepts as would put them in a position to produce similar products. 2


① A-C-B ② B-A-C ③ B-C-A ④ C-A-B ⑤ C-B-A

 


37. 20수특 9-5

 

The way our society views weight and getting into shape is one example of where expectations are often out of touch with reality, and lead to impatience ― one of the top reasons why over 95 percent of all diets fail.


(A) We search for the latest fads and are drawn to products claiming to trim our waistline in thirty days or less. Yet time after time we find ourselves, after attempts to diet, with ever more weight to lose. 1

(B) While there are many factors that go into weight loss and healthy living, one of the hardest things for people to accept is that any real lifestyle change takes time: time to break bad habits and time to form new healthy ones. 2

(C) We are led to believe through various media that dropping pounds is something that can be done quickly and effectively if we try hard enough. The reality show The Biggest Loser, where contestants drop ten or even twenty pounds in one week, implies that rapid results are not only possible but commonplace. 0


① A-C-B ② B-A-C ③ B-C-A ④ C-A-B ⑤ C-B-A

 


38. 20수특 9-6

 

Recent psychological research has revealed that college students who look at a two-second video clip of a professor teaching can predict how students who spend an entire semester with that professor will like that professor by the end of the semester.


(A) That statement is then recorded. At the end of the semester, students who have taken a class with the professor anonymously record whether or not they liked the professor. 1

(B) In other words, a student watching a two-second clip of a professor says, "I like him." Or "I don't like him." 0

(C) With incredible accuracy, those watching the two-second clip predict what the entire class will feel at the end of the semester. It sounds unbelievable, but it is true. One of the big mistakes we make is making a poor impression on others. 2


① A-C-B ② B-A-C ③ B-C-A ④ C-A-B ⑤ C-B-A

 


39. 20수특 9-7

 

Imagination and creativity are the gate keys of fantasy role-playing.


(A) The students exercise their imagination and creativity in countless ways, from taking on the role of their assigned characters to interacting with other creatures and alien environments. 1

(B) If students cannot imagine themselves engaged by the fantasy world described to them, then the game cannot get off the ground. 0

(C) In every case, what is minimally called for is imaginative flexibility in order to react appropriately to the multiple situations the students encounter, while looking ahead to the consequences of various actions and decisions. This means that fantasy role-playing provides an ideal environment to cultivate and test the productive use of imagination, utilizing it to enliven the fantasy narrative, envision alternatives, and empathize with others. 2


① A-C-B ② B-A-C ③ B-C-A ④ C-A-B ⑤ C-B-A

 


40. 20수특 9-8

 

Advertising is a form of persuasion.


(A) I don't have to be the first to break it to you that this is not the main concern of an advertising copywriter for a home appliance company. He's unlikely to suffer a single pang of failure upon finding out that, by buying the washer in his beautifully-crafted ad, you've passed up a wiser purchase. 1

(B) If you really want to read something that's written with your best interests in mind, you pick up a copy of Consumer Reports ― you don't go flipping through Good Housekeeping to find that dishwasher ad you saw last week. When you do read the ad, you take it for granted that any comparisons it makes to the competitions are not necessarily "fair and balanced." 2

(C) This means that from square one, your goals and interests are often very different from those of the advertiser. Let's say your goal is to buy the best dishwashing machine you can afford. 0


① A-C-B ② B-A-C ③ B-C-A ④ C-A-B ⑤ C-B-A

 


41. 20수특 10-1

 

Yale psychologist Irving Janis showed that just about every group develops an agreed-upon view of things — a consensus reality, the "PC" or politically correct view.


(A) In his classic book, Groupthink, Janis explained how panels of experts made enormous mistakes. People on the panels, he said, worry about their personal relevance and effectiveness, and feel that if they deviate too far from the consensus, they will not be taken seriously. 1

(B) Any evidence to the contrary is automatically rejected without consideration, often ridiculed, and may lead to exclusion of the person presenting the un-PC data. So group members are careful not to rock the boat by disagreeing with the consensus — doing so can seriously damage their standing. 0

(C) People compete for stature, and the ideas often just tag along. Groupthink causes groups to get locked into their course of action, unable to explore alternatives, because no one questions the established course. The more cohesive the group, the greater the urge of the group members to avoid creating any discord. 2


① A-C-B ② B-A-C ③ B-C-A ④ C-A-B ⑤ C-B-A

 


42. 20수특 10-2

 

The temperature of 54°F appears to be a magic threshold for several species.


(A) Because to really make a decent sound, the air temperature must be at least 54°F. If it is cooler, you'll barely hear a squeak from these tiny musicians.1

(B) For example, at the height of summer, billowing meadows full of grasses and herbs are the habitat for grasshoppers and crickets, which provide an orchestral backdrop with their chirping. However, this soundscape is by no means constant. 0

(C) As cold-blooded creatures, grasshoppers can't regulate their own body temperature, and only really get going when it's warm enough. Their body movements become faster with rising temperatures, resulting in ever more rapid vibrations of the legs and wings, which produce the chirping sound, depending on the type. This also changes the frequency of the tone produced: the warmer it is, the higher the pitch. 2


① A-C-B ② B-A-C ③ B-C-A ④ C-A-B ⑤ C-B-A

 


43. 20수특 10-3

 

In America we have developed the Corporation Man.


(A) In the areas of management, sales, and public relations, the position of the corporation man is secure only from one stockholders' meeting to the next; a successful rebellion there may sweep out whole cadres of earnest men and replace them with others. 2

(B) His position in the pyramid of management is exactly defined by the size of his salary and bonuses. The pressures toward conformity are subtle but irresistible, for his position and his hopes for promotion are keyed to performance of duties, activities, and even attitudes which make the corporation successful. 1

(C) His life, his family, and his future lie with his corporation. His training, his social life, the kind of car he drives, the clothes he and his wife wear, the neighborhood he lives in, and the kind and cost of his house and furniture are all dictated by his corporate status. 0


① A-C-B ② B-A-C ③ B-C-A ④ C-A-B ⑤ C-B-A

 


44. 20수특 10-4

 

Unfortunately, there are some social scientists who refuse to admit the limitations of their field of study.


(A) The use of mathematical techniques is not an end in itself but only a means to an end, namely, the discovery of what's true about the material world). The use of numbers is one way to be more precise in our effort to rationally understand causes. 2

(B) We cannot really quantify prejudice or love, for instance. When all is said and done, such attempted quantification is in vain. What is often forgotten, even in the physical sciences, is that science is not primarily a matter of quantification. 1

(C) They push hard to make social science imitate physical science. This is usually done by the use of all sorts of numbers, tables, charts, and graphs in order to give the impression of a profound quantification of the subject matter. Now, as a matter of fact, some things can be quantified and some things cannot. 0


① A-C-B ② B-A-C ③ B-C-A ④ C-A-B ⑤ C-B-A

 


45. 20수특 10-5

 

Culture consists of the linked stock of ideas that define a set of commonsense beliefs about what is right, what is natural, what works.


(A) Today, many serious-minded citizens seek to make important issues out of the potential disappearance of the whooping crane, the timber wolf, and other exotic creatures." 2

(B) These commonsense beliefs are not universal, but are instead typically bounded by time as well as by space. Today's orthodoxy may be the heterodoxy of yesterday and tomorrow. 0

(C) Although cultural change is not usually perceptible from day to day, when we look over a longer time span it becomes apparent that even the most fundamental assumptions about morality and the standards by which quality of life should be evaluated are subject to change. In his 1972 paper, Anthony Downs offers a vivid illustration of the extent of cultural change with his observation that "One hundred years ago, white Americans were eliminating whole Indian tribes without a blink. 1


① A-C-B ② B-A-C ③ B-C-A ④ C-A-B ⑤ C-B-A

 


46. 20수특 10-6

 

There is the question of the innate musicality of humans.


(A) Not all can attain professional proficiency;. but then, although all humans can learn to speak, not all can become great orators. 1

(B) We know that all normal humans inherit the ability to learn language — it is somehow "hardwired" in the human brain — but whether the same is true of music, whether all humans are basically musical is not clear, in part because cultures differ so much in their conception of "singing." Yet it seems likely that all humans can learn to sing minimally, to beat rhythms accurately, and to recognize simple pieces. 0

(C) The world's societies differ in the degree to which they encourage individuals to participate in music. In some rural societies, most people are considered about equally good at singing, and everyone participates in music-making at public events. In many urban societies, musical participation is largely limited to listening to live music and even more to recordings, whereas performance is left to professionals. 2


① A-C-B ② B-A-C ③ B-C-A ④ C-A-B ⑤ C-B-A

 


47. 20수특 10-7

 

Graham Allison intriguingly laid out his ideas about the Thucydides Trap, tensions between an established power and a rising one, in an essay for The Atlantic.


(A) As part of the Thucydides Trap Project, a team under Allison's direction examined sixteen cases where a rising power challenged an established one and determined the outcome of such challenges. The results are disturbing. Fourteen out of sixteen cases resulted in war. 0

(B) Both Taiwan and Japan have the potential to draw the US closer to war with China. Arguably, the US's allies might play a bigger role in the Thucydides Trap than fear of a rising power. This results in a situation where a great deal of effort is required to escape the Thucydides Trap. 2

(C) The worrying factor, according to Allison, is the fact that normal events or 'standard crises' that can otherwise be resolved, nevertheless trigger war. In the case of Athens and Sparta, it was the actions of smaller allies that drew them closer to war. This can easily happen in East Asia. 1


① A-C-B ② B-A-C ③ B-C-A ④ C-A-B ⑤ C-B-A

 


48. 20수특 10-8

 

Mummification in Ancient Egypt was developed in response to a gradual change in the burial preferences of its deceased.


(A) The physical features of the body would be retained, and this lifelike appearance of the corpse may have supported the belief of an afterlife. As burial practices became more sophisticated, with the construction of elaborate tombs and monuments to the dead, the bodies of the deceased were no longer buried in the desert sand. 1

(B) However, as belief in the afterlife and rebirth was fundamental to Egyptian burial practices, mummification was developed to artificially preserve the body in readiness for the journey to the underworld and to be judged by Osiris. 2

(C) The artificial preservation of bodies, both human and animal, was practised in Egypt from about 2686 BC until the beginning of the Christian era. The earliest Egyptians were buried in the sand, typically in the foetal position to reduce the size of the hole that would need to be dug, and the hot, dry climate would dehydrate the body. 0


① A-C-B ② B-A-C ③ B-C-A ④ C-A-B ⑤ C-B-A

 


49. 20수특 11-1

 

Fos Whitlock was a successful businessman.


(A) He asked me to talk to his son, Brant. Brant was about to graduate from college and was obsessed with the idea that he had the potential to be a professional golfer. Fos asked me to meet with Brant, obviously hoping that I would steer him toward giving up his dream of being a professional golfer. 0

(B) What was worse: The possibility of risking some money by supporting his son or running the risk of his son's lifelong enmity? To his great credit, Fos did an about-face and made what I think was the right choice. 2

(C) After seeing Brant, I met with Fos and suggested that he had too much to lose. If Brant were deprived of the chance to see if he had the right stuff to become a pro golfer, he would never know if he could have been successful. He would then hate his father for the rest of his life for depriving him of this opportunity. 1


① A-C-B ② B-A-C ③ B-C-A ④ C-A-B ⑤ C-B-A

 


50. 20수특 11-2

 

"Do you like the garden, Ms.


(A) Danby?" Charlotte spoke softly, her voice no more than a dry whisper, but Sarah had no difficulty hearing her. The accent was more noticeable than it had first been on the telephone. "Yes" she said with a thin smile. "I like it very much." 0

(B) Once inside, was instantly drawn to the artful blend of fine decoration in this private sector of being the house. It was just like the garden, filled with simplicity and colour, much like the former owner, she suspected. Each piece of furniture filled its rightful place with pride. The paintings on the walls, every one obviously a masterpiece, were each unique. She recognised a few celebrated names, which meant the collection had grown over the years. 2

(C) Charlotte smiled appreciatively. She pulled open the door and gestured with her hand. "Please, won't you come in?" The smile instantly vanished. Sarah allowed Charlotte to lead the way. 1


① A-C-B ② B-A-C ③ B-C-A ④ C-A-B ⑤ C-B-A

 


51. 20수특 11-3

 

There was Hungry Anderson, who was known to be a tight man with a dollar.


(A) He was called Hungry Anderson from that day on, and people began to say he was a miser. To prove that he wasn't, he bought a shiny Chalmers automobile, but his instincts were too strong for him. He kept the car in a shed in town and came in with his horse and buggy, motored about town, put up the car, and trotted back to his farm. 2

(B) He and his wife lived about a mile out of town. He got his name on an occasion when he had a carpenter working on the roof of his house. 0

(C) At noon, it took the carpenter about six or seven minutes to get down off the roof, and by the time he did, Hungry had eaten his lunch. He explained that when the carpenter was late, he had thought he didn't want to eat. 1


① A-C-B ② B-A-C ③ B-C-A ④ C-A-B ⑤ C-B-A

 


52. 20수특 11-4

 

The Prussian king, Wilhelm I, was a conventional man, and not particularly clever or insightful—.


(A) in other words, the opposite of his minister, Bismarck. He disliked much of what Bismarck did and how he did it. Yet at some level, the king recognized that he and his dynasty needed Bismarck even though, as Wilhelm once mildly complained, "It's hard to be Kaiser under him." 0

(B) He frequently threatened to resign. In the end it was always Wilhelm who backed down saying, "Bismarck mustn't think of resigning." Wilhelm wrote to him after one scene:. "It is my greatest happiness to live with you and thoroughly agree with you!" 2

(C) Since the monarch had the final say over foreign and defence policy, and governments answered only to him, and not the other way around, Bismarck only in the name of Wilhelm was able to exert great control over domestic and foreign affairs. The two men's relationship was marked by terrible arguments. Bismarck would come down with severe headaches and fits of vomiting and claim that he was dying. 1


① A-C-B ② B-A-C ③ B-C-A ④ C-A-B ⑤ C-B-A

 


53. 20수특 12-1

 

You may have noticed that people differ in the schemas they tend to use when evaluating others.


(A) As these examples illustrate, the role of the evaluator or the context in which a target person is encountered often influences which traits or schemas are used. 1

(B) College professors are often concerned with whether someone is smart, sales managers with whether someone is persuasive, and those involved in the entertainment business with whether someone has charisma. 0

(C) But sometimes the schema is simply determined by habit: if a person uses a particular schema frequently, it may become chronically accessible and therefore likely to be used still more frequently in the future. A frequently activated schema functions much like a recently activated one: its heightened accessibility increases the likelihood that it will be applied to understanding a new stimulus. 2


① A-C-B ② B-A-C ③ B-C-A ④ C-A-B ⑤ C-B-A

 


54. 20수특 12-2

 

Not everyone comes to see the game.


(A) This very point was elaborated by William McDougall in a book published in 1908, in which he developed his case for the existence of a gregarious instinct in humans. The question he posed at the turn of the century certainly argues convincingly for the view that we are social creatures. 1

(B) For some, the contest merely provides the setting and opportunities for the expression of other motives. The social contact provided by the crowd itself suggests a reason for people to attend. 0

(C) However, additionally labeling the observed behavior as an "instinct" creates a tautology that adds nothing to our understanding. McDougall asks his readers, "What proportion of the ten thousand witnesses of a football match would stand for an hour or more in the wind and rain, if each man were isolated from the rest of the crowd and saw only the players?" We would guess very few in 1908, fewer today. 2


① A-C-B ② B-A-C ③ B-C-A ④ C-A-B ⑤ C-B-A

 


55. 20수특 12-3

 

There is a widespread belief that creativity is best served through inner peace, stillness, and calmness.


(A) The way she put it to me was, "I have ninety minutes when Sam is napping, and I run to the computer and write like crazy. I'm totally focused." Turns out, my colleague is onto something. In fact, it is better to be aroused when attempting to think creatively. 2

(B) The result? She became prolifically productive. In her words, she was "wired." 1

(C) One of my colleagues was convinced that her own creative writing was best when she had no distractions, quietly sipping tea in a peaceful setting. However, after three months of such languid writing days, she produced nothing that she was proud of. Shortly thereafter, her first baby was born and her schedule went from long, open, peaceful, unstructured days to tightly orchestrated, minute-by-minute slots tightly orchestrated, minute-by-minute slots, punctuated by extreme activity. 0


① A-C-B ② B-A-C ③ B-C-A ④ C-A-B ⑤ C-B-A

 


56. 20수특 12-4

 

During a particularly trying time early in my sales career, a sales manager gave me a poster that read, "If it is to be, it is up to me."


(A) You cannot authoritatively impose effective change upon any society;. rather, it must be a choice that is felt deep within the consciousness of its inhabitants. 1

(B) I realized within that moment that if any changes or improvements were going to occur in my outside world, they needed to begin within my inside world. In life, all meaningful or macrocosmic change within society begins on a microcosmic level — that's you and me. 0

(C) Collectively, society's individuals must band together and say with one voice, "We're mad as hell and we're not gonna take it any more." For this to occur, however, one person must be willing to take the reins of change, with all of the risk those reins entail, and with an iron resolve, lead the charge of effective change. 2


① A-C-B ② B-A-C ③ B-C-A ④ C-A-B ⑤ C-B-A

 


57. 20수특 12-5

 

According to Greek mythology, the Oracle at Delphi was consulted to gauge the risk of waging a war.


(A) The opinions of everyone surveyed are summarized in a report and returned to the respondents, who then have the opportunity to modify their opinions. Because the written responses are kept anonymous, no one feels pressured to conform to anyone else's opinion. If people change their opinions, they must explain the reasons why; if they don't, they must also explain why. 1

(B) In modern times, the term Delphi refers to a group survey technique for combining the opinions of several people to develop a collective judgment. The technique comprises a series of structured questions and feedback reports. Each respondent is given a series of questions (eg, what are the five most significant risks in this project﹖), to which he writes his opinions and reasons. 0

(C) The process continues until the group reaches a collective opinion. Studies have proven the technique to be an effective way of reaching consensus. 2


① A-C-B ② B-A-C ③ B-C-A ④ C-A-B ⑤ C-B-A

 


58. 20수특 12-6

 

What is the basic idea of sociology?


(A) Then, too, neighbors of theirs — white children — had been their friends when they were four and five years old, but by the time they were fourteen and fifteen a barrier had gone up between them. My black undergraduates could see that this racial bias was hardly innate; rather, it showed that social structure affects how people think. Hence they were open to the sociological perspective. 2

(B) It is this: Social structure pushes people around, influences their careers, and even affects how they think. My Tougaloo College students readily understood that social structure pushed people around. 0

(C) Not one of their parents was an architect, for example, because no school in the Deep South in their parents' generation both taught architecture and admitted African Americans. So my Tougaloo students knew how social structure might influence careers. 1


① A-C-B ② B-A-C ③ B-C-A ④ C-A-B ⑤ C-B-A

 


59. 20수특 12-7

 

According to the scholars of the Indian traditions (Vedas), the origin of religion was to be sought in the impressions that natural phenomena made upon man.


(A) The primary stage of religion was not due to the religious nature of man, or to the 'need of the human heart', as O. Müller expressed it in his book, but to man's elementary capability of seeing personal figures in the impersonal phenomena of his surroundings. What finally led to the formation of religion was, thus, the elaboration of a nature mythology, and the veneration of the respective figures. 1

(B) The mythological figures were thought to be personifications of natural objects. The impressive manifestations of nature stimulated the personifying fantasy of man. 0

(C) The beginning of religion was the worship of many natural objects, with a predominance of such phenomena as the sun, the sky, thunderstorms, lightning, rain, and fire. 2


① A-C-B ② B-A-C ③ B-C-A ④ C-A-B ⑤ C-B-A

 


60. 20수특 12-8

 

A well-functioning democracy requires a media system that provides diverse sources of information and encourages civic participation.


(A) The first step is to break up the concentration of media power. Let's give control to a greater number of smaller companies that could legitimately compete with a broader range of information. 1

(B) The government once considered the airwaves such an integral part of our democracy that politicians decided the public should own and control them. It is time for the public to reclaim the responsibility of producing quality media from the corporate conglomerates. 0

(C) Also, we must create and maintain a noncommercial public media system as well as independent alternative media that exist outside the control of transnational corporations and advertisers. The rise of independent political blogs, and that of alternative podcasts, radio networks and television channels are all examples of citizens rising up to take back control of our media. 2


① A-C-B ② B-A-C ③ B-C-A ④ C-A-B ⑤ C-B-A

 


61. 20수특 12-9

 

Consider the question often asked by scientists, including even those who are well disposed toward animals, as to whether the hen suffers from what she has never known.


(A) Somewhat to her surprise, she found that hens who had been confined to battery cages, cages no larger than a sheet of newspaper, when given the choice between a small outside run with grass and the cages they had known all their lives, chose to stay in the cages. 1

(B) The Oxford researcher Marian Dawkins conducted experiments to determine what hens felt about their homes. 0

(C) And fowl expert Valerie Porter points out that chickens taken from a battery cage "will be in a considerable state of what you might call cultural shock if they are deprived of the only type of environment they have ever known. In fact, they will curl up in a corner in a state of terrified agoraphobia and it will take a great deal of time and patient understanding to rehabilitate them to real life." 2


① A-C-B ② B-A-C ③ B-C-A ④ C-A-B ⑤ C-B-A

 


62. 20수특 12-10

 

The cyclical nature of success and failure has been well established in the field of modern bridge design and engineering, in which experience spans about two centuries.


(A) Failures are part of the technological condition. 2

(B) The creative and inherently human process of design, upon which all technological development depends, is in effect timeless. What this means, in part, is that the same cognitive mistakes that were made three thousand, three hundred, or thirty years ago can be made again today, and can be expected to be made indefinitely into the future. 1

(C) Unfortunately, the lessons learned from failures are too often forgotten in the course of the renewed period of success that takes place in the context of technological advance. This masks the underlying fact that the design process now is fundamentally the same as the design process thirty, three hundred, even three thousand years ago. 0


① A-C-B ② B-A-C ③ B-C-A ④ C-A-B ⑤ C-B-A

 


63. 20수특 12-11

 

The popular press deems reshoring to be "bringing manufacturing back home..." from a current location that is not home.


(A) GE, for example, reshored its appliance manufacturing from its production facility in China to the US in its own plant to meet the US demand, whereas the US-based Vaniman Manufacturing decided to no longer buy sheet metal fabrication from an overseas supplier and to instead source from a local supplier to meet demand in the US. 1

(B) The term is agnostic as to whether the manufacturing being brought home occurred in a wholly owned facility in an offshore location or in the factory of an offshore supplier. 0

(C) Both would be considered reshoring more precisely, reshoring back to the US. Reshoring is fundamentally concerned with where manufacturing activities are to be performed, independent of who is performing the manufacturing activities in question — a location decision only, as opposed to a decision regarding location and ownership. 2


① A-C-B ② B-A-C ③ B-C-A ④ C-A-B ⑤ C-B-A

 


64. 20수특 12-12

 

Mark Leary and his colleagues led participants to believe that they were to perform a group task.


(A) Our momentary feelings of self-worth strongly depend on the extent to which others approve of us and include us. 2

(B) The experimenter ignored the participants' actual preferences and randomly assigned some participants to a condition in which they had supposedly been passed over by the others and had to work alone, and other participants to a condition in which they were in high demand by others and worked with a group. Participants in the work-alone condition, who believed they had been excluded, reported lower levels of self-esteem than those involved by the group. 1

(C) Before the task, each participant was asked to write an essay about "what it means to be me" and "the kind of person I would most like to be." The experimenter then gave each person's essay to other participants (in another location) who were asked to indicate who they would like to work with in the group setting. 0


① A-C-B ② B-A-C ③ B-C-A ④ C-A-B ⑤ C-B-A

 


65. 20수특 13-1

 

Movies and cartoons sometimes portray scientists as loners in white lab coats, working in isolated labs.


(A) In reality, science is an intensely social activity. Most scientists work in teams, which often include both graduate and undergraduate students. 0

(B) And, in fact, research papers aren't published until they are vetted by colleagues in what is called the "peer review" process. Most of the examples of scientific inquiry described in science textbooks for college students, for instance, have all been published in peer-reviewed journals. 2

(C) And to succeed in science, it helps to be a good communicator. Research results have no impact until shared with a community of peers through seminars, publications, and websites. 1


① A-C-B ② B-A-C ③ B-C-A ④ C-A-B ⑤ C-B-A

 


66. 20수특 13-2

 

The immune system is the body's defense against foreign invaders such as bacteria.


(A) We know that the immune system begins to decline after adolescence, and the weakening of immune function is linked to age-related vulnerability. 1

(B) According to the autoimmune theory of aging, the system may eventually become defective and no longer distinguish the body's own tissues from foreign tissues. The body may then begin to attack itself, as suggested by the rising incidence of autoimmune diseases with advancing age. 2

(C) The immune system protects and preserves the body's integrity, and it does this by developing antibodies to attack hostile invaders. 0


① A-C-B ② B-A-C ③ B-C-A ④ C-A-B ⑤ C-B-A

 


67. 20수특 13-3

 

When a painter is working for a patron, as Leonardo da Vinci did for the Medici, there can be emotional communication resulting from the artist's intention to produce something that the patron will appreciate and enjoy.


(A) So painting is a social process involving the communication of emotional judgments of the artist to the people who view it. Painters cannot expect viewers to appreciate their work with exactly the same perceptions and emotions that went into their creation, but they can hope to generate some approximation of these. 2

(B) Some artists often take pains concerning the framing and presentation of their work for the benefit of the viewers of their art, such as when Mark Rothko was highly particular about the lighting of his paintings. 1

(C) Even Vincent van Gogh, who was isolated and sold few paintings in his life, cared about the reactions of his brother to the paintings that he produced. 0


① A-C-B ② B-A-C ③ B-C-A ④ C-A-B ⑤ C-B-A

 


68. 20수특 13-4

 

Salt was historically so costly and important in Europe that its consumption was linked to social status.


(A) Among the privileges granted to the elite was access to salt, placed in a container on the high table. This clear expression of social divide is reflected in the phrases "above the salt," referring to someone of high rank, and "below the salt," which means someone of lower rank or less socially acceptable. 2

(B) In the medieval world, with its rigid hierarchy, the way in which people dined and the food that they ate reflected their position in society. 0

(C) Royalty and nobility sat at the high table, positioned on a dais, while their social inferiors ate at lower tables below them. 1


① A-C-B ② B-A-C ③ B-C-A ④ C-A-B ⑤ C-B-A

 


69. 20수특 13-5

 

If you stop to think about all of the tiny parts that make up any one thing, you could be thinking about it for quite some time.


(A) A computer, for example, looks like a big hunk of metal and glass, but if you look closer, moving your way down from the big things to the tiniest parts that make the computer up, you start to see how a computer is really many smaller things combined to make something bigger. 1

(B) A computer isn't just a computer: It's metal, glass and plastic, screws, springs, wires, copper, ink, electricity, and so much more. How those small parts come together to make the bigger parts is what matters, but how often do we take time to think about that fact? 2

(C) There are almost an infinite number of things that influence and impact any larger thing, right down to atoms and smaller molecules. 0


① A-C-B ② B-A-C ③ B-C-A ④ C-A-B ⑤ C-B-A

 


70. 20수특 13-6

 

Taub and his co-workers have already found strong evidence that the brain can be healed by its own plasticity.


(A) Similar results have been produced with speech impairment and even dyslexia. Although this research is still in its early days, Michael Merzenich of the University of California, San Francisco, believes the brain's plasticity may really enable us to protect ourselves against age decline. 2

(B) Amazingly, some people who have lost the use of an arm through a stroke have been trained to use it again by having the good arm restrained and being forced to use the apparently dead one, a technique called 'constraint-induced (CI) movement therapy'. 0

(C) Even though the part of the brain that controlled the arm was damaged, the CI movement therapy forced the brain to open up new areas in order to move the dead limb. 1


① A-C-B ② B-A-C ③ B-C-A ④ C-A-B ⑤ C-B-A

 


71. 20수특 13-7

 

When you watch a documentary on a small standard (4 x 3) video screen and then see it on an equally small wide-screen (16 x 9) receiver, you probably notice relatively little energy change.


(A) Even if you use proper conversion methods for aspect ratios, squeezing such large images into the small video screen reduces not only image size but also, if not especially, event energy. Close-ups, inductive sequencing, and a dense audio track help generate some aesthetic energy on the small video screen, but they cannot compete with the large movie images and high-volume surround sound. 2

(B) This is why some movies that emphasize landscape (from actual landscapes to spaceships or battle scenes) must be seen on the large screen to feel the total impact. 1

(C) When you switch from the small screen to a large HDTV screen, however, the energy change is readily apparent. 0


① A-C-B ② B-A-C ③ B-C-A ④ C-A-B ⑤ C-B-A

 


72. 20수특 13-8

 

Revegetation techniques are put in place with an understanding of the resultant effects on wildlife.


(A) Berry-producing plants should not be selected as they may prove attractive yet fatal to wildlife due to the close proximity of traffic. 2

(B) For instance, the planting of tree lines and woodland areas within roadside landscape may naturally force birds to fly higher above roads when crossing between forest edges, as was shown in the Netherlands. Verges should be widened and landscape planting on bends or curves on a road should be set back, to improve visibility and to discourage crossings by wildlife. 0

(C) In contrast, along straight sections of road carriageway, vegetative cover should extend as close to the road as permitted by road construction and safety standards. Landscaping of centre medians, junctions, roundabouts or interchanges should be sensitive to wildlife. 1


① A-C-B ② B-A-C ③ B-C-A ④ C-A-B ⑤ C-B-A

 


73. 20수특 14-1

 

A diverse garden will become a habitat for a variety of bird species.


(A) Or rather, not a time, but a certain position of the sun. These are relative to sunrise, a precisely definable event. 1

(B) Unfortunately, it changes constantly, as throughout spring, the sunrise takes place a little earlier each day, until the summer solstice on June 21, when it starts getting later again. So, bird song is perhaps not ideal as a genuine replacement for your watch, although each species tends to observe its relative time slot, day by day, with astonishing accuracy. 2

(C) But if all the birds were to sing at the same time, each one's melody would be drowned out in the cacophony of voices. In order for each singer to be adequately appreciated by his rivals or his sweetheart, each species focuses on a specific time in the morning. 0


① A-C-B ② B-A-C ③ B-C-A ④ C-A-B ⑤ C-B-A

 


74. 20수특 14-2

 

With reference to the variable of intensity, it is almost stating the obvious to say that bright lights or loud sounds can attract our attention.


(A) The experiment conducted by LaBarbera and MacLachlan exposed people to five radio commercials that were either normal or time-compressed on the order of 130%. These time-compressed commercials were not "sped up" by making the tape run faster; that would also increase the frequency of the auditory signal, and make the announcer sound like a high-pitched Mickey Mouse. 1

(B) Rather, the time-compression technique involves the shortening of pauses between words, and the reduction of the length of vowel sounds. This results in a message that runs more quickly, without changing the pitch of the announcer's voice. These researchers found that the time-compressed advertisements elicited more interest and better recall than the normal ads. 2

(C) We have all been exposed to countless examples of commercial advertisements that seem to be based solely upon this premise. One unusual example of the use of intensity in advertising contexts is the practice of time-compressed speech in radio commercials. 0


① A-C-B ② B-A-C ③ B-C-A ④ C-A-B ⑤ C-B-A

 


75. 20수특 14-3

 

Recycling means recovery and reprocessing of waste materials for use in new products.


(A) Waste radionuclides recovered from contaminated materials are difficult to recycle in new devices or compounds. Hence, even materials that contain large amounts of radioactive constituents (eg sealed radioactive sources as used in industry, medicine and research) often are immobilized (conditioned) and safely stored and disposed of rather than recycled. 2

(B) Due to their inherent radiation, radionuclides are much more difficult to recover from contaminated materials. Recovery usually presumes concentration of species into a smaller volume even though this may result in more dangerous materials. 1

(C) Recycled waste can be substituted for raw materials, reducing the quantities of wastes for disposal as well as potential pollution of air, water and land resulting from mineral extraction and waste disposal. However, recycling has certain limitations when applied to radioactive materials. 0


① A-C-B ② B-A-C ③ B-C-A ④ C-A-B ⑤ C-B-A

 


76. 20수특 14-4

 

Glass affords transparency.


(A) As a result, birds often try to fly through windows. And every year, numerous people injure themselves when they walk (or run) through closed glass doors or large picture windows. If an affordance or anti-affordance cannot be perceived, some means of signaling its presence is required. 2

(B) To be effective, affordances and anti-affordances have to be discoverable — perceivable. This poses a difficulty with glass. The reason we like glass is its relative invisibility, but this aspect, so useful in the normal window, also hides its anti-affordance property of blocking passage. 1

(C) At the same time, its physical structure blocks the passage of most physical objects. As a result, glass affords seeing through, but not the passage of air or most physical objects (atomic particles can pass through glass). The blockage of passage can be considered an anti-affordance — the prevention of interaction. 0


① A-C-B ② B-A-C ③ B-C-A ④ C-A-B ⑤ C-B-A

 


77. 20수특 14-5

 

In cultural contexts that require polite formulas rather than honest words, language may lose almost all its communicative function, and here food often takes over the role.


(A) More information about the actual social transactions going on at the dinner is transmitted by food choice and distribution. The most valued guest often gets the choicest portion, and so on down. 1

(B) Other aspects of the ritual may communicate even more. Everyone carefully observes who sits next to the host, who sits at the host's table, who is the first one to be greeted, who is served first, who gets the best piece of meat, or who is urged to have seconds. 2

(C) In formal dinners around the world, it is not usually appropriate to send the important social messages verbally. Words are bland and carefully chosen. 0


① A-C-B ② B-A-C ③ B-C-A ④ C-A-B ⑤ C-B-A

 


78. 20수특 14-6

 

Ritualistic behaviour designed to influence future events is not, it seems, limited to humans.


(A) Skinner's explanation for this phenomenon was that the accidental pairing of the release of food early on in the process with whatever the bird happened to be doing was enough to reinforce that particular type of activity. 2

(B) B. F. Skinner's classic research into 'superstition in the pigeon', conducted at Indiana University in 1948, supports this hypothesis. Skinner described an experiment in which pigeons were placed inside a box and were presented with a small piece of food once every fifteen seconds, regardless of their behaviour. 0

(C) After a few minutes the birds developed various little unusual rituals, such as walking round in circles, moving their heads up and down and so on. The pigeons appeared to have concluded that their little routines were causing the release of the food even though in reality there was no relationship whatsoever. 1


① A-C-B ② B-A-C ③ B-C-A ④ C-A-B ⑤ C-B-A

 


79. 20수특 14-7

 

Within the domain of concrete entities, objects and substances have very different properties.


(A) When we say that two objects are identical or the same, we are referring to two objects in their entirety and not to two distinctive parts of a single object. In contrast, when we say that two substances are identical or the same, there is no notion of wholeness. 1

(B) Substances are of scattered existence, and there is no such thing as whole sand, whole water, or whole clay. This portion of sand is identical to that portion of sand, as long as the two portions consist of the same physical constituents. This difference in identity or sameness between objects and substances leads to fundamentally different extension principles for determining category membership across the two ontological kinds. 2

(C) Objects are individuated, whereas substances are nonindividuated. Thus, the two kinds of entities have fundamentally different criteria for the notion of identity or sameness. 0


① A-C-B ② B-A-C ③ B-C-A ④ C-A-B ⑤ C-B-A

 


80. 20수특 14-8

 

Although the efforts to revive dying languages are admirable, the challenges facing those who would reverse the extinction process are intimidating.


(A) The recent revolution in communications technology has provided powerful tools (through the airwaves and cyberspace) for the spread of mainstream Western culture and language. Yet, for some endangered languages, the tide is changing through the digital revolution. 1

(B) Not all of the extinctions are the direct result of hostility and repression from a dominant government, as was the case with American Indians throughout most of U.S. history. But where brutal repression failed to make indigenous languages and culture extinct, intense globalization since the 1980s has been more successful. 0

(C) As Rosenberg points out, digital technology, discussion groups, software companies, and apps are lifelines for language preservation for minority and endangered language communication needs. At one time technology forced some language speakers to adopt the dominant language of their community or nation. Now, new tools create the possibility for revitalizing languages and retaining language speakers of endangered languages. 2


① A-C-B ② B-A-C ③ B-C-A ④ C-A-B ⑤ C-B-A

 


81. 20수특 15-1

 

Several studies have shown that individuals who are ostracized, excluded, or rejected by others behave in ways that will increase their chances of eventually becoming accepted.


(A) Similarly, Williams, Cheung, and Choi observed that ostracized individuals were more likely than others to conform to the opinions of other people. Thus, these studies show that in response to social rejection, people seek to reconnect themselves with their social worlds. 1

(B) In addition, Gardner, Pickett, and Brewer found that individuals who experience social rejection are more likely to remember socially relevant information. Thus, belongingness needs appear to guide the processing and retention of information that is consistent with one's motive. 2

(C) These behaviors range from working harder in group settings, to conforming to group perceptions, or being more sensitive to information about others. For example, Williams and Sommer found that women responded to ostracism by increasing their efforts on a subsequent group task. 0


① A-C-B ② B-A-C ③ B-C-A ④ C-A-B ⑤ C-B-A

 


82. 20수특 15-2

 

David Rock, author of Your Brain at Work, has described in fascinating detail the intricate mechanics of the brain on creativity and stress.


(A) In this condition, we feel curious, open-minded, and interested in what we are doing ― all excellent qualities for thriving on the job. Neuroscience tells us that creativity and engagement are essential to making people happier. 1

(B) We know, for example, that self-described happy people have more new ideas. We now know that stress decreases our cognitive resources, whereas mindfulness induces what is called a toward state in the brain, an openness to possibilities. 0

(C) But the technological onslaught of today's world can also become highly stressful. Long hours, hard work, and high pressure are made worse by our being permanently plugged in. Though the introduction of laptop computers, high-speed Internet, mobile technology, and social media have wonderful advantages in how we connect, they also reinforce behaviors that shut down the toward state and set us on autopilot. 2


① A-C-B ② B-A-C ③ B-C-A ④ C-A-B ⑤ C-B-A

 


83. 20수특 15-3

 

For sea squirts, a two-part life cycle provides a quite obvious advantage.


(A) But then, if the young sea squirts immediately settled down to the bottom, the sea squirt colony would soon be so crowded that they would have to grow on top of each other. There would not be enough food to feed the huge crowds of sea squirts, all jammed into a small area. So instead, the tadpole-like swimming larvae of the sea squirts do not settle down immediately. 1

(B) They swim and drift with the ocean currents. By the time they are ready to change to adults and take up a place on the ocean bottom, they have been scattered over a wide area. 2

(C) Adult sea squirts live very nicely, attached to the sea bottom. All the food they need comes drifting to them in the ocean currents, and they never have to move. They have even solved the problem of getting together to mate by shooting their sperms and eggs out into the water. 0


① A-C-B ② B-A-C ③ B-C-A ④ C-A-B ⑤ C-B-A

 


84. 20수특 15-4

 

The halo effect causes one trait about a person to color your attitude and perceptions of all her other traits.


(A) If the first year of a relationship is deeply fulfilling and life-altering, it can take a long time to notice if things turn sour later. If you like specific aspects of an individual, the halo effect causes the positive appraisal to spread to other measurements and to resist attack. 1

(B) Even stranger, the more noticeable the aspect is when you form your first impression, the more difficult it becomes to change your attitude about that aspect. So, for example, if you are bowled over by the warmth and kindness of a coworker in your first week at a new job, you'll let him get away with a host of obnoxious behaviors later on, maybe even for years. 0

(C) Beautiful people seem more intelligent, strong people seem nobler, friendly people seem more trustworthy, and so on. When they fall short, you forgive and defend them, sometimes unconsciously. 2


① A-C-B ② B-A-C ③ B-C-A ④ C-A-B ⑤ C-B-A

 


85. 20수특 15-5

 

Divers working at high pressures underwater usually breathe "air" that is a mixture of oxygen and helium.


(A) In human bodies, the gas bubbles released in this way often get trapped in the joints, causing extreme pain for the diver. This pain often makes it impossible for the diver to straighten up, which is why this condition is aptly named the bends. 2

(B) Helium is substituted for nitrogen in this mixture because it is less soluble than nitrogen and therefore less likely to dissolve in the bloodstream. This offers better protection against one of the major hazards of diving, called the "bends." 0

(C) If a diver returns to the surface too quickly after a dive, the relatively lower pressure at the surface than deep underwater causes dissolved gases to bubble out of solution in the blood. The effect is similar to the frothing in a bottle of soda water when the cap is removed. 1


① A-C-B ② B-A-C ③ B-C-A ④ C-A-B ⑤ C-B-A

 


86. 20수특 15-6

 

In England in the early 1900s property owners whose land was being eroded by wave action clamored for the Government to take preventive action.


(A) It must be admitted, however, that the land lost probably was good cliffland on the open coast which disappeared in a spectacular way, whereas the land gained was low, sandy and not particularly valuable. Non-geologists are usually not aware that the very existence of a cliff is warning that erosional processes are at work, even though the changes seem to be very slow. 2

(B) After making a careful survey, the commission reported that over a period of thirty-five years England and Wales lost 4,692 acres and gained 35,444 acres, giving a net gain of nearly nine hundred acres a year. This finding seemed to prove that people whose land disappeared complained more loudly than those whose land was increasing. 1

(C) Their island was disappearing beneath the sea! They argued so loudly that a Royal Commission was appointed to study the matter. 0


① A-C-B ② B-A-C ③ B-C-A ④ C-A-B ⑤ C-B-A

 


87. 20수특 15-7

 

Solid objects cohere as wholes.


(A) Rather, the applied force is converted by the liquid into an isotropic one that will urge the liquid to move in any direction open to it, unconstrained by any tendency for the liquid body to cohere. Water will tend to leak from a pipe with equal facility in any direction and not just in the direction of the weight of the head of water bearing down on it. 2

(B) While their shapes can be distorted to some degree to the extent that they are elastic, in the main they accelerate or tend to move as wholes in the direction of an applied force. Bodies of liquid differ in this respect. 0

(C) They freely adapt their shape to a containing vessel or an immersed solid and will simply give way to a solid object moving slowly through them. If a liquid body is subject to a force it will not tend to move as a whole in the direction of that force. 1


① A-C-B ② B-A-C ③ B-C-A ④ C-A-B ⑤ C-B-A

 


88. 20수특 15-8

 

Subjectivity is an integral part of all art, and sometimes, even where it seems the least likely, one finds a comprehensive communication between the artist and the audience very difficult.


(A) Therefore, an audience can never be sure exactly what the artist had in mind. This holds true on all levels, and thus, perfect communication cannot occur between most artists and their audiences through their art alone. 2

(B) No one mistakes that this painting is the portrait of a woman; that much we know. However, the intriguing smile in this painting is interpreted in so many different ways, in terms of what state of the mind this smile depicts. 1

(C) For example, not just in abstract painting, but in the most straightforward painting. Just take one of the best-known paintings, the Mona Lisa, painted by Leonardo da Vinci. 0


① A-C-B ② B-A-C ③ B-C-A ④ C-A-B ⑤ C-B-A

 


89. 20수특 16-01

 

Recent research has shown that trees are not as passive as we long supposed.


(A) However, when the scientists simply broke off new growth without applying any saliva, all the beeches and maples produced were hormones to heal the damage as quickly as possible. 2

(B) To investigate this further, European scientists simulated attacks on small beeches and maples. Whenever a roe deer takes a bite out of the top growth of a young tree, it leaves a little saliva behind in the wound, and it soon became clear that wounded trees can clearly detect the presence of this saliva. 0

(C) To simulate browsing by roe deer, the researchers cut off buds or leaves and dripped roe deer saliva onto the damaged areas. What they noticed was that in response the little trees produced salicylic acid, which in turn led to an increased production of bad-tasting defensive compounds, which discouraged the roe deer from eating them. 1


① A-C-B ② B-A-C ③ B-C-A ④ C-A-B ⑤ C-B-A

 


90. 20수특 16-02

 

Historical linguists study the languages spoken today, and from them make estimates about the ancestral languages from which they descended.


(A) In genetics a person has more and more ancestors as one goes to earlier generations, while a language has a single ancestor at each stage. The "tree model" of languages presents the range of languages descended from an ancestor, and indicates relationships with other languages descended from the same ancestor. Because of the single-ancestor characteristics of the linguistic "tree model," language gives more evidence on path of early human migration than does genetics, because it allows for fewer possibilities. 2

(B) Where possible, linguists also work from written records on languages in earlier times. For linguistics (as for genetics), we assume that present data give us the remnants of earlier communities. 0

(C) But the definition of "earlier community" is different in each case. For language, it is assumed that each language has one parent. 1


① A-C-B ② B-A-C ③ B-C-A ④ C-A-B ⑤ C-B-A

 


91. 20수특 16-03

 

The more things change, the more they stay the same.


(A) For them, what they see around them is "normal," and they have nothing to compare it with until they start to learn about history. It has always been this way. Parents feel the changes in the world; kids don't. 1

(B) Parents often react defensively, and children do not understand what all the fuss is about. Negative parental reactions often originate in hostility toward change. Most adults tend to see their own formative years as normal and what comes afterward as a decline. The only constant is change, and parents and their children experience this in fundamentally different ways. 2

(C) One thing that has remained the same over the centuries is parental disdain for the new kind of world they see around them. This world is only new to us, and it's the only one our children know. They haven't lived long enough to see the kind of social change that has taken place in the last twenty or thirty years. 0


① A-C-B ② B-A-C ③ B-C-A ④ C-A-B ⑤ C-B-A

 


92. 20수특 16-04

 

Because scientific research is so often conducted in the interests of national defense or under the sponsorship of private firms that hope to profit from applications of the findings, the norms of common ownership and publication are often suspended.


(A) The public laboratory had, or claimed to have, no such business interest. Eventually the competing teams compromised and issued a joint publication of the map, but the controversy and legal battles over issuing patents for genetic material have continued. 2

(B) Such situations have led to innumerable conflicts in scientific circles. An outstanding example is the race to publish the complete map and inventory of the human genome ― all the sequences of human DNA that constitute our genes. 0

(C) In 2001, two rival groups raced to be the first to complete the research, one in the private sector, the Celera Genomics Corporation, led by J. Craig Venter, and the other a government-funded laboratory headed by Francis Collins. An argument raged over whether the private corporation was trying to establish patents on human genetic sequences, a violation of the norm of openness. 1


① A-C-B ② B-A-C ③ B-C-A ④ C-A-B ⑤ C-B-A

 


93. 20수특 17-12

 

Looking for patterns works because our memories are organized in terms of what psychologists call 'schema'.


(A) A schema is a familiar pattern of relationships stored in your memory. That way they form memories so strongly linked that they are recalled more or less as a single unit. This is so powerful that it doesn't just influence your way of remembering lists, it actually affects your entire way of thinking. In one experiment, chess grandmasters and masters were tested against ordinary chess players to see how accurately they could remember the position of 20 to 25 chess pieces placed randomly on a board after glancing at the board for 5 to 10 seconds. 0

(B) And if you can reduce complex inputs to simple chunks, you'll find you can think about them much more clearly and effectively. As a general rule, if you ever find yourself forgetting anything, it is not because your brain is declining or your memory is receding. It's simply because you are not using the correct memory technique to help you store and retrieve the information.2

(C) The masters and ordinary players were pretty similar in being able to remember the places of only 6 pieces. Yet if the pieces were arranged in the form of a game (unknown to anyone), the grandmasters and masters could suddenly remember all the positions, while the ordinary players could still manage only 6. It was clear that this was not simply a memory feat 一it was due to the grandmasters' and masters' ability to see the positions as a single chunk or schema. It is clear that the more you develop schema, or patterns, the better you'll remember things. 1


① A-C-B ② B-A-C ③ B-C-A ④ C-A-B ⑤ C-B-A

 


94. 20수특 17-34

 

Novelty compels both humans and animals to engage with the unfamiliar.


(A) Indeed, our strong desire for novelty has evolutionary roots, improving our survival odds by keeping us alert to both friends and threats in our environment. As new parents quickly learn, when given a choice, babies consistently look at, listen to, and play with unfamiliar things. One of my favorite moments from early parenthood was when I watched my infant son notice his hands for the first time. His discovery stands out as a metaphor for learning:. 0

(B) That is, the people who traveled the farthest from home may have had some biological propensity to experience mysterious new places. And yet, while we are born with a strong drive to seek novelty, this drive fades over time. As we grow older, other desires take over, like wanting more predictability. The organizations we build and join reflect this reality: paychecks at the same time each week or month, evaluations according to established processes, jobs that involve a known set of activities. 2

(C) His interest in what those strange, wonderful appendages could do was his first step toward controlling them. The preference for novelty is an efficient way for immature cognitive systems to process information, helping babies cope with changes to their environment before releasing their inner explorer. Interestingly, in human genetics, a preference for novelty has been linked to the migration of early humans to the far reaches of the earth. Recent studies have shown that human groups that migrated the farthest from Africa had more of the genes linked to novelty seeking. 1


① A-C-B ② B-A-C ③ B-C-A ④ C-A-B ⑤ C-B-A

 


95. 20수특 17-56

 

You've no doubt heard the old saying: "when the going gets tough, the tough go shopping!"


(A) Starving E. coli colonies (hyper-)mutate at a rate one thousand times greater than that which would normally be the case in a well-fed colony. You can even find certain hypermutators within a normal or well-fed colony, sitting there, ready to spring into a hypermutational mode at the first sign of serious stress. As Frank T. Vertosick so humorously put it: "these hypermutators are the Van Goghs of the microbial world—somewhat insane, but infinitely creative". 2

(B) In a sporting team, it has been called throwing away the game plan, and this is exactly what hypermutation involves. When a bacterial colony is in a critical situation ― when survival is on the line ― something very strange happens — the bacteria suddenly start mutating at an extraordinarily rapid rate. In so doing, they are consulting the microbial lending library, hoping that they can come up with a mutation that might get them out of their crisis. 1

(C) That is precisely what bacteria do when they find themselves in deep trouble. They go shopping for useful genes that can help to get them out of the mess. There comes a time in the life of any organism or organization, when it has to try something completely different. 0


① A-C-B ② B-A-C ③ B-C-A ④ C-A-B ⑤ C-B-A

 


96. 20수특 17-78

 

Social sanctions vary in degree of formalization.


(A) In most stable associations there are highly formal procedures, such as ceremonies for honoring those whose services are believed to have contributed to the well-being of the membership and for the discredit or exclusion of those whose activities have been considered harmful. In our society, for example, there are courts of law and means of judging criminals which are so complex that only specialists can understand them. Some sociologists attach great importance to such highly formalized sanctions and have even defined the organized group as one in which the social structure is protected and reinforced through formal sanctions. Such norms are without doubt controls on deviant behavior, but for most people the less formal sanctions, the spontaneous displays of approval or disapproval, prove more effective. 0

(B) Since roles consist of reciprocating claims and obligations, they cannot be maintained without the cooperation of others in complementary roles. When a person speaks to a colleague, he ordinarily has a minimal claim upon him to respond in some way. Others may, however, refuse to live up to their obligation to be polite as a way of indicating their disapproval. 2

(C) Those who are about to violate some rule are often stopped short by the show of displeasure on the part of others. Ridicule and gossip are especially effective. In some cases deviant parties may be excluded informally, even when they continue to retain membership in the group. Among the most effective of the informal sanctions is the deprivation of mutual services, the refusal of others to honor the claims of the violator's role. 1


① A-C-B ② B-A-C ③ B-C-A ④ C-A-B ⑤ C-B-A

 


97. 20수특 18-13

 

Cuoi was born to a poor family and he had to work at various jobs to provide for his family.


(A) One day, while gathering wood in the forest far from home, Cuoi came upon a tiger cub and picked it up. Then, he heard a frightful growl. It was the mother tiger. Cuoi threw the cub to the ground and scrambled in terror up into the branches of the nearest tree. A moment later the tigress came crashing through the underbrush and growled as she saw the body of her dead offspring. Cuoi, in his haste to escape, had thrown the cub to the ground with such force it had been killed. Then, a strange thing happened. The tigress walked to a nearby stream and gathered the leaves from a banyan tree. She chewed them into a pulp which she then applied to the head of the dead cub. 0

(B) Cuoi's mother paid no attention to this warning and continued to dump dirty water near the tree. One day the tree began to slowly pull itself from the soil and to fly up into the sky. Returning from his chores, Cuoi noticed this and grasped its roots to keep it from flying away. But he was not heavy enough to keep the tree on the Earth. Instead, he was carried with it into the sky. After many days, Cuoi and the tree reached a strange new world — the Moon. Cuoi planted the tree there and sat down to figure out a way out of his predicament;. but there was no solution. There on the Moon he has sat waiting, year in and year out, even until today. 2

(C) Immediately the young tiger jumped to its feet and ran about as if nothing had happened. When the tigress and her cub had disappeared, Cuoi went to the miraculous banyan tree and gathered a handful of its leaves. On the way home he came upon a dead dog lying by the side of the road. Cuoi then chewed the leaves into a pulp and applied it to the dog's head. After a few minutes the animal was restored to life;. it jumped to its feet. Cuoi went back to the banyan tree, uprooted it, and replanted it in his yard. He warned his mother never to dump dirty water where the tree was planted. "Otherwise," he said jokingly, "it will fly away into the sky." 1


① A-C-B ② B-A-C ③ B-C-A ④ C-A-B ⑤ C-B-A

 


98. 20수특 18-46

 

The older man, a prominent baseball figure named Branch Rickey, suddenly changed his matter-of-fact, businesslike expression and turned on the young African-American athlete.


(A) Moments earlier they had been discussing the possibility of a career in big-time baseball for the young African-American man, Jackie Robinson. The year was 1945, and African Americans were barred from the major leagues. Rickey had warned Robinson that he would need two things to succeed. These were the ability to play baseball with the best, and the ability to stand the massive resentment toward the first African-American player in major-league baseball. In testing Jackie's patience in the latter, Rickey had tried to demonstrate, through pretended anger, the hate Robinson would meet. "Do you think," said Rickey, referring to his dramatic and realistic portrayal of a racist, "that you can get through this kind of thing?" "I can," replied Robinson softly, "if I pray." 1

(B) Rickey's prediction of trouble ahead could not have been more accurate. There were boos in the stadium, constant insults, and opposing players using steel spikes to hurt him. Teammates refused even to give him the traditional handshake and cheer when crossing the plate after a home run. But Jackie endured. And after his winning the Rookie of the Year Award in 1947, fans became aware of the prayer-born patience that made him one of the great men of the game. No one should have to qualify for sainthood in order to play baseball, but he humbled himself. Both he and Rickey were later honored by being elected into baseball's Hall of Fame. 2

(C) "You're just a black man," he shouted, "so you'll never succeed in the big leagues!" The young man, stiff with hurt and surprise, clenched and unclenched his fists threateningly, but the older man moved closer and kicked him with hostility. Then it was over. The older man — the baseball-team owner — stepped back, and the two men studied each other silently. The younger man spoke: "Why do you have to say these things to me, Mr. Rickey?" Branch Rickey, owner of the Brooklyn Dodgers and respected figure in baseball, dropped the false pose he had taken, smiled, and gently touched the young man's shoulder. 0


① A-C-B ② B-A-C ③ B-C-A ④ C-A-B ⑤ C-B-A

 


99. 20수특 18-79

 

The abbot woke up early one morning.


(A) Nothing was unusual in that. But this morning he was awakened by the sound of something moving in the nearby shrine room. That was unusual because most of his monks would normally be practicing their morning "chanting" so he went to investigate. In the darkness he saw a silhouette of a hooded figure. It was a burglar. After a few moments of silence the abbot said kindly, "What do you want, my friend?" "Give me the key to the donation box," said the burglar. The abbot saw a weapon in his hand but felt no fear. He felt only compassion for the young man. "Certainly," he said, slowly handing over the key. As the thief hurriedly emptied the box of cash, the abbot noticed the robber's jacket was torn, his face pale and gaunt. "When was the last time you have eaten, dear boy?" asked the abbot. 0

(B) He was sentenced to ten years in jail. Just over ten years later, the same abbot awoke to the sound of someone in the shrine room. He got up to investigate and, saw the old burglar standing next to the donation box. "Remember me?" shouted the burglar. "Yes" sighed the abbot. "Here's the key." Then the burglar smiled, and said gently, "Sir, put away the key. I couldn't stop thinking about you all those long days in prison. You were the only person in my entire life who was kind to me, who actually cared about me. I have come back to steal again, but this time I have come to take your secret of kindness and inner peace. Please, make me your disciple." 2

(C) "Shut up!" barked the burglar. The abbot nicely responded, "You'll find some food in the cupboard next to the donation box. Help yourself." The thief paused a moment in confusion. He was taken aback by the abbot's consideration for his welfare. Still, he hurriedly filled his pockets with cash from the donation box and food from the cupboard. "And don't call the cops," he shouted. "Why should I call the police?" answered the abbot calmly. "Those donations are to help poor people like you, and I have freely given you the food. You have stolen nothing." A few days later, the abbot read that the burglar had been caught robbing another house. 1


① A-C-B ② B-A-C ③ B-C-A ④ C-A-B ⑤ C-B-A

 


100. 20수특 18-1012

 

It is something of a common idea among animal behaviorists that wild animals do not tolerate disabilities, and that animals who are unfortunate enough to be born with a physical disability or fall ill rarely last very long.


(A) One foot was missing and one of her legs was several inches shorter than the other. Concerned on the first night that Helen might become the object of contempt from the other hens and roosters, Sturla set up a special nest in the barn. But when she opened up the door the following morning, a triumphant Helen greeted her sitting proudly on the top perch. Blind and lame, she had somehow found this spot. Far from feeling contempt for Helen's disabilities, the other birds stood in a kind of admiration for her, and she lives to this day in complete harmony with the rest of the flock, grooming her feathers, enjoying the sun, and dust-bathing with pure delight. 2

(B) I am doubtful. Recent research on many species has shown that young animals born with serious disabilities are nevertheless able to live with the help of their mothers and sometimes other friends and relatives. This is particularly true of elephants but applies to many species. Indeed, animals may have no concept of "disability" in the way humans do. 0

(C) Inspiring in this instance is the account Kim Sturla gave of Helen, a completely tame hen who was found wandering the streets of San Francisco. She was totally blind, and dogs were attacking her when a homeless person took pity on the hen and rescued her. She was taken to the city's animal shelter, where a call was put through to Animal Place to see if they would be willing to give her a home. Helen was born with a condition called cryptophthalmos, meaning that her eyelids had never formed properly and therefore never opened. 1


① A-C-B ② B-A-C ③ B-C-A ④ C-A-B ⑤ C-B-A

 


101. 20수특 19-1

 

William Allingham, poet and editor, is known mainly through a small group of poems that regularly appear in anthologies of Irish verse.


(A) In his own day, however, Allingham was well known to poets such as Robert Browning, Dante Gabriel Rossetti, and Alfred Tennyson, and his work inspired later Irish poets, including W. B. Yeats and John Hewitt. He was born in Ballyshannon, County Donegal, Ireland and his first job was in the local bank, where his father was manager. 0

(B) He befriended the influential poet, critic, and editor, Leigh Hunt, to whom he dedicated his first book of poems in 1850. His most ambitious work was the verse novel, Laurence Bloomfield in Ireland, in which he tried to promote reform of land ownership and tenants' rights. 2

(C) In 1846, he took a post with the national excise service and spent the next twenty-four years working as a customs officer. He first visited England in 1843 and eventually settled there in 1863. 1


① A-C-B ② B-A-C ③ B-C-A ④ C-A-B ⑤ C-B-A

 


102. 20수특 19-2

 

The first tunnel built under the Thames was designed by Marc Isambard Brunel (1769—1849).


(A) When it opened, it was at first targeted by thieves who would lie in wait in the shadows for unsuspecting pedestrians. The East London Railway took over the tunnel in 1869, and it now carries the East London Underground line across the river. 2

(B) Once released, Brunel worked on his observation, and eventually constructed a huge drill with corkscrew blades that passed dug-up material down its length as it moved forward, which was used to bore a tunnel from Wapping to Rotherhithe. The tunnel took 18 years to build, and 10 men died during its construction. 1

(C) Brunel fled the French Revolution for New York and then London, and at one point found himself in debtor's prison. While in prison, he watched a shipworm bore a hole through a piece of wood by passing the chewed-up wood pulp through its body and excreting it as it progressed. 0


① A-C-B ② B-A-C ③ B-C-A ④ C-A-B ⑤ C-B-A

 


103. 20수특 19-3

 

At Greenwich in 1796, the astronomer Maskelyne dismissed Kinnebrook, his assistant, because Kinnebrook observed the times of stellar transits almost a second later than he did.


(A) His attention was called to the "error," and it would seem that he must have striven to correct it. Nevertheless, it increased during the succeeding months until, in January, 1796, it had become about eight tenths of a second. 1

(B) T hen Maskelyne dismissed him. The error was serious, for upon such observations depended the calibration of the clock, and upon the clock depended all other observations of place and time. 2

(C) Maskelyne was convinced that all through 1794 there had been no discrepancy between the two of them. Then in August, 1795, Kinnebrook was found to be recording times about half a second later than Maskelyne. 0


① A-C-B ② B-A-C ③ B-C-A ④ C-A-B ⑤ C-B-A

 


104. 20수특 19-4

 

When her grandmother didn't answer right away, Kay got worried and started to say something, but before she could she watched as Teresa reached into the drawer by her bed and to Kay's astonishment, pulled out a tape recorder.


(A) I need you to listen closely." Teresa looked at Kay to make sure she was listening. Too shocked to say anything, Kay nodded as she sat down in the chair facing her grandmother's wheelchair as Teresa played the tape. 2

(B) What was her grandmother up to now? Where could she have gotten a tape recorder? And from the looks of it, she knew exactly how to use it. Kay had never seen her grandmother with anything like that. 0

(C) Teresa didn't even like using the phone. She once told Kay that she was allergic to the stuff the world made. She never said manmade. She always said "world made" so how and why ..., as the question was beginning to form Teresa said, "I made this recording so I won't forget anything. 1


① A-C-B ② B-A-C ③ B-C-A ④ C-A-B ⑤ C-B-A

 


105. 20수특 20-1

 

For over a million years humans lived in small, mobile groups gathering their food from the wild and hunting animals.


(A) When resources permitted, they came together in larger groups and occasionally, when they could rely on particularly rich sources of food, they became semi-sedentary. Then, in a relatively short period of time after the end of the last ice age, about 10,000 BCE, this stable and well-balanced way of life began to change. 0

(B) Hunting and gathering groups survived but they were increasingly pushed into more marginal areas and those that the farmers could not utilize. By the twentieth century only a few groups of such people survived to be studied by anthropologists. The adoption of farming was the most fundamental change in human history and led on to all that we call civilization and recorded human history. 2

(C) Across the world humans slowly began to settle in one location and replace gathered plants with ones grown on special plots of land while a few animals were also domesticated. In less than ten thousand years this new, agricultural way of life had spread around the globe. 1


① A-C-B ② B-A-C ③ B-C-A ④ C-A-B ⑤ C-B-A

 


106. 20수특 20-2

 

Most historians and philosophers agree that it was the teachings of the seventeenth-century French philosopher Rene Descartes that I ushered in the thinking of the modern age and began the unraveling of the ancient link between emotions and health.


(A) In Descartes's orderly division of the world into rational and irrational — provable and unprovable — emotions and their relationship to health and disease clearly fell into the latter domain. And there they remained until scientific tools powerful enough to challenge the categorization could rescue them. 2

(B) In his reaction to the religious wars and the resulting chaos that spread across Europe for most of his adult life, Descartes formulated the concepts of rationalism and the necessity of visible proof that were to become the founding principles of modern science. 0

(C) In that era, emotions seemed to be a thing of magic, fleeting and undefinable in the framework of the science of the day. 1


① A-C-B ② B-A-C ③ B-C-A ④ C-A-B ⑤ C-B-A

 


107. 20수특 20-3

 

Fashions and social pressures shift.


(A) Needless to say, this was not a widely pursued innovation, but enough affluent people did do so in their quest to maintain their position as societal leaders. As a result, the log house reasserted its position as an American icon, regardless of the ethnic background of its original builders. 2

(B) Throughout almost the first three-quarters of the 20th century, log houses in the United States, as in Norway, were considered to be rough, primitive, and low-class housing. As a consequence, weatherboards were widely used to mask earlier log construction. 0

(C) However, in the prosperous 1960s, when many individuals were seeking a challenge to the status quo, fashions changed and social pressure relaxed. These changes encouraged persons who wished to ride the crest of changing fashion, to seek out hidden log buildings, to remove the siding, and to enjoy the glow of their visual confirmation of society's rediscovery of its heritage. 1


① A-C-B ② B-A-C ③ B-C-A ④ C-A-B ⑤ C-B-A

 


108. 20수특 20-4

 

A suitable way to describe how geologists perceive rocks and landscapes is the metaphor of a palimpsest — the term used by medieval scholars to describe a parchment that was used more than once, with old ink scraped off to allow a new document to be inscribed.


(A) In the same way, everywhere on Earth, traces of earlier eras persist in the contours of landforms and the rocks beneath, even as new chapters are being written. The discipline of geology is similar to an optical device for seeing the Earth text in all its dimensions. 1

(B) Invariably, the erasure was imperfect, and traces of the earlier text survived. These remnants can be read using X-rays and various illumination techniques, and in some cases are the only sources of very ancient documents (including several of the most important writings of Archimedes).0

(C) To think geologically is to hold in the mind's eye what is not only visible at the surface but also present in the subsurface, what has been and will be. 2


① A-C-B ② B-A-C ③ B-C-A ④ C-A-B ⑤ C-B-A

 


109. 20수특 21-1

 

One possible disaster resulting from warmer temperatures in northern areas is a potential tipping point associated with the Greenland ice sheet.


(A) Large amounts of freshwater entering the North Atlantic off Greenland as the ice melts could shut down the conveyor belt because freshwater does not sink like saltwater. 1

(B) The "Atlantic conveyor belt" consists of warm water moving up the eastern coast of North America as the Gulf Stream and then sinking 10,000 feet near Greenland and moving back to the equator, where it warms and rises again. 0

(C) This could trigger an ice age in North America and Europe — an ironic consequence of global warming but one associated in the past with the conveyor-belt shutdown. Climate scientists worried about this possibility several decades ago and then decided about a decade ago that it was very unlikely, but they have now begun to worry about it again. 2


① A-C-B ② B-A-C ③ B-C-A ④ C-A-B ⑤ C-B-A

 


110. 20수특 21-2

 

Dams affect keystone species.


(A) First they block the migration of salmon upstream to their spawning streams. Scientists have created fish ladders as well as management schemes that transport the fish around the obstacles, but these efforts have limited success. As salmon are separated from the ecological functions of a habitat, other keystone species are impacted. 0

(B) If these species disappear, the animals that hunt them disappear. Rivers that do not produce a bonanza of salmon will not attract bears. They migrate to other areas entirely, creating a gap at the center of the ecological arch. 2

(C) In this example, one such keystone species would be the grizzly bear. Without a large amount of salmon, grizzly bears cannot transport the amount of oceanic nutrients inland to help enrich soils for trees. Lack of vegetation cover then affects songbirds and small mammals. 1


① A-C-B ② B-A-C ③ B-C-A ④ C-A-B ⑤ C-B-A

 


111. 20수특 21-3

 

Sixty-five million years ago a meteor the size of New York slammed into the Earth.


(A) The upper ocean became acidified for a brief moment, perhaps only one or two years. But that was enough. More or less every sea creature that built shells or skeletons out of calcium carbonate became either rare or extinct. 1

(B) A handful of corals must have survived, or we would not still have them on Earth today. But they were nonetheless too scarce to leave their imprint; they did not reappear in the fossil record for a full two million years. 2

(C) The environmental chaos that ensued is widely believed to have put paid to the dinosaurs. But it also had a less well-known effect. According to Ken Caldeira at the Carnegie Institution of Washington in California, the meteor also threw up vast amounts of sulfur, which then rained down on the ocean as sulfuric acid. 0


① A-C-B ② B-A-C ③ B-C-A ④ C-A-B ⑤ C-B-A

 


112. 20수특 21-4

 

It is not common for both sides of a noise barrier to look identical because it is usually only one side of the barrier that is required to reflect or absorb noise.


(A) The difference between the two sides of a barrier should be determined on aesthetic as well as acoustic grounds: each side of the barrier should be designed to integrate with the landscape character and the backdrop against which it is to be viewed. Thus, the face of the barrier that may be facing a road corridor may include a pattern or have a bright colour, whereas the other face, which could stand opposite housing, may be treated in a more discreet fashion. 0

(B) Planting on either side of the barrier should also be designed with each separate identity in mind. 2

(C) Here, the facade may be plain and designed to merge in with neighbouring planting. Most barrier types can be designed with this in mind with the obvious exceptions of transparent barriers and many bio-barriers. 1


① A-C-B ② B-A-C ③ B-C-A ④ C-A-B ⑤ C-B-A

 


113. 20수특 22-1

 

The people who came before you invented science because your natural way of understanding and explaining what you experience is terrible.


(A) Those data sit there, naked and exposed, so they can be reflected upon and rearranged by each new visitor. Scientists and laypeople will conjure up new stories using the data, and they will argue, but the data will not budge. They may not even make sense for a hundred years or more, but thanks to the scientific method, the stories, full of biases and fallacies, will crash against the facts and recede into history. 2

(B) You prefer easy-to-understand stories, and thus turn everything in life into a narrative so that complicated problems become easy. Scientists work to remove the narrative, to boil it away, leaving behind only the raw facts. 1

(C) When you have zero evidence, every assumption is basically equal. You prefer to see causes rather than effects, signals in the noise, patterns in the randomness. 0


① A-C-B ② B-A-C ③ B-C-A ④ C-A-B ⑤ C-B-A

 


114. 20수특 22-2

 

As the sun rises in the morning, sunlight warms the ground, and the ground warms the air in contact with it by conduction.


(A) Given their random motion, some air molecules will cross this boundary: The "hot" molecules below bring greater kinetic energy to the cooler air; the "cool" molecules above bring a deficit of energy to the hot surface air. However, on a windless day, this form of heat exchange is slow, and a substantial temperature difference usually exists just above the ground. 1

(B) However, air is such a poor heat conductor that this process only takes place within a few centimeters of the ground. As the sun rises higher in the sky, the air in contact with the ground becomes even warmer, and there exists a thermal boundary separating the hot surface air from the slightly cooler air above. 0

(C) This explains why runners on a clear, windless, summer afternoon may experience air temperatures of over 50'C (122'F) at their feet and only 32'C (90 'F) at their waist. 2


① A-C-B ② B-A-C ③ B-C-A ④ C-A-B ⑤ C-B-A

 


115. 20수특 22-3

 

The idea of using cold temperatures to extend the shelf life of food has been known for centuries.


(A) He caught a chill, which became pneumonia, and he died shortly thereafter while still at Highgate. He died a martyr to his science and sadly history does not record the fate of the world's first oven-ready frozen chicken. 2

(B) The great Francis Bacon, the early seventeenth-century polymath rather than the twentieth-century painter, is generally credited with inventing the frozen chicken. It was not all he did, but certainly the only thing relevant to refrigeration. 0

(C) In the early spring of 1626, while on the way to Highgate in North London, for reasons unrecorded, Bacon decided to buy a chicken whose organs had been removed and stuff it with snow, thus demonstrating that refrigeration was a remarkable way of keeping food fresh for longer. Unfortunately, as the whole escapade was a spontaneous experiment, Bacon must have been unsuitably dressed for the snow. 1


① A-C-B ② B-A-C ③ B-C-A ④ C-A-B ⑤ C-B-A

 


116. 20수특 22-4

 

Genetic diversity, the heritable diversity among individuals and populations within species, provides the basis for evolution.


(A) The vast majority of forest genetic diversity remains undescribed, especially in the tropics. Estimates of the number of tree species vary from 80,000 to 100,000, but fewer than 500 have been studied in any depth. 1

(B) Over millions of years it has enabled forests and trees to adapt to changing conditions. Some tree species have been domesticated, but the management of forest genetic resources mainly involves tree populations that have undergone little selection by humans. 0

(C) Until recently, studies of forest tree genetic resources have concentrated on the few species regarded as the most suitable for domestication for use in plantations and agroforestry systems to produce wood, fibre or fuel. The present and future potential of most tree species to adapt in response to novel climatic conditions or for genetic improvement for human use remains largely unknown. 2


① A-C-B ② B-A-C ③ B-C-A ④ C-A-B ⑤ C-B-A

 


117. 20수특 23-1

 

Today zoo managers recognize the need to provide a place for privacy for their charges.


(A) Most species need the opportunity to be free from the peering faces. The animal's 'flight distance' — the distance at which it will flee an approaching human — must be respected. Since animals spend much of their time foraging, wherever possible food must be hidden so that it is sought. 0

(B) And this would be as distasteful as it would be unnatural. There is no means of replicating the hunt for the large carnivores in captivity. 2

(C) It is unnatural to provide a wild animal with unearned food ― thereby depriving it of one of its predominant activities in the wild — so every effort must be made to replicate that activity. Unfortunately, that can never be done for the large carnivores. Even if, say, one were to introduce a live zebra into a lion's area there would be no hunt — merely a short chase. 1


① A-C-B ② B-A-C ③ B-C-A ④ C-A-B ⑤ C-B-A

 


118. 20수특 23-2

 

Cruise ships generally stay at ports of call between six and twelve hours, although you will find a number of itineraries with overnight port stays.


(A) One solution to this potential disadvantage is to select a cruise that begins and/or ends in a large city ― and this is the majority of Mediterranean cruises. You can then add days at either or both ends of your cruise to allow more time for in-depth sightseeing. 2

(B) This limited time allotment in port is fine for small towns and other "specialized" ports of call, such as ancient historic sites or resort islands, but it can present a problem when visiting larger cities. A pet peeve of mine, for example, is the cruise that has Rome as a day port of call. 0

(C) How can anyone do justice to Rome in a single day? Again, how much of a problem this is depends upon your expectations. 1


① A-C-B ② B-A-C ③ B-C-A ④ C-A-B ⑤ C-B-A

 


119. 20수특 23-34

 

Does an NBA all-star who is young, handsome, rich, and famous need to be told by his coach that he's playing well?


(A) Apparently, the answer is yes. In 1999, Chuck Daly was considered one of the best coaches in the NBA, and Penny Hardaway, one of Daly's players on the Orlando Magic, was one of the stars of the league. Hardaway hit a mild slump in his shooting and the team lost three in a row. Daly said little to him, so Hardaway concluded that the coach had lost confidence in him. 0

(B) He reminded him that he was a great player, told him to have fun, and invited him to come and talk to him any time he wanted. Hardaway broke out of his slump and the Magic went on a winning streak. Hardaway said later, "I'm not saying that was the main reason we haven't lost since, but it definitely helped. He let me know how important I was to this team." 2

(C) In an interview with The New York Times, Hardaway said he had probably jumped to the wrong conclusion from the coach's silence. "I know we make all this money and everything, and you wouldn't think some of us need reinforcement. But you want to hear from your coach that you're doing well, even at this level." Daly, known as a players' coach, solved the problem by having a private, heart-to-heart talk with Hardaway. 1


① A-C-B ② B-A-C ③ B-C-A ④ C-A-B ⑤ C-B-A

 


120. 20수특 24-1

 

Although tone color has a scientific explanation, its function in music is aesthetic.


(A) Even at the most advanced stages of accomplishment, achieving what is considered to be a beautiful tone is a criterion of a good performance. 2

(B) Music is an art of sound, and the quality of that sound has much to do with our response to it. Indeed, the concept of tonal beauty varies considerably in different periods, styles, and cultures. 0

(C) On the other hand, within a particular context, ideals of beauty may be quite firmly established and performers often pay extraordinary prices for instruments that can produce that ideal sound. But no instrument automatically produces a beautiful tone, so the finest violin will produce a rasping, scraping sound in the hands of a beginner. 1


① A-C-B ② B-A-C ③ B-C-A ④ C-A-B ⑤ C-B-A

 


121. 20수특 24-2

 

Although they were internally organized by machines ― cameras — early photographs resembled drawings and paintings because they depicted the world according to linear perspective.


(A) What was being represented remained unchanged. This does not diminish the camera's importance in defining an image. 1

(B) The camera obscura was popular with artists because it automatically modified a scene by compressing form and emphasizing tonal mass according to Western pictorial standards. The camera was not designed as a radical device to unleash a new way of seeing, but evolved to produce a predefined look that took into consideration formulas and procedures such as composition, angle and point of view, quality of light, and selection of subject matter. 0

(C) As with most inventions, unforeseen side effects create unintentional changes. As imagemakers became more sophisticated they routinely used specific cameras and lenses to shape an image, and knowledgeable viewers can often trace the connections between the camera/lens and the resulting picture. 2


① A-C-B ② B-A-C ③ B-C-A ④ C-A-B ⑤ C-B-A

 


122. 20수특 24-3

 

The effectiveness of green architecture depended upon the balance of on-site energy sources with building energy conservation.


(A) In colder climates, construction was focused on solar energy gain, heat retention using higher insulation values and double-glazing, and even movable insulation. In warmer climates attention was given to the solar control of the envelope, especially roofs and western facades, as well as to natural ventilation and daylighting. 1

(B) The more "conservative" the building envelope, insulation and the tightness of construction, the easier it was to match essential energy loads to the availability of on-site resources. The design motto was "conservation first!" 0

(C) Conservation conscious design was in sharp contrast to previous modernist tendencies of spatial generosity, complexity of form, expansive glazing areas, and reliance on abundant fossil fuels and mechanical systems for heating and cooling. 2


① A-C-B ② B-A-C ③ B-C-A ④ C-A-B ⑤ C-B-A

 


123. 20수특 24-4

 

Music tourism sites and attractions generally attract two kinds of visitors: those particularly drawn for whatever reason to the memory or music of a particular performer, composer or genre (most obvious for festivals), and those who are there because the place fits into an itinerary devised for other reasons or because the visit is likely to be enjoyable.


(A) For French tourists, travel to New Orleans in part allows an experience of Francophone identities in the New World, just as it is more likely to be Americans who visit Jim Morrison's grave in Paris. 2

(B) Thus, at Abbey Road, a place identified in many general guidebooks, most visitors were there because they had some appreciation of the Beatles' music, but others were there simply because it was part of an agenda that included a range of obvious London landmarks. Particular links to music sites are, however, much more idiosyncratic and dependent on personal musical taste. 0

(C) Wider cultural and national links may also be evident. New Orleans and the Cajun region are popular with French tourists. 1


① A-C-B ② B-A-C ③ B-C-A ④ C-A-B ⑤ C-B-A

 


124. 20수특 25-1

 

Making a discovery, whether in the laboratory or library archives, about something that no one else in the world knows at a particular time is a thrilling experience for an undergraduate student.


(A) Nearly every U.S. college and university includes undergraduate research experiences in the learning opportunities offered to students. Institutions of higher education of all types are implementing a number of creative approaches to facilitate innovation, problem-solving and discovery, and they see undergraduate research as a critical vehicle for achieving these objectives. 2

(B) They may identify a new bacterial strain, make a connection between previously separate concepts, or uncover long-forgotten archival documents that lead to a new understanding of an issue. 1

(C) College students who have the opportunity to participate in undergraduate research can experience this thrill of discovery. 0


① A-C-B ② B-A-C ③ B-C-A ④ C-A-B ⑤ C-B-A

 


125. 20수특 25-2

 

A professor sent his sociology class to a school in a slum to record case histories of 200 young boys.


(A) Of the 180 still surviving in that neighborhood, all of these men had achieved extraordinary success as doctors, lawyers, and productive members of the community. The professor, determined to find out how these men had beaten seemingly overwhelming odds, followed up with interviews. 1

(B) The sociology students concluded that each boy appeared doomed to a bleak future. More than two decades later, another sociology professor sent his class out to find out what had happened to these 200 boys. 0

(C) Each of the men credited the same particular teacher; someone who had seen great potential in each of them. This teacher's expectations, held with conviction, drove her to actively nurture and support imminent success in all her students. 2


① A-C-B ② B-A-C ③ B-C-A ④ C-A-B ⑤ C-B-A

 


126. 20수특 25-3

 

Samuel and Pearl Oliner found large differences between European Gentiles who harbored Jews from the Nazis and those who did not: Rescuers reported close childhood associations with more people of different social classes and religions.


(A) All this suggests a piece of advice for prospective parents who want their children to develop a broadly charitable nature: Give them positive contact in the home with individuals from a wide spectrum of backgrounds. 2

(B) Not only was this expanded sense of "we" related to their decisions to aid people different from themselves during the war but also, when interviewed half a century later, rescuers were still helping a greater variety of people and causes. 1

(C) Moreover, while growing up, they felt a sense of similarity to a wider and more varied group of people than did nonrescuers. 0


① A-C-B ② B-A-C ③ B-C-A ④ C-A-B ⑤ C-B-A

 


127. 20수특 25-4

 

Have you ever said something to your child like the following?


(A) Your room is a mess; I want you to clean it up before you go to the game. Have you then wondered why she didn't do what she was told to do, right away and with a smile? 0

(B) This attitude, however, fails to consider the child's point of view. When you neglect to consider your child's thoughts, feelings, needs, and possible solutions to getting the room cleaned, you do so at the risk of losing her respect and goodwill. Your child's grumbling resistance is, in effect, a natural consequence of your choice to operate without her input. 2

(C) You made a unilateral decision, and she was expected to carry it out according to your time frame and standards. Because, After all, I'm the parent! 1


① A-C-B ② B-A-C ③ B-C-A ④ C-A-B ⑤ C-B-A

 


128. 20수특 26-1

 

The ancient Greeks had a word that lies at the heart of procrastination: akrasia, which means doing something against our own better judgment.


(A) The pile of important papers you never quite get to. The new job you put off looking for even though going to your current place of employment is the bane of your existence. 1

(B) When we procrastinate, we act against our own self-interests, satisfying the desire for immediate gratification by sacrificing our own longer-term goals and well-being. The essence of procrastination is to willingly put off something even though you expect the delay to make you worse off. 0

(C) Whenever we knowingly delay doing what's necessary in favor of the easier, less important task, we feed the demon of procrastination. 2


① A-C-B ② B-A-C ③ B-C-A ④ C-A-B ⑤ C-B-A

 


129. 20수특 26-2

 

In addition to gender, cultures have many other ways of dividing people into categories.


(A) Rituals, ceremonial occasions marking significant events such as weddings, graduations, and important holidays, are often events which are dressed for, where categories considered important by a society are visually displayed for all to see and appreciate. 2

(B) Common examples include age, class, religiosity (level of religious commitment), political stance, and value system. In all these examples, dress functions as a way of marking that a person is a member of a particular category. 0

(C) For example, older women in both the United States and Europe have created Red Hat Societies, in which red hats and purple clothing are used to formally mark entrance into middle and older age with fun and spirited apparel. These category systems help people living within cultural groups to interact easily, with dress functioning as a sign system allowing for rapid sorting of people into known and accepted groupings important to the given culture. 1


① A-C-B ② B-A-C ③ B-C-A ④ C-A-B ⑤ C-B-A

 


130. 20수특 26-3

 

James Banks states that "worldwide immigration is increasing racial, ethnic, cultural, linguistic, and religious diversity throughout the United States as well as in other Western nations such as the United Kingdom, France, Germany, and Australia."


(A) Christine Bennett adds to this argument by stating currently, more than ever before, there is an urgent need for citizens in the world that have a strong "multicultural competence" and who aim for global goals such as social justice and economic equality in order for there to be a sustainable peace. The intensified movement of people in recent times has not gone unnoticed, and must be addressed if peaceful societies are to exist. 2

(B) Banks argues that this is a time when the world "demands leaders, educators, and classroom teachers who can bridge impermeable cultural, ethnic, and religious borders, envision new possibilities, invent novel paradigms, and engage in personal transformation and visionary action." 1

(C) Indeed, these are not the only countries that are experiencing changing populations. 0


① A-C-B ② B-A-C ③ B-C-A ④ C-A-B ⑤ C-B-A

 


131. 20수특 26-4

 

Literacy is a common end of persons nowadays but reading and writing are recent inventions, only five thousand years old.


(A) It couldn't have. We were not literate for almost the entire history of our species. Furthermore, literacy does not seem to contribute to fitness, since there is an inverse correlation between fitness — as measured by birthrate, a proxy for inclusive fitness — and literacy. 1

(B) The birthrate is at zero or below in many parts of Europe where literacy is almost universal, and over 5 percent in places like Yemen and Niger, where literacy is low. Nonetheless, literacy seems to be really important for something else: human flourishing. 2

(C) There was no selection for literacy. In order to read, we utilize brain areas originally selected to track animals. One way to put the matter is that literacy didn't initially matter one iota for fitness. 0


① A-C-B ② B-A-C ③ B-C-A ④ C-A-B ⑤ C-B-A

 


132. 20수특 27-1

 

Reformers have seen the Internet as a means of moving toward the ideal of responsive democratic governance in the area of public administration.


(A) Just as the Internet has brought dramatic changes to e-commerce, so advocates of e-government, the provision of governmental information and services online, see the Internet as a way to modernize the public sector and democratize the relationship between individual citizens and their government. 2

(B) Information can be accessed and services can be delivered at the convenience of the citizen. No standing in line at a government office, no trying to reach a bureaucrat by telephone, no waiting for forms to arrive in the mail. 1

(C) They view the burgeoning presence of government on the Internet as an attempt to respond to citizens as clients and customers. They believe that conducting public business online provides many of the same advantages as conducting private business online. 0


① A-C-B ② B-A-C ③ B-C-A ④ C-A-B ⑤ C-B-A

 


133. 20수특 27-2

 

Even if we give robots the ability to learn what we want, an important question remains that AI alone won't be able to answer.


(A) How to combine these people's values when they might be in conflict is an important problem we need to solve. AI research can give us the tools to combine values in any way we decide but can't make the necessary decision for us. 2

(B) We can make robots try to align with a person's internal values, but there's more than one person involved here. 0

(C) The robot has an end user (or perhaps a few, like a personal robot caring for a family, a car driving a few passengers to different destinations, or an office assistant for an entire team); it has a designer (or perhaps a few); and it interacts with society — the autonomous car shares the road with pedestrians, human-driven vehicles, and other autonomous cars. 1


① A-C-B ② B-A-C ③ B-C-A ④ C-A-B ⑤ C-B-A

 


134. 20수특 27-3

 

Children, of course, have less control over their attention than adults, but when they do pay attention, they open their minds more fully to the messages presented.


(A) That, in turn, helped drive sales of toys, tickets to Disney's feature films, trips to Disneyland, and so on. And by the late 1950s, toy makers began creating toys specifically designed to be advertised on television — like Mattel's first girl-directed toy, the Barbie doll, whose commercials, which ran during breaks in The Mickey Mouse Club, chronicled glamorous episodes in Barbie's life. 2

(B) By the early 1950s advertisers had come to understand the commercial potential of reaching children by television. The Howdy Doody Show, featuring a clown and a dancing puppet, for instance, was sponsored by a famous food company, and during every show Howdy Doody would dance around a cereal box. 0

(C) But those were the early days. By the late 1950s, programmers had learned to create shows that in a sense served as advertisements themselves, like The Mickey Mouse Club, which helped nurture enduring attachments to characters like Mickey Mouse, Donald Duck, and other friends. 1


① A-C-B ② B-A-C ③ B-C-A ④ C-A-B ⑤ C-B-A

 


135. 20수특 27-4

 

With so many people sharing the most intimate details of their lives with the world, something was bound to disrupt the trajectory of online sharing.


(A) In this way users can control their digital footprints. 2

(B) The year 2013 saw NSA (National Security Agency) leaks, hackers targeting consumer credit cards, and blanket inquiries into individuals' personal lives through their online connections, to name a few. These invasions of privacy and more have inspired whole new platforms based on giving the user a digital experience that can be anonymous, deleted, and secure. 0

(C) For example, app developers have created a photo messaging app that enables users to send a photo or video with text to a specific group of people and control the time limit for how long they can view the sent message from one to ten seconds. When the time limit ends, the message is no longer available and is deleted from the app's servers. 1


① A-C-B ② B-A-C ③ B-C-A ④ C-A-B ⑤ C-B-A

 


136. 20수특 28-1

 

One approach to social facilitation that proposes an influence in social presence is based on the idea that I people generally try to present the best possible appearance to others and to make a favorable impression.


(A) This being the case, observers or coactors may not only motivate individuals to work hard at whatever task is being carried out, but also increase the person's sense of embarrassment when performance leads to failure. 0

(B) Difficult tasks are often failed, however, at least at the beginning. Embarrassment caused by such failure may cause stress and cognitive interference of sufficient intensity to disrupt performance. 2

(C) Failure is not likely to happen when the task is a simple or familiar one, so that the increased motivation is sufficient to produce improvement. 1


① A-C-B ② B-A-C ③ B-C-A ④ C-A-B ⑤ C-B-A

 


137. 20수특 28-2

 

Name a person who's accomplished something great, and you'll find a trail of risk along their personal path to greatness.


(A) If you're not failing, you're probably not stretching yourself. The comfort zone is a place where dreams go to die, and where vitality goes to die with it.2

(B) They weren't afraid to turn the volume up on their lives so they could ultimately turn the volume up on their love. When you're going after anything worthwhile in life, know that you're going to eventually fall, look stupid, and make mistakes;. it's part of the process of achieving. 1

(C) Do you think Martin Luther King, Jr., wasn't afraid? Do you suppose that JFK, Mother Teresa, Gandhi, and Nelson Mandela weren't scared about how their message would be received? These were people who heard the call, felt the fear, and moved forward regardless. 0


① A-C-B ② B-A-C ③ B-C-A ④ C-A-B ⑤ C-B-A

 


138. 20수특 28-3

 

We so often reach the conclusion, soon after we wake up, that "it's just one of those days," and by doing so create one of those days; or we enter a situation and predict that it's going to be bad — boring, irritating, frustrating, or annoying — and our prediction becomes a self-fulfilling prophecy.


(A) And while I should certainly allow myself to experience painful emotions at times, in some situations it may be appropriate to "fake it till I make it" and imbue myself and others with positive energy. 2

(B) Just as I am affected by other people's moods, other people are affected by mine. If I choose to enter a room with a sense of joy and excitement, my positive mood will spread and will affect those present. 1

(C) In fact, however, we can make most days and most encounters cheerful, lively, positive, and pleasant — if we choose to bring these kinds of emotions to the situation. Emotions are contagious. 0


① A-C-B ② B-A-C ③ B-C-A ④ C-A-B ⑤ C-B-A

 


139. 20수특 28-4

 

You can steer a conversation in any direction that you choose.


(A) what response might you get? She would tell you about its quality and the craftsmanship that went into it. When you say it's expensive, she'll talk about the quality. If you say that it looks beautiful, she'll tell you about the cost. By asking the right questions, you can steer the conversation in any direction you want and elicit the information that you need. 2

(B) Take this example. Let's say that while you are at a friend's house, she shows you her brand-new dining room table. If you want to know if it was really expensive, would asking directly be your best bet? Usually not, because she may get a little defensive. 0

(C) But if you said to your friend that it's the most gorgeous table you have ever seen, what might she respond with? You guessed it — how expensive it was! If you said, "This looks like it cost a fortune. How could you spend so much on a table?" 1


① A-C-B ② B-A-C ③ B-C-A ④ C-A-B ⑤ C-B-A

 


140. 20수특 29-1

 

In the United States, the coming of the railroads opened up local and regional markets to competition.


(A) Local producers had difficulty competing with large manufacturers, and many local companies went out of business or merged with other firms producing similar foods. 0

(B) In 1860, New York City had more than 2,000 butchers; only a few hundred survived in 1900. These butchers had counted on their customers' appreciation of the quality and flavor of their meats, but shoppers readily gave up locally butchered beef and pork for cheaper cuts shipped from the Midwest. 2

(C) The shipment of dressed beef from the Midwest to eastern cities, for instance, all but eliminated local slaughterhouses and reduced the number of butchers. 1


① A-C-B ② B-A-C ③ B-C-A ④ C-A-B ⑤ C-B-A

 


141. 20수특 29-2

 

There are limits to the idea of "timeless" fashion.


(A) Perhaps, then, being cool is also a matter of having freedom to exercise one's tastes, and the resources to be able to do this. 2

(B) It's hard to imagine a woman dressed in Jane Austen-era clothes as "cool" even though she might be fashionable given the standards of the time. Historical figures who might qualify as "cool," such as Lord Byron, are individuals who had a degree of financial and/or social freedom (usually arising from wealth and leisure) that allowed them to exercise their aesthetic choices in a way that was not possible for individuals who were more constrained by convention and circumstance. 1

(C) Fashions from the Victorian era or from Elizabethan times do not appear cool or fashionable now, no matter who wears them. Indeed, it is interesting to consider whether the concept of "cool" can be applied to historical eras in which fashion is constrained by social and gender conventions to a far greater degree than is the case in most modern liberal societies. 0


① A-C-B ② B-A-C ③ B-C-A ④ C-A-B ⑤ C-B-A

 


142. 20수특 29-3

 

Politicians, especially those in the national spotlight, are often jokingly accused of being narcissists but, in all seriousness, their profession lends itself to this particularly destructive personality trait.


(A) Finally, you constantly have people relying on you, believing in you, and holding you responsible as the sole representative of a cause. All of this power can lead to an exaggerated sense of self-importance that can cause some individuals to believe that the world revolves around them. 1

(B) That's when their out-of-control behaviors become easier to rationalize. 2

(C) For example, in order to be a successful candidate, you have to be unnaturally optimistic even in the face of probable defeat and possess high levels of self-esteem despite the constant criticism that comes with the territory. Furthermore, you are constantly given sole credit for successes — even though those successes were achieved, in part, by the work of many aides and assistants. 0


① A-C-B ② B-A-C ③ B-C-A ④ C-A-B ⑤ C-B-A

 


143. 20수특 29-4

 

Many of those who oppose globalization reserve their highest loyalties to the sovereign state, which they believe exists to protect their interests.


(A) Thus, anti-globalizers argue, globalization has created a democratic deficit by empowering institutions in which people have no voice and unleashing economic and cultural forces over which they have no control. 1

(B) Globalization, they believe, is eroding the rights and capacity of people to determine their own future. The result is alienation and anxiety, as people's lives are troubled by remote forces beyond their control or understanding. 2

(C) They argue that in democratic states, such as those in Europe and North America, citizens have a voice in determining their own fates but have little or no voice in the boardrooms of giant transnational corporations, remote international bureaucracies like the EU or WTO, or economic markets, and such institutions are not accountable to citizens. 0


① A-C-B ② B-A-C ③ B-C-A ④ C-A-B ⑤ C-B-A

 


144. 20수특 30-1

 

A middle-aged woman sat in a clinic's sitting room waiting to be fitted for a prosthesis to replace the leg she lost in an accident.


(A) The word "pirate" transformed her immediately and changed her entire outlook. She vividly imagined the romantic thrill of sailing the high seas in search of adventure. Some time later, the little boy asked what happened to her leg. She replied proudly "I' m a pirate too." 2

(B) She struck up a casual conversation and eventually asked what happened to his eye. He replied, "Oh nothing. I'm a pirate." 1

(C) Although considerable time had passed since the accident and she had adjusted well, she continued to feel emotionally devastated by her loss. The woman watched as a young boy with a patch over his eye played with some toys. The thought of losing an eye at such a tender age made her own difficult situation seem minor by comparison. 0


① A-C-B ② B-A-C ③ B-C-A ④ C-A-B ⑤ C-B-A

 


145. 20수특 30-2

 

The definition of a calorie does not refer to something that produces fat, but to something that produces energy.


(A) Calories are an important source of energy for the body and the brain. They are necessary for the growth of nails and hair and to replace skin cells. 0

(B) Consuming enough calories is necessary not only for healthy physical functioning but also for psychological well-being. The intake of sufficient calories is a necessary condition to be in a positive frame of mind. 2

(C) Calories produce the energy needed for walking, bicycling, reading, talking over the phone, texting and so on. Without the intake of calories, the body cannot develop energy for physical, psychological and social activities and becomes exhausted. 1


① A-C-B ② B-A-C ③ B-C-A ④ C-A-B ⑤ C-B-A

 


146. 20수특 30-3

 

The anti-aging industry promotes a particular image of longevity that is reduced to biomechanical processes at the cellular level.


(A) Gaining broader cultural knowledge of medicinal foods may offer alternatives to anti-aging discourse in which bodies are subject to biomechanical processes. Traditional systems of medicine offered insights based on observation of the subtle interactions of food and environment of human bodies. 1

(B) Such discourse focuses on longevity as an individual body's battle against aging rather than considering aging in a broader social and historical context. In an era when care of the self comprises a vast industry, healing through nutrition and healthy diets may seem too low-tech or slow. 0

(C) We can greatly influence our well-being through diet and nutritional knowledge, not just consuming dietary supplements. Longevity is not guaranteed, but the possibility of accessible self-managed care on a daily basis through attention to one's food can enhance the quality of one's life. 2


① A-C-B ② B-A-C ③ B-C-A ④ C-A-B ⑤ C-B-A

 


147. 20수특 30-4

 

As early as 1907, one prominent Russian zoologist, Ilya Metchnikoff, began to question if the "all bacteria are bad" orthodoxy might be flawed.


(A) He observed that the oldest villagers were drinking fermented yogurt containing the bacteria Lactobacillus bulgaricus. Metchnikoff suggested that one secret to longevity is consuming healthy bacteria. History would prove him right. 2

(B) He was also struck by the fact that some people lived to a ripe old age despite harsh rural conditions and poor hygiene associated with poverty. In Bulgaria, he noted, there were peasants in the Caucasus Mountains who lived beyond one hundred years. 1

(C) During the cholera epidemic of 1892 in France, Metchnikoff mixed bacteria together in a petri dish and found that some bacteria could stimulate cholera growth, but to his surprise found that other bacteria hindered it. This led him to speculate whether swallowing some types of helpful bacteria might be useful for preventing deadly diseases. 0


① A-C-B ② B-A-C ③ B-C-A ④ C-A-B ⑤ C-B-A

 


148. 20수특 T1-1

 

We received your application for the Steven Christopher Scholarship.


(A) This experience will greatly benefit your qualifications the next time you apply for the same scholarship. We encourage you to reapply for next year's slot, and wish you the best of luck in your academic career. 2

(B) However, we regret to inform you that the scholarship has been granted to someone else who has a wider experience in the industry. Considering that you are a few years younger than the others who are also applying, we encourage you to gain more experience and expose yourself more to different challenges and situations in the corporate world. 1

(C) We sincerely appreciate your interest. Each year hundreds of students apply for the scholarship, and your outstanding academic record placed you among the finalists. 0


① A-C-B ② B-A-C ③ B-C-A ④ C-A-B ⑤ C-B-A

 


149. 20수특 T1-2

 

I signaled to David, my diving teacher, that I was going down into the sea.


(A) I couldn't see David, I couldn't see the bottom, and I could barely see the surface. I was breathing too fast, and my suit was too tight and confining. My panic grew at a rate almost too fast to control. 1

(B) I worked to calm myself: Slow your breathing, adjust your buoyancy, move slowly. It worked; I didn't dash to the surface, I looked at David, and out of habit only I gave him a grin and a thumbs-up, but it was enough to refocus and follow my training, adjust my buoyancy, take a slow breath, and proceed down. Now I was looking at my old invertebrate friends on the cave wall that had been the site of my very first underwater job. 2

(C) As I slowly sank away from the lighted sea surface, I swam toward the cave wall. With a growing uneasiness, I realized that the cold was not nearly as much a problem as the dark. Underwater, it was completely, disorientingly dark. 0


① A-C-B ② B-A-C ③ B-C-A ④ C-A-B ⑤ C-B-A

 


150. 20수특 T1-3

 

People are distracted.


(A) But only multichannel information can somehow guarantee that a piece of information really reaches the audience. So, do not hesitate to repeat your content in emails, intranet, staff meetings and so on. At some point it will reach the minds of your people and stick in their memory. 2

(B) They are distracted because information is everywhere and the selection of the relevant messages becomes increasingly difficult. So, even if a piece of information was successfully delivered, it doesn't mean it's been noticed, understood, internalised and pondered. It may just be forgotten or neglected. 0

(C) This is why Consul Marcus Tullius Cicero said 'Repetita iuvant' (repetitions help). And the purpose is clear: ensuring the reception of that message or piece of information. It's best to repeat messages in different channels, even if it seems a redundant exercise. 1


① A-C-B ② B-A-C ③ B-C-A ④ C-A-B ⑤ C-B-A

 


151. 20수특 T1-4

 

The foundation of an effective team is the recognition that each member needs every other member, and no individual can be successful without the cooperation of others.


(A) As a young boy I was a very enthusiastic baseball fan. My favorite player was the Hall of Fame pitcher Robin Roberts of the Philadelphia Phillies. During the early 1950s, his fastball dominated the National League. 0

(B) I told my uncle that I knew how to stop Robin Roberts:. "Make me his catcher." 2

(C) My uncle, who took me to my first ball game, explained that opposing batters were so intimidated by Roberts's fastball that they were automatic "outs" even before they got to the plate. My uncle claimed that Robin Roberts was unstoppable. Even as a young boy I intuitively knew that no one was unstoppable by himself. 1


① A-C-B ② B-A-C ③ B-C-A ④ C-A-B ⑤ C-B-A

 


152. 20수특 T1-5

 

People or firms that purchase services come in contact with other consumers as well as the service employees.


(A) The two groups are significantly different in behavior, and the expectation is that they would not mix well within the facilities at the same time. Similarly, restaurants separate smokers and nonsmokers, and they should try to separate other patrons that show some potential for conflict. 2

(B) For example, a hotel guest waits in line at the front desk or the concierge desk with other guests. In addition, the guests share facilities such as the pool, the restaurant, and the fitness center. 0

(C) Therefore, service firms must manage consumer interactions to the best of their abilities to ensure customer satisfaction. For example, a hotel's sales office would not want to book group business with a nondrinking religious group at the same time as a reunion of military veterans. 1


① A-C-B ② B-A-C ③ B-C-A ④ C-A-B ⑤ C-B-A

 


153. 20수특 T1-6

 

Great coaches know that they can get their athletes to perform well by drumming certain ideas into their heads.


(A) This is basically the technique Jack Nicklaus, perhaps the greatest golfer of the past several decades, used to enhance his performance. Before every shot, he formed a mental picture in which he saw three things: (1) the target area the ball would land in, (2) the flight path of the ball to the target area, and (3) himself using the appropriate swing for that particular shot. 1

(B) Foremost is the idea that the players are winners, so that they will think only of winning and never about the possibility of losing. Chances are high that they indeed will win because the image of themselves as winners will force them to concentrate only on the moves that ensure winning. 0

(C) In short, if athletes define themselves as winners, they are more likely to win. By the same token, if athletes define themselves as losers, they will very likely lose. 2


① A-C-B ② B-A-C ③ B-C-A ④ C-A-B ⑤ C-B-A

 


154. 20수특 T1-7

 

A lichen is an organism consisting of a fungus and an alga living together, usually in an interdependent relationship.


(A) In contrast, areas with clean air can support larger varieties of lichens. Some lichen species are sensitive to specific air-polluting chemicals. Old man's beard and yellow Evemia lichens, for example, can sicken and die in the presence of excessive sulfur dioxide (S02), even if the pollutant originates far away. 1

(B) These hardy species are good biological indicators of air pollution because they continually absorb air as a source of nourishment. A highly polluted area around an industrial plant might have only gray-green crusty lichens or none at all. An area with moderate air pollution might support only orange crusty lichens. 0

(C) For this reason, scientists discovered S02 pollution on Isle Royale, Michigan, in Lake Superior, an island where no car or tall factory chimney has ever existed. They used Evemia lichens to point the finger northwest toward coalburning facilities in and around the Canadian city of Thunder Bay, Ontario. 2


① A-C-B ② B-A-C ③ B-C-A ④ C-A-B ⑤ C-B-A

 


155. 20수특 T1-8

 

Belize is the second smallest country in Central America in landmass, the smallest in population, yet is one of the most diverse in several ways.


(A) A coral barrier reef, the world's second longest, protects the coast from high sea tides, but not from frequent destructive hurricanes. The northern region is also lowland and heavily forested with old-growth hardwoods such as mahogany. These have been harvested for many years and have been important in the Belizean economy. 1

(B) Subtropical in climate with a wet and a dry season, the country is located on the Caribbean Sea with Mexico to the north and Guatemala to the south and west. Most of Belize is low-lying country. The long coastline is often swampy with lagoons and many small islands. 0

(C) For such a small country, there is surprising biodiversity. Belize is recognized as one of the world's ecological treasures, and as a result, ecotourism is a growing part of their economy. 2


① A-C-B ② B-A-C ③ B-C-A ④ C-A-B ⑤ C-B-A

 


156. 20수특 T1-9

 

The above table, provided by Modesto Junior College in Modesto, California, shows enrollment counts and success rates of the students who took the English basic skills course in two terms, the fall of 2014 and 2015, by their ethnic group.


(A) The number of Hispanic students accounted for more than 50% of the students who enrolled in the English basic skills course in both terms. Pacific Islander students had the highest success rate in both years while the students of Multi-Ethnicity had the lowest success rate in 2014 and 2015. 1

(B) In three ethnic groups, the enrollment count had fewer students in the fall of 2015 than that of 2014. 2

(C) Both the total number of enrolled students and their success rate in the fall of 2015 were higher than in the fall of 2014. Nearly all of the ethnic groups recorded higher success rates in 2015 than their previous year's success rates: only one ethnic group, Multi-Ethnicity, didn't have an increase in their success rate. 0


① A-C-B ② B-A-C ③ B-C-A ④ C-A-B ⑤ C-B-A

 


157. 20수특 T1-10

 

2020 Caroline County Art Show & Contest The 2020 Caroline County Art Show & Contest is just around the corner.


(A) ■ You must be 18 years and older to enter the contest. ■ The artwork will be judged by a guest art curator. 1

(B) Come and visit to view art created by local artists, or register your own work in the art contest! Art contest guidelines ■ Drop off your artwork at Pine Tree Art Gallery by December 27. 0

(C) Opening reception: Friday, January 3, 6-8 p.m. ■ General public ― $20 ■ Gallery members ― $10 ■ Participating artists ― Free Guest curator talk: Saturday, January 4, 10-11 a.m. ■ This year's guest curator is John Raymond. ■ No charge and open to all! For more information about the art show & contest, click here. 2


① A-C-B ② B-A-C ③ B-C-A ④ C-A-B ⑤ C-B-A

 


158. 20수특 T1-11

 

Automated External Defibrillators Automated External Defibrillators (AEDs) are portable devices that can detect life-threatening cardiac conditions and apply electrical shocks to the heart to restore heart rhythm, allowing blood and oxygen to flow to the heart, brain and body.


(A) 3- The machine will analyze the heart rhythm. 4- If an abnormal rhythm is detected, the AED will charge and instruct you to push the shock button. 5- Do not touch the victim or their clothing while the AED is analyzing or delivering a shock. 1

(B) 6- After the AED delivers a shock, immediately begin chest compression with your hands for two minutes. 7- Check the victim and repeat the analysis, shock, and chest compression as needed until medical professionals arrive. 2

(C) AEDs require no training, which means anyone nearby can provide treatment during cardiac arrest in those critical minutes before medical professionals arrive. How to use an AED 1- Place the AED near the victim's head, power on the unit, and wait for vocal instructions. 2- Apply the sticky pads to the victim's chest. 0


① A-C-B ② B-A-C ③ B-C-A ④ C-A-B ⑤ C-B-A

 


159. 20수특 T1-12

 

We behave (perform roles) as we think the role or roles should be performed (eg, daughter or son, mother or father, etc); this is known as role-taking.


(A) If someone smiles back at you, your assumption is that he or she has the same intentions as you ― to make your acquaintance. Your behavior then may be to initiate a conversation with the person based on the smile being an assumed symbol or sign of cordiality. 2

(B) As we try to guess the intentions of the other, the other is impacting our behavior. For example, if you smile at someone while at a party, it may be your way of attempting to make the person's acquaintance. 1

(C) We also assume that others we encounter will behave as we would behave if we were in their roles, that they will conform to a community standard or model. By adapting our behavior accordingly, the expected behavior or the perspective of the other is acting upon us. 0


① A-C-B ② B-A-C ③ B-C-A ④ C-A-B ⑤ C-B-A

 


160. 20수특 T1-13

 

Which comes first, the site or the plan for the house?


(A) I believe a house should "grow" from its site and not look like it has been imported and dropped there haphazardly. If you have already selected your architect, bring him along to offer his opinion on the sites you are considering. He may well see things about a site that you may not see. 1

(B) I advise my clients to choose the site first. This allows you to design a house to fit the land. You wouldn't buy a rug and then figure out what room it fits in later on. 0

(C) The site you have in mind might not be appropriate for the house you desire. How many times have we seen houses placed awkwardly on a slope when the house design would clearly have been more comfortable on a flat site? Developments of tract houses are fertile ground for examples of house designs that were poorly fitted to their sites. These mismatches of house to site are akin to wearing a tuxedo with tennis shoes. 2


① A-C-B ② B-A-C ③ B-C-A ④ C-A-B ⑤ C-B-A

 


161. 20수특 T1-14

 

Office workers can sometimes choose their own desk setups, integrating exercise on an individual basis.


(A) In the laboratory, regular exercise improves problem-solving abilities, fluid intelligence, and even memory — sometimes dramatically so. It's worth finding out whether the same is true in business settings, too. 2

(B) But businesses have compelling reasons to incorporate such radical ideas into company policy as well. Business leaders already know that if employees exercised regularly, it would reduce health-care costs. There's no question that halving someone's lifetime risk of a debilitating stroke or Alzheimer's disease is a wonderfully humanitarian thing to do. 0

(C) But exercise also could boost the collective brain power of an organization. Fit employees are more capable than sedentary employees of mobilizing their God-given IQs. For companies whose competitiveness rests on creative intellectual horsepower, such mobilization could mean a strategic advantage. 1


① A-C-B ② B-A-C ③ B-C-A ④ C-A-B ⑤ C-B-A

 


162. 20수특 T1-15

 

It is very important in the information age to understand the difference between knowledge and information.


(A) What is accessible by computer and, indeed, what is published in the journals is information. Knowledge is something that has to be constructed in the mind of the expert reader. This is what scholarship is about. 0

(B) It would probably take many years, despite the fact that their research was all published. To take another example, what do producers of science documentaries for television programmes do when they are researching their subjects? They talk to the experts rather than trying to read the journals. Quite rightly, as that is the only place that knowledge is to be found — inside the heads of the scholars. 2

(C) Information is, these days, instantly accessible, but knowledge still takes years of dedicated study to acquire. Imagine that a freak accident wiped out an entire field of experts on a subject while all were attending a conference. How long would it take to reconstruct expertise in the field so that research could once again progress? 1


① A-C-B ② B-A-C ③ B-C-A ④ C-A-B ⑤ C-B-A

 


163. 20수특 T1-16

 

When people want to react correctly to a persuasive message but don't have the motivation or ability to think about it deeply, there is a shortcut they can take.


(A) In addition, the more consensus you witnessed among audience members, the more likely you would be to follow their lead, even if you didn't initially agree with them. It's for this reason that interrogators are taught to say to a suspect "We believe you are guilty" rather than "I believe you are guilty." 2

(B) They can observe the responses of others to the message. 0

(C) For example, if under such conditions you heard a political speech and everyone in the audience around you responded enthusiastically to it, you might well conclude that the speech was a good one and become persuaded in its direction. 1


① A-C-B ② B-A-C ③ B-C-A ④ C-A-B ⑤ C-B-A

 


164. 20수특 T1-17

 

Popular participation in conflict in the nineteenth century cannot be understood without examining the role of empire.


(A) Scots, for example, were excluded from membership of the English community, but by participating in the wars against France and the empire in India, they became part of the British nation, increasing the war-fighting capacity of the latter. Empire was a crucial mechanism in constructing the exchange in the nineteenth century. 2

(B) Hannah Arendt later dismissively characterized imperialism as "the export of superfluous men and superfluous capital." She was correct that colonial functionaries were often from marginal populations — Cecil Rhodes, after all, would advocate imperialism "to settle surplus population" and thus "avoid civil war" — but these marginalized individuals increasingly used their participation in colonial conflicts to make claims to membership in the home nation. 1

(C) Individuals had always gone to the colonies for profit or to settle. But individuals also participated in nineteenth-century imperial campaigns for a novel reason. 0


① A-C-B ② B-A-C ③ B-C-A ④ C-A-B ⑤ C-B-A

 


165. 20수특 T1-18

 

Agricultural effects on water quality can occur at local, regional, and national scales.


(A) For example, increased levels of nutrients from agricultural fertilizers can stimulate algal blooms and affect the ecology of local streams. Nitrate and some chemical weed killers can move through the soil to groundwater and, eventually, to local streams. 0

(B) Elevated nutrient inputs stimulate harmful algal blooms along the US coasts causing negative economic impacts. 2

(C) Farther downstream, these elevated nutrients can increase costs associated with treating the water so that it is suitable for drinking. Ultimately, chemicals associated with agricultural activities (such as nutrients and pesticides) and sediment (eroded soil) empty into our river mouths and can harm valuable commercial and recreational fisheries. 1


① A-C-B ② B-A-C ③ B-C-A ④ C-A-B ⑤ C-B-A

 


166. 20수특 T1-19

 

Many sociologists identify themselves as researchers to the people they study.


(A) On the surface, the neighborhood looked like a badly organized place, an urban jungle of its period. Yet Gans discovered that it was a well-organized community ― an urban village rather than a jungle — where the residents enjoyed close social relationships with one another. 2

(B) They do not worry that revealing their true identity will change their subjects' behavior. They are not overly concerned that subjects will hide secrets from them. Usually, they strive to minimize these problems by not getting too deeply involved with their subjects while simultaneously establishing a good rapport with them. 0

(C) This is not easy to accomplish, though. Nevertheless, such efforts have paid off, as indicated by some sociological insights that have emerged from their work. Herbert Gans, for example, became a participant observer in a poor Italian neighborhood in Boston in the late 1950s. 1


① A-C-B ② B-A-C ③ B-C-A ④ C-A-B ⑤ C-B-A

 


167. 20수특 T1-20

 

Only 10 percent of the midlatitude rainforest in the Pacific Northwest remains untouched.


(A) Throughout the tropics, a pattern of replacing numerous species with a few favored ones is common. In particular eucalyptus has been preferred over existing local species because it is fast growing and, when cut for coppicing, its shoots quickly develop into new tree growth. However, the oil in its leaves results in a ground litter that inhibits undergrowth with the result that soil erosion occurs. Hence, reforestation utilizing eucalyptus trees is not environmentally friendly. 2

(B) Forests in their natural condition once masked the area. This has been replaced by a checkered mosaic composed of plots of bare ground, recently replanted commercial saplings, and some mature forests. This type of land cover threatens the habitat of many plants and animals. 0

(C) Current conditions represent a chronically degrading environmental condition. The U.S. Forest Service, under public pressure and court orders, has altered the former approved clear-cutting strategy. Now some mature trees are left standing with the hope of encouraging a more natural regeneration of forest lands. 1


① A-C-B ② B-A-C ③ B-C-A ④ C-A-B ⑤ C-B-A

 


168. 20수특 T1-21

 

One particular problem that people sometimes struggle with is avoiding the use of masculine pronouns to refer to both men and women.


(A) Students often argue that using "he" or "his" is less cumbersome than writing "he or she" or "his or her." As an alternative, you can rephrase a sentence to use a gender-neutral plural pronoun. 0

(B) Instead of saying "Man cannot live without water," you could say "One cannot live without water" or "People cannot live without water." Avoiding sexist language isn't all that difficult, but it may take conscious effort; the pay-off is more effective interpersonal communication. 2

(C) The sentence "A doctor should be polite to his patients" excludes the possibility of female doctors, but "Doctors should be polite to their patients" makes the same point without the gender bias. In the same way, you can replace masculine terms to refer to humans with gender-inclusive alternatives. 1


① A-C-B ② B-A-C ③ B-C-A ④ C-A-B ⑤ C-B-A

 


169. 20수특 T1-22

 

The need for distinctiveness is a basic human need to which sport fandom can contribute.


(A) Given that brand consumption is associated with desires to be unique, it seems reasonable that individuals use sport fandom as an opportunity to meet their need for distinction by selectively choosing to follow non-mainstream sports or less popular teams. For instance, individuals can partially meet their need for uniqueness by identifying with a distant team or rooting for an underdog. 2

(B) According to this framework, individuals strive for two sometimes opposing social goals: inclusion and differentiation. These goals are best satisfied "through identification with distinctive groups that satisfy both needs simultaneously." 1

(C) Although humans want to feel a sense of belonging with those around them, they simultaneously have a need to be different and unique. The fundamental need for uniqueness is best understood through Brewer's Optimal Distinctiveness Theory. 0


① A-C-B ② B-A-C ③ B-C-A ④ C-A-B ⑤ C-B-A

 


170. 20수특 T1-23

 

Chunking is vital for cognition of music.


(A) you don't reel off the names of roads as an abstract list, but have to construct your route by mentally retreading it. When musicians make a mistake during rehearsal, they wind back to the start of a musical phrase ('let's take it from the second verse') before restarting. 2

(B) If you ask a pianist to start a Mozart sonata from bar forty-one, she'll probably have to mentally replay the music from the start until reaching that bar —. the score is not simply laid out in her mind, to be read from any arbitrary point. It's rather like describing how you drive to work:. 1

(C) If we had to encode it in our brains note by note, we'd struggle to make sense of anything more complex than the simplest children's songs. Of course, most accomplished musicians can play compositions containing many thousands of notes entirely from memory, without a note out of place. But this seemingly awesome accomplishment of recall is made possible by remembering the musical process, not the individual notes as such. 0


① A-C-B ② B-A-C ③ B-C-A ④ C-A-B ⑤ C-B-A

 


171. 20수특 T1-2425

 

What is wrong with pseudoscience?


(A) Differences of opinion among its believers, when such disagreements arise at all, lead to endless sect splitting rather than to progress. Fourth, the primary aim of pseudoscience is not to set up, test, and correct systems of hypotheses (theories) mapping reality, but to influence things and men:. it has, like magic and like technology, a primarily practical aim rather than a cognitive one but, unlike magic, it presents itself as science and, unlike technology, it does not enjoy the backing of science. 2

(B) the pseudoscientist, like the fisherman, exaggerates his catch and neglects his failures or excuses them. Third, pseudoscience lacks a self-correction mechanism:. it cannot learn from either fresh empirical information (which it swallows without digesting it), new scientific discoveries (which it looks down on), or criticism (which it rejects furiously). It can make no progress because it manages to interpret every failure as confirmation and every criticism as an attack. 1

(C) Certainly not just that it is basically false, since anyway all our factual theories are at best partially true. What is wrong with pseudoscience is, first, that it refuses to ground its doctrines and could not do it because pseudoscience makes a total break with our scientific heritage - which is not the case of scientific revolutions, all of which are partial since every new idea has to be judged by means of others that are not questioned in the given context. Second, pseudoscience refuses to test its doctrines by experiment proper;. moreover, it is largely untestable because it tends to interpret all data in such a way that its theses are confirmed no matter what happens:. 0


① A-C-B ② B-A-C ③ B-C-A ④ C-A-B ⑤ C-B-A

 


172. 20수특 T1-2628

 

On a beautiful spring afternoon, a softball game was held between rivals Central Washington University and Western Oregon University.


(A) The only home run of Tucholsky's four-year career would be erased. The coach didn't know what to do. That's when Mallory Holtman stepped in. Holtman played first base for the other team. She was also Central Washington's all-time home run leader. She knew that if her team lost the game, their playoff hopes would probably be gone. But after listening to the coach and umpire for a while, she asked a question. Is the other team allowed to carry her around the bases? The umpires said there was nothing in the rule book against it. So Holtman and Central's shortstop, Liz Wallace, walked over and helped Tucholsky up. Carrying her, they resumed the home run walk. At each base, they paused to let Tucholsky touch it with her uninjured leg. 1

(B) Holtman said, "I wonder what this must look like to other people." The three players burst out laughing. The other people in the stadium weren't laughing, though. They were shedding tears at seeing such a moving act of sportsmanship. Tucholsky's Western team ended up winning the game 4 to 2. Afterward, someone realized that the umpires had made a mistake. The rules did allow a substitute runner, after all. But it didn't really matter. "In the end, it's not about winning and losing so much," said Holtman. "It was about this girl. She hit it over the fence and was in pain. And she deserved a home run." 2

(C) Western's Sara Tucholsky had never hit a home run - not even in batting practice. Then, in the second inning she smashed the ball over the center field fence. With two players on base, it would be a three-run shot. Thrilled, Tucholsky sprinted toward first base. But as she watched the ball clear the fence, she missed first base. When she stopped quickly to go back and touch it, something in her knee gave out. She fell to the ground. In terrible pain, Tucholsky crawled through the dirt back to first base. The Western coach rushed onto the field. The umpires told the coach it was against the rules for Tucholsky's Western teammates to help her around the bases. The coach could substitute a runner for her. But then the hit would be judged a single. 0


① A-C-B ② B-A-C ③ B-C-A ④ C-A-B ⑤ C-B-A

 


173. 20수특 T2-1

 

Our school library is very nice, and I am proud of being a member of the library maintenance team.


(A) And as a student and a member of the team, I would like to suggest a few improvements that would make our library even better. First, a library should be a quiet place, but noise from the school cafeteria can easily be heard in the school library. 0

(B) Finally, a magazine and newspaper section should be created so that students can come to the library to read about current events. Thank you in advance for considering my suggestions. I look forward to talking with you about them. 2

(C) Therefore, I think the library should be moved to the first floor so that the noise can be minimized. There is also a need for additional bookshelves to accommodate the books that have been piled up in a corner of the library. 1


① A-C-B ② B-A-C ③ B-C-A ④ C-A-B ⑤ C-B-A

 


174. 20수특 T2-2

 

The day of the salsa contest arrived and practically all the local dance schools took part.


(A) The hostess introduced Annette and Reiner, from Diego Santiago's dance school, on stage. They walked up onto the dance floor. While they bowed to the jury and the audience, Reiner noticed that Annette was trembling with anxiety. 0

(B) He squeezed her hand and Annette felt the warmth that radiated from his hand flowing like calming energy through her body. Her goosebumps disappeared and were replaced by excitement and joyful anticipation. Annette stood in the spotlight in front of a large crowd and felt not afraid but absolutely supported by her partner. It was as though, when he held her hand, he absorbed all her fears, leaving her with only positive feelings. 2

(C) "Don't worry," he encouraged her in a low voice. "You were born to dance. We'll be OK!" 1


① A-C-B ② B-A-C ③ B-C-A ④ C-A-B ⑤ C-B-A

 


175. 20수특 T2-3

 

A big mistake many new college students make is just sitting in their rooms.


(A) Walk down your residence hall corridor and look into other rooms where doors are propped open. If you see a student sitting alone in a room, knock lightly, say hello, and introduce yourself. If nothing else, get out and walk around campus. 1

(B) Some students stay in their rooms waiting for friends to come find them, convinced that it will happen on its own. Other students remain in their rooms because they feel shy or are uncertain about how to approach other people. For the first few weeks of the semester, you should try to spend as little time as possible in your room. 0

(C) Look for groups of new students who are hanging out together and join them. Whatever you do, don't isolate yourself. Most students form friendships within the first couple of months of college, and it is much harder to join existing groups than it is to meet new people one-on-one. 2


① A-C-B ② B-A-C ③ B-C-A ④ C-A-B ⑤ C-B-A

 


176. 20수특 T2-4

 

The freedom to choose one's identity is critical, since the sources of identity are shifting from "belonging" to "achievement."


(A) A good society recognizes and does not pit roots and wings against one another. If roots are the necessary condition for happiness, then wings are the sufficient condition. A good life is not possible without both. 2

(B) Speaking for many, Kymlicka says that "identification is more secure, less liable to be threatened, if it does not depend on accomplishment." But this is absurd. Achievement increasingly is the basis for a satisfying life. To be sure, some people want unconditional acceptance by their "in" group. 0

(C) But more and more, people in rich countries achieve many of their identities. They choose their careers, friendships, allies, mixing and matching pieces and styles. Even their ethnic, racial and national affiliations are forged in various ways, despite the fact that a person's self-image depends partly on how he's viewed by others. Indeed, the ability to make one's own self is the essence of freedom. 1


① A-C-B ② B-A-C ③ B-C-A ④ C-A-B ⑤ C-B-A

 


177. 20수특 T2-5

 

The efficient and valuable use of big data needs the personal and organizational capacity of asking the right questions and in the right way.


(A) The arts/humanities are important in the age of digital transformation and big data because they dominate the knowledge domains of the creation and communication of narratives as well as meanings of human life. In other words, the arts and humanities are capable of embedding into big data the aesthetic human-based dimensions that ultimately make them relevant in order to identify, address, and solve key questions for sustainable societal, economic, and environmental wealth creation. 1

(B) Big data is powerful only if is generated, combined, or supported by the creation of strong narratives, organizationally and contextually framed. This means that the big data has to be "thick," i.e., not only quantitatively but most importantly qualitatively relevant. 0

(C) The arts and humanities are essential in order to make big data, analytics, data mining, and digital transformation significant for stakeholders. 2


① A-C-B ② B-A-C ③ B-C-A ④ C-A-B ⑤ C-B-A

 


178. 20수특 T2-6

 

Once you have firmly established the habit of placing a pair of commas around a nonessential element that interrupts or changes the normal order of the English sentence, you can consider a few situations in which this mark of punctuation may safely be omitted.


(A) You are doubtless aware that the tendency of modern writers is to make considerably less use of punctuation than their predecessors did. 0

(B) These marks were never marks of actual punctuation, in the sense that they clarified meaning, and today they have almost disappeared from printing. A more important reason for the diminishing amount of punctuation in modern writing is that our writers are learning to construct their sentences in such a way that the word-symbols themselves communicate the meaning clearly. 2

(C) One reason for this, of course, is that we have ceased to use the "musical notation" that was once fashionable, probably because most modern prose is designed to be read silently, to be taken in by the eye and not by the ear. 1


① A-C-B ② B-A-C ③ B-C-A ④ C-A-B ⑤ C-B-A

 


179. 20수특 T2-7

 

Like the downtown office complex, tourism has frequently developed as islands of renewal in seas of decay.


(A) For tourists, the city can be reduced to a simulacrum, a set piece representing the city in its entirety. Thus, reduced to Harborplace or the Renaissance Center and Greektown, both Baltimore and Detroit can be presented as gleaming new places to play. 2

(B) In a hostile environment, zones of demarcation can solve seemingly insolvable problems of image and social control. Tourists who visit converted cities are unlikely to see the city of decline at all, except on their way from an airport. 1

(C) The strategy of carving out sharply demarcated and defended zones for middle-class consumers of entertainment and leisure came naturally to older cities confronted with problems of crime, poverty, and physical neglect. Creating a "tourist bubble" was tempting ― some might say necessary ― as a way not only of securing a space for development, but for achieving an efficient application of scarce resources. 0


① A-C-B ② B-A-C ③ B-C-A ④ C-A-B ⑤ C-B-A

 


180. 20수특 T2-8

 

Henry Cavendish was born in 1731 in Nice, where his parents had gone because of his mother's health, which continued to fail.


(A) From there, he proceeded to St. Peter's College (Peterhouse), Cambridge University in 1749, leaving after three years without a degree. For the next thirty-odd years, he lived at his father's house on Great Marlborough St., London. Freed from the need to support himself, he followed his inclinations, which were to study and carry out researches in the physical sciences. 1

(B) Around the time his father died, in 1783, he acquired two houses of his own, one in and one outside London, both of which he adapted to his scientific habit. He was a prominent member and administrator of the Royal Society of London. His manner of living was modest, and over time he accumulated an immense fortune. He died in 1810, at age 78. 2

(C) She died two years later, after giving birth to a second son. Henry's father never remarried. When Henry was 11, he was enrolled in Hackney Academy, a progressive school outside London. 0


① A-C-B ② B-A-C ③ B-C-A ④ C-A-B ⑤ C-B-A

 


181. 20수특 T2-9

 

The graph above shows U.S. adults' evaluation of the health risks of food additives, sorted by gender.


(A) The lowest percentage of both men and women say food and drinks with artificial coloring pose a serious health risk. 2

(B) Women (39%) are more wary than men (25%) of meat from animals given antibiotics or hormones. Men and women show the smallest difference in their evaluation of serious health risks when it comes to food and drink with artificial preservatives. 1

(C) Women are more inclined than men to view food additives as a serious health risk: More women (55%) than men (46%) say that additives in the food people eat every day pose a serious risk to their health. About four in ten women say fruits and vegetables grown with pesticides pose a great deal of health risk, while about one in four men say the same. 0


① A-C-B ② B-A-C ③ B-C-A ④ C-A-B ⑤ C-B-A

 


182. 20수특 T2-10

 

2020 Jacksonville's Annual Aisle of Lights One night a year, Jacksonville shines with thousands of twinkling lights for the Aisle of Lights celebration.


(A) The luminaria kit includes candles, paper bags, and sand. 3. Arrange the bags, each with a candle inside and weighted with sand in the bottom, along your sidewalk. 4. Light the candles on Sunday, December 13, around 6 pm and enjoy! 1

(B) 5. Don't worry about cleaning up the luminarias. Volunteers will collect them the next morning. Other Winter Activities: In addition to the Aisle of Lights celebration, there will be other festive activities around town throughout the weekend of December 12 and 13. See a listing of the activities at www.jacksonville.org/WinterActivities. 2

(C) Since 1990, on every second Sunday of December, Jacksonville residents and businesses have celebrated this annual community tradition. And on December 13 this year, every sidewalk of Jacksonville will shine again. How to Participate 1. Visit the Jacksonville Community Center from December 6 to December 12 to fill out a brief registration form and get a free luminaria kit. 2. 0


① A-C-B ② B-A-C ③ B-C-A ④ C-A-B ⑤ C-B-A

 


183. 20수특 T2-11

 

City of London Public Bike Rental Service 1.


(A) 3. RETURN YOUR BIKE You can return your bike to any station. To return your bike, find your chosen bike station and push the front wheel of your bike into an empty spot. If you have inserted the bike correctly, a green light will come on above the keypad next to the front wheel. If your bike is not returned within 24 hours, or if you have damaged the bike, you may be subject to a fine. 2

(B) OBTAIN YOUR BIKE Choose a bike and enter your printed code into the keypad. When the green light appears, pull your bike free. Adjust the seat height. Now you're ready to ride! 1

(C) HIRE YOUR BIKE At your chosen bike station, first touch the screen and follow the basic instructions for payment. Have your debit or credit card ready, as this is the only accepted method of payment. Upon payment, you will be given a printed code to unlock your bike. 2. 0


① A-C-B ② B-A-C ③ B-C-A ④ C-A-B ⑤ C-B-A

 


184. 20수특 T2-12

 

Vagueness is an obstacle to efficient communication.


(A) Sometimes people who want to avoid committing themselves to a particular course of action use vagueness as a ploy. 0

(B) A good journalist would then press for further information about precisely how this efficiency was to be achieved, forcing him to come out from behind this veil of vagueness. Or someone who was late for an appointment but didn't want to admit that this was because he'd stopped for a drink on the way might say 'Sorry I'm late, I had something I needed to do on the way here and it took slightly longer than I expected', deliberately leaving the cause of the delay vague, and exercising a particular kind of economy with the truth. 2

(C) For instance, a politician asked how precisely he intends to save money in the public sector might make vague generalisations about the need for improved efficiency, which, while true, don't commit him to any particular way of achieving this. 1


① A-C-B ② B-A-C ③ B-C-A ④ C-A-B ⑤ C-B-A

 


185. 20수특 T2-13

 

When opposites blend, they are placed on the far ends of a continuum, and between the two extremes there is a gradation that mixes the two opposites.


(A) For instance, black and white blend into each other through shades of gray. As the amount of white decreases through shades of gray, the amount of black increases. 0

(B) In a blend, pure black and pure white are diluted when combined into gray. They both lose their identity; gray is not black and it is not white. 2

(C) The two opposites are always in a proportional relationship, but at any point along the continuum there is some amount of each (except at the very extremes). In contrast, a zero-sum game where the winner takes all also establishes a proportional relationship, but at any point along the continuum there is only one or the other, black or white, and each retains its full identity.1


① A-C-B ② B-A-C ③ B-C-A ④ C-A-B ⑤ C-B-A

 


186. 20수특 T2-14

 

In sports, attendance is nearly always (98-99 percent of the time) with at least one other person.


(A) The excitement of the competition and the aura of the star power of the players on the team are such that the experience is best enjoyed in the presence of others. 2

(B) The sports fan pays a price for the right to enjoy an emotional experience with others. The fan goes to the game to be with others, to share the experience in this social exchange. 0

(C) More broadly speaking, unlike most other retail settings, large crowds have positive psychological effects. No line at the grocery checkout will make most shoppers happy, but no line to see a ballgame is a definite hint to a fan either that this is a terrible sporting event or that the fan has arrived at the stadium on the wrong date. 1


① A-C-B ② B-A-C ③ B-C-A ④ C-A-B ⑤ C-B-A

 


187. 20수특 T2-15

 

As writers, we need to share our experiences of writing and to encourage one another, but we also need to work independently.


(A) Writing needs that stocked reservoir of real but aimless sensations. When the time comes and you need them, they'll find you. 2

(B) So the point you might consider is this: if you have a piece of writing in mind, or in some half-finished state — which is actually the case for almost all writers much of the time — stop, go for a walk, draw a picture, enjoy something different, go window-shopping, or even fly-fishing, but observe, take notice, bring to your surroundings and other people an extra special level of attention, see them as unfamiliar and worth all your attention, store up what you see, hear or think, and let it lie in that part of your mind which has no obvious practical function, which isn't continually worrying about what you have to do today or next week. 1

(C) We may also need to do certain things which, paradoxically, do not involve words. 0


① A-C-B ② B-A-C ③ B-C-A ④ C-A-B ⑤ C-B-A

 


188. 20수특 T2-16

 

When asked, "what was one of your best days at work?"


(A) It was not because of the hardship, per se, but because the hardship was shared. It is not the work we remember with fondness, but the fellowship, how the group came together to get things done. 2

(B) For most of us, we have warmer feelings for the projects we worked on where everything seemed to go wrong. We remember how the group stayed at work until 3 a.m., ate cold pizza and barely made the deadline. Those are the experiences we remember as some of our best days at work. 1

(C) very few of us recount the time everything went smoothly and the big project we were working on came in on time and under budget. Considering how we work so hard to make things go well, that example should count as a pretty good day at work. But strangely, the days everything goes smoothly and as planned are not the ones we remember with fondness. 0


① A-C-B ② B-A-C ③ B-C-A ④ C-A-B ⑤ C-B-A

 


189. 20수특 T2-17

 

If you're a small herbivore, it's a bad idea to run in the face of danger.


(A) And because herds are easier to spot than single animals, roe deer live alone. But another reason for their solitary existence is the lack of food in ancient undisturbed forests. A herd of deer would have to cover a lot of territory to find sufficient food. 1

(B) Travelling long distances, however, increases the risk of coming across a pack of wolves. And so the single life is better. 2

(C) Wolves could easily follow you and attack, so it's better for you to hide. Roe deer don't run very far before they tum around and try to return to their original location, and when they do, they cross their own tracks, which confuses their pursuers ― which trail should they follow? Once they're safely back on home grass, roe deer hide in groups of small trees. 0


① A-C-B ② B-A-C ③ B-C-A ④ C-A-B ⑤ C-B-A

 


190. 20수특 T2-18

 

It was not until the advent of the Industrial Revolution — the historical period beginning in the late 1700s when the economies of the United States and many nations in Europe shifted from manual labor and hand tools to machines and factory manufacturing ― that human-produced garbage became a critical issue.


(A) As people were able to buy more and more new things, they stopped seeing value in broken and used items and began seeing these old items as trash. New forms of colorful advertising and packaging encouraged this burgeoning consumer culture. 2

(B) This economic change produced many more products for people to purchase, and it also created jobs that helped to increase wealth that could be used to purchase products. 1

(C) The full effect of the Industrial Revolution, however, was not felt until around the turn of the twentieth century, when systems of mass production and mass distribution were developed. 0


① A-C-B ② B-A-C ③ B-C-A ④ C-A-B ⑤ C-B-A

 


191. 20수특 T2-19

 

The reasons for the deficiencies in human-machine interaction are numerous.


(A) But in fact, we humans are amazingly complex. Those who have not studied human behavior often think it is pretty simple. Engineers, moreover, make the mistake of thinking that logical explanation is sufficient: "If only people would read the instructions," they say, "everything would be all right." 2

(B) Why this deficiency? Because much of the design is done by engineers who are experts in technology but limited in their understanding of people. "We are people ourselves," they think, "so we understand people." 1

(C) Some come from the limitations of today's technology. Some come from self-imposed restrictions by the designers, often to hold down cost. But most of the problems come from a complete lack of understanding of the design principles necessary for effective human-machine interaction. 0


① A-C-B ② B-A-C ③ B-C-A ④ C-A-B ⑤ C-B-A

 


192. 20수특 T2-20

 

When you buy from large corporations, you support the increasing consolidation of wealth and power in the hands of the few.


(A) Paying in cash, rather than by credit card, can also help local businesses as they are often the ones least able to afford the hefty fees the credit card companies charge them for each and every transaction. Look in the phone book for local alternatives to large corporate chains. 2

(B) Chain businesses often take those dollars directly away from smaller local businesses that cannot afford to lose the income. By making your purchases at local businesses, you spread that wealth out to more local people and increase your community's standard of living. 0

(C) This is because local businesses rely more on local suppliers and service providers, forming a kind of local economic web of interdependence that creates jobs and a thriving community. Therefore, every dollar you spend at a local business helps your community maintain its individual character, uniqueness, and diversity while supporting your neighbors in their quest for the good life. 1


① A-C-B ② B-A-C ③ B-C-A ④ C-A-B ⑤ C-B-A

 


193. 20수특 T2-21

 

Nitrogen in its gaseous form is often used in situations in which it is important to keep other, more reactive atmospheric gases away.


(A) In addition to these applications, nitrogen is used in oil production, in which it is pumped in compressed form underground to force oil to the surface. Ordinary air cannot be used for this purpose because some of the gases that make up air would react with the oil, producing undesired by-products. 2

(B) Nitrogen has recently also been used in blanketing fruit after it has been picked to protect it from rotting. Apples, for example, can be stored for up to 30 months if they are kept at low temperatures in an atmosphere of nitrogen. 1

(C) It serves industry as a blanketing gas, for example, in protecting materials such as electronic components during production or storage. To prevent the oxidation of wine, wine bottles are often filled with nitrogen after the cork is removed. 0


① A-C-B ② B-A-C ③ B-C-A ④ C-A-B ⑤ C-B-A

 


194. 20수특 T2-22

 

The bigger the group, the greater the benefits — up to a point.


(A) Even with the benefit of gossip to circulate information about other people's reputations, to boost our social capacities, if a group of humans has more than 150 members, we end up losing track of who's who. That makes the maintenance of social harmony within the group much more challenging. For human cooperatives to remain stable across populations larger than 150 people, we needed to invent God (or gods). 2

(B) It is thought to reflect limitations in how much social information a human brain can keep track of, not just regarding their own relationships, but other people's too. Our capacity to sustain larger cooperative groups than any other primate probably stems from our ability to learn not just from our own personal experience, but also from other people's experiences. 1

(C) Communities of humans tend to be relatively stable up to around 150 people. This appears to be the optimal size for a cooperative group of humans both across the world and throughout history. 0


① A-C-B ② B-A-C ③ B-C-A ④ C-A-B ⑤ C-B-A

 


195. 20수특 T2-23

 

How can we make decisions in the face of scientific uncertainty?


(A) An approach currently favored by many natural resource managers is called adaptive management, or "learning by doing." In adaptive management, policies are designed from the outset to use scientific principles to examine alternatives and assess outcomes. 1

(B) The answer is that our plans generally have to be conditional and contextual. Scientific information can help us understand environmental issues, but the policies we create based on this understanding will always depend on further study and more confirming evidence. 0

(C) Rather than assume that what seems the best initial policy option will always remain so, adaptive management sets up scientific experiments to monitor how conditions are changing, and what effects our actions (or inactions) are having on both target and nontarget elements of the system. The goal of adaptive management is to enable us to live with the unexpected. It aims to yield understanding as much as to produce answers or solutions. 2


① A-C-B ② B-A-C ③ B-C-A ④ C-A-B ⑤ C-B-A

 


196. 20수특 T2-2425

 

Recent research on solutions to social dilemmas provides an example of the positive value of regulatory authorities.


(A) In a social dilemma, a society must prevent citizens from engaging in actions that are individually beneficial in the short term but that hurt society in the long term. Studies suggest that one solution that groups voluntarily adopt when faced with social dilemmas is to designate formal leaders who are empowered to control the behavior of the group's members. Similarly, groups develop rules governing members ' conduct to preserve valuable social relationships. 0

(B) It cannot be assumed that authorities will be compassionately motivated and will use their power and legitimacy to promote the positive objectives outlined above. Although they can facilitate the productive exchange of resources to the benefit of all members of society, it is not inherent in the nature of authority that it will function in this way. The effects of authority depend on the motives of those exercising it. 2

(C) These informal rules are the precursors of formalized law. It is also important to recognize the potential dangers of giving authorities the power to affect public behavior. Authorities may use that power to advance limit their own interest, or the interest of a particular group or individual, over the interest of others. 1


① A-C-B ② B-A-C ③ B-C-A ④ C-A-B ⑤ C-B-A

 


197. 20수특 T2-2628

 

There was a young boy who used to go for regular soccer practice but always played in reserves and never made it to the playing 11.


(A) He said, "OK, son, go, play. But remember, I am going against my better judgment and the reputation of the school is at stake. Don't let me down." The game started and the boy played like a house on fire. Every time he got the ball, he scored a goal. Needless to say, he was the best player and the star of the game. His team had a spectacular win. When the game finished, the coach went up to him and said, "Son, how could I have been so wrong in my life? I have never seen you play like this before. 1

(B) While he was practicing, his father used to sit at the far end, waiting for him. The matches had started and for four days, he didn't show up for practice or quarter or semifinals. All of a sudden he showed up for the finals, went to the coach and said, "Coach, you have always kept me in the reserves and never let me play in the finals. But today, please let me play." The coach said, "Son, I'm sorry, I can't let you. There are better players than you and besides, it is the finals, the reputation of the school is at stake and I cannot take a chance." The boy pleaded, "Coach, I promise I will not let you down. I beg of you, please let me play." The coach had never seen him plead like this before. 0

(C) What happened? How did you play so well?" The boy replied, "Coach, my father is watching me today." The coach turned around and looked at the place where the boy's father used to sit. There was no one there. He said, "Son, your father used to sit there when you came for practice, but I don't see anyone there today." The boy replied, "Coach, there is something I never told you. My father was blind. Just four days ago, he died. Today for the first time he is seeing me (from above)." 2


① A-C-B ② B-A-C ③ B-C-A ④ C-A-B ⑤ C-B-A

 


198. 20수특 T3-1

 

It has come to my attention that you were responsible for this past weekend's "Halloween Scare" email that was circulated through our corporation's intranet mail system.


(A) Thank you in advance for your cooperation. 2

(B) Your expertise as a systems analyst is vital to us, but your outstanding ability does not justify your singular poor judgment. In the future, please limit your use of The Dogwood Firm's technology to work-specific business. 1

(C) The rather large graphic file you created and mass-mailed to all departments and all personnel was not only thematically and visually inappropriate, but it overloaded our servers and crashed key hubs in Charlotte, Biloxi, and Mobile, temporarily crippling business in the Southern Region. You must know that this kind of behavior cannot be condoned at The Dogwood Firm. 0


① A-C-B ② B-A-C ③ B-C-A ④ C-A-B ⑤ C-B-A

 


199. 20수특 T3-2

 

One evening, just three days before the Olympics were to begin, my father called me into the kitchen.


(A) I guess I'll just have to get a job since I didn't get into any of the schools I wanted to go to." "Well," he said, "you might not want to start filling out any job applications too soon." When I gave him a confused look, he smiled broadly. 1

(B) With a very stern look, he said, "Lisa, what do you plan to do after the Olympics are over?" His question was like a bucket of ice-cold water being poured on me. Forced to face the reality beyond the summer, I answered rather weakly, "I don't know, Dad. 0

(C) "Lisa, after you go to the Olympics — you're going to college!" Then he handed me a letter that stated that I had been accepted and was receiving financial grants to go to a four-year university! I screamed and jumped up and down. My father gave me a huge hug and we danced around the kitchen together, singing joyfully, "Go, Li-sa, Go, Li-sa" 2


① A-C-B ② B-A-C ③ B-C-A ④ C-A-B ⑤ C-B-A

 


200. 20수특 T3-3

 

Globalization and technological innovation are key drivers of socio-economic transformations.


(A) An important point is that globalization and technological innovation are not natural processes that societies must either endure or stop. 1

(B) Quite to the contrary, the particular ways in which globalization and innovation unfold can be shaped by policies and it is important to steer them in the direction of social inclusion. Therefore, not only should we make sure to support those who lose from the globalized economy and technological disruptions and ease their adaptation and transition to the new opportunities offered by these developments, but we can work to make the changes themselves occur in a way that generates less loss and more gain for all. 2

(C) Experts (not always decision-makers, unfortunately) know the virtues and dangers of the former, but there is much uncertainty about how the latter will affect quality of life and social inequalities. 0


① A-C-B ② B-A-C ③ B-C-A ④ C-A-B ⑤ C-B-A

 


201. 20수특 T3-4

 

Firms in the same industry may be able to imitate and copy one another more readily if they are located together.


(A) However, managers may not know which firm will develop leading innovations. On average, the "sharing" of information may benefit the group. 1

(B) Therefore, they may be able to respond to changes in their industry more quickly than if they were isolated from their competitors. Of course, the firm that is copied may be harmed, so in this instance, it would be better off in an isolated location, where copying would be more difficult. 0

(C) In industries with numerous and scattered innovations, such as fashion or computer games, all firms may be better off if they have locations that allow them to imitate quickly. Furthermore, a firm that copies two changes is in a better position to innovate additional changes by combining or modifying changes that were taken from other firms. Thus, particularly in fast-changing industries, economies from industrial imitation, modification, and innovation tend to be important sources of localization economies. 2


① A-C-B ② B-A-C ③ B-C-A ④ C-A-B ⑤ C-B-A

 


202. 20수특 T3-5

 

Workers are united by laughing at shared events, even ones that may initially spark anger or conflict.


(A) One team told repeated stories about a dumpster fire, something that does not seem funny on its face, but the reactions of workers motivated to preserve safety sparked laughter as the stories were shared multiple times by multiple parties in the workplace. Shared events that cause laughter can indicate a sense of belonging since "you had to be there" to see the humor in them, and non-members were not and do not. 1

(B) Instances of humor serve to enact bonds among organization members. Understanding the humor may even be required as an informal badge of membership in the organization. 2

(C) Humor reframes potentially divisive events into merely "laughable" ones which are put in perspective as subservient to unifying values held by organization members. Repeatedly recounting humorous incidents reinforces unity based on key organizational values. 0


① A-C-B ② B-A-C ③ B-C-A ④ C-A-B ⑤ C-B-A

 


203. 20수특 T3-6

 

As Marshall McLuhan famously said in the 1960s, we are now a post-literate society.


(A) Secondary schools and universities are trying desperately to keep these activities at the centre of schooling, but even some of the most conservative Ivy League schools have replaced listening to lectures and writing essays with field work, role-playing games, online discussion, and other forms of experiential, interactive "e-learning." Having a book published was once seen as the ultimate way of getting your message out to a wide audience. 1

(B) Today, a blogger can get millions of hits a day or an online video can have millions of views. If a book sells five thousand copies in Canada it is considered a bestseller. The Kony 2012 video now stands at over 96 million views — a shocking statistic for a thirty-minute video! 2

(C) Our culture has returned to a kind of medieval attitude toward print. Extended reading and writing is something small elite groups do. 0


① A-C-B ② B-A-C ③ B-C-A ④ C-A-B ⑤ C-B-A

 


204. 20수특 T3-7

 

The table above shows the American public's thoughts on which issues the president and Congress should prioritize in January 2011 and January 2019.


(A) The percentage of the public who considered improving the job situation a top priority fell by 34 percentage points over the eight-year period. Similarly, the percentage of the public who considered strengthening the nation's economy and reducing the budget deficit a top priority also declined, yet not as much as improving the job situation. 0

(B) The percentage of the public who said the president and Congress should prioritize the issues of improving education and securing Social Security did not change much during the period between 2011 and 2019. 2

(C) The percentage of the public who felt dealing with climate change should be prioritized increased substantially 一 18 percentage points 一 during the eight-year period. Additionally, reducing health care costs, dealing with problems of poor and needy people, and protecting the environment were the issues that had a higher percentage of support as top priorities in 2019 than in 2011. 1


① A-C-B ② B-A-C ③ B-C-A ④ C-A-B ⑤ C-B-A

 


205. 20수특 T3-10

 

An Ashanti funeral is often held several weeks after a death.


(A) They notify distant relatives, clean and tidy up the deceased person's house, and memorialize the departed family member on posters, T-shirts, and in other images that will be distributed to mourners. When all is ready, the body is brought back to the house and prepared for the funeral, covered with kente cloth and decorated with gold jewelry. 1

(B) During this period, while the body is preserved in a mortuary, the family of the deceased works to prepare for the event. Funerals are traditionally the responsibility of a person's abusua (matriclan); these are relatives who are linked through common maternal ancestors. 0

(C) The ceremonies can last several days. They begin with a lying-in-state at the family home, which culminates in the presentation of burial gifts for use by the deceased in his or her journey to asamando, the land of the dead. Then come the burial and finally the great funeral, a public event honoring the deceased person with music, dancing, food, and drinks. 2


① A-C-B ② B-A-C ③ B-C-A ④ C-A-B ⑤ C-B-A

 


206. 20수특 T3-11

 

Some scientists compare the brain to a relay station that merely coordinates incoming signals and outgoing responses, whereas others see it as an immense computer that processes information and then arrives at an appropriate response.


(A) When the mind selects and orders incoming information into meaning, it is telling itself a story. 2

(B) Francois Jacob, the French molecular biologist and Nobel laureate, suggests that the human mind is far more; it has a built-in need to create order out of the constant flow of information coming from its sensory organs. In other words, the brain creates a narrative, with a beginning, a middle and an end — a temporal sequence that makes sense of events. 0

(C) The brain selects and discards information to be used in the narrative, constructing connections and relationships that create a web of meaning. In this way, a narrative reveals more than just what happened; it explains why. 1


① A-C-B ② B-A-C ③ B-C-A ④ C-A-B ⑤ C-B-A

 


207. 20수특 T3-12

 

In general, searching online for health information can be valid, eye-opening, educational, and even useful.


(A) If you think that juice cleanses are the way to better health and well-being, it's easy to find websites supporting this. If delaying vaccines is your cup of tea, online sources abound. If you're debating whether to eat only organic food, plenty of available information will support this. 2

(B) While many doctors roll their eyes when they hear, "I did my research," from a patient, sometimes that research can be sound. If a patient has a rare disease and presents articles about it, many of us will be grateful that we were saved some extra work. But the Web becomes entangled when sites angled with opinions, personal anecdotes, exaggeration, and false claims manipulate the navigator to believe what is posted. 0

(C) People also run into trouble when looking for information online based on preconceived notions. Here comes the Curse of the Original Belief. If you believe that megadosing on vitamin C will prevent colds, you will seek out (and easily find) sites promoting this notion. 1


① A-C-B ② B-A-C ③ B-C-A ④ C-A-B ⑤ C-B-A

 


208. 20수특 T3-13

 

Beethoven himself raised the cash to visit Vienna at the age of 16 in the hope of persuading Mozart to give him lessons.


(A) If Beethoven had been able to prolong his stay in Vienna, he would surely have met Mozart in the end. But a summons home to his mother's deathbed prevented his persistence from being put to the test. 2

(B) Yet Beethoven may well have been right. Mozart, for all his youthfulness, already had such crookedly arthritic fingers that observers remarked on his inability to cut his meat. 1

(C) Of an actual meeting with Mozart, there is no evidence outside the fields of romantic biography. But he did, or so it seems, hear Mozart play, and complained of the 'choppiness' of his style ― a description which has to be matched against Mozart's own statement that his piano music was meant to 'flow like oil'. 0


① A-C-B ② B-A-C ③ B-C-A ④ C-A-B ⑤ C-B-A

 


209. 20수특 T3-14

 

It appears that Internet customers rarely secure the lowest price.


(A) Similarly, buyers normally use one online grocer because of the trouble of getting to know another site. In short, habits take over. 2

(B) According to a popular price comparison website, 80 percent of Internet customers pay more than they have to. It seems that use of the Internet to obtain better value is restrained by loyalty to particular websites. 0

(C) Once they are familiar with a site, consumers may return to it later because it is easy to use and saves time. A consumer might agree that a book might be cheaper elsewhere but still use the online bookstore they're familiar with because of convenience — this convenience can be seen as considerable, as often customers allow trusted sites to store their credit card and delivery details, so purchasing really is a single click. 1


① A-C-B ② B-A-C ③ B-C-A ④ C-A-B ⑤ C-B-A

 


210. 20수특 T3-15

 

Findings from several studies on nonpatient groups such as university students suggest that simply looking at everyday nature, as compared to built scenes that lack nature, is significantly more effective in promoting restoration from stress.


(A) One early study focused on students who were experiencing mild stress because of a final course exam. 0

(B) Results suggested that the nature views fostered greater psychological restoration as indicated by larger reductions in negative feelings such as fear and anger/aggression and much higher levels of positive feelings. Also, the scenes with vegetation sustained interest and attention more effectively than did the urban scenes without nature. 2

(C) A self-ratings questionnaire was used to assess restorative influences of viewing either a diverse slide sample of unblighted built settings lacking nature, or slides of undistinguished nature settings dominated by green vegetation. 1


① A-C-B ② B-A-C ③ B-C-A ④ C-A-B ⑤ C-B-A

 


211. 20수특 T3-16

 

Leonardo da Vinci had a keen interest in the reality and the wonders of nature as a broad and dynamic whole.


(A) This can be readily seen both in his drawings of anatomical structures in biology and his refined representations of mechanical structures in physics. He published amazingly detailed drawings of human anatomy, where, as one biographer noted, he paid "attention to the forms of even very small organs and hidden parts of the skeleton." 1

(B) Da Vinci is even credited with being the first in the modern world to introduce the idea of controlled experimentation 一 the core concept of science — and, for this, he has been considered by some writers to be the Father of Science. Probably more than any other scholastic luminary of that time, he recognized the relationship between the whole and its parts. 2

(C) The subject matter of his inspired paintings was almost more wondrous than reality. Da Vinci was also deeply curious about the small details that might be able to explain the human-perceived wonders he painted. 0


① A-C-B ② B-A-C ③ B-C-A ④ C-A-B ⑤ C-B-A

 


212. 20수특 T3-17

 

Many traditional sports remain important elements of contemporary national sporting cultures.


(A) In many cases, however, what are commonly assumed to be traditional sporting practices actually represent hybrid amalgamations of traditional games and imported sporting values and practices. Judo, for instance, dates from only 1882 and was developed as a modernization of traditional styles of jujitsu. 1

(B) Sumo, despite recent image problems, remains immensely popular and important in Japan. Similarly, sepak takraw and combat sports like muay thai, silat, and arnis remain permanent fixtures of Southeast Asia's sporting landscape. 0

(C) Similarly, muay thai adopted the ring, system of rounds, gloves, and weight divisions after concerns about high levels of death and injuries during the early 20th century, while sepak takraw took its current form in the 1930s, with the addition of a net and court adopted from badminton to the traditional pastime of kicking a rattan ball. 2


① A-C-B ② B-A-C ③ B-C-A ④ C-A-B ⑤ C-B-A

 


213. 20수특 T3-18

 

Although Mobile Healthcare Network applications provide numerous opportunities and benefits, they raise various security and privacy issues.


(A) Users may also worry about their critical health data being tampered with when their health data are stored in the untrusted cloud servers. 1

(B) Since the health information, eg, phenomena, health condition, emergency, is relatively sensitive for users, any inappropriate disclosure may violate user privacy and even result in property loss. 0

(C) Moreover, some malicious attackers misbehave in MHNs to disrupt the effectiveness or mislead other users' preferences. Without appropriate security and privacy protections, users may not accept MHN applications. 2


① A-C-B ② B-A-C ③ B-C-A ④ C-A-B ⑤ C-B-A

 


214. 20수특 T3-19

 

I remember as a student going to a speed-reading course of the type that was in vogue in the 1970s.


(A) The idea that you can take in the full meaning of large numbers of words in a single glance has been shown to be wrong:. yes, you can quickly understand the main point of a whole block of text at a single glance, and you can race through a book getting a pretty good idea of what the author is saying. 1

(B) We were led to believe that you could train your eye and brain to take in whole blocks of text — scores of words at a time — and that reading word by word was primitive and inefficient. It was the course that was a waste of time, however. 0

(C) But the faster you go, the more you miss. So if friends boast that they can read thick novels in an afternoon, test them on what they remember of the details. You probably can gather the plot by racing through a novel, but you'll miss much of the subtleties of the language, the scenes and the narrative. 2


① A-C-B ② B-A-C ③ B-C-A ④ C-A-B ⑤ C-B-A

 


215. 20수특 T3-20

 

Strangely enough, volcanic events have been linked to changes in the El Nifio cycle.


(A) It is not entirely clear at this point how volcanoes trigger El Ninos, but the answer may lie in the global cooling that accompanies large eruptions. According to climatologist Michael Mann, the cooling effect of volcanoes would be more notable in the western part of the Pacific Ocean, thus reducing the temperature differential between the eastern and western Pacific and setting the stage for El Nifio. 1

(B) Scientists at the National Center for Atmospheric Research have discovered a strong statistical link between El Nifio and tropical volcanic eruptions. In the years following a major eruption, they argue, the chance of El Nifio doubles compared with a standard year. 0

(C) Since the El Nifio in turn influences the formation of tropical cyclones, these findings suggest that tropical volcanoes may play a role in cyclonic weather as well. 2


① A-C-B ② B-A-C ③ B-C-A ④ C-A-B ⑤ C-B-A

 


216. 20수특 T3-21

 

On his long journey home after the Trojan War, the hero Odysseus came to an island where the goddess Circe advised him to avoid the Sirens, beautiful winged monsters whose irresistible song lured mariners to their death.


(A) The cunning hero packed his men's ears with beeswax and commanded them to tie him to the ship's mast. There he stood as they sailed into dangerous waters; the Sirens called to him, and he heard their song. 1

(B) As Circe had predicted, he longed to go to them, to cast away everything he held dear. He shouted at his men, ordered, then begged them to set him free, but the mast was strong, the rope held fast, and his men couldn't hear his pleas. And so Odysseus did not perish, but emerged on the other side of the Siren song wiser, more sensible, and prepared to complete his journey home. 2

(C) Forewarned but undaunted, Odysseus sailed into peril anyway. His plan: He would listen but not give in. 0


① A-C-B ② B-A-C ③ B-C-A ④ C-A-B ⑤ C-B-A

 


217. 20수특 T3-22

 

The concentration of large-scale economic activity has resulted in the formation of multinational companies.


(A) Such multinational companies possess considerable influence over the operations of the government of the countries in which they invest, thereby undermining the economic and political independence of such countries. In return for providing jobs and revenue derived from taxing their operations, multinational companies may demand concessions from governments as the price for their investment in that country. 1

(B) These have their headquarters in one country but their commercial activities are conducted throughout the world. Incentives for them to do this include access to raw materials and (in the case of firms locating in the third world) the availability of cheap labour. 0

(C) They may seek direct or indirect control over a country's political system to ensure that government policy is compatible with the needs of the company. If these conflict, the government may suffer: in Guatemala, for example, President Jacobo Arbenz's quarrels with the American United Fruit Company resulted in his replacement by an American-backed military government in 1954. 2


① A-C-B ② B-A-C ③ B-C-A ④ C-A-B ⑤ C-B-A

 


218. 20수특 T3-23

 

Although from very early on infants show discrimination of their mother's voice and scent, they do not exhibit a clear preference for any particular caregiver.


(A) Yet, from the beginning, babies contribute to their interactions and exchanges with others. Built-in bias to orient tow따.d, look at, and listen to certain stimuli will contribute to paying attention to and eventually developing preference for those who interact with him and provide care on a regular basis. 1

(B) Infants are not attached to their caregivers at birth. Any caregiver responding to their needs would be as effective; infants tend to respond similarly to any individual who tends to their signals or interacts with them. 0

(C) The infant uses characteristic reflexive responses in his behavioral repertoire (eg, crying, head-turning, reaching, grasping) when interacting with others. These behaviors typically have as a consequence to increase the time the baby is in proximity with those around him. 2


① A-C-B ② B-A-C ③ B-C-A ④ C-A-B ⑤ C-B-A

 


219. 20수특 T3-2425

 

Perhaps one of the most iconic memory studies was by Neisser and Harsch in 1992, in which they looked at flashbulb memories related to the Challenger explosion.


(A) Of the seven details they previously recorded, on average the students could remember only 2.95 of them. A quarter of the students scored zero out of seven, and half scored two or less. In fact, only a quarter of the students even remembered taking the survey previously. Despite their terrible recall of the event, the average score of confidence in the accuracy of their memories was as high as 4.17 out of 5. 1

(B) Other studies have also shown the lack of any relationship between confidence in a memory and its accuracy. We tend to think that vividness and confidence predict accuracy, but they don't. The clear lesson here is that we all need to be humble when it comes to the accuracy of our own memories. Failure to appreciate the true nature of memory can create great mischief. 2

(C) They gave 106 students in an introductory psychology class a questionnaire asking them to recall how they heard about the Challenger explosion, which had happened within the last twenty-four hours. They were given seven specific questions about what they were doing and how they felt at the time. Two and a half years later, the same students were given a follow-up questionnaire. In this survey they were also asked to rate their confidence in the accuracy of their memory on a scale from 1 to 5. 0


① A-C-B ② B-A-C ③ B-C-A ④ C-A-B ⑤ C-B-A

 


220. 20수특 T3-2628

 

One of the most extraordinary Olympic marathons was the one held during the 1908 London games.


(A) When John Hayes crossed the line moments later, it was he who was declared the winner. The gold medal was given to the American runner. The officials' help had disqualified Pietri. The crowd was horrified. The Olympic officials 一who should have known better 一had made a serious blunder, having been caught up in the excitement and drama of the moment. But rules are rules. Pietri may not have won the gold medal (or silver or bronze, come to that), but he was eventually presented with a special silver cup on the following day by Queen Alexandra, who was so taken by the plight and bravery of the little Italian. 2

(B) The first to cross the finishing line — in what later became known as the 'Dorando marathon' 一 was Italian pastry cook Dorando Pietri. Pietri, a small man, was an amazing runner. At 17, he once delivered an urgent message for his employer by running the 15 miles! This was the start of an impressive running career. He ran, and won, a number of marathons and then came the 1908 London Olympics. The day of the race was a hot one. 0

(C) The course of just over 26 miles ran from Windsor Castle to the White City Stadium. The streets were lined with a quarter of a million spectators. Dorando Pietri was up with the leaders from the outset and, with the finishing line in sight, he streaked into the lead, with American runner John Hayes close behind. As he entered the stadium, Pietri was suffering from heat exhaustion and fell several times inside the stadium. He kept falling and struggling back to his feet and falling down again. It was now that Olympic officials (obviously impressed by Pietri's get-up-and-go) rushed across the track, picked him up and helped him across the finishing line. 1


① A-C-B ② B-A-C ③ B-C-A ④ C-A-B ⑤ C-B-A

 


[ANSWER]
1. ② 2. ① 3. ③ 4. ⑤ 5. ⑤ 6. ⑤ 7. ④ 8. ② 9. ⑤ 10. ④


11. ④ 12. ⑤ 13. ② 14. ⑤ 15. ⑤ 16. ① 17. ④ 18. ④ 19. ② 20. ③


21. ① 22. ① 23. ④ 24. ⑤ 25. ② 26. ① 27. ② 28. ③ 29. ② 30. ③


31. ④ 32. ④ 33. ① 34. ④ 35. ① 36. ② 37. ④ 38. ② 39. ② 40. ④


41. ② 42. ② 43. ⑤ 44. ⑤ 45. ③ 46. ② 47. ① 48. ④ 49. ① 50. ①


51. ③ 52. ① 53. ② 54. ② 55. ⑤ 56. ② 57. ② 58. ③ 59. ② 60. ②


61. ② 62. ⑤ 63. ② 64. ⑤ 65. ① 66. ④ 67. ⑤ 68. ③ 69. ④ 70. ③


71. ⑤ 72. ③ 73. ④ 74. ④ 75. ⑤ 76. ⑤ 77. ④ 78. ③ 79. ④ 80. ②


81. ④ 82. ② 83. ④ 84. ② 85. ③ 86. ⑤ 87. ③ 88. ⑤ 89. ③ 90. ③


91. ④ 92. ③ 93. ① 94. ① 95. ⑤ 96. ① 97. ① 98. ④ 99. ① 100. ③


101. ① 102. ⑤ 103. ④ 104. ③ 105. ① 106. ③ 107. ③ 108. ② 109. ② 110. ①


111. ④ 112. ① 113. ⑤ 114. ② 115. ③ 116. ② 117. ① 118. ③ 119. ① 120. ③


121. ② 122. ② 123. ③ 124. ⑤ 125. ② 126. ⑤ 127. ① 128. ② 129. ③ 130. ⑤


131. ④ 132. ⑤ 133. ③ 134. ③ 135. ③ 136. ① 137. ⑤ 138. ⑤ 139. ③ 140. ①


141. ⑤ 142. ④ 143. ④ 144. ⑤ 145. ① 146. ② 147. ⑤ 148. ⑤ 149. ④ 150. ③


151. ① 152. ③ 153. ② 154. ② 155. ② 156. ④ 157. ② 158. ④ 159. ⑤ 160. ②


161. ③ 162. ① 163. ③ 164. ⑤ 165. ① 166. ③ 167. ③ 168. ① 169. ⑤ 170. ⑤


171. ⑤ 172. ④ 173. ① 174. ① 175. ② 176. ③ 177. ② 178. ① 179. ⑤ 180. ④


181. ⑤ 182. ④ 183. ⑤ 184. ① 185. ① 186. ③ 187. ⑤ 188. ⑤ 189. ④ 190. ⑤


191. ⑤ 192. ③ 193. ⑤ 194. ⑤ 195. ② 196. ① 197. ② 198. ⑤ 199. ② 200. ④


201. ② 202. ④ 203. ④ 204. ① 205. ② 206. ③ 207. ③ 208. ⑤ 209. ③ 210. ①


211. ④ 212. ② 213. ② 214. ② 215. ② 216. ④ 217. ② 218. ② 219. ④ 220. ③


 

728x90
반응형

728x90
반응형

enk1 | Since 2005 임희재 | 블루티쳐학원 | 01033383436 | 200710 22:53:52

 

순서배열

 

1. 영능김1 1-1

 

Can I Swim?


(A) His name was Diego Gonzales. Diego was a shy boy who seemed to be frightened of his own shadow. He struggled with his classes, and none of the after-school activities seemed to suit him. He was neither strong nor athletic. On the rugby field, nobody passed the ball to him or involved him in the game, except to make fun of him. 2

(B) Tom Michell is a British teacher. In the 1970s, he worked at an English language boarding school in Argentina. One day he visited a beach and found a penguin that was injured and alone. Michell decided to help the bird. 0

(C) He cleaned it, fed it, and even gave it a name—Juan Salvado. From that day on, Juan Salvado lived on the terrace of his room in the school dormitory. The following is part of the story, written by Tom Michell, about the penguin and a boy at the school. From the first day that I brought a penguin to live at the school, one student in particular wanted to help with his care. 1


① A-C-B ② B-A-C ③ B-C-A ④ C-A-B ⑤ C-B-A

 


2. 영능김1 1-2

 

Diego's early education had not prepared him well for life at his new school.


(A) One day, I took Juan Salvado to the school swimming pool with the boys. As soon as the other swimmers left, we brought Juan Salvado to the water to see if he would swim. Juan Salvado had been living at the school for several months by then. However, in all that time, he had never been able to swim because his feathers had been damaged. 2

(B) His knowledge of English was limited, so he avoided conversation. However, Diego enjoyed the company of Juan Salvado. Indeed, on the terrace, Diego could relax. 0

(C) He had some friends who also had trouble fitting in. Looking after Juan Salvado was good for those boys. They fed him fish, swept the terrace, and spent time with him. 1


① A-C-B ② B-A-C ③ B-C-A ④ C-A-B ⑤ C-B-A

 


3. 영능김1 1-3

 

"Go on!" I said.


(A) I had never had the opportunity to study a penguin in the water before. I was familiar with the awkward way that Juan Salvado walked on land, but now I watched in awe. Using only a stroke or two, he flew at great speed from one end of the pool to the other, turning swiftly before touching the sides. It was amazing! 1

(B) The penguin stared at me and then at the pool, like he was asking, "Is this where the fish come from?" Without further encouragement, he jumped in. With a single movement of his wings, he flew like an arrow across the water and knocked into the wall on the opposite side. Luckily, he was not hurt! 0

(C) Everyone could see how much he was enjoying himself. "Ooh", The boys shouted, as though they were watching a fireworks display. After a while, Diego came over and asked quietly, "Can I swim, too?" 2


① A-C-B ② B-A-C ③ B-C-A ④ C-A-B ⑤ C-B-A

 


4. 영능김1 1-4

 

I was astonished.


(A) Not only could Diego swim, but he swam magnificently! He chased after Juan Salvado, and they swam in perfect harmony. It was like a duet written for violin and piano. Sometimes Juan Salvado took the lead and Diego followed after him. At other times Diego went ahead and the penguin swam around the boy. Occasionally they swam so close that they almost touched. 2

(B) He had never gone near the pool before. I was not even sure if he could swim. "The water is cold, and it's getting late. Are you sure you want to go in?" I asked. "Please!" 0

(C) "All right then," I said, "but be quick!" I had never seen him so excited before. His eyes were shining with joy, and he seemed to be truly alive for the first time. Without hesitating, he dived into the cold water. I was ready to jump in and rescue him if he could not swim. However, I soon realized that I did not have anything to worry about. 1


① A-C-B ② B-A-C ③ B-C-A ④ C-A-B ⑤ C-B-A

 


5. 영능김1 1-5

 

I was almost speechless.


(A) Suddenly Diego was not the sad little boy we had become used to. He was a very normal boy with a very special talent. "Diego! You can swim!" 0

(B) He asked without looking directly at me, but I saw a smile on his face. As we returned to the dormitory, Diego told me that his father had taught him how to swim in the river by their home. It was the first time he had talked about his life. I listened in silence, without making any corrections to his English, as he talked nonstop all the way back to the dormitory. 2

(C) "Yes, I can swim." "I mean you are able to swim really well. Brilliantly, in fact!" "Do you think so?" 1


① A-C-B ② B-A-C ③ B-C-A ④ C-A-B ⑤ C-B-A

 


6. 영능김1 1-6

 

The events of that day were extraordinary.


(A) He had earned the respect of his classmates. Over the next few weeks, his grades improved and he became more popular. Thanks to a swim with a penguin, a lonely boy's life was changed forever. 2

(B) Diego's confidence grew quickly after that day. When the school had a swimming competition, he won every race he participated in. The encouragement and acknowledgement given by the other boys was genuine. 1

(C) A child had gone down to the water to swim with a penguin, and shortly afterward, a young man had emerged. The ugly duckling had become a swan. It was definitely a turning point. 0


① A-C-B ② B-A-C ③ B-C-A ④ C-A-B ⑤ C-B-A

 


7. 영능김1 2-1

 

Opposite Personalities, Great Partnerships Everybody is unique.


(A) Introverts recharge their batteries by spending some time alone; extroverts need to recharge when they do not socialize enough. Extroverts are good at performing tasks under pressure and coping with multiple jobs at once. Introverts, on the other hand, like to focus on one task at a time and can concentrate very well. 1

(B) However, there have been many attempts to categorize people's personalities. One of the most common methods divides people into two types, introverts and extroverts. According to this division, introverts tend to be drawn to the internal world of thoughts and feelings, while extroverts are drawn to the external world of people and activities. 0

(C) Extroverts tend to do assignments quickly. They make fast decisions and are comfortable with taking risks. Introverts often work more slowly and deliberately. They think before they act, give up less easily, and work more accurately. 2


① A-C-B ② B-A-C ③ B-C-A ④ C-A-B ⑤ C-B-A

 


8. 영능김1 2-2

 

Based on all this information, you might think that introverts and extroverts do not get along.


(A) At that time in Montgomery, buses were divided into two zones: one for black people and the other for white people. She took a seat in the black zone and watched quietly as more and more passengers got on the bus. Soon, all the seats in the white zone were taken. Then the driver ordered her to give her seat to a white passenger. 1

(B) Rosa Parks was a shy, mild-mannered introvert. She avoided standing out in public or drawing attention to herself. However, she had the courage to resist injustice, so she answered calmly with a single word—"No." The furious driver called the police, and she was arrested. 2

(C) However, they actually work well together because their personalities complement each other. Sometimes they can even accomplish great things when they collaborate. Let's take a look at some famous examples! Case One: Working Together for Civil Rights On December 1, 1955, in the American city of Montgomery, Alabama, a black woman named Rosa Parks got on a bus. 0


① A-C-B ② B-A-C ③ B-C-A ④ C-A-B ⑤ C-B-A

 


9. 영능김1 2-3

 

Parks's calm response to the situation impressed many people.


(A) He then praised Parks's bravery and hugged her. She stood silently. Her mere presence was enough to strengthen the crowd. 2

(B) Soon after, her quiet resistance came together with the inspirational speechmaking of Martin Luther King Jr. When 5,000 people assembled at a rally to support Parks's act of courage, King made a speech to the crowd. He was an extrovert—assertive, sociable, and good at motivating people. 0

(C) "There comes a time when people get tired of being trampled," he told them. "There comes a time when people get tired of being pushed out of the sunlight." King was an amazing speaker, and his words filled the people with pride and hope. 1


① A-C-B ② B-A-C ③ B-C-A ④ C-A-B ⑤ C-B-A

 


10. 영능김1 2-4

 

Rosa Parks's act and Martin Luther King Jr.'s speech inspired Montgomery's black community to boycott the buses, a crucial turning point in the struggle for civil rights.


(A) The boycott lasted for 381 days. It was a difficult time for everyone, but eventually the buses were integrated. 0

(B) Similarly, Rosa Parks could not have excited the crowd at the rally with her words. When their introverted and extroverted traits were combined, however, his charisma attracted attention to her quiet bravery. In the end, this partnership had a huge impact on society. 2

(C) Think about how the partnership of these two people accomplished this. A powerful speaker refusing to give up his seat on a bus would not have had the same effect. 1


① A-C-B ② B-A-C ③ B-C-A ④ C-A-B ⑤ C-B-A

 


11. 영능김1 2-5

 

Case Two: A Business Partnership On June 29, 1975, Steve Wozniak tapped a few keys on his keyboard, and letters appeared on a screen.


(A) He had just created a personal computer that allowed people to type on a keyboard and see the results on a monitor simultaneously. At the sight of the brilliant device, Steve Jobs suggested to Wozniak that they start a business. 0

(B) In fact, the two men formed one of the most famous partnerships of the digital era. Wozniak would come up with a clever engineering idea, and Jobs would find a way to polish, package, and sell it. 2

(C) Wozniak was a great inventor. When he partnered with Jobs, however, he was able to do much more. 1


① A-C-B ② B-A-C ③ B-C-A ④ C-A-B ⑤ C-B-A

 


12. 영능김1 2-6

 

The two men had opposite personalities.


(A) Wozniak hated small talk and often worked alone. It was these features of his introverted personality that enabled him to focus on inventing things. Jobs, on the other hand, had outstanding social skills. 0

(B) Obviously, the answer is neither. The world needs both introverts and extroverts, and they often make a terrific team. We simply need to respect different personalities as well as our own. Then, when we have a chance to work together, we might be able to do great things! 2

(C) According to Wozniak, he was good at communicating with people. Wozniak was a shy inventor, whereas Jobs was a daring entrepreneur, but they were alike in that neither was afraid to face challenges that seemed impossible. So which personality type is better? 1


① A-C-B ② B-A-C ③ B-C-A ④ C-A-B ⑤ C-B-A

 


13. 영능김1 3-1

 

Turn Off the Lights and Save a Tiger.


(A) How does this work? Let's take a look. A Species in Danger Tigers, one of the world's largest feline species, have long been the kings of Asia's forests. 1

(B) Despite being the dominant predators of their habitats, they move silently and remain unseen most of the time. Imagine how ancient people must have felt when encountering tigers in the wild! It is no surprise that tigers have been feared and worshipped by humans for centuries, standing as symbols of power and courage. The fact that ancient rock paintings feature images of tigers shows how closely tigers have been related to humans throughout history. 2

(C) You can protect tigers simply by switching off the lights. This may sound strange, but it is actually true. An everyday action that helps us save energy can also help save an endangered species. 0


① A-C-B ② B-A-C ③ B-C-A ④ C-A-B ⑤ C-B-A

 


14. 영능김1 3-2

 

At one time, tigers were found all across Asia, from Korea to Turkey.


(A) However, the world's tiger population has been shrinking rapidly. Illegal hunting and habitat loss are the main reasons behind this decrease. 0

(B) In fact, it is now estimated that there are fewer than 4,000 tigers living in the wild. Some experts even predict that the last of the world's wild tigers will disappear within the next 10 years. 2

(C) At the start of the 20th century, it was estimated that there were approximately 100,000 wild tigers. In recent years, however, three of the nine subspecies of tigers have become extinct. 1


① A-C-B ② B-A-C ③ B-C-A ④ C-A-B ⑤ C-B-A

 


15. 영능김1 3-3

 

Our Interconnected World.


(A) This is because all of Earth's species are interconnected. Think about what would happen if tigers became extinct. Existing at the top of the food chain, they maintain the populations of animals they prey on, such as deer and boar. Without tigers, these species would rapidly increase in number. As a result, their food source, vegetation, would begin to disappear. 1

(B) This would cause birds and insects to lose their homes, and bigger animals that prey on them would soon run out of food. Eventually, the entire ecosystem would be affected. Humans are no exception, as we rely on nature for everything we need to survive, including air, food, and water. This is how the disappearance of a single species can threaten the whole planet. 2

(C) It would be very sad if there were no more wild tigers. Would it really matter, though? After all, we could still see them in zoos or watch programs about them on TV. Shouldn't we be more worried about protecting human beings? The fact is, however, that we need to protect tigers in order to protect ourselves. 0


① A-C-B ② B-A-C ③ B-C-A ④ C-A-B ⑤ C-B-A

 


16. 영능김1 3-4

 

Now imagine what would happen if we made the effort to save tigers.


(A) This is an ecological term referring to species that live in a large area containing a variety of different ecosystems. 1

(B) If we choose to protect these species, we must conserve their habitat. As a result, the other species that share this habitat, including trees and insects, are protected too, as if there were a large umbrella being held over them. 2

(C) Tigers are considered an "umbrella species." 0


① A-C-B ② B-A-C ③ B-C-A ④ C-A-B ⑤ C-B-A

 


17. 영능김1 3-5

 

Small Efforts with Big Results.


(A) If Earth's oceans continue to rise, this area could be wiped out and its tiger population could be reduced by as much as 96%. By conserving energy, however, we can slow climate change, and this will slow the rise of the oceans. So keep switching off the lights whenever you're the last person to leave a room! 2

(B) When fossil fuels are burned, carbon dioxide is released into the air, and this contributes to climate change. Climate change has a number of negative effects, including rising sea levels that threaten many parts of the world. One of these places, called the Sundarbans, is an area on the coast of Bangladesh inhabited by a large number of tigers. 1

(C) Now, it is obvious that we must protect tigers. You may, however, still wonder how switching off the lights helps. Well, the lights in our homes require electricity, and more than half of the world's electricity is created by burning fossil fuels. 0


① A-C-B ② B-A-C ③ B-C-A ④ C-A-B ⑤ C-B-A

 


18. 영능김1 3-6

 

You can also protect tigers when shopping.


(A) You could volunteer at a nonprofit organization or share important information on social networking sites. However small your actions may seem, they can help make a big difference. Most importantly, you must remember that we all share the same planet. If a single species disappears, every other living creature, including human beings, could be affected. 2

(B) Many popular products, including chocolate, instant noodles, and soap, are made with palm oil. Unfortunately, forests where tigers live are being destroyed to build more and more palm oil plantations. Some palm oil, however, is produced in a more sustainable way. 0

(C) Products that use this environmentally friendly palm oil usually have a special mark on the label. Look for it the next time you go shopping! Get Involved There are many other things you can do to protect tigers and other endangered species. 1


① A-C-B ② B-A-C ③ B-C-A ④ C-A-B ⑤ C-B-A

 


19. 영능김1 4-1

 

A Protector of Our National Heritage.


(A) Yunju, a high school student, went to a Korean art exhibition. She wrote a report about her experience to share with her class. Last week, I visited an exhibition of artwork and ancient items selected from the Kansong Art Museum's collection. The exhibition included information about the man who gathered all of the artwork displayed there. 0

(B) This decision was greatly influenced by his mentor, Oh Sechang, who was an independence activist and had keen insight into Korean art. With Oh's guidance and his own convictions, Kansong devoted most of his fortune to acquiring old books, paintings, and other works of art. He considered these items the pride of the nation and believed they represented the national spirit. Without his actions, they would have been destroyed or taken overseas. 2

(C) His name was Jeon Hyeongpil, but he is better known by his pen name, Kansong. He was born into a rich family in 1906 and lived through the Japanese occupation of Korea. At the age of 24, he inherited a massive fortune. After carefully thinking about what he could do for his country, he decided to use the money to protect Korea's cultural heritage from the Japanese. 1


① A-C-B ② B-A-C ③ B-C-A ④ C-A-B ⑤ C-B-A

 


20. 영능김1 4-2

 

As soon as I walked in, I could not help but admire some ink-and-water paintings by Jeong Seon, a famous Korean artist also known as Gyeomjae.


(A) Fortunately, it was rescued at the last minute and later purchased by Kansong. Knowing that these beautiful paintings were nearly turned to ashes made me feel very sad. I am thankful that these paintings are still around so that future generations can also appreciate them. 2

(B) The way Gyeomjae painted the mountains, rivers, and valleys makes them look very inviting. I was shocked when the museum tour guide said that the album was almost burned as kindling. 1

(C) These paintings were kept in an album called the Haeak jeonsincheop. They depict the beautiful scenery of Geumgangsan Mountain and its surrounding areas. 0


① A-C-B ② B-A-C ③ B-C-A ④ C-A-B ⑤ C-B-A

 


21. 영능김1 4-3

 

The next item that impressed me was a gorgeous porcelain vase called the Celadon Prunus Vase with Inlaid Cloud and Crane Design.


(A) With the money he spent on it, Kansong could have bought 20 nice houses! Later, a different Japanese collector offered double the price Kansong had paid for the vase. However, Kansong refused to part with it because he knew that it was the most magnificent vase of its kind. 1

(B) Today it is listed as one of Korea's National Treasures. Seeing it in person was an absolutely breathtaking experience! 2

(C) It is a pleasant shade of green, with a lovely pattern of clouds and cranes encircling the entire vase. The cranes seem to be alive and stretching their wings in search of freedom. Kansong bought the vase from a Japanese art dealer in 1935. 0


① A-C-B ② B-A-C ③ B-C-A ④ C-A-B ⑤ C-B-A

 


22. 영능김1 4-4

 

Finally, I saw the one item in the museum that I will never forget – an original copy of the Hunminjeongeum Haerye.


(A) Schools were forbidden to teach lessons in Korean, and scholars who studied Korean were arrested. 2

(B) At that time, however, Korea was still occupied by Japan. The Japanese colonial government intended to get rid of the Korean language. 1

(C) This ancient book was written in 1446, and it explains the ideas and principles behind the creation of Hangeul, the writing system of the Korean language. It was found in Andong in 1940. 0


① A-C-B ② B-A-C ③ B-C-A ④ C-A-B ⑤ C-B-A

 


23. 영능김1 4-5

 

From the moment he heard that the Hunminjeongeum Haerye had been discovered, Kansong couldn't stop thinking about it.


(A) It has since been designated a National Treasure of Korea and included in the UNESCO Memory of the World Register. Looking at the ancient book, I could feel Kansong's strong commitment to preserving Korean history. 2

(B) He knew he had to protect it at all costs. After years of waiting, he was finally able to obtain the book. He purchased it at ten times the price the owner was asking and carefully hid it in his house. 0

(C) When the Japanese were finally defeated, he was able to share it with the rest of Korea. The guide said that the Hunminjeongeum Haerye is the museum's most precious treasure. Without it, the origins and fundamentals of Hangeul would have been lost to history. 1


① A-C-B ② B-A-C ③ B-C-A ④ C-A-B ⑤ C-B-A

 


24. 영능김1 4-6

 

Standing in the middle of the exhibition hall, surrounded by Korean art, I could not stop thinking about Kansong.


(A) He used the building as a place to store all of the important cultural items he had collected over the years. Kansong died in 1962, and Bohwagak was renamed the Kansong Art Museum in 1966. It now holds about 5,000 items, including 12 Korean National Treasures. 2

(B) He was an amazing person! He did not collect art for his personal enjoyment. He did it to protect Korea's cultural identity during the harsh Japanese colonial period. After Korea regained its independence, he stopped collecting art, as he knew it would safely remain in Korea. 0

(C) During our country's worst time, a single man was able to defend Korea's national spirit and pride. Thanks to him, we are still able to experience an essential part of Korean culture today. Founded in 1938, the Kansong Art Museum was Korea's first private museum. When Kansong built it, he named it Bohwagak. 1


① A-C-B ② B-A-C ③ B-C-A ④ C-A-B ⑤ C-B-A

 


25. 영능김1 5-1

 

Nature: The Great Inspiration of Architects Nature is all around us.


(A) The act of creating things based on nature is called "biomimicry." This term is derived from the Greek words bios, meaning "life," and mimesis, meaning "imitation." 1

(B) It impresses us with its beauty and supplies us with everything we need to survive. It also provides some people with the inspiration to create things in a new way. 0

(C) Architects who use biomimicry look at nature as an incredibly successful engineer who has already come up with answers to some of the problems they now face. They carefully study plants, animals, and other aspects of nature to learn how they work. As a result, they have been able to find some innovative solutions to engineering and architectural challenges. 2


① A-C-B ② B-A-C ③ B-C-A ④ C-A-B ⑤ C-B-A

 


26. 영능김1 5-2

 

The Curving Beauty of Nature.


(A) The Sagrada Familia is an enormous church in Barcelona, Spain. Designed by the world-famous architect Antoni Gaudi, the church is one of the most prominent buildings in the world. Construction of this remarkable building began in 1882, and Gaudi took over responsibility for its design in 1883. Believe it or not, the building is still under construction. 0

(B) Many parts of the church incorporate images and forms from nature. For example, the church's spires are topped with spheres that resemble fruits. There are also turtles carved into the stone bases of columns and spiral stairs that resemble the shells of sea creatures. 2

(C) Some people love the Sagrada Familia and others hate it, but nearly everyone is fascinated by its unique design. Gaudi believed that all architects should look to nature for inspiration. He preferred the curves found in natural objects to the straight lines found in artificial ones. This preference can be seen in all his buildings, including the Sagrada Familia. 1


① A-C-B ② B-A-C ③ B-C-A ④ C-A-B ⑤ C-B-A

 


27. 영능김1 5-3

 

Perhaps the most impressive feature of the Sagrada Familia is the ceiling.


(A) These tree-like columns are not just for decoration, though. Inspired by trees, Gaudi gave the columns a single base that splits off into branches near the top. 1

(B) This allows them to support the roof better by distributing its weight evenly. Because Gaudi recognized the superiority of natural forms, he was able to design a building that is both beautiful and functional. 2

(C) Gaudi designed the columns inside the church to resemble trees and branches, so visitors who look up can feel as if they were standing in a great forest. The light that comes through the small holes all over the ceiling even resembles the light beaming through leaves in a forest. 0


① A-C-B ② B-A-C ③ B-C-A ④ C-A-B ⑤ C-B-A

 


28. 영능김1 5-4

 

A Lesson from Insects.


(A) It has a large central chimney and smaller outer chimneys that are close to the ground. The heat generated by the daily activity of the termites rises up through the central chimney, eventually escaping through the top of the mound. In the meantime, cooler air is pulled in through the smaller chimneys, keeping the termites' home at a comfortable temperature during the hot day. 2

(B) To solve this problem, the building's architect, Mick Pearce, turned to termite mounds for an alternative. Termite mounds are large structures built by certain termite species. Scientists believe that the mounds stay cool due to a constant flow of air. Each mound has a network of holes referred to as chimneys. 1

(C) The Eastgate Centre is an office building and shopping complex in Harare, Zimbabwe. Built in 1996, it might not be as visually impressive as the Sagrada Familia. However, the building is an excellent example of biomimicry. Due to the hot climate of Harare, air conditioning systems can be very costly to install, run, and maintain. 0


① A-C-B ② B-A-C ③ B-C-A ④ C-A-B ⑤ C-B-A

 


29. 영능김1 5-5

 

Also, the soil surrounding the mound absorbs heat in the hot daytime hours.


(A) Therefore, the temperature inside the mound does not increase greatly and stays relatively cool. At night, when the outside temperature goes down, the heat is finally released. This process inspired Pearce to design an innovative climate control system. The Eastgate Centre was constructed without a conventional cooling system. 0

(B) There are openings near the base of the building, and outside air comes into the building through them. This air is moved through the building by a system of automatic fans. Eventually, the air, along with heat generated by human activity during the day, rises upward through the building's internal open spaces and is released through chimneys on the roof. 2

(C) Instead, Pearce used building materials that can store large amounts of heat. The floors and walls of the building absorb heat during the day, just like the soil of a termite mound. The heat is released at night, and the walls cool down, ready to store heat again by the next morning. The structure of the building also helps keep the building cool. 1


① A-C-B ② B-A-C ③ B-C-A ④ C-A-B ⑤ C-B-A

 


30. 영능김1 5-6

 

As a result, the building has not only cool temperatures but also fresh air.


(A) Using biomimicry in architecture is just one way that humans are utilizing the lessons of nature to improve the way we do things. Biomimicry is also being used to solve problems in the fields of robotics, agriculture, and many others. 1

(B) More importantly, the Eastgate Centre uses far less energy than other buildings, which saves money and helps protect the environment from pollution. Without the inspiration Pearce received from tiny termites, none of this would have been possible. 0

(C) Imitating the ideas of nature not only helps solve problems, but it also makes us feel closer to nature. As a result, humans are more likely to stop destroying the environment and start becoming part of it instead. 2


① A-C-B ② B-A-C ③ B-C-A ④ C-A-B ⑤ C-B-A

 


31. 영능김1 SP-1

 

Coach Ken Carter takes over the head coaching job for the Richmond High School basketball team.


(A) Maybe I should speak louder. I'm Ken Carter, your new basketball coach. (One of the players tosses a ball to another player, ignoring Coach Carter.) You, what's your name, sir? Lyle: Jason Lyle, but I'm not a sir. 2

(B) He comes up with an idea. Carter: Good afternoon, gentlemen. I'm your new basketball coach, Ken Carter. (The players ignore him and chat with each other.) 1

(C) The school is located in a poor neighborhood. Many students at the school drop out and end up living difficult lives. The team has had a losing record for several years, and the team members keep blaming each other for losing. Carter finds out that their problem is not how they play basketball, but that they lack respect for themselves. 0


① A-C-B ② B-A-C ③ B-C-A ④ C-A-B ⑤ C-B-A

 


32. 영능김1 SP-2

 

Carter: Starting today, you are a sir.


(A) Cruz: This is crazy! By the way, why are you wearing a suit and tie? Carter: What's your name, sir? Cruz: Timo Cruz, sir. Carter: Well, Mr. Cruz, when we treat ourselves with respect. (Cruz does not listen and starts bouncing the ball.) All right, Mr. Cruz, leave the gym right now. 1

(B) You all are. "Sir" is a term of respect. All of you will have my respect until you abuse it. (He gives out some pieces of paper.) These are contracts. If you sign and honor this, we'll be successful. It states that you will attend all of your classes and maintain a 2.3 grade point average. 0

(C) Cruz: Fine. I don't need to listen to you anymore. I quit. (He walks out of the gym.) Carter: Is there anybody else who doesn't want to sign this contract? (Students look at each other. Two more students leave the gym and the others stay.) 2


① A-C-B ② B-A-C ③ B-C-A ④ C-A-B ⑤ C-B-A

 


33. 영능김1 SP-3

 

Carter teaches the players to stay focused and play together as a team.


(A) Other players: That's impossible! (Cruz starts doing push-ups. Carter continues coaching.) (On Friday) Carter: It's Friday, but you still have 100 push-ups and 100 laps to go. It's time to give up, Mr. Cruz. Lyle: I'll do them for him, Coach. 1

(B) You said we're a team. When one person struggles, we all struggle. When one player triumphs, we all triumph, right? Kenyon: I'll do some, too. Come on, guys, let's help him out! (Other players do push-ups together with Cruz. Carter seems impressed.) 2

(C) Cruz realizes how tough life is off the court and how much basketball means to him. Later, the team wins an inspiring victory while Cruz watches from the crowd. Eventually, he decides to come back. (Cruz comes into the gym.) Cruz: How can I get back on the team? Carter: If you want to rejoin the team, you need to do 1,000 pushups and 1,000 laps around the gym—by Friday. 0


① A-C-B ② B-A-C ③ B-C-A ④ C-A-B ⑤ C-B-A

 


34. 영능김1 SP-4

 

With Cruz back on the team, they go undefeated for the rest of the season, and they even win a big tournament.


(A) What's more important than winning is respecting the rules and taking responsibility for your behavior. If you don't realize that, you'll never succeed or even adjust to the real world. I believe honoring this contract is the first step for you to take responsibility in your life. Now, I want you to go home, look at your lives tonight, and ask yourself, "Do I want a better life?" If the answer is yes, then I promise you I will do everything in my power to get you to a better life. (The athletes are deep in thought.) 2

(B) However, Carter later discovers that some of the students have been skipping classes and getting failing grades. He decides to do something. (In front of the gym door) Kenyon: What's up, Cruz? Cruz: I don't know. This note says that practice has been canceled and the coach is waiting in the school library. Kenyon: The library? I don't even know where the library is! 0

(C) (In the school library) Carter: Gentlemen, in this hand, I have the contracts you signed. In this hand, I have academic progress reports from your teachers. The gym will stay locked until we all satisfy the terms of this contract. Cruz: But why? We're undefeated, and we won the tournament. Didn't you want us to win? Carter: That's not the point. 1


① A-C-B ② B-A-C ③ B-C-A ④ C-A-B ⑤ C-B-A

 


35. 영능김1 SP-5

 

(In Carter's office) Principal: You locked the gym?


(A) Principal: So you take away the one thing that they're good at? We both know that for some of these boys, this basketball season will be the best part of their lives. Carter: Don't you think that's the problem? 2

(B) Are you crazy? Carter: Nobody expects them to go to college, Ms. Garrison. 0

(C) Nobody even expects them to graduate from high school. We need to stop neglecting them and make them realize that they can do more. 1


① A-C-B ② B-A-C ③ B-C-A ④ C-A-B ⑤ C-B-A

 


36. 영능김1 SP-6

 

The coach's act angers the parents who expected the team to keep winning.


(A) Other parents: (shouting) No! Board Chairman: Quiet! Let's hear from Coach Carter. 1

(B) Eventually, the school board confronts Carter. Parent 1: Basketball is all these boys have. Should Carter be allowed to take that away from them? 0

(C) Carter: You really need to consider the message that you're sending these boys: that they are above the law. I'm trying to teach them discipline. If these kids don't honor a simple contract, it won't be long before they're out there in society breaking laws. 2


① A-C-B ② B-A-C ③ B-C-A ④ C-A-B ⑤ C-B-A

 


37. 영능김1 SP-7

 

Despite Carter's speech, the board votes to unlock the gym.


(A) We're going to fulfill the contract, sir. Now we know what's important in life, and it's all because of you. Thank you, sir. Carter: Gentlemen, there's only one way to say this: We've achieved our goal. 1

(B) The team members study hard and raise their grades enough to fulfill the contract. Later, the team competes in the state basketball tournament. The players fail to win the final, but they achieve something far more important:. Belief in themselves, as well as hope for a brighter future in life beyond the basketball court. 2

(C) Carter is about to resign, but he decides to visit the gym one last time. There, he sees something shocking. All the players are studying together. Cruz: Sir, they can open the doors of the gym, but they can't make us play. 0


① A-C-B ② B-A-C ③ B-C-A ④ C-A-B ⑤ C-B-A

 


[ANSWER]
1. ③ 2. ③ 3. ② 4. ③ 5. ① 6. ⑤ 7. ② 8. ④ 9. ③ 10. ①


11. ① 12. ① 13. ④ 14. ① 15. ④ 16. ④ 17. ⑤ 18. ③ 19. ① 20. ⑤


21. ④ 22. ⑤ 23. ③ 24. ③ 25. ② 26. ① 27. ④ 28. ⑤ 29. ① 30. ②


31. ⑤ 32. ② 33. ④ 34. ③ 35. ③ 36. ② 37. ④ 

728x90
반응형

728x90
반응형

enk | Since 2005 임희재 | 블루티쳐학원 | 01033383436 | 200710 22:49:58

 

PARA

 

1. 영능김 1-1

 

The Final Touchdown With only two minutes to play, both teams were fighting for the football.


(A) One of the seniors, Ethan, was especially happy. He had never played in any of the games before. Now, Ethan was finally getting the chance to step onto the grass. 1

(B) It was the last home game for the seniors of Winston High, and they were determined to win. Since it had been a close game the whole evening, the best players of each team hadn't left the field. Once Winston High's coach finally knew that victory was theirs, all the seniors on the sidelines were allowed to play for the last few seconds. 0

(C) When the rival team dropped the ball, one of our players recovered it and quickly ran down the field with it. Ethan ran right after him to catch up. As our player got closer to the end zone, he saw Ethan behind him on his left. Instead of running straight ahead, the player kindly passed the ball to Ethan so that he could score a touchdown. 2


① A-C-B ② B-A-C ③ B-C-A ④ C-A-B ⑤ C-B-A

 


2. 영능김 1-2

 

All eyes were on Ethan.


(A) Ethan's touchdown didn't win the game, but it will be worth remembering. By now you're probably wondering why. 2

(B) With the ball in his hands, everything seemed to be moving in slow motion, like in a Hollywood movie. People kept their eyes on him as he made his way to the end zone. They saw him cross the goal line right before the clock ran out. 0

(C) Unexpectedly, everyone in the crowd leapt to their feet with their hands in the air. They were bursting with excited shouts and unending cheers for Ethan. In this moment, all of Ethan's hard work and dedication was being rewarded with glory. 1


① A-C-B ② B-A-C ③ B-C-A ④ C-A-B ⑤ C-B-A

 


3. 영능김 1-3

 

Well, Ethan is only five feet tall, and his legs unnaturally bend away from each other.


(A) It is difficult for him to walk, run, or move around. Because of his condition, he decided to leave his crowded high school in the big city. 0

(B) The coach wasn't sure at first, but in the end he allowed Ethan to come to practice. Regardless of his physical difficulties, Ethan worked just as hard as every other player on the team. Although he knew he would never be a valuable player in any of the team's games, he poured his heart and soul into practice every day. 2

(C) He moved to our school in the middle of his first year in high school. That following summer, he asked the coach if he could join the football team as a sophomore. 1


① A-C-B ② B-A-C ③ B-C-A ④ C-A-B ⑤ C-B-A

 


4. 영능김 1-4

 

Over time, however, Ethan became valuable to the team in different ways.


(A) His passion for the game was an inspiration to all his teammates. Because Ethan motivated and encouraged them, they became his most passionate fans. Day in and day out, seeing Ethan's smile, positive attitude, and hard work lifted everyone's spirits. 0

(B) He always observed each play carefully from the sidelines. Although he wasn't the one making the actual plays on the field, Ethan's mind was always right there with his teammates. Everyone could sense his love for football, and the coaches admired his commitment. 2

(C) Right before every game, Ethan would always be in the middle of the group offering motivational words. He had a special talent for calming people down and bringing out the best in them. Ethan was also Winston High's loudest supporter. 1


① A-C-B ② B-A-C ③ B-C-A ④ C-A-B ⑤ C-B-A

 


5. 영능김 1-5

 

For the past three years, Ethan has been schooling us all in the game of life.


(A) Yes, sometimes there is something better than being the best. 2

(B) He always reminds us that everyone is important to a team's success, though their role on the team may be small. Instead of putting all his efforts into trying to be the team's best player, he has done everything he can to make the team better. 0

(C) As Ethan has shown us, lifting up those around us is also of great worth. When we help others shine, their light will shine on us in return. 1


① A-C-B ② B-A-C ③ B-C-A ④ C-A-B ⑤ C-B-A

 


6. 영능김 2-1

 

From Trash to Treasure Every day during lunch, Jamie enjoys a soft drink and has a decision to make:.


(A) So, what about trying to creatively reuse, or "upcycle," them instead? This new approach is becoming more popular since it is even more environmentally friendly than recycling. What's more, it can also be fun! Here are some inspiring examples of how people have creatively upcycled old, used things. 2

(B) What should he do with the empty can? Many people would answer, "Recycle it!" Obviously, recycling is good for many reasons. 0

(C) We can reduce the amount of trash thrown away, use less energy than we would to make new products, and conserve natural resources by recycling. However, recycling is not a perfect way to manage waste. It still requires large amounts of energy to purify used resources and convert them into new products. 1


① A-C-B ② B-A-C ③ B-C-A ④ C-A-B ⑤ C-B-A

 


7. 영능김 2-2

 

Through upcycling, a seemingly useless object can be transformed into something completely different that is useful for everyday life.


(A) To solve this problem, Parsons and his team are turning them into sandal bottoms. They then use canvas and natural materials to make the other sandal parts. What a great reuse of resources! 2

(B) These bags are perfect for bicyclists going to work every day in all kinds of weather. Similarly, a man named Kyle Parsons and his partners have been creatively reusing old motorcycle tires from Bali, Indonesia. A shocking number of tires get thrown away there every year, and they are a serious environmental problem since they cannot decompose or be recycled. 1

(C) What do you think can be done with old truck tarps, car seat belts, and bicycle inner tubes? Individually, these things look like trash, but with a little imagination the Freitag brothers, Markus and Daniel, repurpose them for something totally new:. very strong bags. 0


① A-C-B ② B-A-C ③ B-C-A ④ C-A-B ⑤ C-B-A

 


8. 영능김 2-3

 

Along with small everyday items, much bigger things can also be upcycled—.


(A) Can you believe a building for melting metal is now a viewing platform with a gorgeous 360-degree view? The final result is the Landscape Park Duisburg Nord. It has almost 570 acres of land filled with gardens, cycling paths, and pretty lights at night, in addition to its creatively repurposed buildings. This park proves that it's possible to preserve the heritage of a place as well as the environment. 2

(B) Many of the buildings kept their original shapes, but received extra equipment and new designs in their surrounding areas. For instance, old gas tanks became pools for divers. Concrete walls of iron storage towers were turned into ideal training fields for rock climbers. 1

(C) even old buildings that cannot be used for their original purpose anymore. The German government showed us an excellent example of this with a former steel plant that closed in 1985. Rather than destroy the plant's buildings or abandon the entire facility, they decided to give it new meaning as a series of useful public structures. 0


① A-C-B ② B-A-C ③ B-C-A ④ C-A-B ⑤ C-B-A

 


9. 영능김 2-4

 

When artists add their own creative touches, things that most people consider junk are reborn as beautiful works of art.


(A) From farther away, however, they appear to blend together into marvelous landscapes or other paintings. There is also an artist who shows that even disposable cups can be reused as artistic material. 1

(B) The giant pictures made from trash by environmental artist Tom Deininger are one of a kind. Up close, these brightly colored creations look like a mixed-up mess of broken plastic, unwanted toys, and bent wire—all things that cannot be recycled. 0

(C) For years, Gwyneth Leech has turned used coffee cups into brilliant art exhibits. After a cup is used by someone, she paints a unique design on it and hangs it with many other painted cups in front of a window or pretty background. These works from Leech and Deininger are not only pleasing to the eye, but they also naturally provoke an interest in environmental conservation in people. 2


① A-C-B ② B-A-C ③ B-C-A ④ C-A-B ⑤ C-B-A

 


10. 영능김 2-5

 

As you can see, creative thinking has the power to make many positive changes to the environment.


(A) Perhaps he could upcycle them to make lanterns, toys, or sculptures for his friends and family. The options are endless, and all he needs is a little creativity to think of them. 1

(B) By giving old products more value, we can lessen the amount of waste in a way that is even more eco-friendly than recycling. So what would you say to Jamie now as he decides what to do with his cans? 0

(C) In the same way, stop and think before you throw something out. Who knows? Maybe you can turn that trash into treasure. 2


① A-C-B ② B-A-C ③ B-C-A ④ C-A-B ⑤ C-B-A

 


11. 영능김 3-1

 

Art Heals Has a painting, a movie, or a novel ever made you feel better?


(A) When you are angry or irritated Henri Matisse, Harmony in Red. 2

(B) As you will see, the use of color, different perspectives, and engaging plots can have an uplifting effect on your mind, body, and soul. 1

(C) Taking medicine can help you deal with your emotions and relieve your worries, but sometimes art might actually be the cure you're looking for. 0


① A-C-B ② B-A-C ③ B-C-A ④ C-A-B ⑤ C-B-A

 


12. 영능김 3-2

 

Many people think that they should look at calming colors when they are angry or irritated.


(A) Because the color red excites the emotions, they may think that looking at it will make them angrier. However, this picture shows that the opposite can be true. By looking at the color red here, you can release your anger. This picture is actually helping you calm down. 0

(B) At the same time, the green and blue space outside the window causes healing and relaxing feelings. The existence of these cool colors actually makes the "heaviness" of the red colors appear a bit lighter. Although the color red is dominant, it works together with the various contrasting colors to form a harmony. Seeing this balance keeps you from becoming overwhelmed by your emotions and helps you overcome your anger. As the painting's title suggests, this must be the power of the harmony in red. 2

(C) The woman in the picture is standing in a vividly red room and is placing fruit in a bowl. She seems to be carrying on her work in silence. As you watch the woman working dutifully at her task in this red room, your anger melts away instead of getting worse. In addition, the yellow fruit on the table brings out positive and cheerful emotions. 1


① A-C-B ② B-A-C ③ B-C-A ④ C-A-B ⑤ C-B-A

 


13. 영능김 3-3

 

When you lack confidence The Secret Life of Walter Mitty Walter Mitty has developed the pictures used on the front cover of Life magazine for the past sixteen years.


(A) Unfortunately, the picture for the final cover is missing. Walter decides to hit the road to find the picture. He believes the photographer still has it and follows his trail. 1

(B) This is how Walter's wild dash across Greenland, Iceland, and the Himalayas begins. During this adventure, he survives a volcanic eruption and a fall from a helicopter. Although he finally finds the photographer, Walter realizes he has, more importantly, become the person he always imagined he could be. 2

(C) Other than that, he leads a boring life filled with daydreams. However, his world is about to change:. Life will soon become an online-only publication. 0


① A-C-B ② B-A-C ③ B-C-A ④ C-A-B ⑤ C-B-A

 


14. 영능김 3-4

 

Learn from Walter.


(A) Once you begin making bold choices, courage will follow. All you need is motivation and this is already inside of you. So start living! This movie will remind you that your dreams are ready whenever you are. 2

(B) There is no such thing as the right moment. Create your own opportunities, and everything will fall into place. Don't worry about not being brave enough—. 1

(C) Don't sit around and dream about your next adventure—. Just go ahead and make it happen. Don't wait for the right moment—. 0


① A-C-B ② B-A-C ③ B-C-A ④ C-A-B ⑤ C-B-A

 


15. 영능김 3-5

 

When you feel like you don't fit in Richard Bach, Jonathan Livingston Seagull Jonathan Livingston Seagull knows that he's different from others.


(A) Instead, they look at him coldly as they now consider him unfit to be a part of the flock. Jonathan tries his best to rejoin the flock, but he's no longer satisfied flying in formation with the other seagulls as he knows how wonderful soaring above the clouds really feels. If you have ever felt a little bit different, take Jonathan Livingston Seagull's message to heart. Don't be afraid of being different. 1

(B) There is no need to apologize for being the way you are. This is what makes you special. Once you embrace what makes you different, learn as much as you can about it. Keep perfecting that special skill that makes you different from the rest of the crowd. 2

(C) Instead of fighting over food with the other seagulls, Jonathan spends all his time learning about flying. Every day, he practices new skills by rolling, spinning, and diving high above the sea. During one of his practices, Jonathan flies through his flock. He expects the others to praise his amazing ability. 0


① A-C-B ② B-A-C ③ B-C-A ④ C-A-B ⑤ C-B-A

 


16. 영능김 4-1

 

Put a Roof over Someone's Head Sometimes you may feel upset when you wake up suddenly from a nightmare, but you can always let out a sigh of relief.


(A) Many people around the world I don't wake up in a soft and comfortable bed. Instead, they open their eyes to see a dirt floor or a leaking roof. I didn't realize for a long time that something I take for granted could be someone else's biggest dream. 2

(B) No matter how scary the dream was, at least you've woken up safe and sound in your own home. You can go back to sleep because you know your loved ones are just around the corner in their own beds. 0

(C) Your home is a special place that protects you and your family from everything. Unfortunately, this is not a reality that everyone shares. 1


① A-C-B ② B-A-C ③ B-C-A ④ C-A-B ⑤ C-B-A

 


17. 영능김 4-2

 

One day, I learned about a program that needed volunteers to go to different parts of the world to help build houses for the poor.


(A) I was going to find out soon enough. The volunteers and I got to meet the family that would be moving into the home we were going to build the day after we arrived in Karjat, India. This family of five was living in a one-room hut. It was quite clear that this place was not big enough to house all the family members. 1

(B) In addition to that, there were jars everywhere to catch all the rainwater dripping from the roof. With all these jars on the floor, there was hardly any space to sit, let alone lie down. It was shocking to see how they were living. After meeting them, I felt even more determined to build them a beautiful home. 2

(C) After watching a presentation about it, I was really touched by the mission, so I decided to take part in the next volunteer trip. I had my doubts, though. I didn't know anything about construction, and we'd only be there for two weeks. Could we really change these people's lives as the presentation had suggested? 0


① A-C-B ② B-A-C ③ B-C-A ④ C-A-B ⑤ C-B-A

 


18. 영능김 4-3

 

There were clear skies and temperatures over 30˚C that first week.


(A) It was the best way to end such an incredible experience. The entire community was there. Everyone was dancing or crying tears of joy. I will never forget the looks on the faces of the family as they waved goodbye to us. 2

(B) Under the burning sun, we carried bricks, mixed concrete, and dug holes for pipes. We had to slowly take apart the family's hut to get more bricks and other materials for the new house. Every morning, the family and their neighbors would provide breakfast for everyone. Then, all of us—the volunteers, community members, and the family—would start working. 0

(C) We would share food and stories from time to time. When evening came, we were all very tired and sweaty, but we were happy. After another week, the home was finally finished. On the last day, we had a party to celebrate the completion of the new house. 1


① A-C-B ② B-A-C ③ B-C-A ④ C-A-B ⑤ C-B-A

 


19. 영능김 4-4

 

As I look back on this trip, I find it amazing that so many different people came together to build a house for a family they'd never met.


(A) I thought I was there to give, but I received so much more in return. This experience has inspired me to continue building houses for others. I hope it will also encourage my friends and family members to help out in the future.2

(B) I'm thankful for the friendships I've made through this trip. In addition, I learned so much from the other volunteers, the community members, and this family. 1

(C) For many of us, it was the first time we'd ever built a house. The work was hard, but not one person ever stopped smiling or even complained. 0


① A-C-B ② B-A-C ③ B-C-A ④ C-A-B ⑤ C-B-A

 


20. 영능김 4-5

 

Although two weeks may not seem like a long time, I had the chance to make new friends, learn about a different culture, and, most importantly, do something that makes the lives of others better.


(A) Right now, there are still many people around the world without a place to live. They are the reason why ordinary people like me want to go out there and help put a roof over their head. 2

(B) I am so glad that this family now has a safe place where they can lay their heads. 0

(C) I believe everyone deserves a decent home. 1


① A-C-B ② B-A-C ③ B-C-A ④ C-A-B ⑤ C-B-A

 


21. 영능김 5-1

 

Three Questions There was once a king who wanted to know three things; the right time to do everything, the most necessary people to pay attention to, and the most important thing to do.


(A) Many people traveled to his palace, but they all provided different answers to his questions. Regarding the first question, some said that the king should make a detailed schedule so that he could always know the perfect time to do everything. 1

(B) He thought that knowing these things would ensure his success. He announced that he would reward anyone who could teach him these things.0

(C) Others claimed that he should take every situation into account and wait for the precise moment to act. Still others suggested that he should consult wise men. 2


① A-C-B ② B-A-C ③ B-C-A ④ C-A-B ⑤ C-B-A

 


22. 영능김 5-2

 

Equally various were the answers to the second question.


(A) The hermit lived in a forest and met only ordinary folk, so the king disguised himself as a simple peasant. He ordered his bodyguards to stay behind while he went on alone to seek out the hermit. Reaching the hermit's hut, the king found the old man digging in his garden. The hermit greeted the king and continued digging. 1

(B) Some said that the members of the king's council were the most important people, while others mentioned priests, doctors, or warriors. In response to the third question, some replied that science was the most important thing, whereas others insisted that it was war or religious worship. The king was not pleased with any of the answers. Still wishing to find the best answers to his questions, the king decided to visit an old hermit who was famous for his wisdom. 0

(C) The king said, "I have come here to ask you three questions. How can I learn the right time to do everything? Whom do I most need to pay attention to? Finally, what is the most important thing to do?" 2


① A-C-B ② B-A-C ③ B-C-A ④ C-A-B ⑤ C-B-A

 


23. 영능김 5-3

 

The hermit listened carefully but declined to respond.


(A) "You must be tired," the king said,"Let me help you with that." The hermit thanked him, handing the king his spade. The king worked for a long time, while the hermit watched him silently. Eventually, as the sun was setting, the king stopped and said, "I came to you for answers to my questions. 0

(B) When the man reached the king, he fell down. The king could see blood flowing from a large wound in his stomach. The king washed and bandaged the wound, and then he and the hermit carried the man into the hut. The man closed his eyes and fell asleep. Because he was completely exhausted, the king also lay down and slept. 2

(C) If you can give me none, please let me know so that I can return home." "Someone is running toward us," the hermit said. "Let's see who it is." The king turned and saw a man running out of the woods. 1


① A-C-B ② B-A-C ③ B-C-A ④ C-A-B ⑤ C-B-A

 


24. 영능김 5-4

 

When he awoke in the morning, the man was staring at him.


(A) "Forgive me," the man begged. "I don't know you, and I have no reason to forgive you," the king replied. "You might not know me, but I know you," the man answered. 0

(B) Although I escaped, I would have died if you hadn't saved my life. Now, my sons and I will serve you forever." Pleased to have made friends with his enemy so easily, the king forgave the man and promised to restore his property. After the man left, the king went outside to talk to the hermit. 2

(C) "During the last war, you killed my brother and took my property. When I knew that you had gone to see the hermit, I decided to kill you on your way back. However, I came across your bodyguards, who recognized me and wounded me. 1


① A-C-B ② B-A-C ③ B-C-A ④ C-A-B ⑤ C-B-A

 


25. 영능김 5-5

 

"For the last time, I beg you to answer my questions," the king said.


(A) Later, the most important time was when you cared for the man. If you hadn't helped him, he would've died, so he was the most important person, and helping him was the most important thing. Remember, there is only one time that is important; now! The person that you are with is the most important person, and doing that person good is the most important thing."2

(B) "They've already been answered," said the hermit. "What do you mean?" the king asked. 0

(C) "If you hadn't helped me, you would've left, and that man would've attacked you. Therefore, the most important time was when you were digging. I was the most important person, and helping me was the most important thing. 1


① A-C-B ② B-A-C ③ B-C-A ④ C-A-B ⑤ C-B-A

 


26. 영능김 6-1

 

Against All Odds Towering over the East River, the Brooklyn Bridge is one of New York City's famous landmarks.


(A) This is the story of how the Roeblings built the Brooklyn Bridge. In the 1860s, the populations of Manhattan and Brooklyn were rapidly increasing, and so was the number of the commuters between them. Thousands of people took boats and ferries across the East River every day, but these forms of transport were unstable and frequently stopped by bad weather. 1

(B) Many New Yorkers wanted to have a bridge directly connecting Manhattan and Brooklyn because it would make their commute quicker and safer. Unfortunately, because of the East River's great width and rough tides, it would be difficult to build anything on it. It was also a very busy river at that time, with hundreds of ships constantly sailing on it. 2

(C) Nowadays, many people take it for granted, but back in the 1800s it was something people only dreamt of. The bridge's construction cost more than $320 million in today's money and the lives of over two dozen people. In particular, the family in charge of the project paid a high price to create the impressive bridge we all know and love today. 0


① A-C-B ② B-A-C ③ B-C-A ④ C-A-B ⑤ C-B-A

 


27. 영능김 6-2

 

Any bridge over the river would need to be a very high suspension bridge.


(A) However, he was involved in a ferry accident later that year and died of an infection not long after. It looked like his ambitious plan was destined to fail shortly after it had been launched. 2

(B) In addition, he planned to build two large stone towers to hold up the bridge's road and allow people to walk across it. If his ideas worked, the final result would be the longest, strongest suspension bridge ever built. John's ambition inspired people, so construction began in 1869. 1

(C) Considering the limited technology in those days, building such a bridge seemed impossible. That is, people thought it was impossible until John Roebling, an expert at building suspension bridges, accepted the challenge. John proposed the use of steel cables - instead of iron ones - that would be six times stronger than needed to support the bridge. 0


① A-C-B ② B-A-C ③ B-C-A ④ C-A-B ⑤ C-B-A

 


28. 영능김 6-3

 

Luckily, John's role as chief engineer was succeeded by his son, Washington Roebling.


(A) Many died or were permanently injured by a serious disease called "the bends," including Washington Roebling. In 1872, he developed this disease and was unable to move easily or visit the construction sites throughout the rest of the project. Other people would have quit at that point, but not Washington. 1

(B) Because he had built bridges with his father and studied bridge construction in Europe, he believed in John's dream. At that time, the foundations for the bridge's two towers were being built in the East River, which was extremely difficult and dangerous work. Workers had to stay at the bottom of the river in a waterproof box with little light and constant danger. 0

(C) He continued to supervise the bridge building for years by watching it through a telescope from his bedroom. However, there were still many things he could not do despite all his efforts. Once again, the project seemed likely to be abandoned. 2


① A-C-B ② B-A-C ③ B-C-A ④ C-A-B ⑤ C-B-A

 


29. 영능김 6-4

 

To everyone's amazement, yet another Roebling stepped in to save the bridge.


(A) By the time the bridge was finished in 1883, Emily was carrying out many of the chief engineer's duties, which was unprecedented for a woman in those days. Many people praised her contributions to the project, and she became the first person to cross the bridge. It was the moment she, Washington, John, and everyone else who built the bridge had worked so hard for. 2

(B) As her husband's health failed, though, Emily began passing his instructions to the assistant engineers and bringing back their construction reports. In the process, she naturally picked up a lot of information about bridge building. With his assistance, she also studied higher mathematics and engineering so hard that she became an expert in them without ever going to college! 1

(C) This time, it was Washington's wife, Emily Warren Roebling. She believed in what her family had started, and she was determined to see it through. Before marriage, she knew almost nothing about engineering. 0


① A-C-B ② B-A-C ③ B-C-A ④ C-A-B ⑤ C-B-A

 


30. 영능김 6-5

 

To this day, the Brooklyn Bridge stands as evidence of the Roebling family's persistence.


(A) These amazing people made incredible sacrifices and overcame all obstacles to complete a project that seemed impossible to others. 0

(B) 2

(C) Not only did they prove their doubters wrong, but they also achieved an accomplishment that inspired us to do the impossible. 1


① A-C-B ② B-A-C ③ B-C-A ④ C-A-B ⑤ C-B-A

 


31. 영능김 7-1

 

Hanji, Korea's Paper Until 1966, no one knew that the Mugujeonggwang Daedaranigyeong, the world's oldest printed document, lay inside a container at Bulguksa Temple in Gyeongju, Korea.


(A) Experts around the world were shocked that a document printed more than 1,200 years ago could still be around. They were even more surprised when the paper was removed from the container. 0

(B) How could this paper last for more than 1,000 years without breaking down or becoming damaged? The secret lies in hanji's amazing physical properties. 2

(C) Although the document was printed before 751 CE, it was still in perfect condition. This discovery proved that the paper-making technology of the Unified Silla Kingdom era (676–935) was more advanced than that of either Japan or China, both of which also had highly developed paper-making technology. 1


① A-C-B ② B-A-C ③ B-C-A ④ C-A-B ⑤ C-B-A

 


32. 영능김 7-2

 

Hanji is traditionally made from the bark of the mulberry tree.


(A) Through a number of complex processes, the tree bark is made into a paper that is very durable and hard to tear. On the other hand, Western paper, which is made from pulp, begins to fall apart and becomes unusable after 100 years. 0

(B) The paper also absorbs water and ink very well, so there is no bleeding. 2

(C) It's easy to understand why Koreans created the old saying about hanji; "Paper lasts a thousand years, while silk endures five hundred." In addition to lasting a long time, hanji keeps heat and sound in but allows air to flow through it easily. 1


① A-C-B ② B-A-C ③ B-C-A ④ C-A-B ⑤ C-B-A

 


33. 영능김 7-3

 

Because of hanji's characteristics, Koreans could see early on that it was more than just something to write on.


(A) Back then, people would put many layers of hanji together to make suits of armor. This armor, called jigap, was tough enough to stop arrows. With so many uses, hanji is naturally considered an invaluable part of Korean history and culture. 1

(B) Though the tide of modernization seems to have made people forget about this paper's outstanding qualities, hanji has endured and remains relevant today. In recent years, finding even more functions and purposes for hanji has become a trend. 2

(C) Since ancient times, they have glued this paper to the walls, door frames, and floors of their homes. Koreans have also used it to make furniture, lanterns, wedding accessories, and boxes. Due to its durability, hanji was even used in battle. 0


① A-C-B ② B-A-C ③ B-C-A ④ C-A-B ⑤ C-B-A

 


34. 영능김 7-4

 

Lately, designers have been using hanji to make clothes, socks, and ties.


(A) It seems that hanji clothing is here to stay. One of hanji's newest uses is a treat for the ears. Customers can now buy speakers that use vibration plates and outside panels made of hanji. Compared to regular speakers, the sound that comes from hanji speakers is stronger and sharper. 1

(B) The fabric these designers are using is a blend of hanji yarn with cotton or silk. This blend is almost weightless and keeps its shape better than other materials. It is also washable and eco-friendly. Not only is hanji clothing practical, but it's also making waves at domestic and international fashion shows. 0

(C) The paper's thickness and ability to absorb sound help the speakers pick up the smallest vibrations. In addition, the fact that the sound will not change over time because of the strength of hanji makes these speakers a great purchase. Serious music lovers will really be able to appreciate the great sound quality of these speakers. 2


① A-C-B ② B-A-C ③ B-C-A ④ C-A-B ⑤ C-B-A

 


35. 영능김 7-5

 

And now, in one of its most innovative uses yet, hanji may soon be traveling into outer space.


(A) Scientists hope to use hanji in the future for space probes since it is less expensive and lighter than the currently used materials. As you can see, people keep transforming hanji for countless uses. 1

(B) Its ability to adapt to the needs of every generation has led to the revival of this traditional paper. So long as hanji continues to be treasured, there may be no limit to how it will be enjoyed in the future. 2

(C) Korea and the U.S. are planning to use hanji on robots and spacecraft through a joint project supported by NASA. The paper has special properties that will help protect spacecraft from the harmful rays of the sun. 0


① A-C-B ② B-A-C ③ B-C-A ④ C-A-B ⑤ C-B-A

 


36. 영능김 8-1

 

How Teens Make Decisions Host; Hello, everyone!


(A) Teens are more likely to make these types of decisions than adults. With the help of our guest, we'll learn why teens tend to act before thinking everything through. Now, here's our guest for tonight, Dr Jenny Clarkson! Thank you for joining us, doctor! Dr C; Thank you for having me, Joseph! 2

(B) You say some harsh things that you normally wouldn't say. You're so angry that you don't care. When you're about to push "send," you think about whether it's a good idea. Before you know it, you've sent the message anyway. 1

(C) Welcome to The Dr Brain Show. I'm your host, Joseph Emerson. Can you think back to a time when a friend upset you? Let's imagine that you decided to write an angry message to that friend. 0


① A-C-B ② B-A-C ③ B-C-A ④ C-A-B ⑤ C-B-A

 


37. 영능김 8-2

 

Host; Dr Clarkson, could you first tell us about how we make decisions?


(A) However, studies show that some parts of the brain continue to develop until the early twenties. That means teens' brains are still maturing and not completely developed. This may be why teens seem to make risky decisions. 2

(B) Dr C; Sure, It seems like we make decisions almost immediately, but our brain actually has to go through several steps before deciding anything. Neurons, which are special brain cells, make up different structures in our brains. These structures send signals to each other. After the structures finish evaluating all the signals, they will send out a response that will tell our body what to do. 0

(C) Host; I see. Does this process happen exactly the same way in everyone's brain? Dr C; People basically go through the same decision-making process, but there is a slight difference between teens and adults. Scientists used to think that the brain was done growing by the time you turned 12 since the brain reaches its maximum size around that age. 1


① A-C-B ② B-A-C ③ B-C-A ④ C-A-B ⑤ C-B-A

 


38. 영능김 8-3

 

Host; Very interesting!


(A) Please tell us more about the relationship between their brains and their decision-making. Dr C; Well, the region that controls emotions matures faster than the part of the brain that helps you think ahead and measure risk.0

(B) Host; So what you're saying is teens are likely to make choices based on their feelings since their brains are not fully developed. 2

(C) Teens therefore rely on it heavily, which means they are influenced more by feelings and instincts than by reason when making decisions. In other words, teens are usually not inclined to consider all the consequences of their actions, so they make choices that they end up regretting. 1


① A-C-B ② B-A-C ③ B-C-A ④ C-A-B ⑤ C-B-A

 


39. 영능김 8-4

 

Dr C; That's right.


(A) For example, if teens aren't reading, doing experiments, or solving problems, then the brain will get rid of the connections that are related to those activities. Once those are gone, their brains will put more energy into making other connections stronger. Host; So, do you mean that the activities teens are involved in can shape the way their brains develop? 1

(B) However, this is not the whole story. Teens' brains are also going through other important changes. Their brains are constantly identifying and removing any weak connections between neurons. 0

(C) Dr C; Exactly, This is why the types of activities teens choose to participate in are especially important. If a teen decides to play sports or learn an instrument, then the brain will strengthen those connections. On the other hand, if he or she chooses to surf the Internet or play online games all day long, then those connections will survive instead. The harder teens work at building good habits, the stronger those connections in their brains will be. 2


① A-C-B ② B-A-C ③ B-C-A ④ C-A-B ⑤ C-B-A

 


40. 영능김 8-5

 

Host; I see.


(A) We hope the information you've shared will help our viewers at home make more reasonable choices in the future. That's it for The Dr Brain Show tonight. Good night, everyone! 2

(B) Do you have any final comments for our viewers? Dr C; If we view the adolescent period as merely a process of becoming mature, then it's easy to dismiss it as a passing phase. However, we shouldn't look at the changes that occur in teens' brains only in terms of maturity. 0

(C) Adolescence is also a period when significant changes happen in the brain that help new abilities appear. Therefore, adolescence is not a stage to simply get through, but an important stage in people's lives where they can develop many qualities and abilities, and shape their future. Host; Thank you for your insight, Dr Clarkson! 1


① A-C-B ② B-A-C ③ B-C-A ④ C-A-B ⑤ C-B-A

 


[ANSWER]
1. ② 2. ③ 3. ① 4. ① 5. ③ 6. ③ 7. ⑤ 8. ⑤ 9. ② 10. ②


11. ⑤ 12. ① 13. ④ 14. ⑤ 15. ④ 16. ③ 17. ④ 18. ③ 19. ⑤ 20. ③


21. ② 22. ② 23. ① 24. ① 25. ③ 26. ④ 27. ⑤ 28. ② 29. ⑤ 30. ①


31. ① 32. ① 33. ④ 34. ② 35. ④ 36. ⑤ 37. ③ 38. ① 39. ② 40. ③


 

728x90
반응형

728x90
반응형

evh1 | Since 2005 임희재 | 블루티쳐학원 | 01033383436 | 200710 22:49:58

 

PARA

 

1. EVH1 1-1

 

From time to time, you are told by parents or teachers to "think outside the box."


(A) Looking at a problem from a different perspective can lead you to a new approach to handle the problem. It truly works. 1

(B) To this, you may say, "Easier said than done." But try it. 0

(C) The following stories show how changing your perspective can change your life. 2


① A-C-B ② B-A-C ③ B-C-A ④ C-A-B ⑤ C-B-A

 


2. EVH1 1-2

 

A professor walked around on a stage while teaching.


(A) If I hold it for a minute or two, it's fairly light. If I hold it for an hour, it may make my arm ache. If I hold it for an entire day, my arm will likely cramp up, which will force me to eventually drop the glass. In each case, the weight of the glass doesn't change, but the longer I hold it, the heavier it feels to me." 1

(B) "Some of your worries are like this glass of water. Think about them for a short while, and nothing happens. Think about them a bit longer, and you begin to ache a little. Think about them all day long, and you will feel numb and be incapable of doing anything else until you drop them." 2

(C) She raised a glass of water and asked, "How heavy is this glass of water?" Students shouted out various answers. She replied, "From my perspective, the weight of this glass doesn't matter. It all depends on how long I hold it. 0


① A-C-B ② B-A-C ③ B-C-A ④ C-A-B ⑤ C-B-A

 


3. EVH1 1-3

 

One day, a daughter complained to her father about the problems she was struggling with.


(A) Shortly after, she found that the potatoes became soft. After peeling off the shells of the eggs, she noted that the inside of the eggs became firm. Finally, she took a sip of the coffee and smiled at the aroma. 2

(B) She was frustrated because it seemed that just as one problem was solved, another followed. Her father took her to the kitchen. He filled three pots with water and placed potatoes, eggs, and ground coffee beans in each of the three pots. He then let them boil. 0

(C) After twenty minutes, he turned off the stove. Turning to her, he asked, "What do you see?" "Potatoes, eggs, and coffee," she hastily replied. "Yes, but look closer," he said, "and tell me what happened to each." 1


① A-C-B ② B-A-C ③ B-C-A ④ C-A-B ⑤ C-B-A

 


4. EVH1 1-4

 

"Alright, Dad.


(A) The ground coffee beans stayed intact but changed the water itself, making it fragrant and flavorful." "Which are you?" he asked his daughter, "When adversity knocks on your door, how do you respond? You can lose your strength though you started strong. 1

(B) You can be tough with a stiff spirit though you started weak. Or, you can change the circumstance that was brought about by adversity for the better. Are you a potato, an egg, or a coffee bean?" 2

(C) What are you trying to tell me?" she asked. He explained that each of these objects had faced the same adversity, but each one reacted differently. "The potato was strong and hard, but became soft and weak after sitting in the boiling water. The inside of the egg was fragile but became hardened after being boiled in the water. 0


① A-C-B ② B-A-C ③ B-C-A ④ C-A-B ⑤ C-B-A

 


5. EVH1 1-5

 

Once upon a time, there lived a young athletic boy who prepared himself for a running competition.


(A) He ran with determination, strength, and power, and always beat other competitors. Whenever the boy was running for a prize, a large crowd gathered together to watch the race. A wise old man, upon hearing of the boy, had traveled far to watch his races. Though the crowd cheered loudly for the boy, the wise man stayed still and calm, expressing no sentiment. 0

(B) "Race!" said the wise man. The race began, and the boy was the only finisher because the elderly lady and the blind man were left standing at the starting line. The little boy noted that the crowd did not cheer for him this time, which puzzled him. In the middle of the crowd was sitting the wise old man, looking calmly at the boy. He ran to the wise man and asked, "Why don't people cheer for me now?" 2

(C) The little boy felt proud and important. "I can beat anyone!" cried the little boy. After hearing this, the wise man said to him, "Now I present two new challengers to you." Behind him were an elderly lady and a blind man. 1


① A-C-B ② B-A-C ③ B-C-A ④ C-A-B ⑤ C-B-A

 


6. EVH1 1-6

 

"Race again," replied the wise man, "but this time, finish together."


(A) The wise man smiled. He said, "Well done, little boy. You've won a lot more in this race than in any other you've run before. 1

(B) The little boy stood between the blind man and the elderly lady, holding them arm in arm. The race began, and the little boy walked slowly with the others. When they crossed the finishing line, the crowd cheered loudly and waved at the boy. 0

(C) You are a real winner." Now the little boy was greatly delighted, and he felt proud and important again. 2


① A-C-B ② B-A-C ③ B-C-A ④ C-A-B ⑤ C-B-A

 


7. EVH1 2-1

 

Our intangible cultural heritage is a bridge linking our past and our future.


(A) Intangible cultural heritage is a precious asset for communities, groups, and individuals across the world. Only they can protect it and pass it on to tomorrow's generations. 1

(B) The following pages offer an insight into the great diversity of humanity's living heritage across the world. At a time of rapid and profound change, we must strengthen our resolve and take action to preserve this heritage for the benefit of future generations. 2

(C) It is the way we understand the world and the means by which we shape it. It is rooted in our cultural identities and provides a foundation of wisdom and knowledge upon which to build sustainable development for all. 0


① A-C-B ② B-A-C ③ B-C-A ④ C-A-B ⑤ C-B-A

 


8. EVH1 2-2

 

In the months between spring and autumn, people in Catalonia, Spain build castells during street festivals.


(A) When the last person reaches the top and waves his or her hands, the crowd below shouts and cheers. 2

(B) A castell is a tower of people standing on each other's shoulders, often raised as high as ten levels. The formation of the bottom base or pinya is the first step in building a castell. 0

(C) Anybody who is willing to help, from children to senior citizens, can be part of the pinya by supporting the tower and protecting it from collapsing. When the pinya is formed, each level is added one after another. 1


① A-C-B ② B-A-C ③ B-C-A ④ C-A-B ⑤ C-B-A

 


9. EVH1 2-3

 

Building a castell means a lot to Catalonians.


(A) The knowledge about how to build castells is accumulated over generations and handed down from generation to generation within a community, and can only be learned by practice. Building one provides people with a strong sense of belonging and a heightened spirit of teamwork and cooperation. 0

(B) At a time when individual-based activities dominate most people's leisure time, this Catalan practice does seem to deserve serious attention. 2

(C) A castell requires between 75 and 500 people to build, all of whom wear a group color. The color is a proud element of a community, and young children long for the day when they can wear a shirt and play their role in building their group's castell. 1


① A-C-B ② B-A-C ③ B-C-A ④ C-A-B ⑤ C-B-A

 


10. EVH1 2-4

 

Gingerbread broadly refers to any type of baked treat that is typically flavored with ginger and honey.


(A) Bake the shaped dough until it turns light yellow in color. Put a string between two baked licitars. Dip the stringed licitars into the glaze and hang them until they dry. Decorate each licitar with a picture, a mirror, or flowers. 2

(B) The custom of giving licitars is deeply rooted in Croatian tradition. Licitars are not only a tasty dessert, but they are also an artistic expression. How to Make Licitars. Prepare dough and let it mature for a few days. Then roll the dough out and shape it in molds. 1

(C) The process of making a licitar, a type of Croatian gingerbread, is special because it requires skill and endurance. The dough matures for a few days, then is shaped in molds and baked, and then left for two weeks to dry. Coloring is the next step, after which a second drying takes place. Once dried, the licitars are decorated. Licitars are given as gifts for wedding guests, or they can be used as decorations. 0


① A-C-B ② B-A-C ③ B-C-A ④ C-A-B ⑤ C-B-A

 


11. EVH1 2-5

 

A licitar is uniquely Croatian because of the long history and the social role it has played.


(A) In this way, every generation is able to leave its mark in each gingerbread they make. Licitar makers play an important social function, too. 1

(B) Gingerbread craftspeople appear at most festivals and fairs in Croatia. People watch cookies being made and socialize. In a time when local cultures are losing their unique identities, Croatian gingerbread craftsmanship provides us with a new perspective on traditional culture. 2

(C) Croatians learned the recipe during the 16th century from their neighbors in the eastern Alps and went a step further to develop their own. Since then, making licitars has become a family tradition where secrets are passed down from generation to generation. 0


① A-C-B ② B-A-C ③ B-C-A ④ C-A-B ⑤ C-B-A

 


12. EVH1 2-6

 

Tugging has long been practiced as a kind of ritual in many agricultural regions across Asia.


(A) It takes place during the New Year holidays in April in the open space of a Buddhist temple or a village center. In the Philippines, punnuk is held in the Hapao River, where people tug young trees that are tightly bundled with vines and hooked to a straw figure in the middle. 1

(B) The way the ritual is performed varies from place to place. In Cambodia, the ritual involves men tugging against women. 0

(C) In Vietnam, bamboo poles can be used for tugging. In Korean juldarigi, making a rice straw rope as well as moving it to a ritual site is part of the tradition. The rope stands for a dragon which is believed to bring rain to the region. 2


① A-C-B ② B-A-C ③ B-C-A ④ C-A-B ⑤ C-B-A

 


13. EVH1 2-7

 

The tugging rituals in Asia are mostly held by rice farmers, who wish for enough rainfall and abundant harvests, so the rituals can be regarded as a form of prayer in the region.


(A) They are also performed as a way of expressing their appreciation for a good harvest. 0

(B) While preparing for the rituals and performing them, participants realize the importance of solidarity, cooperation, and harmony among community members. 2

(C) There is no intentional competitive element inherent in the rituals, and winning or losing is not emphasized. 1


① A-C-B ② B-A-C ③ B-C-A ④ C-A-B ⑤ C-B-A

 


14. EVH1 3-1

 

Human beings are capable of doing amazing things through sports.


(A) Whatever we do on earth, however, is governed by the rules of nature. In fact, science can explain a lot about the magic in sports. 2

(B) The precision and power they show seem like magic. 1

(C) Look at sprinters running through the field or baseballs flying into the catcher's glove. 0


① A-C-B ② B-A-C ③ B-C-A ④ C-A-B ⑤ C-B-A

 


15. EVH1 3-2

 

How Fast Can a Human Run 100 Meters?


(A) So what is the limit to how fast a human can run 100 meters? Sports scientists say that sprinters' speed is influenced by how hard they hit the ground with their feet and by how many steps they take. 1

(B) Cheetahs do not hit the ground as hard as humans, but they can take twice as many steps as we do. This is why they can run fast. 2

(C) Usain Bolt, the Jamaican sprinter, broke the world record for the 100-meter sprint in 2009, clocking a time of 9.58 seconds. Animals like the adult cheetah can run 100 meters in about six seconds. 0


① A-C-B ② B-A-C ③ B-C-A ④ C-A-B ⑤ C-B-A

 


16. EVH1 3-3

 

The problem for humans is that the harder they hit the ground, the more time they spend in the air, which makes their steps longer but reduces the number of steps they can take.


(A) Therefore, maximum running speed is attained when the combination of force applied to the ground and stride frequency is optimal. 0

(B) Many experts predict, however, that a human being could probably run the 100-meter sprint in just over nine seconds. Maybe future technology will help runners run even faster than they do today. 2

(C) Unfortunately, this combination also depends on the runner's size, leg length, and other factors that vary among individuals. 1


① A-C-B ② B-A-C ③ B-C-A ④ C-A-B ⑤ C-B-A

 


17. EVH1 3-4

 

Why Do Curlers Sweep the Ice?


(A) The sweepers stop as the rock settles into the house. "Nice job!" All four players bump fists indicating success. Have you guessed the sport? 1

(B) It's called curling. One might expect that a curling stone sliding across the ice would continue in a straight line according to Newton's first law of motion. However, the stone will begin to curl when it loses its speed even though it is three or four times as heavy as a bowling ball. This is why players have to direct the stone's path carefully, and they do this by sweeping. 2

(C) "Hurry! Hurry up!" Two people with brooms furiously sweep the ice in front of a rock that slides across a sheet of ice. "Whoa!" 0


① A-C-B ② B-A-C ③ B-C-A ④ C-A-B ⑤ C-B-A

 


18. EVH1 3-5

 

By rubbing the ice with the broom, the sweepers are generating heat to reduce friction between the stone and the ice.


(A) If the sweepers stop, the stone begins to curl again. 1

(B) This results in the stone going straighter and farther. 0

(C) Although the house—the area where the curlers aim the stone—is often blocked by an opponent's stone, curling teams can actually guide one stone behind another, which is one of the major strategic elements to the game. The precision with which the team can control the direction and speed of the stone is a major factor in the strategy of the game. 2


① A-C-B ② B-A-C ③ B-C-A ④ C-A-B ⑤ C-B-A

 


19. EVH1 3-6

 

Why Does a Baseball Have Stitches?


(A) The batter swings and hits a foul ball into the stands. You reach up with your glove and catch it as other spectators clap and cheer. You look at the baseball with its smooth leather covering and bright red stitches. 0

(B) The stitches on a baseball actually affect the flight of the ball. Without them, a pitcher couldn't throw it as fast or as accurately. 2

(C) Do the stitches serve a purpose other than to keep the leather covering over the ball? A typical baseball has 108 double hand stitches. They serve more than just a decorative function. 1


① A-C-B ② B-A-C ③ B-C-A ④ C-A-B ⑤ C-B-A

 


20. EVH1 3-7

 

When a round object like a ball is thrown, the front part of the ball hits the air, increasing the air pressure there while the air becomes thinner at the back of the ball, making the air pressure low.


(A) The stitches more or less scoop the air and direct it inwards towards the back of the ball. This effectively increases the net air pressure in the back of the ball, which reduces the drag. 1

(B) Further, as a baseball is released, the pitcher can put spin on the ball. As the ball spins, the stitches help the ball to fly in the direction the pitcher intends. 2

(C) This difference in air pressure creates an unwanted effect called "drag" that literally drags the ball backward and slows it down. This is where the stitches come into play. 0


① A-C-B ② B-A-C ③ B-C-A ④ C-A-B ⑤ C-B-A

 


21. EVH1 4-1

 

[MARY SEACOLE, AN UNSUNG HERO OF THE CRIMEA].


(A) He had no idea who the lady was, and when searching for some kind of clue, he only found the author's initials written on the back side of the painting. After passing through various auctions, the person in the portrait was finally identified as Mary Seacole. 1

(B) A portrait of an old, dark-skinned lady wearing three medals on her left breast was discovered in 2003. An antique dealer accidentally found it behind a framed print at a garage sale in Burford, U.K. 0

(C) The National Portrait Gallery in London confirmed that the painting was genuine and purchased it, quoting that "as a woman and as a West Indian of mixed race, she broke many barriers to make a huge contribution to Victorian society." The portrait has been displayed there since 2004. 2


① A-C-B ② B-A-C ③ B-C-A ④ C-A-B ⑤ C-B-A

 


22. EVH1 4-2

 

Mary Seacole was born in Kingston, Jamaica in 1805.


(A) By observing her mother, Mary learned the practice of traditional Jamaican medicine. Mary also traveled widely to take care of sick people. 1

(B) When she visited Panama in 1851, Mary managed to save her first cholera patient, and in so doing she gained extensive knowledge of this disease. In fact, she herself contracted and recovered from it while in Panama. Having overcome many difficulties, she gained a reputation for her work in treating cholera and other diseases. 2

(C) Her father was a Scottish soldier, and her mother practiced traditional Jamaican medicine. Her mother ran a boarding house while caring for wounded soldiers as if they were her own family members. 0


① A-C-B ② B-A-C ③ B-C-A ④ C-A-B ⑤ C-B-A

 


23. EVH1 4-3

 

It was while she was in London in 1853 that she heard about the Crimean War and the collapse of the nursing system down there.


(A) Thousands of men were dying of cholera, dysentery, cold, and battle wounds because of a lack of proper medical care. She immediately applied to the War Office to go to the Crimea and help the sick and wounded soldiers. In the application process, she stressed that she had considerable experience treating sick soldiers in Jamaica. 0

(B) Mary persevered and founded a firm with Thomas Day, a distant relative of hers, and they went to the Crimea with a large stock of medicines. Mary, who was then 50, was supposed to work officially as a sutler, someone who was allowed to sell goods to soldiers near the front. 2

(C) To her disappointment, her application was rejected. She asked herself, "Did these ladies shrink from accepting my aid because my blood flowed beneath a somewhat duskier skin than theirs?" Nothing, however, could stop her from following her aspirations. 1


① A-C-B ② B-A-C ③ B-C-A ④ C-A-B ⑤ C-B-A

 


24. EVH1 4-4

 

As soon as she arrived in the summer of 1855, however, Mary started to take care of the sick and wounded as if she were their mother.


(A) This is why the soldiers of the British army came to call her "Mother Seacole." One of the army doctors testified with admiration that despite the numbing coldness on the front line, Mary Seacole would care for the soldiers and provide them with tea, food, and words of comfort. 0

(B) She is always in attendance near the battle field to aid the wounded and has earned many a poor fellow's blessings." 2

(C) She also braved enemy fire near the front lines trying to find soldiers in need of help. One news reporter described her as "a warm and successful physician, who doctors and cures all manner of men with extraordinary success. 1


① A-C-B ② B-A-C ③ B-C-A ④ C-A-B ⑤ C-B-A

 


25. EVH1 4-5

 

At the end of the war in 1856, Mary turned out broke.


(A) Some army officers organized a benefit festival to raise funds in her honor. It was reported that to their surprise, thousands of contributors gathered and her name was shouted by a thousand voices. She was also awarded three medals for her bravery and her work from England, Turkey, and France, which is vividly depicted in her portrait. 1

(B) In 1857, with the help of her supporters, Mary published her autobiography, Wonderful Adventures of Mrs. Seacole in Many Lands. The last 25 years of her life, however, were spent quietly outside of public attention. She died on May 14th, 1881. 2

(C) She had spent all her personal wealth in the Crimea. She returned to England ill and poor. Fortunately, there were veterans who never forgot the amazing things that Mary had done on the battle field. 0


① A-C-B ② B-A-C ③ B-C-A ④ C-A-B ⑤ C-B-A

 


26. EVH1 4-6

 

During the Crimean War (1853-1856), Mary Seacole was from time to time compared to Florence Nightingale.


(A) Many people believe that she reappeared in history when a hero like her was needed most. 2

(B) In fact, she risked her life to aid the wounded and bring comfort to dying soldiers. While Florence Nightingale became a legend, Mary Seacole was largely forgotten until the year 2003 when her portrait was accidentally discovered. 1

(C) Unlike Nightingale, Seacole had to overcome prejudice related to her race. Being a born healer, she made her own way to the war where she was needed. 0


① A-C-B ② B-A-C ③ B-C-A ④ C-A-B ⑤ C-B-A

 


27. EVH1 5-1

 

Let Your Creativity Unfold!


(A) Is this really true? 1

(B) Can't ordinary people like you and me be creative? The following are some examples that tell us creativity is not a matter of talent or a gift, but is a matter of perspective nourished by passion and hard work. 2

(C) Many think that creativity is a gift given to few people. 0


① A-C-B ② B-A-C ③ B-C-A ④ C-A-B ⑤ C-B-A

 


28. EVH1 5-2

 

Tiny but Shiny You would probably think of using a pencil to write or draw something.


(A) One day in his twenties, however, he saw the beauty in small living things such as ants and spiders, and suddenly wanted to share this perspective with others. He saw a pencil on his desk, picked it up, and started carving its lead with a sewing needle and a very sharp blade. The lead was very fragile and easily snapped or broke as he applied his tools. 1

(B) Sculpting with such tiny tools on the lead was hard work, and it took weeks and sometimes months or years of concentration. However, he slowly and steadily improved his technique and turned his inspirations into pieces of artwork. He carved whatever he could think of, from a farm house to a framing hammer, all at the very tip of a pencil. A means of writing turned into an object of wonder because Dalton took a new perspective. 2

(C) You might not imagine a pencil actually being a creation itself. In fact, a man named Dalton M. Ghetti looked at a pencil as art, not just as a means to create it. Dalton was like most other sculptors in that he worked with large objects. 0


① A-C-B ② B-A-C ③ B-C-A ④ C-A-B ⑤ C-B-A

 


29. EVH1 5-3

 

Some people can use their body parts as tools for creative art by moving them in imaginative ways.


(A) His artistic experiments with hands began in 1990 when he started to use body painting techniques for a variety of purposes like advertising and exhibitions. He was an art major, and his interest in wildlife protection led him to be increasingly involved in "handimals." 1

(B) The animals he creates on hands look so realistic that whoever looks at them may not realize at first that they are just painted hands. 2

(C) From the right angle, a hand can be seen as two soccer players competing for a ball or a sprinter anxiously waiting for the starter's gun to fire, as illustrated in Annie Ralli's works. The Italian artist, Guido Daniele, can create amazing illusions with his hand art. 0


① A-C-B ② B-A-C ③ B-C-A ④ C-A-B ⑤ C-B-A

 


30. EVH1 5-4

 

[Nature in Play].


(A) He is famous for his beautiful dress designs. However, he seldom draws dresses with pens or brushes. His designs come from everyday objects. He cuts out a hole in paper, and the gorgeous dress designs are borrowed from the real world. He gets his inspiration from golden leaves in an autumn garden to the blue sky on a summer day to cars and buildings on a busy city street. 2

(B) Creative minds know no limits. They catch every clue in nature and respond in novel ways. They observe their surroundings with keen eyes, get inspired, add a little artistic touch, and put into life what we don't see. Take a look at the billboard on the left. 0

(C) A woman with shiny golden hair smiles at you, but when the sun sets, her hair glows red. The color of her hair changes depending on the time of day and the position of the sun. At night, her hair will look pitch-black, which could never be reproduced by any artificial means. Edgar Artis goes even further in his work. 1


① A-C-B ② B-A-C ③ B-C-A ④ C-A-B ⑤ C-B-A

 


31. EVH1 5-5

 

[The Giant Canvas].


(A) A green frog looks as if it were lifting the road and raking fallen leaves under it. The street comes alive and tells us an astonishing story, energizing people who are leading busy lives in bleak, urban environments. 2

(B) For many street artists, the city where they live is the canvas for their artwork. Though their work is not considered mainstream, their ideas are bright and innovative. 0

(C) A manhole cover, which we can see on any city street, changes into a pineapple with some yellow paint. Who would have thought of using train tracks as a music sheet? 1


① A-C-B ② B-A-C ③ B-C-A ④ C-A-B ⑤ C-B-A

 


32. EVH1 5-6

 

Lyon, a city in France, is famous for its murals.


(A) Creative people are not magicians. They are ordinary people like you and me. The only difference is that they try to find new things in themselves and their surroundings. They seek to make connections that have never been made. 1

(B) For these reasons, we can see the expressive creativity of many different forms of art in everyday objects we see around us. Who knows? You can be inspired, too! 2

(C) The city greets its visitors with its story-telling murals scattered all around it. At a glance, there seem to be many buildings that line the stairs in the mural above. However, the picture is a giant mural painted on a building wall. It looks so real that whoever happens to see it may be tempted to take the steps up to visit with those painted men and women. 0


① A-C-B ② B-A-C ③ B-C-A ④ C-A-B ⑤ C-B-A

 


33. EVH1 6-1

 

[Livestock and the Environment].


(A) Sold in stores everywhere, bacon may be a triumph of modern living, but there is a dark side to the bacon industry. Pigs are raised in confined pens and fed corn that has been grown in vast fields that stretch as far as the eye can see. To grow the corn, huge amounts of fertilizer are used, which eventually gets washed into rivers and streams. 0

(B) To meet growing demand, livestock industries have turned to large-scale "manufacturing" of meat in huge "factories." Such a move, however, puts strain on the environment, in particular on the land and water resources of the earth. 2

(C) This affects the ecosystems of such bodies of water, and the fertilizer runoff ends up in fish and other marine life for kilometers around. This is the process necessary to get bacon onto our breakfast tables. Factory Farming That Dries Up Our Planet Annual meat consumption has reached an average of over 40 kilograms per person globally, and the demand is increasing. 1


① A-C-B ② B-A-C ③ B-C-A ④ C-A-B ⑤ C-B-A

 


34. EVH1 6-2

 

Every year, over one billion tons of grain are consumed by farm animals.


(A) The bed of the Colorado River in the U.S. is expected to dry up soon;. Too much water has been drained from the river as neighboring farms need to grow feed for cows. 1

(B) More than two thirds of all agricultural land is used to raise grains and vegetables for livestock while merely eight percent of the agricultural land is used to grow food for direct human consumption. Moreover, about one third of the world's fresh water is used for meat production, which cuts into the supply of water for humans. 0

(C) In short, a large portion of our natural resources is used for meat production. 2


① A-C-B ② B-A-C ③ B-C-A ④ C-A-B ⑤ C-B-A

 


35. EVH1 6-3

 

[From Land to the Ocean].


(A) Livestock is said to produce 130 times as much manure as humans in the U.S. When not managed properly, the manure and water containing it cause severe harm to the ecosystems of rivers and oceans. 1

(B) When rain comes, manure is washed away with the runoff, polluting nearby soil, streams, and rivers all the way until it reaches the ocean. 2

(C) As more people want more meat, it is inevitable that land for livestock be extended to meet the demand. More meat production results in increased water pollution in both rivers and oceans since cows and pigs eat a lot and at the same time produce a lot of manure. 0


① A-C-B ② B-A-C ③ B-C-A ④ C-A-B ⑤ C-B-A

 


36. EVH1 6-4

 

Once it reaches the ocean, it contributes to the formation of what is known as a "dead zone."


(A) When too many plants grow in the water, they use up the ocean's oxygen, suffocating other plants and animals. The largest known dead zone occurred in the Gulf of Mexico in 2002, when runoff from the Mississippi River resulted in over 20,000 square kilometers of the Gulf area to become uninhabitable for oceanic life. 2

(B) Dead zones occur around the world, primarily near areas where heavy farming and industrial activity spill nutrients into the water. 0

(C) Concentrated levels of nutrients in water result in the blooming of algae, which are simple forms of water plants. 1


① A-C-B ② B-A-C ③ B-C-A ④ C-A-B ⑤ C-B-A

 


37. EVH1 6-5

 

[The Unknown Contributor to Global Warming].


(A) However, about 15 percent is emitted by the growing number of cows and pigs. Statistics vary about the exact amount of methane emitted by a cow, but it is generally agreed that a single cow releases up to 120 kilograms of methane per year, more than any other domestic animal. That is about 1,000 times the amount of methane emitted by an average person. 2

(B) The increasing demand for meat and meat products brings trouble not only to the natural resources under our feet but also to those overhead in the atmosphere. Even though carbon dioxide is a major cause of global warming in the air, recent research has shown that methane can have a greater impact. Methane is responsible for about 16 percent of global greenhouse gas emissions. 0

(C) That doesn't seem like much, but experts warn that methane is over 20 times as powerful as carbon dioxide in trapping the sun's heat in the atmosphere. Where does all the methane come from? Much of the methane in the atmosphere is released naturally in damp areas. 1


① A-C-B ② B-A-C ③ B-C-A ④ C-A-B ⑤ C-B-A

 


38. EVH1 6-6

 

Methane is produced in a cow's special digestive system.


(A) A cow has a stomach with four rooms to break down the food it eats. The food digested in the first two rooms is sent to the mouth to be chewed again, and then sent to the third and the fourth rooms. A cow does this for about eight hours on average every day during which it keeps burping and passing gas, giving off methane into the atmosphere. 0

(B) Koreans ate an average of 11 kilograms of meat per person in 1980, but the average increased to 51 kilograms in 2014. As people prefer increased animal protein in their meals, our land, rivers, and oceans are bound to suffer with no solution in sight. Maybe it is time we give our diets a second thought. 2

(C) Imagine the enormous amount of methane emitted from the burping mouths of 1.5 billion cows on six continents. The world's livestock industries are growing at an unprecedented rate due to population growth and rising incomes. The world's population is predicted to reach around nine billion by 2050. 1


① A-C-B ② B-A-C ③ B-C-A ④ C-A-B ⑤ C-B-A

 


[ANSWER]
1. ② 2. ④ 3. ③ 4. ④ 5. ① 6. ② 7. ④ 8. ③ 9. ① 10. ⑤


11. ④ 12. ② 13. ① 14. ⑤ 15. ④ 16. ① 17. ④ 18. ② 19. ① 20. ④


21. ② 22. ④ 23. ① 24. ① 25. ④ 26. ⑤ 27. ④ 28. ④ 29. ④ 30. ③


31. ③ 32. ④ 33. ① 34. ② 35. ④ 36. ③ 37. ③ 38. ① 

728x90
반응형

728x90
반응형

evh | Since 2005 임희재 | 블루티쳐학원 | 01033383436 | 200710 22:49:58

 

PARA

 

1. 영비홍 1-1

 

Homin: Jinho invited me to his magic show the other day.


(A) He has set his mind on studying psychology in college as magic basically deals with people's minds. I wish I had a plan for my future. Eunseo: Why don't you visit the career development office? 1

(B) He performed wonderful card tricks in front of a huge audience. I am envious of him as he has a plan for his future dream of becoming a professional magician. He watches magic performance shows almost every day and keeps practicing magic tricks until he can perform them perfectly. 0

(C) You can get some important information from the office. Here's what I received from them. 2


① A-C-B ② B-A-C ③ B-C-A ④ C-A-B ⑤ C-B-A

 


2. 영비홍 1-2

 

To make the right career choice, you have to learn about yourself first.


(A) Your values, interests, and personality will make some careers more suitable for you and others less appropriate. You can use self-assessment tests, often called career tests, to better understand yourself. Free career tests are available at our office. If the career test results are not clear and don't solve the problem, don't give up. 0

(B) STEP 3 Get information about the jobs. STEP 4 Narrow down the list. STEP 5 Learn more about the jobs. 2

(C) A hobby you really enjoy can be an important part of choosing a career. 5 Steps to Make a Career Choice. STEP 1 Take a career test. STEP 2 Make a list of jobs. 1


① A-C-B ② B-A-C ③ B-C-A ④ C-A-B ⑤ C-B-A

 


3. 영비홍 1-3

 

Now, you should make a list of occupations based on your self-assessment test results.


(A) Get yourself involved in various activities that you enjoy and do your best today. Your best possible future will unfold when you make today the best it can be. 2

(B) You now have to gather more in-depth information from people who have first-hand knowledge. You should identify who they are and conduct interviews with them or ask them for a job-shadowing opportunity. If you are still not sure, don't worry. 1

(C) For each job on your list, you should do some careful research in order to learn about the occupation's educational requirements, outlook, and earnings potential. After researching, you can eliminate careers in which you are no longer interested. At this point you may only have two or three occupations left on your list. 0


① A-C-B ② B-A-C ③ B-C-A ④ C-A-B ⑤ C-B-A

 


4. 영비홍 1-4

 

I am a music lover, so yesterday I job-shadowed my uncle who is a music producer at X-Music.


(A) I left home very excited. Not only was I getting the day off of school, but I was also getting a chance to see in person how a music album is made. 0

(B) He also explained what needs to be done to create an album, from planning the album to arranging, recording, and editing the music. More than half of what he said sounded like a foreign language to me. 2

(C) The studio was full of the latest equipment, whose size and complexity amazed me. My uncle gave me a short explanation of the equipment. 1


① A-C-B ② B-A-C ③ B-C-A ④ C-A-B ⑤ C-B-A

 


5. 영비홍 1-5

 

We headed to the recording booth, where two singers were recording music for their rock band.


(A) Outside of the booth, the composer and a technician were waiting for us. My uncle sat down in the middle next to the composer when the recording started. It didn't take long, however, until it was stopped by my uncle, then by the composer and the technician. 0

(B) My day at X-Music helped me realize that love of music is not the only thing required to become a music producer: patience, leadership, and communication skills on top of a good ear for music are important skills to learn, too. I'll have to come up with a plan to develop these skills. 2

(C) They didn't like this or that about the music and had the singers repeat the same line again and again, all of which sounded perfect to me. When everyone got exhausted, my uncle cheered them up with occasional funny stories. It took a whole day to finish one song! 1


① A-C-B ② B-A-C ③ B-C-A ④ C-A-B ⑤ C-B-A

 


6. 영비홍 1-RM1

 

I taste chocolate and evaluate its quality, telling good chocolate from bad, faulty chocolate.


(A) Then I breathe out through my nose to identify the smell. I always record everything like smells, sounds, tastes, and textures on a computer. As a chocolate taster, the most important part of my job is keeping my mouth objective. 2

(B) Then comes the actual tasting. I put a small piece into my mouth, press it lightly against the roof of my mouth, and wait until it melts. 1

(C) The evaluation process begins with smelling samples and breaking them to see if they sound crisp. Old or improperly stored chocolate doesn't make a crisp sound. 0


① A-C-B ② B-A-C ③ B-C-A ④ C-A-B ⑤ C-B-A

 


7. 영비홍 1-RM2

 

I have to keep my mouth ready and lively so that my evaluation may be as objective as possible.


(A) Keen observation skills and a good understanding of basic concepts of chemistry are required, and a degree in Food Technology is recommended. 2

(B) Since I taste an average of thirty samples a day, one should not affect the taste of another. As soon as one is done, I spit it back out and wait for a while. 0

(C) Then I eat a cracker and drink some warm water to get my senses back. For those who want to become chocolate tasters, I would say love of chocolate is not all it takes to succeed. 1


① A-C-B ② B-A-C ③ B-C-A ④ C-A-B ⑤ C-B-A

 


8. 영비홍 2-1

 

Wide pants, a striped shirt, a baseball cap: walk down the street and you'll see what's in fashion.


(A) Here are some tips from three teenage fashion leaders on how to experiment and find your own style. 2

(B) Teens, like adults, tend to follow popular trends. That's fine if you're happy with the current style, but you can also create your own style if you're not. 0

(C) Think of Steve Jobs, and a figure of a man wearing a black shirt and blue jeans might pop into your head. Likewise, you can showcase your own charm and individuality with your own unique style, which can become a part of your personal image: a signature style. 1


① A-C-B ② B-A-C ③ B-C-A ④ C-A-B ⑤ C-B-A

 


9. 영비홍 2-2

 

What is your favorite color?


(A) One day, I realized that green has many shades, so I experimented with various shades of green and eventually, I found that a deep green helps me look great. If you like a certain color, you can try various shades like I did, and find one that suits you. 2

(B) Do you often wear fashion items in that color? 0

(C) As for me, my favorite color is green, but I felt it did not go well with my skin tone, so I used to avoid wearing it. 1


① A-C-B ② B-A-C ③ B-C-A ④ C-A-B ⑤ C-B-A

 


10. 영비홍 2-3

 

Once you decide on a color, you may discover the color wheel useful to find other colors that go well together.


(A) Choosing a mixture of analogous and complementary colors together, called split complementary colors, can be tricky but results in a calmer look than a combination of complementary colors. Unless your signature style is to wear colors that clash, using the color wheel will help you choose colors that are natural and pleasing to the eye. {The color wheel} 1 Analogous colorsPick one color on the color wheel, skip one, and choose the next one. 1

(B) 2 Complementary colorsSelect two colors on opposite sides of the color wheel. 3 Split complementary colorsChoose two analogous colors and the complementary color of the one that is found between them. 2

(C) There are three simple ways to use the color wheel: combining analogous colors, choosing complementary colors, and mixing together analogous and complementary colors. Combining analogous colors is one of the easiest ways of matching colors that will give you an elegant look, for example, a yellow shirt on top of green pants. Choosing complementary colors, like wearing a green skirt with red shoes, creates a bold impression. 0


① A-C-B ② B-A-C ③ B-C-A ④ C-A-B ⑤ C-B-A

 


11. 영비홍 2-4

 

I spend a lot of time at school in my uniform, so I don't pay too much attention to what I wear.


(A) Mixing and matching clothes is a lot of fun! My point is: If you have a favorite fashion item, and it goes well with your other clothes, you can create a new outfit every single day. 2

(B) I like to wear it under a denim shirt, a leather jacket, or a cardigan. Putting on clothes in layers like this keeps my style fresh even though I don't have a lot of clothes. 1

(C) But there is one thing I often wear outside of school: a black striped shirt. It's very casual with a simple design. 0


① A-C-B ② B-A-C ③ B-C-A ④ C-A-B ⑤ C-B-A

 


12. 영비홍 2-5

 

One of the simplest ways to develop your own style is to pay attention to accessories like shoes, hats, glasses, or watches.


(A) If your face is round, however, you may look better in angular or square frames. Here's another important tip. People tend to make purchasing decisions about frames in terms of material or color, but not many think about their eyebrows. 1

(B) Eyebrows can be so expressive, and covering them with your frames may hide the feelings you express, eventually making you look strict. BEFORE/ The frames cover the eyebrows, making him look strict. AFTER/ The frames do not cover the eyebrows, making him look less strict. 2

(C) I spend a lot of time choosing my eye glasses because they can be an important part of my look. When I choose frames, I think of my face shape. Since my face is rather angular, I usually choose round frames. 0


① A-C-B ② B-A-C ③ B-C-A ④ C-A-B ⑤ C-B-A

 


13. 영비홍 2-6

 

Climate plays a key role in what we wear, and the way we live as a whole.


(A) Pajamas have followed a similar route to become trendy fashion wear in the bedroom or on the streets. In India, where staying cool is very important, a pair of wide pants made of light and soft fabric seemed to be the perfect way to stay cool. The British rulers at the time liked the idea and spread the Indian pajamas to the rest of the world, helping them to arrive in our bedrooms. The old saying that necessity is the mother of invention does make sense, and some good inventions seem to spread quickly and easily. 2

(B) In the Netherlands, for example, where one fourth of its land area is located below sea level, an interesting form of shoes developed as a way of getting around the muddy roads. Wooden shoes, called klompen, were the perfect tool to keep farmers' feet dry while working in wet areas. A few artistic souls went further to begin dancing in klompen, which makes a pleasant series of rhythms, eventually providing a basis for modern tap dancing. 0

(C) The Inuit had to fight the cold and the wind, and came up with the idea of wearing a heavy jacket called an anorak. It was originally made from seal skin, and its surface had to be cared for regularly with fish oil for constant water resistance. Now, it is a modern winter necessity, coming in a variety of styles and colors. 1


① A-C-B ② B-A-C ③ B-C-A ④ C-A-B ⑤ C-B-A

 


14. 영비홍 3-1

 

"Going green" seems to be all the rage these days.


(A) Probably not. Some innovators have taken on the challenge of "going green," and have come up with some brilliant ideas to make "going green" easier and simpler. 2

(B) In your home, for example, do you always unplug your TV when you aren't watching it? Do stores in your neighborhood always keep their doors shut when the heaters or air conditioners are on? 1

(C) Stores and companies use phrases like this to promote their businesses as eco-friendly. It makes sense to try to live in a way that does not harm the environment, but is it easy to go green? 0


① A-C-B ② B-A-C ③ B-C-A ④ C-A-B ⑤ C-B-A

 


15. 영비홍 3-2

 

Reuse Oldies but goodies!


(A) It is a special garbage bag that can be filled with used, but still usable items. Placing the bag outside on the pavement makes whatever is in it available to anyone in the community. Goedzak's bright color attracts attention while the transparent side of the bag reveals its contents. 1

(B) Everyone has things that are no longer of any use, and chances are, those things will eventually get thrown away. However, some of the things that get thrown out are still useful to other people. Goedzak is a Dutch way of allowing people to get second-hand things that might otherwise be thrown away. 0

(C) People can help themselves to anything they like. What an idea! These transparent garbage containers have helped many Dutch people go greener by reducing the amount of trash going to landfills. 2


① A-C-B ② B-A-C ③ B-C-A ④ C-A-B ⑤ C-B-A

 


16. 영비홍 3-3

 

Share Talk to the community.


(A) According to the Food and Agriculture Organization (FAO) of the United Nations, a third of global food production goes into trash bins annually. In Germany alone, around eleven million tons of food are wasted every year. 0

(B) The only rule is not to pass anything on to others that you wouldn't eat yourself. This project may change the way people think about food: if food is not shared, it is wasted. 2

(C) Having been founded to tackle this problem on a local scale, the online platform, "foodsharing∙de" allows extra food in your fridge or cupboard to be distributed to neighbors. The basic concept is simple: people sharing food. 1


① A-C-B ② B-A-C ③ B-C-A ④ C-A-B ⑤ C-B-A

 


17. 영비홍 3-4

 

Waste Not Drink and eat it!


(A) Using disposable cups may be convenient, but it is not necessarily eco-friendly. They are a massive source of waste. Every year, people in the U.S. use over 100 billion disposable cups, and Koreans dispose of over 15 billion cups each year. 0

(B) You can think of it as a treat for coffee! You may have to consume extra sugar, but it will definitely create less waste. 2

(C) That's what drove a few novel designers to come up with edible coffee cups. A cookie forms the main structure, with a white chocolate layer on the inside and a thin layer of sugar paper on the outside. This structure allows you to drink coffee without finding yourself holding a soaked mess. 1


① A-C-B ② B-A-C ③ B-C-A ④ C-A-B ⑤ C-B-A

 


18. 영비홍 3-5

 

Use Less Holes mean a lot.


(A) In fact, Ecofont uses about a fifth less ink than traditional fonts without ruining readability. The brilliance of Ecofont is the different perspective it takes on going green: the use of less ink by the font. 2

(B) What can you do to go green when you have 500 photocopies to make? Many green strategies focus on using less paper, like printing on both sides. Another green strategy is to use less ink, which is what many people already do. 0

(C) But what if you could take it a step further? That's what Ecofont is. A designer thought that if he could create fonts that have tiny holes in them, he might be able to make more efficient use of the amount of ink used. 1


① A-C-B ② B-A-C ③ B-C-A ④ C-A-B ⑤ C-B-A

 


19. 영비홍 3-6

 

An eco-friendly way of life is not about changing the entire world overnight.


(A) When you do these things, you are keeping your environmental bank account full. When you go one step further and help others do them too, you are an activist and big changes can happen. 2

(B) It is about becoming aware of your own wasteful ways and then helping others become aware of theirs. 0

(C) After awareness comes the process of adopting slightly different ways of doing some daily tasks. 1


① A-C-B ② B-A-C ③ B-C-A ④ C-A-B ⑤ C-B-A

 


20. 영비홍 4-1

 

I was thrilled when my cousin, Suji, invited me to Italy, a country in southern Europe that looks like a boot.


(A) Since Italy is seven hours behind Seoul, I was quite exhausted and sleepy when I got there. However, I forced myself to stay awake to begin my trip in earnest. 2

(B) She had a few days off from studying music in a conservatory, so we could spend a week together in Rome and Venice. 0

(C) I had never been abroad by myself, and I was a bit worried, but after the long flight for twelve hours, I was pleased to be greeted by my cousin at Leonardo da Vinci International Airport in Rome. 1


① A-C-B ② B-A-C ③ B-C-A ④ C-A-B ⑤ C-B-A

 


21. 영비홍 4-2

 

Rome was like a giant museum to me.


(A) It amazed me to think that people could build such a structure without modern construction equipment. The Colosseum has eighty arches through which about fifty thousand people could go in and out in fifteen minutes! 1

(B) We began our sightseeing at the Colosseum. Nowadays, we can only see a part of what was once the greatest structure in the ancient world. 0

(C) As I reached the top of the stairs inside, I looked down and I could almost hear the cheer of the crowd. 2


① A-C-B ② B-A-C ③ B-C-A ④ C-A-B ⑤ C-B-A

 


22. 영비홍 4-3

 

Suji and I walked along a path leading away from the Colosseum and heard the sound of falling water.


(A) We knew instantly that we were near the famous Trevi Fountain. 0

(B) I threw one over my shoulder, wishing a return to Italy someday. I found it interesting that even on the other side of the world, people still wish for simple things like happiness, love, and marriage. 2

(C) Legend says that a single coin thrown into the fountain will ensure a return to Rome, a second coin will bring true love, and a third coin marriage!1


① A-C-B ② B-A-C ③ B-C-A ④ C-A-B ⑤ C-B-A

 


23. 영비홍 4-4

 

Who in Rome could miss out on a chance to visit Vatican City, the place where the Pope lives?


(A) Suji convinced me to wait in line for over twenty minutes saying that it would be worth it. She was right: the gelato was out of this world. 2

(B) The Creation of Adam, one of Michelangelo's masterpieces, on the ceiling of the Sistine Chapel, still lingers in my mind. Although I knew photographs are not allowed, the masterpiece was so impressive that I almost took one. After looking around, we walked out to see many people lined up in front of a small store where green apple gelato was served. 1

(C) It is known as the smallest state in the world. In fact, it takes only thirty minutes to walk from one border to the other! I was overwhelmed, however, by the collection of sculptures and paintings in the Vatican Museums. 0


① A-C-B ② B-A-C ③ B-C-A ④ C-A-B ⑤ C-B-A

 


24. 영비홍 4-5

 

After a couple of more days in Rome, we headed to Venice.


(A) Suji said she had to visit a friend, so she suggested that I explore the city on my own for a few hours. 1

(B) The seaside city was a lot more romantic than Rome, but a lot more humid as well. 0

(C) We could meet up at the train station later in the afternoon. 2


① A-C-B ② B-A-C ③ B-C-A ④ C-A-B ⑤ C-B-A

 


25. 영비홍 4-6

 

I decided to go to the Rialto Bridge first, so I started walking.


(A) They were so used to being around people that they would wait until people weren't paying attention and then steal their crackers! But what truly made me stop and stare in wonder were the beautiful buildings surrounding all three sides of the square. Along the buildings were shops selling beautiful glass pieces, gloves, and much more. After looking around for a while, I bought small glasses for my parents. 2

(B) Within moments, the winding alleys made my map almost useless. After some wandering, I was lucky enough to meet a group of tourists my age from Britain. They were headed to the Rialto Bridge too! 0

(C) The bridge itself was as elegant as people say it is, but I was more impressed by the beautiful view of the canal from the steps of the bridge. I said goodbye to my British friends and walked to St_ Mark's Square, one of the prime attractions of Venice. I had never seen so many pigeons in my life. 1


① A-C-B ② B-A-C ③ B-C-A ④ C-A-B ⑤ C-B-A

 


26. 영비홍 4-7

 

My trip to Venice would not be complete without a gondola ride along the Grand Canal, which snakes through the city in a large S shape.


(A) We shared the fare and we commented on the unique differences of the buildings along the canal. We had a nice chat, took some great pictures, and exchanged email addresses before we got off the gondola. 2

(B) I was disappointed to find out the fare to ride the Grand Canal by myself was so expensive that I could not afford it. 0

(C) The moment I was turning back, I saw my British tourist friends walking toward the ticket office. 1


① A-C-B ② B-A-C ③ B-C-A ④ C-A-B ⑤ C-B-A

 


27. 영비홍 4-8

 

My trip to Italy was definitely an experience of a lifetime.


(A) So far, I have read two pages. I hope I have opportunities to read many more pages. 2

(B) People say that the world is a book, and that those who do not travel read only one of the pages in it. 1

(C) I hope my next trip can be to England to visit my British friends. 0


① A-C-B ② B-A-C ③ B-C-A ④ C-A-B ⑤ C-B-A

 


28. 영비홍 5-1

 

The Amazing Story of the 33 / Alone in the Dark.


(A) The miners locked in below did not give up in the dark, nor did their families above ground surrender to despair. 2

(B) On October 13, 2010, thirty three Chilean miners who had been buried inside the San José mine for 69 days were finally rescued. 0

(C) It was a triumph of engineering and a victory of faith. 1


① A-C-B ② B-A-C ③ B-C-A ④ C-A-B ⑤ C-B-A

 


29. 영비홍 5-2

 

On August 5, 2010, at around lunch break, miners digging for copper and gold started to feel vibrations in the earth.


(A) As the days passed, Chileans grew increasingly uncertain that any of the miners had survived. A small exploratory hole was drilled on August 22, and the camera captured a message that said, "We are still alive. "It was the first sign of hope. 1

(B) Soon, a video camera was sent down 700 meters deep and captured the first images of the miners, all clearly in good health. The discovery sparked joyful celebrations nationwide, and rescue efforts gave a light of hope that the miners could be saved. 2

(C) Almost immediately after the vibrations began, they heard a sudden huge explosion, and the whole mine filled up with dust and rock. A massive piece of the nearby mountain had broken off, burying almost all the layers of the mine. For seventeen days after the initial collapse, there was no word on their fate. 0


① A-C-B ② B-A-C ③ B-C-A ④ C-A-B ⑤ C-B-A

 


30. 영비홍 5-3

 

The miners were lucky to have an air tunnel that allowed enough fresh air to reach them.


(A) For eighteen days, each person had to live on two spoonfuls of tuna, a mouthful of milk, bits of crackers, and a bite of canned fruit every other day. Another factor which bothered the miners severely was the high heat and humidity of the shelter. Each miner had lost an average of 8 kilograms by the time they were rescued. 2

(B) Until the tunnel to deliver food and medicine was operational, food was the most critical issue in the shelter. They only had enough food for two days. 1

(C) They also had broken trucks from which they could charge the batteries of their head lamps. In addition, they were able to drink water from storage tanks nearby. 0


① A-C-B ② B-A-C ③ B-C-A ④ C-A-B ⑤ C-B-A

 


31. 영비홍 5-4

 

The miners united as a group soon after the collapse.


(A) They organized themselves into a society where each person had one vote. They all knew that if their social structure broke down, their problems would become more serious and did what they could do best. 0

(B) It created a tunnel large enough to lift them one by one. For this purpose, a specially designed capsule was built. More than 1,400 news reporters from all over the world, together with the family members of the miners, gathered to watch the rescue process. 2

(C) For example, José Henríquez, a religious man, tried to keep morale up, and Yonni Barrios, who had had some medical training, helped other miners with their health problems. On October 9, a rescue hole was finally drilled through to the miners in their shelter. 1


① A-C-B ② B-A-C ③ B-C-A ④ C-A-B ⑤ C-B-A

 


32. 영비홍 5-5

 

On October 12, the first rescue worker was sent downward to the miners, who greeted him with nervous relief.


(A) They had been trapped for so long that their first priority was to get medical attention. Luis Urzúa, who had taken a major role as a democratic leader while underground, was the last one to come up to the surface on October 13. "The 69 days during which we tried so hard were not useless. 1

(B) Soon, the first trapped miner was raised to the surface. One by one, the miners were brought up in the capsule to see the sunlight. Upon leaving the capsule, each miner was enthusiastically greeted, but they could not see their families right away. 0

(C) We wanted to live for our families, and that was the greatest thing," Urzúa said to the Chilean people after his rescue. Then, the rescuers and the rescued began singing the Chilean national anthem with the thousands of joyous people who came to support the operation, celebrating the heroics and the humanity of all those involved. 2


① A-C-B ② B-A-C ③ B-C-A ④ C-A-B ⑤ C-B-A

 


33. 영비홍 5-RM

 

The rescue capsule Fénix was designed by the Chilean Navy in collaboration with NASA to rescue the 33 miners trapped underground in Chile's San José copper mine.


(A) In an emergency, such as the capsule getting stuck in the rescue hole, the bottom could be opened from the inside so that the miner could be lowered back down to the shelter. Thanks to the capsule and the organized rescue efforts, the miners put an end to their life of darkness. When they stepped out of the capsule door to meet their loved ones, the designers of the Fénix shared in the joy;. their efforts resulted in the saving of 33 lives. 2

(B) The bottom of the capsule held three tanks of air, which would provide enough air for one person to breathe for about 90 minutes. This amount of air was more than needed for the 15 to 20 minutes that the trip to the surface was expected to take. It also had a communication system so that the miners could speak to the surface while being lifted up. 1

(C) Painted in the red, white, and blue colors of Chile, the Fénix looked like a military missile. The device was nearly 3.95 meters high on the outside, and the inside was 1.9 meters high and about 51 centimeters across, which was just enough room for an adult male. When mobilized by the rescue team, the Fénix was equipped with an emergency oxygen supply. 0


① A-C-B ② B-A-C ③ B-C-A ④ C-A-B ⑤ C-B-A

 


34. 영비홍 6-1

 

A Jungle of Choice / Decisions, decisions, decisions.


(A) Since we can't have everything we want, we have to make the resources that we have go as far as possible. Imagine that you go to a shopping mall to buy a pair of jeans. Let's take a look at some of the things that may affect your decisions while you are there. 2

(B) Stores are full of attractive products. Advertisements cover cars and buildings, TV commercials shout slogans, and pop-up promotions on the Internet can be annoying. 1

(C) Being a consumer is tough these days. At the same time, without decisions to make, being a consumer would be easier but much less interesting. 0


① A-C-B ② B-A-C ③ B-C-A ④ C-A-B ⑤ C-B-A

 


35. 영비홍 6-2

 

"Hey, these jeans are on sale!


(A) If the jeans were originally $100 but are now on sale for $80, the lower price would lead more customers to consider buying the jeans and spending another $20 on a T-shirt, too. The bottom line is that sales attract customers that might not have made purchases at the regular price, and they motivate customers to spend because their money can now buy more. 2

(B) "Have you ever wondered why retail stores put items on sale? 0

(C) Sales reduce inventory size, making room for the store to buy more stuff to sell, and they attract customers. 1


① A-C-B ② B-A-C ③ B-C-A ④ C-A-B ⑤ C-B-A

 


36. 영비홍 6-3

 

"Become the person you've always wanted to be.


(A) When advertisers associate appealing images with certain products, consumers may buy the products to associate themselves with those images. You're still the same you, but you feel better about yourself because you are wearing Brand X's new jeans. Is this worth paying 25%, 50%, or even 100% more? Well, that's up to each individual to decide on his or her own. 2

(B) "Jeans are jeans, right? Well, no! There are ordinary jeans and there are designer jeans. 0

(C) As the TV ads prove, beautiful people wear Brand X, don't they? And you feel you'll be more beautiful if you wear it, too. This is the power of association. 1


① A-C-B ② B-A-C ③ B-C-A ④ C-A-B ⑤ C-B-A

 


37. 영비홍 6-4

 

"How do you like these sneakers to go with those jeans?


(A) "Have you ever been offered to buy something that you had not planned on buying? A sales clerk may make suggestions to you about what else to buy in addition to your originally planned purchase. This is called up-selling and it's designed to be not only helpful for you, but also for the store's bottom line. 0

(B) No one can tell you that you shouldn't buy something that really suits you, but remember that the arrangement of items in a store is not random. Product placement seems to have been designed to give subtle suggestions to consumers while they shop. 2

(C) Have you also noticed that shoes, hats, and socks are displayed together next to one another? They are mostly inexpensive items strategically placed there. Since you've already decided to buy a pair of jeans, why not buy a pair of sneakers too? 1


① A-C-B ② B-A-C ③ B-C-A ④ C-A-B ⑤ C-B-A

 


38. 영비홍 6-5

 

What Most People Do Why are you influenced by these marketing strategies?


(A) Furthermore, if a cashier recommends something, you may feel as if you "needed" it all along. 2

(B) What's going on in your head? Well, when your brain is loaded with too many decisions to make, it may go on "autopilot. 0

(C) "Instead of deliberating, you choose the easy way and make your decisions automatically. For example, many people may simply assume that buying an item on sale will save them money, or that something with a higher price tag is better in quality. 1


① A-C-B ② B-A-C ③ B-C-A ④ C-A-B ⑤ C-B-A

 


39. 영비홍 6-6

 

Notice What's Out There!


(A) Don't worry, though. Being a smart consumer is not something that comes naturally. Once you start noticing what's out there, your experience and wisdom will guide you to smart consuming. 2

(B) If there are so many choices and marketing strategies out there, how can you become a smart consumer? There isn't a "right" answer for everyone because we have different tastes and different values, but the first step is to be aware of your "autopilot" mode. To prevent this, ask yourself these questions before you make any purchase. 0

(C) Do I really need the product or do I simply want it? Would my money be better spent on something else? In the jungle of information, you may feel overwhelmed. 1


① A-C-B ② B-A-C ③ B-C-A ④ C-A-B ⑤ C-B-A

 


40. 영비홍 6-RM

 

Street Food Business Around the World.


(A) These recipes are often handed down from the mother of the family to her children. Meals from the Sky. Melbourne, Australia has many small buildings and narrow alleys between them. This gave rise to a unique restaurant called Jafflechutes, which sells sandwiches by parachute. Three young men wanted to sell sandwiches, but they didn't have the money to rent a ground floor space. Instead of customers coming up, they thought why not send sandwiches down? 1

(B) Customers can pay online, select a time, stand on an 'X' marked outside the building, and wait for the parachute with their name on it. Food that Moves. In the United States, food trucks have become very popular among workers in large cities. Food trucks can move around, and many sell food that originated in other countries like Mexico, China, Brazil, and Turkey. For example, one of many recently popular food trucks, called Kogi, sells a mix of BBQ from Korea and food from Mexico. Kogi's marketing strategy uses social networking to announce where it will turn up next. 2

(C) People run street food booths around the world for a number of reasons, such as to get an opportunity to open their own business on a budget, to sell food in a lively place, or to promote their own special recipe. The advantage of these kinds of small businesses is that they provide opportunities for owners on smaller budgets. At the same time, consumers benefit because of the variety of food that is available. Special Family Recipes. In the crowded cities in Vietnam, people usually keep their doors open, so cooking and eating food on the street is natural. Street food businesses in Vietnam are usually owned by the family, and recipes are kept in the family. 0


① A-C-B ② B-A-C ③ B-C-A ④ C-A-B ⑤ C-B-A

 


41. 영비홍 7-1

 

Discover from Nature Do you remember your mom taking you to a doctor's office?


(A) What if a shot given in the arm didn't hurt? Well, maybe in the future, it won't. Some scientists have been studying how the lowly mosquito is able to bite us without us knowing. 1

(B) When they figure out the secrets, the doctor's shot might become a painless procedure. This is what "biomimetics" is about. 2

(C) The doctor decided to give you some medicine. She rolled up your sleeves and gave you a shot in the upper arm. That hurt, didn't it? 0


① A-C-B ② B-A-C ③ B-C-A ④ C-A-B ⑤ C-B-A

 


42. 영비홍 7-2

 

People who study biomimetics look to solve human problems by copying how nature does it.


(A) Let's look at some examples of biomimetics we see today or will see in the near future. 1

(B) 2

(C) It might sound complicated, but it is much easier once you understand. 0


① A-C-B ② B-A-C ③ B-C-A ④ C-A-B ⑤ C-B-A

 


43. 영비홍 7-3

 

Natural Air Conditioning Termites are simple creatures, but when they work together, they can build incredible natural structures.


(A) even though the temperature outside can vary from 40 degrees Celsius during the day to 1 degree Celsius at night, it is always about 30 degrees Celsius inside. Termite mounds are built in a way that hot air rises out and cool air comes in through the bottom. Inspired by termite mounds, Mike Pearce, an African architect, constructed a building in Zimbabwe and another in Australia using the same passive cooling techniques. 1

(B) Some termite mounds can reach 7 meters in height. They even go 3 meters underground. There is another amazing fact about termite mounds:. 0

(C) These buildings were 10% cheaper to build because less money was spent on air moving equipment, and this design reduced cooling energy costs by 35%. Now that's a cool idea! 2


① A-C-B ② B-A-C ③ B-C-A ④ C-A-B ⑤ C-B-A

 


44. 영비홍 7-4

 

Airplane Winglets If you look at an airplane's wings, you can sometimes see that the tips are turned upwards.


(A) They found that the tips smooth the flow of air, which helps them conserve energy when flying. The engineers thought that if it worked for birds, why not for airplanes? 1

(B) The end result is that the airplane winglets help keep planes smaller, saving about 10% in fuel costs. This is beneficial not just for the environment but for passengers' wallets, too. 2

(C) These are called "winglets" and they may look neat, but they have measurable benefits. When engineers studied birds, they observed that birds' wings have tips that turned up in flight. 0


① A-C-B ② B-A-C ③ B-C-A ④ C-A-B ⑤ C-B-A

 


45. 영비홍 7-5

 

Sea Urchins Sea urchins may be eaten in some parts of the world, but they can also damage parts of the sea environment with their bony mouths.


(A) The efficiency of this natural design is now being tested for incorporation into missions in space. When small robots are sent to another planet to collect soil samples, the standard method is to use something inefficient like a small shovel. 1

(B) A sea urchin mouth looks a lot like a five-fingered claw you might see while trying to pick up prizes at the arcade. This design is surprisingly efficient at grabbing and grinding. 0

(C) By copying the design of a sea urchin mouth, scientists believe it will be easier to collect samples. Amazingly, a design developed naturally in the deep sea may soon be seen in deep space. 2


① A-C-B ② B-A-C ③ B-C-A ④ C-A-B ⑤ C-B-A

 


46. 영비홍 7-6

 

When we observe nature, we can be amazed by its beauty and its grand scale.


(A) Now, as we observe nature on all levels big and small, we have the technology and the means to copy what nature has perfected. 1

(B) Because there is still so much we do not know about nature, there is still much left to discover. Maybe one day you will be inspired by nature to invent a product that will change the world. 2

(C) We should remember that nature has had millions of years to experiment. 0


① A-C-B ② B-A-C ③ B-C-A ④ C-A-B ⑤ C-B-A

 


47. 영비홍 8-1

 

Where Sound, Color and Letters Meet A piece of work in one field can inspire artists in another field to create something new.


(A) Likewise, a painting can inspire a musician to create music in which you can almost see different colors and shapes. 1

(B) Furthermore, lines from a novel or a poem can inspire painters or musicians to create visual or auditory art that gives life to a story. These interactions between artists can have unexpected results, producing works of art that have strong visual, auditory or emotional influences on people. 2

(C) Music can inspire a painter to create a visual representation of something he or she has heard. 0


① A-C-B ② B-A-C ③ B-C-A ④ C-A-B ⑤ C-B-A

 


48. 영비홍 8-2

 

Music Drawn on the Canvas Music has played a key role in the creation of some artwork.


(A) In his paintings, for example, yellow is linked with the sound of the trumpet and blue with that of the cello. In addition, certain shapes in his paintings were associated with particular feelings. The triangle represents aggressive feelings and the square calm moods. Each time he stroked the canvas with his brush, he might have intended to turn a series of musical notes into visual forms. 2

(B) "I saw all my colors before my eyes," he said. He felt as if wild and powerful lines appeared in front of him. As a result, he gave up his law career to study painting. For Kandinsky, music and color were closely tied together. 1

(C) The influence of music on the visual arts can be best seen with the expressionist painter Wassily Kandinsky. Kandinsky studied law and economics and was successful in his law career. However, in his early 30's, he had an unusual visual experience while looking at Monet's Haystacks. He also was influenced by the melody of Wagner's Lohengrin. 0


① A-C-B ② B-A-C ③ B-C-A ④ C-A-B ⑤ C-B-A

 


49. 영비홍 8-3

 

Melodies Reflecting Colors and Shapes Musicians have also found inspiration from painters and their works of art.


(A) After visiting a memorial exhibition of Hartmann's works, Mussorgsky composed a piano suite in 10 movements to describe each of Hartmann's paintings displayed at the exhibition. Anyone who listens to the movements can associate the melodies with what they see in Hartmann's paintings. 1

(B) While Mussorgsky was writing the melodies, he must have wanted to translate the stories in the paintings into his musical language. 2

(C) Modest Mussorgsky was a composer who is famous for his descriptions of colors in his music. One of his most frequently performed piano works, Pictures at an Exhibition, was composed in his efforts to capture what he felt about the paintings of an artist friend named Viktor Hartmann, who died at the early age of 39. 0


① A-C-B ② B-A-C ③ B-C-A ④ C-A-B ⑤ C-B-A

 


50. 영비홍 8-4

 

Words Living in Melodies and Images A novel or a play often inspires musicians and painters.


(A) The Wedding March is one of the best known pieces from the suite. Marc Chagall, known for his use of dreamy colors, was also moved by the play and drew a painting with the same title, Midsummer Night's Dream. 1

(B) The figures in the painting recreate the dreamlike atmosphere of the play. Although Chagall and Mendelssohn lived in different times, they both translated Shakespeare's words and sentences into their own artistic languages. 2

(C) For example, Felix Mendelssohn was inspired after reading Shakespeare's play, A Midsummer Night's Dream, at the age of 17 and began to compose a piece of music to capture the magic and fantasy in Shakespeare's imaginary world. It became part of his famous work, A Midsummer Night's Dream. 0


① A-C-B ② B-A-C ③ B-C-A ④ C-A-B ⑤ C-B-A

 


51. 영비홍 8-5

 

The English word "inspire" originally meant "to breathe in."


(A) Maybe, they knew we would interpret their works accordingly, noticing the melodies, colors, shapes, and the words influencing each other. 2

(B) Kandinsky, Mussorgsky, Mendelssohn, and Chagall were great breathers because they turned their intakes into artwork that stimulates us in novel ways. 1

(C) Air breathed in has to be breathed out in one way or another. 0


① A-C-B ② B-A-C ③ B-C-A ④ C-A-B ⑤ C-B-A

 


[ANSWER]
1. ② 2. ① 3. ⑤ 4. ① 5. ① 6. ⑤ 7. ③ 8. ③ 9. ③ 10. ④


11. ⑤ 12. ④ 13. ③ 14. ⑤ 15. ② 16. ① 17. ① 18. ③ 19. ③ 20. ③


21. ② 22. ① 23. ⑤ 24. ② 25. ③ 26. ③ 27. ⑤ 28. ③ 29. ④ 30. ⑤


31. ① 32. ② 33. ⑤ 34. ⑤ 35. ③ 36. ③ 37. ① 38. ③ 39. ③ 40. ④


41. ④ 42. ④ 43. ② 44. ④ 45. ② 46. ④ 47. ④ 48. ⑤ 49. ④ 50. ④


51. ⑤ 

728x90
반응형

728x90
반응형

esh | Since 2005 임희재 | 블루티쳐학원 | 01033383436 | 200710 22:49:58

 

PARA

 

1. 영시한 1-1

 

Shonda Lynn Rhimes is a famous American television producer and writer.


(A) Don't stop dreaming until your dreams come true. I think that's nonsense. 2

(B) When people give graduation speeches, they usually tell you: Follow your dreams. Dream and dream big! 1

(C) Rhimes delivered the following speech at the graduation ceremony for Dartmouth College in 2014. President Hanlon, staff, honored guests, parents, students, families, and friends, good morning and congratulations to the graduating class! 0


① A-C-B ② B-A-C ③ B-C-A ④ C-A-B ⑤ C-B-A

 


2. 영시한 1-2

 

A lot of people dream.


(A) You don't have to know. You just have to keep moving forward. You just have to keep doing something, seizing the next opportunity, and staying open to trying something new. It doesn't have to be the perfect job or the perfect life that you dream of. Perfect is boring, and dreams are not real. Just do! If you're dreaming of traveling, leave right now. 1

(B) Do you want to be a writer? Start writing. A writer is someone who writes every day. If you don't have a job, get one. Get any job. Don't sit at home waiting for the magical opportunity. Do something until you can do something else. 2

(C) And while they are busy dreaming, the really happy and successful people are busy doing. Dreams are lovely, but they are just dreams. Dreams do not come true just because you dream them. It is hard work that makes things happen and creates change. So, this is my first lesson for you: Be a doer, not a dreamer. Maybe you know exactly what you want to do, or maybe you have no idea what you truly want to do. It doesn't matter. 0


① A-C-B ② B-A-C ③ B-C-A ④ C-A-B ⑤ C-B-A

 


3. 영시한 1-3

 

My second lesson is this: Don't be self-centered.


(A) Crime and violence are part of the everyday lives of these people. So before you complain, remember that you have been given a gift. Your whole life so far has been a gift. It's time to pay for it by doing something for the world. 2

(B) Right after graduation, you may have the worst days of your lives. But don't act like you're the most miserable person in the world, because you are not. We are already a lot luckier than most people on the earth. We live in the country where we are free to speak our own mind and most people believe that everyone should be treated equally regardless of gender or race. 0

(C) In some parts of the world, girls are harmed simply because they want to get an education. Slavery still exists. Children still starve to death. Even in the United States, there are countless people who are living much more difficult lives than we can ever imagine. 1


① A-C-B ② B-A-C ③ B-C-A ④ C-A-B ⑤ C-B-A

 


4. 영시한 1-4

 

Accept that you cannot do everything.


(A) Since I'm a very successful woman and a mother with three children, many people ask me, "How do you do it all? How do you become successful at your job while having a family?" Now I'm going to give you an entirely honest answer: I don't. Whenever I'm succeeding in one area of my life, I'm failing in another area. 0

(B) It will happen to you when you become successful. We all achieve one thing by failing to achieve something else. Anyone who tells you they are doing it all perfectly is a liar. 2

(C) If I'm writing really exciting stories for television, I'm not spending enough time with my children. If I'm enjoying a family holiday with my children, I'm not finishing the script that I should. If I'm succeeding at one, I'm inevitably failing at the other. This happens with all truly successful people. 1


① A-C-B ② B-A-C ③ B-C-A ④ C-A-B ⑤ C-B-A

 


5. 영시한 1-5

 

I hope I didn't scare you.


(A) You may sometimes feel like you're failing at work or at your home life. The real world is hard. 1

(B) I just wanted to say that it is okay, even if it doesn't look okay. Your dreams may not all come true. 0

(C) And yet, you can still wake up in the morning and be amazed at everything you have and everything you have achieved. And then 20 years from now, one of you can find yourself right here where I stand, giving a graduation speech. 2


① A-C-B ② B-A-C ③ B-C-A ④ C-A-B ⑤ C-B-A

 


6. 영시한 2-1

 

Darwin's "survival of the fittest"is universally regarded as a simple reality in the game of life.


(A) Often, animals help each other and work together for the greater good. Cooperation prevails at every level of the animal kingdom. 1

(B) In the following three examples, it is inspiring to see how animals cooperate to help each other. 2

(C) Successful play sometimes requires a degree of selfishness. However, across the animal kingdom, selfishness is only a very small part of an animal's life. 0


① A-C-B ② B-A-C ③ B-C-A ④ C-A-B ⑤ C-B-A

 


7. 영시한 2-2

 

Have you ever observed ants marching in and out of their nest?


(A) When there are a few ants moving in a single direction, some ants are likely to move slowly, some fast, and others even faster. As their number increases, however, they communicate with each other so that they all move at the same speed. The slow speed up while the fast and the superfast slow down. There are times when ants leaving the nest for food encounter other ants coming back with food on their backs. 1

(B) If so, it may have reminded you of a highway packed with cars. But this comparison is not appropriate. Ants never get into a traffic jam, no matter how fast they move or how many of them occupy the road. This is because their movements are based on cooperation. 0

(C) When this happens, the inbound ants, the ones carrying a burden, always have the right of way. Outbound ants turn to get out of the way, while inbound ants continue in a straight line. This arrangement makes sense because it is more difficult for inbound ants to change direction due to their heavy loads. Following these rules and cooperating in this way, ants improve the traffic flow and make the delivery of food to the nest more efficient. 2


① A-C-B ② B-A-C ③ B-C-A ④ C-A-B ⑤ C-B-A

 


8. 영시한 2-3

 

Fish also cooperate.


(A) First, the cleaners are too small to satisfy their hunger. Second, it is hard to find good cleaners. Once the cleaners and the clients establish trust, the clients want to cooperate with the cleaners and keep them around. 2

(B) After the cleaners do their job, the clients let them go, instead of swallowing them. The cleaners get a meal, and the clients get a healthier mouth. Sometimes the cleaner fish chew on the wall of the client fish's mouth. Although this may cause pain, the big fish still do not eat the little ones for two reasons. 1

(C) As everyone knows, big fish often eat little fish. Sometimes, however, little fish help the big fish, so the big fish do not eat the little fish. Tiny fish, which scientists refer to as "cleaners," swim into the mouths of bigger fish, which are referred to as "clients." The cleaners eat parasites in the clients' mouths. 0


① A-C-B ② B-A-C ③ B-C-A ④ C-A-B ⑤ C-B-A

 


9. 영시한 2-4

 

Pied flycatchers are small birds that cooperate in dangerous situations.


(A) Bats die if they go two nights without food. As hunting for blood—heir only food—s not easy, younger bats fail to find food one night out of three. But hunger is rare because bats that find blood share it with bats that don't. 2

(B) They do not answer the calls of birds that have refused to help others before. More Info. Do you know how bats cooperate to survive? One kind of bat has a system of food sharing. 1

(C) When a predator enters a flycatcher's area, the flycatcher makes a loud sound for help. Other flycatchers hear the call, and they come to help. As more and more birds join the group, the predator becomes frightened and runs away. It is important to note that flycatchers go to the aid of another bird in danger only if that bird has helped other birds in the past. 0


① A-C-B ② B-A-C ③ B-C-A ④ C-A-B ⑤ C-B-A

 


10. 영시한 2-5

 

Cooperation is a process that started with the first single-celled creatures.


(A) We human beings cooperate in plenty of ways, from lining up at a bus stop to sharing knowledge on a website. 1

(B) Sometimes, however, human cooperation can break down. To foster cooperative success among humans, we should perhaps look to nature for inspiration. 2

(C) Living things, from their beginnings more than three billion years ago, took over the planet by learning to cooperate. 0


① A-C-B ② B-A-C ③ B-C-A ④ C-A-B ⑤ C-B-A

 


11. 영시한 2-6

 

There are many team sports which require cooperation among the team members in order to win.


(A) Let' see the origins of a few of these sports. Field hockey has a long history. Four-thousand-year-old drawings of men playing a simple version of a hockey game have been found in tombs in Egypt. In the Middle Ages variations of hockey-like games were played in European countries. Later England modernized it. 0

(B) The first reference to rowing as a sport comes from 15thcentury BC Egypt. It later spread throughout Europe. Modern rowing as a competitive sport goes back to England in the 18th century. Now one of the most famous competitions is the Oxford and Cambridge Boat Race. 2

(C) Soccer A Chinese game called cuju meaning "kick a ball" existed during the 2nd and 3rd centuries BC and has been recognized as the first version of soccer with similar rules to the modern game. Players could use any part of their body except their hands. Later soccer-like games were played in Japan Australia and several countries in Europe. Now 211 countries around the world are registered as members of FIFA. Rowing has existed since humans first traveled on water by boat. 1


① A-C-B ② B-A-C ③ B-C-A ④ C-A-B ⑤ C-B-A

 


12. 영시한 3-1

 

When you eat fast food, such as pizza, hamburger, or fried chicken, what do you drink with it?


(A) People all over the world love sodas. Every year, Americans consume 170 liters of soda per person, and the British 100 liters. Of all age groups, teenagers drink the most soda. 1

(B) Most people, however, understand that sodas are not good for their health due to the ingredients. What is in a soda? Which ingredients are really not good for you? 2

(C) Most people have a soda. When you go to the movies or when you just want something refreshing to satisfy your thirst, a soda is often the first choice. Soda, also called a soft drink, is a sweet drink with carbonation. 0


① A-C-B ② B-A-C ③ B-C-A ④ C-A-B ⑤ C-B-A

 


13. 영시한 3-2

 

Everyone knows that soft drinks are loaded with large amounts of sugar.


(A) Artificial sweeteners provide a sweet, sugary taste with fewer calories. This may seem like a good thing, but it really isn't. According to a recent study, artificial sweeteners make you want more sweet foods. This means that you may easily gain weight if you keep drinking diet soda. In addition, artificial sweeteners are known to generate headaches, emotional disorders, and sleeping problems. 2

(B) Having extra sugar means adding unnecessary calories. Over time, this can lead to obesity and other health problems, such as heart disease and diabetes. Even diet sodas are not safe. Diet sodas replace the sugar with artificial sweeteners. 1

(C) A typical 250ml can of soda contains 30 grams of sugar. WHO, the World Health Organization, recommends that people consume less than 25 grams of sugar a day. Therefore, if you drink one small can of soda, your daily sugar intake already exceeds the recommended amount. What is worse, people usually don't stop with one can. 0


① A-C-B ② B-A-C ③ B-C-A ④ C-A-B ⑤ C-B-A

 


14. 영시한 3-3

 

Most soda contains several types of acids.


(A) Acid is a chemical substance with a sour taste. When added to water, it produces a sharp flavor. 0

(B) Also, the acid in sodas interacts with stomach acid, slowing digestion and blocking nutrient absorption. Furthermore, it can damage tooth enamel, the hard substance that protects your teeth. If you drink sodas regularly, your teeth decay more easily. 2

(C) Acid also delays the growth of bacteria, which extends the expiration date of a product. However, acid interferes with the body's ability to absorb calcium, and as a result, bone softening occurs. 1


① A-C-B ② B-A-C ③ B-C-A ④ C-A-B ⑤ C-B-A

 


15. 영시한 3-4

 

When you hear the word "caffeine," you most likely think of coffee.


(A) For example, the artificial color Yellow No_6, used in some pineapple juices, adds nothing to the taste. It is just there to make the drink look pretty. Also, the artificial flavor that gives the drink its pineapple taste is not real pineapple juice at all. Studies have shown that excessive consumption of these kinds of chemicals can create problems such as kidney disease and ADHD. 2

(B) But some sodas, especially colas and carbonated energy drinks, also contain caffeine. Caffeine makes you feel more awake, but it may bring about an irregular heartbeat. A single can of cola may not affect you much, but if you consume colas regularly, they can make you feel more nervous and keep you from sleeping well at night. 0

(C) Artificial Colors and Artificial Flavors Soft drink companies attract consumers by adding bright colors and sweet flavors to their products. Most of these colors and flavors, however, are not natural. They are artificial chemicals. 1


① A-C-B ② B-A-C ③ B-C-A ④ C-A-B ⑤ C-B-A

 


16. 영시한 3-5

 

You can see how the sugar, the acid, the caffeine, and the artificial chemicals in sodas are really not good for you.


(A) Of course, the best choice would be water. Water has no calories, no acid, no caffeine, and no chemicals of any kind. And if you want something to add a boost of flavor, put a slice of lemon in the water. 1

(B) If you can't break your soda drinking habits in a short period of time, don't be too hard on yourself. Once you start cutting back, you are heading in the right direction. 2

(C) Perhaps you might now understand that you should stop drinking them. However, if quitting sodas altogether sounds too difficult, cut down gradually. For example, replace one regular soft drink or one diet soda per day with another more healthy drink. 0


① A-C-B ② B-A-C ③ B-C-A ④ C-A-B ⑤ C-B-A

 


17. 영시한 4-1

 

Imagine you are standing in a rainforest.


(A) Insects, birds, and monkeys are responsible for much of this noise. And sometimes there is another sound, one that does not belong in the forest at all. It is the buzz of a chainsaw. Every year some 13 million hectares of rainforest, an area about the size of England, disappears. 2

(B) Do you think it is a quiet, peaceful place? If so, you are wrong. The rainforest is actually a very noisy place. 1

(C) You are surrounded by tall trees, many of which are more than 40 meters tall. You are a hundred kilometers away from the nearest city. What do you hear? 0


① A-C-B ② B-A-C ③ B-C-A ④ C-A-B ⑤ C-B-A

 


18. 영시한 4-2

 

This loss destroys the habitats for millions of species and has a major effect on the jungle's biodiversity.


(A) The surprised loggers fled, but White was shocked. Despite the fact that they were still fairly close to the ranger station, it had been impossible to hear anything from back there. It is because the forest was so full of other sounds.2

(B) Also, it increases the amount of CO² in the air. Destruction of the rainforest is caused by logging, farming, mining, and other human activities. Among these, logging is the main reason for nature's loss. Some 70 to 80 percent of the logging in the rainforests is thought to be illegal. 0

(C) To address this problem, a young American engineer has invented a simple device that detects illegal logging the moment it occurs. It all started in 2011, when Topher White visited Indonesia as a volunteer. One day, he and some of the other volunteers set out from the ranger station on a walk into a protected rainforest. After walking only five minutes, his group came upon people who were cutting down trees illegally. 1


① A-C-B ② B-A-C ③ B-C-A ④ C-A-B ⑤ C-B-A

 


19. 영시한 4-3

 

White started thinking about ways to help.


(A) This device would be placed high up in a tree. When it picked up the buzz of a saw, it would send a message to a ranger's cell phone. White knew that he had to protect the cell phone so that it could survive in the hot and wet rainforest environment. His solution was to put the phone in a plastic box. 1

(B) Since there was no electricity where the phone needed to be placed, the device had to be able to power itself. White attached solar panels to the cell phone. He was sure that the panels would work, even under the shade of the thick tree leaves. 2

(C) He knew that even in the jungle, far from the city, there was good cell phone service. He thought that perhaps cell phone technology could solve the problem. After he returned home to the U.S., in his father's garage he developed a small listening device using an old cell phone. He attached a sensitive microphone to the cell phone so that it could detect chainsaw noise from up to three kilometers away. 0


① A-C-B ② B-A-C ③ B-C-A ④ C-A-B ⑤ C-B-A

 


20. 영시한 4-4

 

White returned to Indonesia to test the device.


(A) When they approached the logging spot, the illegal loggers ran away. White published his story on the Internet and word quickly spread. 1

(B) Surprisingly, on only the second day after he installed the device, it picked up chainsaw noises. An alert message was immediately sent to White and the forest rangers. 0

(C) People living in other countries contacted White and asked if they could use the device. Others, from around the world, started sending him their old cell phones so he could build more devices. These devices, called Rainforest Connection (RFCx), are now being used in the rainforests in Africa and South America. 2


① A-C-B ② B-A-C ③ B-C-A ④ C-A-B ⑤ C-B-A

 


21. 영시한 4-5

 

One RFCx can protect 300 hectares of forest.


(A) Preventing this amount of CO² from being released has the same effect as taking 3,000 cars off the road for a year. 1

(B) If a forest of this size is cut, 15,000 tons of CO² are released into the air. 0

(C) These devices are saving rainforests and providing new life for thousands of discarded cell phones. Thanks to Topher White and his RFCx devices, the earth is now a better place to live. 2


① A-C-B ② B-A-C ③ B-C-A ④ C-A-B ⑤ C-B-A

 


22. 영시한 5-1

 

How to Handle Haters.


(A) If you have ever been confused about what to do, you are not alone. Luckily for us, Benjamin Franklin, the American politician and inventor, came up with an answer more than 200 years ago. 2

(B) Have you ever known someone you just did not get along with? Maybe it was someone you had difficulty working with or it could have been someone you were always arguing with. 0

(C) This kind of situation can leave you wondering what you should do. Should you ignore them, be extra nice to them, or stand up to them? 1


① A-C-B ② B-A-C ③ B-C-A ④ C-A-B ⑤ C-B-A

 


23. 영시한 5-2

 

Benjamin Franklin was a very smart man, not just in politics and science, but also in his ability to deal with people.


(A) Franklin decided to try to change his enemy's opinion of him. To do this, he sent a letter to the man asking for a favor. 1

(B) He asked the man to lend him a very rare and valuable book, one that he knew the man had in his personal library. The man, Franklin's enemy, immediately sent him the book. Franklin read the book and returned it with a note of thanks. 2

(C) Once, when Franklin was serving as a representative for the people of the state of Pennsylvania, a powerful opponent made a long speech criticizing him. This speech was so powerful that it threatened Franklin's political career. 0


① A-C-B ② B-A-C ③ B-C-A ④ C-A-B ⑤ C-B-A

 


24. 영시한 5-3

 

The next time Franklin met his enemy, the man was much nicer to him.


(A) "By asking a small favor of his enemy, Franklin gave the man a chance to do him a kindness. This turned his enemy into a friend. Today this phenomenon is known as the Benjamin Franklin effect. 1

(B) It is commonly thought that if you want people who do not like you to like you, you should do favors for them. However, if you want to try to create a connection with someone who does not like you, asking that person to do you a favor is more effective. To test this phenomenon, a psychologist performed an experiment and came to the same conclusion. 2

(C) They eventually became friends and remained so for the rest of their lives. In his autobiography, Franklin explained what happened. Franklin wrote, "He that has once done you a kindness will be more ready to do you another. 0


① A-C-B ② B-A-C ③ B-C-A ④ C-A-B ⑤ C-B-A

 


25. 영시한 5-4

 

About 80 students were invited to participate in an experiment and were told they would be given some money after a series of tests.


(A) Group 1 was asked by the researcher to return some of the prize money. He explained that he had used his own money for the tests and the study was in danger of running out of funds. Group 2 was asked by a secretary to return some of the money to the institution because the budget was running low. 1

(B) A researcher was hired and made the students dislike him by being rude to them. The students took the tests and were all awarded the promised money. They were then divided into three groups. 0

(C) Group 3 was not asked to return any money. Afterward, all three groups were asked to rate how much they liked the researcher. The people in Group 1, who had done the researcher a personal favor by returning some of the money, rated the researcher the highest. Despite the fact that they had had a bad impression of him during the experiment, they had more positive feelings for him after returning "his money." 2


① A-C-B ② B-A-C ③ B-C-A ④ C-A-B ⑤ C-B-A

 


26. 영시한 5-5

 

How can it be that we feel more positively toward people for whom we do favors?


(A) He justified doing the favor by telling himself that Franklin was not a bad person after all and convinced himself that he actually liked Franklin. Is there someone with whom you would like to have a better relationship? Try asking that person for a small favor, such as lending you a pencil or watching your bag while you go to the restroom. Then see if their feelings toward you change. The chances are they will. 2

(B) It is because of what psychologists refer to as "cognitive dissonance. "We want our actions and our thoughts to be in harmony. When we do something that we do not like, the balance is broken, and we feel unhappy. If there is a conflict between our thoughts and our actions (cognitive dissonance), we will change one or the other to relieve the conflict. 0

(C) After lending Franklin the book, Franklin's enemy experienced cognitive dissonance. He felt troubled because he was doing something nice for someone he disliked. Feeling anxious and unhappy, he had to find a way to resolve his feelings. The easiest way for him to do that was to change how he thought about Franklin. 1


① A-C-B ② B-A-C ③ B-C-A ④ C-A-B ⑤ C-B-A

 


27. 영시한 5-6

 

Every culture has proverbs about human relationships.


(A) Show me your friends and I'l tell you who you are - Russia. As distance tests a horse' strength, so does time reveal a person' character - China. Laughter is a language everyone understands - Chad. 1

(B) Read the following proverbs different countries and cultures and think about the meaning of each one. He who gets close to a good tree will have good shade - Spain. Kind words take the snake out of its hole - Turkey. 0

(C) A friend will cause you to weep, an enemy to laugh - Afghanistan. If you want to go fast, go alone. If you want to go far, go together - Africa. Lose your temper and you lose a friend; lie and you lose yourself - Native American. 2


① A-C-B ② B-A-C ③ B-C-A ④ C-A-B ⑤ C-B-A

 


28. 영시한 6-1

 

Dabbawalas: Mumbai's Lunch Box Delivery Men In India, many traditions are today being challenged as a result of globalization.


(A) Dabbawalas are delivery men who carry hot lunch boxes for Mumbai office workers from their homes to their offices. The word dabbawala comes from the Hindi dabba meaning "lunch box," and wala meaning "the person who carries it." 2

(B) In Mumbai, many people who work in offices far from home rely on an express food delivery service run by dabbawalas. 1

(C) The practice of eating a home-cooked meal for lunch, however, continues. 0


① A-C-B ② B-A-C ③ B-C-A ④ C-A-B ⑤ C-B-A

 


29. 영시한 6-2

 

Most dabbas, or lunches, reach their destination after passing through several pairs of hands.


(A) Another person loads each dabba onto the right train, sending them off in different directions. A third person rides on each of the trains with the dabbas. 1

(B) Finally, a fourth person picks up the boxes at the receiving station and distributes them, again by bicycle, to each customerʼ office, all by 12:30 p.m. The empty dabbas are picked up by 5:00 p.m. and are returned to their original addresses by the same team, following the same procedure in reverse. 2

(C) A typical scenario would have one person on a bicycle pick up a dabba by 9:00 a.m. from about thirty different addresses. After collecting all of the dabbas assigned to him, he takes them to the nearest train station. 0


① A-C-B ② B-A-C ③ B-C-A ④ C-A-B ⑤ C-B-A

 


30. 영시한 6-3

 

The dabbawala service began around 1890, when a banker hired a young man to deliver a lunch box from his home to his Mumbai office.


(A) Those who use the dabbawala service are mostly middle-class office workers who live in one of Mumbai's suburbs. They have to leave for the office early in the morning, riding on a packed train. It would be difficult for them to carry their own dabba. Also, they often have dietary restrictions, depending upon their religion. 1

(B) Hindus do not eat beef, Muslims do not eat pork, and Jains do not eat onions and potatoes. As a result, it is not easy for workers to find the right food in restaurants near their office. To meet their dietary needs, Indian workers prefer their home-cooked meals, made especially for them. 2

(C) Other people liked the idea and copied it. When demand for the service expanded, a businessman started the lunch-delivery service in its present format. Now, more than 120 years later, the dabba is a unique part of the Mumbai culture. Why do Mumbai workers not take their lunch boxes with them in the morning, when they leave for the office? 0


① A-C-B ② B-A-C ③ B-C-A ④ C-A-B ⑤ C-B-A

 


31. 영시한 6-4

 

Today approximately 5,000 dabbawalas cover 70 square kilometers in and around Mumbai.


(A) It is an amazing record, considering that most of the dabbawalas are illiterate and that not a single piece of paper is used in the delivery process. The secret to this error-free system is in a coding system. Each dabba carries a code, painted with different colors, numbers, and symbols. These codes tell the dabbawala where the food comes from and which railway stations it must pass through on its way to a specific office in a specific building, in Mumbai. 2

(B) They conduct about 400,000 transactions daily. 200,000 lunch boxes are delivered to offices every morning, six days a week, and 200,000 are returned home every afternoon. They even deliver in the pouring rain and during political strife. 0

(C) Surprisingly, hardly any cases of late or mistaken delivery are ever reported. Their motto is "error is horror. "The dabbawalas are proud of their 99.99 percent accuracy rate, which means just one mistake in every six million deliveries. 1


① A-C-B ② B-A-C ③ B-C-A ④ C-A-B ⑤ C-B-A

 


32. 영시한 6-5

 

The dabbawala organization is an outstanding example of an efficient distribution system.


(A) Today, most people cannot imagine an efficient delivery system operating without the benefit of technology. Dabbawalas, however, demonstrate that with no form of technology, some of the old ways may still be the best ways.2

(B) It is simple, relies on teamwork, has a low operating cost, and delivers almost 100 percent customer satisfaction. 0

(C) It is not surprising, then, that business schools and large corporations around the world learn from the dabbawala system. 1


① A-C-B ② B-A-C ③ B-C-A ④ C-A-B ⑤ C-B-A

 


33. 영시한 6-6

 

SWITZERLAND Watchmaking School Switzerland is world-renowned for its high-quality watches.


(A) Its first watchmaking school opened in the mid-1800s. Now Switzerland has several watchmaking schools whose students take the traditional watchmaking course. The class sizes are kept small to ensure that each student receives extensive individualized lessons from master watchmakers. ITALY Central Institute of Restoration Italy is a country filled with valuable cultural assets. 0

(B) They are recognized for their abundance of color, different patterns, and weaving techniques. With the aim to preserve and promote this living art, the Royal Textile Academy of Bhutan was created in 2005. Its mission is to educate, promote and preserve Bhutanese textiles. 2

(C) Therefore, the restoration and protection of the nation' cultural heritage is guaranteed by the Italian Constitution. The Central Institute of Restoration was founded in l939 to conserve and restore artworks and archaeological findings using scientific technology. Each year the four-year course accepts about 20 students who are selected through exams that are held by the Ministry of Cultural Heritage. BHUTAN Royal Textile Academy of Bhutan Bhutan is famous for its hand-woven textiles. 1


① A-C-B ② B-A-C ③ B-C-A ④ C-A-B ⑤ C-B-A

 


34. 영시한 7-1

 

Gaudi Architect Inspired by Nature Most industrialized countries have produced a number of good architects.


(A) As a child, he was fascinated by the natural wonders of the surrounding countryside. Gaudi took an interest in architecture at a young age and studied architecture in Barcelona, the city that would become home to his greatest works. Gaudi's designs were deeply influenced by forms in nature. He understood that the natural world is full of curved lines, rather than straight lines. 1

(B) As a result, most of his constructions use curved stones and animal- or plant-shaped designs. Gaudi's work is also known for its use of bold colors. He decorated many of his buildings with colorful tiles. This combination of natural designs and bright colors creates a breathtaking visual experience. Here are some of Gaudi's greatest works, all of which are found in the city of Barcelona. 2

(C) Few countries, however, can claim to have produced a great architect. One country that can make that claim without embarrassment is Spain. There, at the summit of the nation's architectural genius, stands Antoni Gaudi (1852-1926). Gaudi was born in Catalonia, Spain, in 1852. 0


① A-C-B ② B-A-C ③ B-C-A ④ C-A-B ⑤ C-B-A

 


35. 영시한 7-2

 

Parc Güell is a public park and is one of Gaudi's most decorative works.


(A) Known as "El Drac," which means "the dragon" in the language of Catalonia, this colorful fountain is a symbol of Parc Güell. On a hill within the park are curved terraces and multicolored tile seats where visitors can enjoy wonderful views of Barcelona and the sea beyond. 2

(B) At the entrance to the park are two buildings, both with curved roofs. These buildings seem to be taken directly from "Hansel and Gretel. "The park is also home to a famous dragon fountain, covered with beautiful colored tiles. 1

(C) Gaudi completed this park in 1914. It was built for Eusebi Güell, a rich businessman who admired Gaudi's style. Parc Güell contains amazing stone structures, gorgeous tiles, and beautiful buildings. 0


① A-C-B ② B-A-C ③ B-C-A ④ C-A-B ⑤ C-B-A

 


36. 영시한 7-3

 

Casa Batllo is a house renovated by Gaudi between 1904 and 1906.


(A) Everything is curved, including the walls, the ceilings, and the wooden doors. Because of the curves, the Batllo family could not fit their traditional, straight furniture against the curved surfaces. Thus, Gaudi had to design special furniture for the family. 2

(B) Built in 1877, the original building was very different from what it is today. When it was purchased by Joseph Batllo, he asked Gaudi to add his impressive touch to the design. From the outside, Casa Batllo looks as if it has been made from skulls and bones. The "skulls" are in fact balconies and the "bones" are supporting stone columns between the windows. 0

(C) These details have given the house the nickname, the House of Bones. Gaudi decorated the building with colors and shapes found in the sea. Indeed, the design of the green and blue tiles on the wall remind people of the sea, while the curved window frames were inspired by ocean waves. The interior of the house is even more impressive. 1


① A-C-B ② B-A-C ③ B-C-A ④ C-A-B ⑤ C-B-A

 


37. 영시한 7-4

 

Sagrada Familia is the most widely-known symbol of Barcelona and one of the world's largest churches.


(A) When completed, the highest tower will reach a height of 170 meters. The walls are decorated with sculptures that describe events in the Bible. When you step inside the church, the large columns supporting the ceiling immediately catch your eye. The columns branch out at the top so that each column looks like a huge tree. 1

(B) Begun in 1882, it has been under construction for more than one hundred years. It is expected that the church will be completed in 2026, on the 100th anniversary of Gaudi's death. Despite its incomplete state, Sagrada Familia's incredible design draws an estimated 2.5 million tourists every year. On the outside, the church will have 18 high towers. 0

(C) Between the columns are skylights to let natural light in. The colorful stained glass windows filter the sunlight and project red, blue, and green light all over the interior space. The tree-like columns and the different light at different times of the day transform the inside of the church into a stone forest. 2


① A-C-B ② B-A-C ③ B-C-A ④ C-A-B ⑤ C-B-A

 


38. 영시한 7-5

 

These are only three of Gaudi's many works in or near Barcelona.


(A) All are recognized as important works of architecture, and seven have been named UNESCO World Heritage Sites. 0

(B) Even today his imaginative, creative, unique architecture characterizes the city of Barcelona. No visitor to Barcelona would want to leave without seeing these works of Antoni Gaudi. 2

(C) These extraordinary works show how important Gaudi was to the development of modern architecture in the late 19th and early 20th centuries. 1


① A-C-B ② B-A-C ③ B-C-A ④ C-A-B ⑤ C-B-A

 


39. 영시한 8-1

 

Robots May Rescue You from Future Disasters In 2011, an earthquake and its accompanying tsunami destroyed Japan's Fukushima nuclear power plant.


(A) The robots the Japanese were using, however, were not up to the task. Eventually, humans had to do most of the extremely dangerous work. 2

(B) Since it was impossible for humans to work in this environment, the Japanese government considered sending in robots to handle the situation. 1

(C) The resulting nuclear disaster released large amounts of radioactive material into the surrounding area. 0


① A-C-B ② B-A-C ③ B-C-A ④ C-A-B ⑤ C-B-A

 


40. 영시한 8-2

 

Since then, there has been renewed emphasis on developing robots that can serve in dangerous situations.


(A) During the competition, the robots had to solve a series of problems they might come upon in a disaster situation. The tasks were: driving a vehicle, getting out of the vehicle, opening a door, locating and closing a leaking valve, using a drill to cut through a wall, pulling a plug out of a wall socket and then plugging it in, navigating rough terrain, and climbing stairs. HUBO completed all eight tasks in the shortest time of all the competitors – 44 minutes and 28 seconds. 1

(B) The key to HUBO's success was its ability to move from a standing position to a kneeling position. HUBO had wheels attached to its knees and feet. When kneeling, HUBO was able to use these wheels to move around quickly and decisively. 2

(C) In response to Japan's nuclear disaster, the 2015 DARPA Robotics Challenge was created to speed up the development of robots that could work in disaster-stricken areas. The competition attracted 25 teams from around the world. The winner was a Korean team from KAIST who developed a robot called HUBO. 0


① A-C-B ② B-A-C ③ B-C-A ④ C-A-B ⑤ C-B-A

 


41. 영시한 8-3

 

From the beginning, HUBO was better than the other robots at performing the tasks.


(A) The task that took the longest time for HUBO was the sixth one, pulling a plug out of a wall socket and putting it back into another. It takes a human less than 10 seconds to perform the task, but it took HUBO 13 minutes and 30 seconds. 2

(B) It was able to drive a vehicle fast and when it encountered a barrier, it was able to turn the vehicle smoothly to avoid it. Next, it was able to get out of the car in less than four minutes and, once out of the vehicle, got on its knees and sped away. As the series of eight tasks became progressively more difficult, HUBO's performance on the tasks reflected the growing difficulty. 0

(C) On the fifth task, for which it had to use a drill to cut through a wall, HUBO failed on its first attempt. Generally speaking, it was difficult for a robot to hold a drill in the right position and simultaneously press an on/off button. On the second trial, however, HUBO successfully completed the task.1


① A-C-B ② B-A-C ③ B-C-A ④ C-A-B ⑤ C-B-A

 


42. 영시한 8-4

 

For the final task, climbing stairs, it was important that the robot be able to see its feet.


(A) Other robots had difficulty doing this because they had to bend their bodies forward to see over their knees to scan the stairs. This awkward move caused them to lose their balance. HUBO solved this problem in a clever way.0

(B) That way, the robot's knees did not block the camera's view of either the feet or the floor. After scanning the stairs, the robot set off to climb to the top, completing the task effortlessly. 2

(C) It climbed the stairs backward. But how did it see the steps if it was moving backwards? By rotating its upper body 180 degrees. 1


① A-C-B ② B-A-C ③ B-C-A ④ C-A-B ⑤ C-B-A

 


43. 영시한 8-5

 

This amazing robot was not made in a day.


(A) The KAIST team had already built four HUBOs and had been improving them for years. They practiced outdoors, in good weather and bad, and on rough terrain. They burned up motor after motor, but never gave up. 0

(B) They will be called upon to perform complicated tasks that will be too dangerous for humans. Scientists expect that these robots will save lives and reduce the damage caused by future disasters. 2

(C) They approached each failure as a challenge to make a faster, stronger, and better robot. The DARPA Robotics Challenge eventually ended, but it is only the beginning. In the future, there will be other robots like HUBO. 1


① A-C-B ② B-A-C ③ B-C-A ④ C-A-B ⑤ C-B-A

 


44. 영시한 SP1-1

 

How Volunteering Abroad Changed My Life by Susan Shain I was about to board a plane, and I was scared.


(A) The year was 2005. The plane's destination? Tanzania. 1

(B) I'd never traveled anywhere so different before, and I had no idea what to expect. That scared me. But I had never been one to turn away from adventure, so when the rest of my group started boarding, I did, too. Little did I know that the next six weeks would change my life forever. 2

(C) Not because I was afraid of flying, but because I was afraid of what awaited me at my destination. "I could turn around now," I said to myself. "I could just go back to my part-time job and have a nice quiet summer." 0


① A-C-B ② B-A-C ③ B-C-A ④ C-A-B ⑤ C-B-A

 


45. 영시한 SP1-2

 

What Volunteering in Tanzania Was Like I traveled to Tanzania as part of a university program the summer after my freshman year.


(A) I learned to cook ugali, milk a cow, and wear a kanga –. inspiring to witness their close family ties and endless positivity. Volunteering abroad taught me so much about the world and about myself. I grew more in those few weeks than I did my entire first year of college. 2

(B) By volunteering, I became part of the lives of local families — sharing meals, playing with their kids, and visiting their homes. These experiences were eye-opening, educational, and inspiring:. – eye-opening to see how few possessions they had, yet how much love and personal strength they had –. educational to experience their culture:. 1

(C) We spent several weeks volunteering at a community center and an orphanage. Though we did many other exciting things like going on safari and climbing Mt Kilimanjaro, it was our volunteering time that stuck with me. Volunteering allowed me to truly connect with the people in Tanzania. If I had not volunteered, I would never have got an inside look at their way of life. 0


① A-C-B ② B-A-C ③ B-C-A ④ C-A-B ⑤ C-B-A

 


46. 영시한 SP1-3

 

How Volunteering Abroad Changed Me These are what volunteering gave me:.


(A) Volunteering gave me deep gratitude for the comforts and opportunities I used to take for granted. Appreciation for new cultures The local people are welcoming, strong, and overwhelmingly positive. Their language, clothing, and culture are beautiful. 1

(B) Growing up in rural New York State, I had never before experienced people so different from me. During my time in Tanzania, I learned to appreciate those differences. Volunteering abroad taught me the beauty of diversity. 2

(C) Gratitude for what I have When I returned home, I literally hugged my toilet, and I cried during my first hot shower. I didn't have these things in Tanzania. To this day, I sometimes say a silent "Thank you" as I turn on the tap to get a drink of water. 0


① A-C-B ② B-A-C ③ B-C-A ④ C-A-B ⑤ C-B-A

 


47. 영시한 SP1-4

 

Hunger for exploration Though I was happy to return home, I was confident it would not be my last adventure.


(A) After experiencing this in Tanzania, I decided to help people wherever and whenever I could for them and for myself. It is something I try to do every day. 2

(B) Commitment to helping people I will be honest: Volunteering feels good. Giving back to others and seeing the smiles on their faces is a feeling unmatched by anything else. 1

(C) I knew I wanted to continue exploring, meeting people from different backgrounds, trying new foods, and discovering unique landscapes. It made me a travel addict. 0


① A-C-B ② B-A-C ③ B-C-A ④ C-A-B ⑤ C-B-A

 


48. 영시한 SP1-5

 

It has now been 10 years since I traveled to Tanzania, and these principles still guide me.


(A) I will never know the answers to these questions. But I do know I will be forever grateful I got on that plane. Volunteering abroad led me to the life I have today — which I wouldn't change for anything. 2

(B) Sometimes I wonder what my life would be like if I had not got on that plane to East Africa. Would I have ever tasted ugali, danced with the Masai, or bathed an elephant? Would I be the person I am today? 1

(C) I graduated from college in 2008, and ever since, I have been working and traveling around the world. I have volunteered everywhere from Korea to France to Nicaragua to North Carolina. I practice gratitude daily and am always trying to meet new people and learn about different ways of life. 0


① A-C-B ② B-A-C ③ B-C-A ④ C-A-B ⑤ C-B-A

 


49. 영시한 SP1-6

 

The Secret to Happiness.


(A) Scientific research provides convincing data to support the evidence that giving is a powerful path to lasting happiness. Researchers have found that the areas in our brains that feel pleasure become activated when we give —meaning that besides doing good, donating our money or time actually makes us feel good. This feeling is similar to a "runner's high," a term created for that feel-good sensation that rushes through your body after a run. They use the term "helper's high" to describe what happens to your body and your brain when you are kind to another person or give to them in some way. Helping others may just be the secret to living a life that is not only happier but also healthier, wealthier, more productive, and meaningful. 2

(B) For centuries, the greatest thinkers have suggested the same thing:. Happiness is found in helping others. And so we learn early: It is better to give than to receive. But is there a deeper truth behind this statement? The answer is yes. 1

(C) There is a Chinese saying that goes:. "If you want happiness for an hour, take a nap. If you want happiness for a day, go fishing. If you want happiness for a year, inherit a fortune. If you want happiness for the rest of your life, help somebody." 0


① A-C-B ② B-A-C ③ B-C-A ④ C-A-B ⑤ C-B-A

 


50. 영시한 SP1-7

 

< CULTURE > Volunteers around the World Mealshare Mealshare is a non-profit organization founded by two young men from Canada in 2013.


(A) Volunteers gather for the clean-up and are provided with cleaning tools such as plastic gloves and bags. Volunteers record what they pick up and the data is used to find out the sources of trash, and to study how to reduce trash. 2

(B) With "uy One, Give One"motto, Mealshare partners with restaurants, and places its symbol next to a few menu items. When a customer orders a Mealshare item, the restaurant donates $1 to Mealshare. With the collected money, meals for people in need are served. 0

(C) Teens for Jeans Started by a non-profit organization, it is a campaign in which teenagers in the U.S. collect pairs of jeans and give them to local homeless youth. Many schools and teenagers have donated over five million pairs of jeans since the campaign started in 2008. COASTSWEEP It is an international clean-up campaign organized in the U.S. 1


① A-C-B ② B-A-C ③ B-C-A ④ C-A-B ⑤ C-B-A

 


51. 영시한 SP2-1

 

To Kill a Mockingbird In the early part of the novel To Kill a Mockingbird, the heroine and her brother are told by their father never to kill a mockingbird.


(A) Miss Maudie answers:. "Your father's right. All mockingbirds do is make music for us to enjoy. 1

(B) "I hope you shoot only tin cans in the garden, but I know you'll want to hunt birds," he says to his children, who are learning how to shoot a gun. "Shoot as many bluejays as you want, but remember that it's a sin to kill a mockingbird. "Puzzled, the heroine asks Miss Maudie, her kind and clever neighbor, why she should not kill a mockingbird. 0

(C) They don't eat up people's gardens or steal their food. They only sing their hearts out for us. That's why it's a sin to kill a mockingbird." 2


① A-C-B ② B-A-C ③ B-C-A ④ C-A-B ⑤ C-B-A

 


52. 영시한 SP2-2

 

In To Kill a Mockingbird, there are characters like mockingbirds, who are innocent and harmless.


(A) Lee wrote To Kill a Mockingbird based on her own childhood experiences. Just like the heroine of the novel, she grew up in Alabama and saw many African-American people treated cruelly and unfairly. Her criticism against racism and her wish for a more fair and understanding world are well expressed in this book. 1

(B) However, they are often hurt and sometimes even killed by others. Through their stories, the author Harper Lee shows how an unfair society harms innocent people and prevents others from helping them. At the same time, she also shows how important it is to remain just, fair, and understanding in such a society. 0

(C) For this reason, To Kill a Mockingbird has long been a book loved not only by Americans but also by many people around the world. It has been translated in 40 different languages, and more than 40 million copies have been sold. 2


① A-C-B ② B-A-C ③ B-C-A ④ C-A-B ⑤ C-B-A

 


53. 영시한 SP2-3

 

To Kill a Mockingbird is the story of a young girl growing up in a southern town during the 1930s, when African-American people were facing severe discrimination.


(A) He has not been seen by anyone for many years, and the adults of Maycomb do not want to talk about him. Curious about what he looks like and why he remains hidden, the children share rumors about him and try to get him to come out of his house. While they are acting out the story of his life one day, Atticus stops them, telling them to see things from other people's points of view. 2

(B) Scout, Jem, and their friend Dill are fascinated by the haunted house in their neighborhood and the man who lives there, Arthur "Boo"Radley. Boo is known as a "Monster" for stabbing his father in his leg when he was a boy, but no one actually witnessed the accident. 1

(C) The six-year-old heroine, Scout Finch, lives with her brother Jem and their father Atticus in Maycomb, a little town in Alabama where things do not change quickly. Here, the same families have been doing the same things for generation after generation, and Atticus is a lawyer just like his father was. 0


① A-C-B ② B-A-C ③ B-C-A ④ C-A-B ⑤ C-B-A

 


54. 영시한 SP2-4

 

Despite the children's playful actions, Boo shows signs of affection for them.


(A) Meanwhile, Atticus agrees to defend an African-American man named Tom Robinson, who has been accused of attacking a young white woman, Mayella Ewell. Most of the white people in Maycomb do not understand why Atticus would defend this African-American man. 1

(B) He leaves small gifts in a tree outside the Radley home, fixes and returns Jem's lost pants, and puts a blanket on Scout's shoulders, while she is out in the cold weather looking at a neighbor's house that is on fire. However, he never shows himself, so Scout grows more and more curious about him. 0

(C) They are angered by Atticus's effort to give Tom the best defense possible. Jem and Scout are also bullied by other children because of this. 2


① A-C-B ② B-A-C ③ B-C-A ④ C-A-B ⑤ C-B-A

 


55. 영시한 SP2-5

 

One day during the trial, Atticus provides clear evidence that Tom never attacked Mayella and that she and her father, Bob Ewell, are lying.


(A) Nevertheless, the all-white jury decides that Tom is guilty. Tom later tries to escape from prison and is shot to death, and Scout finds that most of her neighbors don't care about this. Both Atticus and his children are shocked by the injustice of what has happened to Tom. 0

(B) Scout realizes that this man is Boo Radley. She and the wounded Jem are brought back to their house by Boo. 2

(C) Despite the fact that Tom has been convicted and is now dead, Bob Ewell feels that he has been insulted and tries to take his revenge. He threatens Tom's wife, tries to break into the judge's house, and finally attacks Jem and Scout as they walk home from a Halloween party. However, a strange man comes to rescue them. 1


① A-C-B ② B-A-C ③ B-C-A ④ C-A-B ⑤ C-B-A

 


56. 영시한 SP2-6

 

When the sheriff arrives, he discovers that Ewell was stabbed to death during the fight.


(A) However, he does not charge anyone with murder. He knows that any further investigation will harm Boo, and he doesn't want to do that because he knows that Boo has risked himself to save the children. 0

(B) Standing on the porch of the Radley house, she sees the village as Boo has always seen it. She imagines seeing herself and her brother from Boo's perspective and finally understands what her father meant when he told her to put herself in other people's places. 2

(C) After the sheriff leaves and Jem is safely put to bed, Scout and Boo walk arm-in-arm back to Boo's house. When Boo disappears into the house, never to be seen again, Scout thinks about all the things he has done for her and Jem and regrets that they have never given him anything in return. 1


① A-C-B ② B-A-C ③ B-C-A ④ C-A-B ⑤ C-B-A

 


57. 영시한 SP2-7

 

"Atticus was right," she thinks.


(A) "One time he said you never really know a man until you stand in his shoes and walk around in them. 0

(B) "The various events that she experienced taught Scout how unjust and ugly the adult world was. However, they also taught her the importance of sympathy, understanding, and a sense of justice. 2

(C) Just standing there on the Radley porch was enough. 1


① A-C-B ② B-A-C ③ B-C-A ④ C-A-B ⑤ C-B-A

 


[ANSWER]
1. ⑤ 2. ④ 3. ③ 4. ① 5. ② 6. ④ 7. ② 8. ⑤ 9. ⑤ 10. ④


11. ① 12. ④ 13. ⑤ 14. ① 15. ③ 16. ④ 17. ⑤ 18. ③ 19. ④ 20. ②


21. ② 22. ③ 23. ④ 24. ④ 25. ② 26. ③ 27. ② 28. ⑤ 29. ④ 30. ④


31. ③ 32. ③ 33. ① 34. ④ 35. ⑤ 36. ③ 37. ② 38. ① 39. ⑤ 40. ④


41. ③ 42. ① 43. ① 44. ④ 45. ⑤ 46. ④ 47. ⑤ 48. ⑤ 49. ⑤ 50. ③


51. ② 52. ② 53. ⑤ 54. ② 55. ① 56. ① 57. ① 

728x90
반응형

728x90
반응형

esh1 | Since 2005 임희재 | 블루티쳐학원 | 01033383436 | 200710 22:49:58

 

PARA

 

1. 영1시한 1-1

 

Failure Is Our Friend.


(A) He was one of the world's greatest inventors, and he acquired over 1,000 patents. Like a wizard, he seemed to take ideas from thin air. However, he also faced tremendous difficulties. Edison is said to have failed 9,999 times before creating a perfect light bulb. Unlike the average person, Edison saw these mistakes not as failures but as an inevitable part of the invention process. 1

(B) In response to a question about his errors, he once said, "I have not failed 9,999 times. I've successfully found 9,999 ways that will not work." Of course, he was right. He was able to achieve success after failing repeatedly. 2

(C) No one wants to fail. We all want to succeed in everything we try and to avoid failure. Nonetheless, failing and learning from our bitter experiences is necessary for success. In fact, the most successful people are often people who have experienced many more failures than others. One of the most famous examples of someone who learned from failure was Thomas Edison.0


① A-C-B ② B-A-C ③ B-C-A ④ C-A-B ⑤ C-B-A

 


2. 영1시한 1-2

 

These failures can not only motivate us to find a successful way to accomplish our goal but also help us to grow in wisdom and in spirit.


(A) Later in her life, Morrison wrote masterpieces such as Song of Solomon, Beloved, and Jazz. In 1993, she became the first African-American woman to receive the Nobel Prize in Literature. 2

(B) Toni Morrison began writing when she was in college, but she did not produce anything good enough to publish for many years. Her troubled marriage, divorce, and life as a single mother made it even harder for her to write. 0

(C) At 39, she published her first novel, The Bluest Eye, which received mixed reviews and did not sell well. However, all her hardships added depth to her novels and encouraged her to improve her writing skills. 1


① A-C-B ② B-A-C ③ B-C-A ④ C-A-B ⑤ C-B-A

 


3. 영1시한 1-3

 

Sometimes failure can lead us to success by showing us that we are not good at something.


(A) She experienced a brutal letdown. While staying in Paris, she happened to discover her passion for and talent in fashion design, which led her to move to fashion as a career. After returning to America, she started working as a salesperson in a clothing store. 1

(B) The fashion designer Vera Wang, for example, found a path to success, thanks to her failure. At the age of seven, she began devoting herself to figure skating, hoping to compete in the Olympics. When she was 19, however, she failed to make the U.S. Olympic team and quit figure skating. 0

(C) Developing her career from editor of a fashion magazine to design director for a global brand for about 20 years, she eventually ended up designing clothes on her own. Today, Vera Wang is a world-famous designer whose elegant dresses are sought after by celebrities the world over. 2


① A-C-B ② B-A-C ③ B-C-A ④ C-A-B ⑤ C-B-A

 


4. 영1시한 1-4

 

Still, wouldn't it be even better if we could be successful without ever failing?


(A) As a result, the always successful person or organization could suddenly encounter a disaster. Between the 1960s and the 1980s, NASA, the U.S. agency in charge of researching and exploring space, completed one successful mission after another with no significant failure. 1

(B) We may think so, but in the end, performing without failure doesn't necessarily produce lasting success. All success with no failure often leads a person to arrogance and carelessness. 0

(C) In 1968, it launched Apollo 8, the first manned spacecraft to fly around the moon. In 1969, Apollo 11 landed on the moon, and the NASA astronaut Neil Armstrong became the first person in history to walk on the moon. 2


① A-C-B ② B-A-C ③ B-C-A ④ C-A-B ⑤ C-B-A

 


5. 영1시한 1-5

 

During the following years, NASA successfully sent five other rockets to the moon.


(A) In 1986, NASA planned to send its second space shuttle, Challenger, into orbit. Right before the launch, engineers expressed concerns about mechanical malfunctions and advised that the launching be postponed. 1

(B) Even when the oxygen tank in Apollo 13 exploded on its way to the moon in 1970, its entire crew was rescued, and they were able to return home safely. This continued series of successes made the decision-makers at NASA too self-assured and unable to imagine failure. 0

(C) However, NASA managers did not take their warnings seriously and launched Challenger as planned. Challenger broke apart 73 seconds after it was launched, resulting in the deaths of its seven crew members. 2


① A-C-B ② B-A-C ③ B-C-A ④ C-A-B ⑤ C-B-A

 


6. 영1시한 1-6

 

Edison, Morrison, and Wang all suffered big failures, but they never gave up.


(A) She remarked, "Failure taught me things about myself that I could have learned no other way." By risking and confronting failures and learning from them, we can become wiser and stronger. 2

(B) After all, the only way to avoid failures is to do nothing. JK Rowling said that suffering many failures in her life eventually enabled her to write the Harry Potter series. 1

(C) They learned from their mistakes and went on to experience even greater triumphs. Like many successful people, we should view failure not as the opposite of success, but as an indispensable step on the path to reach success. 0


① A-C-B ② B-A-C ③ B-C-A ④ C-A-B ⑤ C-B-A

 


7. 영1시한 2-1

 

The Evolution of the Camera.


(A) In the movie Argo, a group of American embassy staff members try to escape from a foreign country before the enemy finds them. To prevent the enemy from identifying them, they attempt to eliminate all the photographs inside the embassy along with all embassy documents. While they are hiding and waiting for rescue, the enemy reassembles the torn pieces of the photographs to find out what the escapees look like. 0

(B) There would be too many pictures floating around on the Internet to find and erase. This example points out how differently we handle photographs today from how we did in the past. At the center of this change is the ever-evolving technology of the camera. 2

(C) These staff members must leave the country before their photographs are reconstructed. This story only makes sense because it is set in 1980. If a similar incident happened today, it would be impossible for the embassy employees to destroy their photographs in time. 1


① A-C-B ② B-A-C ③ B-C-A ④ C-A-B ⑤ C-B-A

 


8. 영1시한 2-2

 

From the Film Camera to the Digital Camera.


(A) As a result, most people are taking far more photographs than ever they would have before. In the past, when a family took a trip to the countryside for the weekend, they might have taken 10 to 20 photographs. Today, many families will take 100 to 200 pictures during the same countryside trip. When they return home, they can choose the shots they want to keep and discard the rest. 2

(B) A photographer needed to learn how to decide an adequate composition for the picture, to adjust the focal length, and to set the film and the shutter speed before shooting. It was important to prepare carefully if a person was going to take a good photograph, because it was nearly impossible to change anything once the photograph had been taken. The digital camera has changed all that. Since today's digital cameras store photographs in the form of digital images, no film is needed. In other words, taking pictures with a digital camera is practically free. 1

(C) Before the digital camera was invented, we had the film camera. It captured an image on a piece of film every time we took a picture. Once used, the film could not be erased or reused. To take pictures, photographers needed not only a camera, but also film, which they had to buy every time they wanted to take another photograph. Furthermore, taking a photograph required knowledge, skill, and practice. 0


① A-C-B ② B-A-C ③ B-C-A ④ C-A-B ⑤ C-B-A

 


9. 영1시한 2-3

 

The discarded pictures cost them nothing, except perhaps a little time.


(A) Thus, people took pictures mostly on special occasions, such as weddings, graduations, and trips to exotic places. Now we can take pictures of anything we want, whenever we want. No one thinks that taking a picture of tonight's dinner, a lovely pet, or a book you like, is strange or a waste of money. 2

(B) Moreover, the digital camera has made it astonishingly easy to take photographs by making all the shooting processes automatic. People who know next to nothing about photography can still produce high-quality photographs. 0

(C) The transition from the film camera to the digital camera has changed our idea about what to take pictures of. When photographers used film in the past, each photo cost money. 1


① A-C-B ② B-A-C ③ B-C-A ④ C-A-B ⑤ C-B-A

 


10. 영1시한 2-4

 

The Smartphone and Citizen Journalism.


(A) By integrating the mobile phone and the camera, the smartphone has further transformed the way we use and think about photography. Nowadays, whenever we want to take a picture, our camera is already in our hand, in our smartphone. 0

(B) Taking a photograph of oneself was difficult and not often done in the past. However, the smartphone made it so convenient and popular that taking a photograph of oneself, or a selfie, is now one of the most common types of photography on the Internet. It can be said that the smartphone lets everyone use photography as a tool for self-expression. 2

(C) The smartphone has also made it easier to share our photographs with others since smartphones come with a wireless Internet connection and social networking apps. These features of the smartphone have resulted in new picture-taking habits. 1


① A-C-B ② B-A-C ③ B-C-A ④ C-A-B ⑤ C-B-A

 


11. 영1시한 2-5

 

The smartphone also enables ordinary people to gather, report, and spread news without relying on the mainstream media, such as newspapers or television.


(A) Internationally, citizen journalism has played a crucial role in letting the world know the realities of major world events, such as the earthquake in Haiti and the Arab Spring. 2

(B) This practice is called citizen journalism. 0

(C) Thanks to citizen journalists, inappropriate or inconsiderate behaviors such as leaving one's pet's droppings on the street or abusing a bus driver are quickly photographed and reported online. 1


① A-C-B ② B-A-C ③ B-C-A ④ C-A-B ⑤ C-B-A

 


12. 영1시한 2-6

 

By any measure, the world today is far different from the world as it was in the film Argo.


(A) Some people worry that these changes have made us preoccupied with taking pictures and flooding the Internet with too many unimportant images.1

(B) There may be some truth in that, but it cannot be denied that technology today has made photography an essential part of our culture and that it has altered the way people see the world. 2

(C) There are infinitely more photographs and amateur photographers now than there were 40 years ago. 0


① A-C-B ② B-A-C ③ B-C-A ④ C-A-B ⑤ C-B-A

 


13. 영1시한 3-1

 

Kim Whanki, Painter of Korean Beauty.


(A) Kim Whanki, one of the most beloved Korean artists, is considered the leading figure in the first generation of Korean Abstract painters. He is more than just that, however. All his life, he devoted himself to discovering the essence of Korean beauty. Moreover, he was a man who never ceased to challenge himself and grow as an artist. 0

(B) When the Korean War broke out in 1950, he went to Busan for safety. His paintings Shanty and Refugee Train portrayed the hard life of the common people during wartime. However, these paintings are not depressing images. Bright colors and simple lines show hope for life among people who did not give in to their difficult circumstances. 2

(C) Kim Whanki was born in 1913, on a small island in South Jeolla Province. During the Japanese occupation, he spent most of his adolescent years in Seoul and then studied painting in Japan. It was during this time that he became familiar with new trends in Western art, including Abstract painting. After Korea achieved its independence from Japan, he became both a promising painter and respected professor at an art college in Seoul. 1


① A-C-B ② B-A-C ③ B-C-A ④ C-A-B ⑤ C-B-A

 


14. 영1시한 3-2

 

Returning to Seoul after the war, Kim Whanki felt a strong desire to capture the essence of the beauty found in traditional Korean art.


(A) He collected old Korean paintings and pottery. He especially loved large white porcelain moon jars. When he was asked why moon jars attracted him so much, he said, "Korean jars have broadened my idea of beauty. 0

(B) At the time, his painting style was semi-abstract;. Viewers could see certain forms in his paintings although the objects were simplified. 2

(C) The jars are like curious textbooks for my paintings." Along with jars, he often included mountains, plum blossoms, and the moon as subjects in his paintings. Paintings like Jars and Plum Blossoms and Jar and Moon represent his love of Korean pottery and other traditional objects. 1


① A-C-B ② B-A-C ③ B-C-A ④ C-A-B ⑤ C-B-A

 


15. 영1시한 3-3

 

By the early 1950s, although Kim Whanki had become successful as an artist, he was not satisfied with being an outstanding painter only within Korea.


(A) During this period, objects were given more simplified shapes, and his art became more abstract. In his famous work Song of Eternity, he included natural objects from the sipjangsaeng, the 10 symbols of eternal life, such as water, stones, mountains, clouds, and pine trees. 1

(B) In 1956, he resigned from the college and went to Paris, where he lived for three years. He concentrated on delivering the beauty of traditional Korean objects on canvas to the people of France. 0

(C) It was praised by art critics for combining Asian concepts and ideals with abstraction. 2


① A-C-B ② B-A-C ③ B-C-A ④ C-A-B ⑤ C-B-A

 


16. 영1시한 3-4

 

Kim Whanki returned to Seoul in 1959 and quickly regained his fame as a prominent artist.


(A) Once an art dealer lied to him and sold his paintings without paying him. On another occasion, a group of paintings were lost during an exhibition. With loneliness and financial difficulty, he tried to find a way to survive in New York as an artist. 2

(B) In 1963 at the age of 50, he became the first Korean artist to be invited to the São Paulo Art Biennale in Brazil. Meeting artists from all over the world and seeing their works, he decided to go directly from São Paulo to New York to challenge himself and extend his artistic boundaries. 0

(C) Life in New York was not easy. In the beginning, he received negative reviews from critics. 1


① A-C-B ② B-A-C ③ B-C-A ④ C-A-B ⑤ C-B-A

 


17. 영1시한 3-5

 

During his 11 years in New York, Kim Whanki's style eventually reached complete abstraction.


(A) Kim Whanki explained, "The dots are the faces of all my friends in Korea. The faces I was longing for became the stars in my mind and the dots in my painting." Starting with that work, he began to use only dots and created his own unique style called "dot painting." 1

(B) In dot paintings, he filled big canvases with countless dots. Most of these paintings were in his characteristic blue tones, which represented the color of the sea near his hometown. 2

(C) He gradually took away figures and filled his canvases with basic elements such as dots and lines. His masterpiece, completed in 1970, Where, in What Form, Shall We Meet Again?, is covered with thousands of blue dots. The title was taken from a famous poem by Kim Kwangsup, which is about looking at the stars and longing for loved ones. 0


① A-C-B ② B-A-C ③ B-C-A ④ C-A-B ⑤ C-B-A

 


18. 영1시한 3-6

 

In the Evening Kim Kwangsup Of all those numerous stars, one in particular looks down upon me.


(A) As the night grows deeper, it fades into brightness and I disappear into darkness. 1

(B) Of all these countless people, my eyes are set upon one particular star. 0

(C) Where, in what form shall the two of us... you, one so warm, and me, one so tender, meet again? 2


① A-C-B ② B-A-C ③ B-C-A ④ C-A-B ⑤ C-B-A

 


19. 영1시한 3-7

 

Kim Whanki passed away in New York in 1974.


(A) Meanwhile, his life story helps us realize how important it is to be true to and passionate about our dreams and ideals. 2

(B) Almost half a century has passed since his death, but his art is still highly respected. 1

(C) Throughout his entire life, his artistic passion never grew old or became weak. 0


① A-C-B ② B-A-C ③ B-C-A ④ C-A-B ⑤ C-B-A

 


20. 영1시한 4-1

 

Last winter, my family and I visited Penang Island, Malaysia.


(A) I had always been interested in Malaysia because it is a multicultural country, where Chinese, Indian, Islamic, and traditional Malay cultures coexist with European influences. In addition, we chose Penang as our destination because it is famous for its street art, something that both my mother and I are interested in. 0

(B) As a center of trade between the East and the West, George Town has been home to a diverse population for a long time and has many beautiful, historically important buildings. For these reasons, it is listed as a UNESCO World Heritage Site. 2

(C) Penang is a tropical island located off the northwest coast of Malaysia. Its capital, George Town, was built by British settlers in the late 18th century and named after King George III. 1


① A-C-B ② B-A-C ③ B-C-A ④ C-A-B ⑤ C-B-A

 


21. 영1시한 4-2

 

In the last week of January, my family and I flew to Penang.


(A) Laksa is a spicy noodle soup that blends Chinese and Malay cuisine. People from different regions and cultures have invented different types of laksa. Penang asam laksa is made with fresh seafood and has a sour, spicy taste. At first I thought it looked like Kalguksu in a Korean Ramyeon sauce, but with a strange smell. 1

(B) As soon as we entered George Town, I was struck by its unique beauty. European-and Asian-style buildings stood side by side, and many buildings looked neither completely Western nor completely Eastern but a mixture of the two. Having left our luggage at the hotel, my family and I walked around the city. First, we stopped at a nearby restaurant and had asam laksa for lunch. 0

(C) When I tasted it, however, I found it was really delicious. The soup was very rich, and its sour, spicy taste was very refreshing. After finishing lunch, we visited some famous, old buildings. Among them, my favorite was Blue Mansion, a large, luxurious house built in the 19th century. Painted in vivid blue and built in a style that combined Chinese and European architecture, it was both beautiful and unique at the same time. 2


① A-C-B ② B-A-C ③ B-C-A ④ C-A-B ⑤ C-B-A

 


22. 영1시한 4-3

 

The next day, we went to see some impressive historical temples.


(A) It was terrific to see how all these different religious temples coexist in the same area. On the third day, we went to Armenian Street. It is called that because many Armenian immigrants moved to this district in the 19th century. However, they did not stay long, and soon Chinese traders replaced them. 1

(B) As a result, many of the buildings on Armenian Street have unusual appearances. They appear to have been designed and begun by Europeans but decorated and finished by Chinese. Although there were many white buildings, some badly in need of paint, there were also many other buildings painted in yellow, pink, orange, and brick-red. The colorfulness and slightly different decorations of the buildings made the whole street look really delightful. 2

(C) First, we stopped by the Kapitan Keling Mosque, the largest Mosque in George Town. Visitors wearing shorts are not allowed to enter, so I had to borrow a long skirt to get in. Then, we went to see Sri Maha Mariamman Temple, which is covered with colorful carvings of countless Hindu gods and goddesses. Lastly, we visited Kek Lok Si, the largest Buddhist temple in Malaysia located in a suburb of George Town. 0


① A-C-B ② B-A-C ③ B-C-A ④ C-A-B ⑤ C-B-A

 


23. 영1시한 4-4

 

We also saw some of Penang's famous street art on Armenian Street and more on nearby Cannon Street.


(A) I was amazed by the diverse styles of murals. Some of them were quite realistic, while others were artistically simplified or somewhat abstract. 0

(B) A famous mural of two children riding a bicycle, for example, was created by a famous artist named Ernest Zacharevic. My favorite pieces were the mural of a very large cat and another painting of two cute little owls. 2

(C) A few combined actual things such as a bicycle or a cart with paintings on the wall. I was also surprised to learn that these works of art were created not only by local people but also by artists from all parts of the world. 1


① A-C-B ② B-A-C ③ B-C-A ④ C-A-B ⑤ C-B-A

 


24. 영1시한 4-5

 

On the last day of our trip, we visited the Little Penang Street Market, a street market that is held on the last Sunday of each month.


(A) The seller explained that it was traditional artwork made by the Orang Asli, the inhabitants who have lived in Malaysia longer than anyone else. After having lunch at a food stall, we left for the airport. 2

(B) The atmosphere was so lively and exciting that the entire event felt more like a small festival than a market. My parents bought a small wooden sculpture of a monkey. 1

(C) It was one of the reasons why we decided to take this trip near the end of January. The Little Penang Street Market is known as the best place to see local crafts, and it really was full of beautiful ethnic goods, including clothes, shoes, wood carvings, woven baskets, and much more. 0


① A-C-B ② B-A-C ③ B-C-A ④ C-A-B ⑤ C-B-A

 


25. 영1시한 4-6

 

On our way home, I thought that Malaysia and Korea might have more in common than I had originally imagined.


(A) I was deeply impressed by how the Malaysians have embraced their diversity and how it has made its culture richer. 1

(B) Like Korea, Malaysia was occupied by other countries but overcame this tragic history to achieve economic and cultural development. 0

(C) Korea is also becoming an increasingly diverse country. The trip to Penang makes me wonder what Seoul will be like in the future. 2


① A-C-B ② B-A-C ③ B-C-A ④ C-A-B ⑤ C-B-A

 


26. 영1시한 5-1

 

French fries, which are often served with fast foods, are one of the best known American side dishes.


(A) Instead of French fries, the French have traditionally enjoyed roasted potatoes with chicken for their Sunday lunch. Potato pancakes and potato soups are popular traditional dishes in Germany, while fish and chips is one of the most popular street foods in England. 2

(B) The real inventors of the so-called "French" fries were not the French but the Belgians. 1

(C) It is said that Thomas Jefferson, the third President of the United States, tasted these fried potatoes in France and brought the recipe home to America. 0


① A-C-B ② B-A-C ③ B-C-A ④ C-A-B ⑤ C-B-A

 


27. 영1시한 5-2

 

The potato has become such an important part of Western food culture that it is hard to believe that it has had such a relatively short history in Europe.


(A) For this reason, potatoes were most often given to animals, while people ate them only when there was nothing else to eat. The potato gradually but steadily spread across Europe, as people learned how it could help them in times of war, bad weather, and desperate hunger. 2

(B) Most people thought potatoes looked too ugly for people to eat. Some even believed that potatoes were created by devils. 1

(C) The potato has been grown by South American Indians in the Andes Mountains for nearly 10,000 years, but it was only 500 years ago when the Europeans learned about this vegetable and brought it to their homelands. At first, Europeans did not like this unfamiliar and alien food. 0


① A-C-B ② B-A-C ③ B-C-A ④ C-A-B ⑤ C-B-A

 


28. 영1시한 5-3

 

Hunger was a common part of everyday life in 17th- and 18th-century Europe.


(A) Not only did potatoes grow well in much of Europe, but they were also very nutritious. Once Europeans began to grow potatoes, their food supplies doubled in quantity and increased dramatically in quality. Throughout Europe, potatoes quickly became the main source of nutrition for the majority of common people. 1

(B) By the end of the 18th century, potatoes had taken over most European farms, and famine had become a rare and unusual incident. At long last, Europe was able to feed itself. 2

(C) The average European not only often went hungry, but sometimes even starved to death. Many countries suffered nationwide famines at least once every decade. The potato changed all that. 0


① A-C-B ② B-A-C ③ B-C-A ④ C-A-B ⑤ C-B-A

 


29. 영1시한 5-4

 

Thanks to the fact that the potato was inexpensive and nutritious, many Europeans escaped famines, and the populations in European nations increased rapidly during the 19th century.


(A) It was easier to grow potatoes than many other crops, so fewer people were needed for farming. 1

(B) Furthermore, the potato indirectly led to the Industrial Revolution in 19th century England. 0

(C) People who no longer worked on farms came to the cities and became factory workers, enabling new manufacturing industries to grow more quickly. With the larger populations and more advanced technologies, European nations became powerful enough to venture into other parts of the world and colonize them. 2


① A-C-B ② B-A-C ③ B-C-A ④ C-A-B ⑤ C-B-A

 


30. 영1시한 5-5

 

The impact of the potato in Europe was not entirely positive.


(A) Many countries were affected, but Ireland suffered the most. In two months, the disease wiped out almost half of the potato crop in Ireland. 1

(B) It was much more complicated. Because the people of Ireland were excessively dependent upon the potato for food, when a potato disease hit Europe in 1845, it led to a disastrous situation. 0

(C) The situation continued to worsen. By 1852, more than one million Irish people, about 20 percent of the population, had starved to death, while another two million people had fled Ireland, and almost three-quarters of them had migrated to the United States. It was one of the deadliest famines in history. 2


① A-C-B ② B-A-C ③ B-C-A ④ C-A-B ⑤ C-B-A

 


31. 영1시한 5-6

 

Despite the terrible outcome of the potato disease in Europe, it was perhaps less important in the potato's history than the Colorado potato beetle was in North America.


(A) After a series of desperate attempts, one man eventually found that Paris green, a toxic chemical used for making green paint, killed the beetles. Soon farmers were spraying it on their potatoes. Farmers were not the only people to find a new use for Paris green. Chemists saw what the farmers were doing and decided to try Paris green on other pests. 1

(B) In the 1860s, the potato beetle was carried by horses and cows from Mexico to the state of Colorado in the United States. There it attacked potatoes and spread quickly when potatoes were shipped to other parts of the country on trains and steamships. Until that time, American farmers had only planted a few varieties of potatoes, and the potatoes that they planted were extremely vulnerable to the pest. As a result, farmers were practically defenseless against this pest. 0

(C) They then decided to try other chemicals for other problems. Scientists began to do research and discover chemicals and substances that were effective for many different agricultural problems. That was the beginning of the modern pesticide industry. 2


① A-C-B ② B-A-C ③ B-C-A ④ C-A-B ⑤ C-B-A

 


32. 영1시한 5-7

 

Often, the course of history is changed by seemingly insignificant things.


(A) The potato changed Europe first by freeing the European people from hunger. 1

(B) It then went on to prepare the ground for the Industrial Revolution, the rise of the European empires, and new techniques in farming. As a result, this simple vegetable, which was once unwanted and thought to be ugly, has shaped the modern world as we know it. 2

(C) In this case, it was changed by the potato. 0


① A-C-B ② B-A-C ③ B-C-A ④ C-A-B ⑤ C-B-A

 


33. 영1시한 6-1

 

When Jeff Bush, a 37-year-old Florida man, screamed out for help, his brother Jeremy ran into Jeff's bedroom.


(A) I tried everything I could, but I couldn't get him out," Jeremy cried. It was a sinkhole that instantly pulled Jeff into the earth. A sinkhole is a hole that opens up in the ground caused by a collapse of the land surface. 1

(B) Seeing that Jeff and all his furnishings had vanished into the earth, Jeremy desperately tried to rescue him. He didn't stop until the police arrived. "I tried so hard. 0

(C) Sinkholes vary in size from 1 to 600 meters deep. Some sinkholes are large enough to swallow entire buildings. Sinkholes have been around for a long time. It is said that the ancient Maya believed sinkholes were passageways to the world of the dead. 2


① A-C-B ② B-A-C ③ B-C-A ④ C-A-B ⑤ C-B-A

 


34. 영1시한 6-2

 

Unlike the sinkhole in Florida, not all sinkholes are dangerous, destructive, and ruinous.


(A) The Great Blue Hole in Belize is one of the most spectacular; it is 300 meters wide and 124 meters deep. 2

(B) Some sinkholes attract tourists who are interested in the mystery and the beauty of nature. Some notable sinkholes include Sima Humboldt in Venezuela, which is 314 meters deep and Xiaozhai Tiankeng in China, 662 meters deep and 626 meters wide. 0

(C) Sinkholes can form underwater, too. Underwater sinkholes are called blue holes. 1


① A-C-B ② B-A-C ③ B-C-A ④ C-A-B ⑤ C-B-A

 


35. 영1시한 6-3

 

Sinkholes typically occur in areas where the rock beneath the surface soil is limestone, carbonate rock, sandstone, or some other soft rock that is easily worn away by water.


(A) Slowly and continually, sometimes taking hundreds or thousands of years, the water erodes small parts of the rock, creating cracks underground. As the process continues, the soil above is gradually washed into the cracks, and it creates a space in the soil. It is not seen on the surface. 1

(B) These kinds of rocks are found in many parts of Florida, Texas, and Kentucky in the U.S. They are also common in the U.K., Mexico, Belize, Slovenia, Croatia, and China. When rainwater seeps down through surface soil, it eventually reaches bedrock below. 0

(C) As the space becomes too large to support the soil above, it collapses, and a hole opens up on the surface. Though sinkholes form gradually, the final collapse of the surface happens suddenly. 2


① A-C-B ② B-A-C ③ B-C-A ④ C-A-B ⑤ C-B-A

 


36. 영1시한 6-4

 

Rainwater erodes the bedrock, creating cracks underground.


(A) As the space cannot support the soil above, it collapses, and a hole opens up. 2

(B) The space in the soil grows over time. 1

(C) The soil above the cracks is washed into the cracks, creating a space in the soil. 0


① A-C-B ② B-A-C ③ B-C-A ④ C-A-B ⑤ C-B-A

 


37. 영1시한 6-5

 

The most common factor that triggers a collapse is a change in underground water levels.


(A) Most sinkholes we are seeing these days are indirectly related to human activities, such as drilling, mining, or construction. They may also occur because of broken water pipes, heavy weight on soft soil, and the removal of groundwater. Collapses from these man-made causes can have especially dramatic effects because they often occur in the middle of a street or in housing areas. 2

(B) For example, during long periods of drought, underground water levels will fall. When this happens, the space that was once supported by the water may become weaker and collapse. Another trigger is a rapid increase in surface water. 0

(C) A sudden heavy rain can add dramatically to the weight of the surface soil, making it too heavy for the space beneath to bear. This, too, can bring about a collapse. Sometimes the trigger can be man-made. 1


① A-C-B ② B-A-C ③ B-C-A ④ C-A-B ⑤ C-B-A

 


38. 영1시한 6-6

 

Recently, many sinkholes have opened up in urban areas around the world, sometimes resulting in tragic accidents.


(A) In 2010, in Guatemala City, an area approximately 20 meters wide and 30 meters deep collapsed, swallowing a three story factory and killing 15 people. The sinkhole was caused by the increase of surface water due to tropical storm Agatha and leakage from a local waste water pipe. 0

(B) Sinkholes caused by human carelessness can often be prevented. Thorough soil testing prior to construction and maintaining the underground water and plumbing systems are the best ways to prevent the formation of a sinkhole in the middle of a city. 2

(C) Generally speaking, naturally occurring sinkholes cannot be prevented. However, if you live in an area where sinkholes have happened before, look around for possible warnings, such as leaning trees, doors or windows that don't close properly, or rainwater collecting in unusual spots. 1


① A-C-B ② B-A-C ③ B-C-A ④ C-A-B ⑤ C-B-A

 


39. 영1시한 6-7

 

Sinkholes are not widespread events like hurricanes or earthquakes.


(A) However, this does not mean that we should ignore the possibility of sinkholes. 1

(B) As our population grows and we develop in areas where sinkholes are common, we seem to be seeing more sinkholes and more stories about humans being affected by them. If we look for ways to detect and prevent sinkholes, it may be easier to rescue another Jeff from a sinkhole somewhere else in the world. 2

(C) They are very localized. 0


① A-C-B ② B-A-C ③ B-C-A ④ C-A-B ⑤ C-B-A

 


40. 영1시한 SP1

 

Provide Access to Clean Water When Samuel Taylor Coleridge wrote "water, water, everywhere, nor any drop to drink," he did not have the 21st century's global water situation in mind.


(A) But he wasn't far from correct about the present world's water situation.0

(B) 2

(C) Today, the availability of water for drinking and other uses is a critical problem in many areas of the world. 1


① A-C-B ② B-A-C ③ B-C-A ④ C-A-B ⑤ C-B-A

 


41. 영1시한 SP2

 

How serious is our water challenge?


(A) If sufficient water for sanitation were available, these children would be saved. Is water really scarce? Earth is sometimes called "the Blue Planet" because it is covered by so much water. In fact, Earth does contain over a billion trillion liters of water. However, very little of that water is safe to drink.1

(B) Over 97% of the water on Earth is salt water. Of the fresh water that remains, over two thirds is locked away in ice caps and glaciers. Most of the rest is trapped in the soil or underground. But that amount is enough for now. 2

(C) Nowadays, the lack of clean water causes more deaths than war. One out of every six people living today does not have adequate access to water, and more than double that number live in unhealthy environments because they lack the water they need for cleaning. In some countries, half the population does not have access to safe drinking water. As a result, people living in these countries have many health problems. Nearly 5,000 children globally die every day from diseases caused by the lack of clean water. 0


① A-C-B ② B-A-C ③ B-C-A ④ C-A-B ⑤ C-B-A

 


42. 영1시한 SP3

 

Then why is the water crisis so severe?


(A) Even within specific countries, such as Brazil, some regions have more than enough fresh, unpolluted water while other regions often suffer from droughts. And it is not just drinking water that is needed. Water for drinking and personal use is only a small portion of society's total water needs — household water usually accounts for less than 5 percent of total water use. 1

(B) Because groundwater, mostly deposited in earlier times, is being exhausted far faster than it is being replaced. Moreover, it is not always located where it is needed. For example, Canada has far more water than its people need, while the Middle East, North Africa, and many other regions are always in want of water. 0

(C) Most of the fresh water we use is for agriculture and industrial activities. In addition, water is also inevitably needed for ecological processes not directly related to human use. Overcoming the crisis in water and sanitation is one of the toughest human challenges of the early 21st century. 2


① A-C-B ② B-A-C ③ B-C-A ④ C-A-B ⑤ C-B-A

 


43. 영1시한 SP4

 

So what solutions are out there?


(A) Another approach that can be applied is desalination. Desalination is extracting the salt from seawater. Desalination is not a new idea and is already used in many regions, particularly in the Middle East. Saudi Arabia produces about one tenth of the water produced by desalination in the world. 1

(B) Israel uses desalination technology to provide about a fourth of its domestic water needs. More than 12,000 desalination plants are now in operation in the world. But desalination plants are expensive to build and require lots of energy to operate, which makes desalination suitable mainly for seaside cities in rich countries. It therefore has limited value for poor countries, where water supply problems are the most serious. If desalination is going to be a realistic source of fresh water, countries need either abundant, cheap energy or a way to make the process more efficient. 2

(C) Today, we need more sophisticated and controlling water supply methods that do not only meet our increased needs but also prevent damage to the environment and ecosystems. One large-scale approach used in the U.S., China, India, and other countries has been to divert the flow of water from regions where it is plentiful to where it is scarce. Such diversion projects provide some short-term relief for cities, but they are not practical as widespread or long-term solutions and have ecologically damaging effects. Another problem is that diverting water to some people often means less for others, and this can become an explosive political issue potentially. 0


① A-C-B ② B-A-C ③ B-C-A ④ C-A-B ⑤ C-B-A

 


44. 영1시한 SP5

 

A different technological approach is used for irrigation systems.


(A) However, this is not a perfect solution because it does not provide enough water to cleanse the soil. Technologies are being developed, for instance, to improve recycling of waste water so that it can be used for irrigation or industrial purposes. Hopefully, recycled water may someday even be returned to groundwater. However, very effective purification methods and thorough safeguards are necessary to ensure the safety of recycled water. 2

(B) Agricultural irrigation consumes enormous quantities of water;. In developing countries, irrigation often exceeds 80 percent of total water use. One of the earliest and most widespread forms is flood irrigation. It involves pumping or diverting water to fields so that water flows along the ground among the crops. 0

(C) It is simple and cheap, but also inefficient and wasteful. Improved technologies such as "drip irrigation" can drip water directly onto plants. It can provide crops with water more efficiently and significantly reduce agricultural water demand. Already some countries, such as Jordan, have dramatically reduced water use with drip technology. 1


① A-C-B ② B-A-C ③ B-C-A ④ C-A-B ⑤ C-B-A

 


45. 영1시한 SP6

 

Above are large-scale solutions for providing a huge amount of water.


(A) Traditional approaches to water use and conservation cannot solve the water crises of the future. We must, as Einstein points out, do something different. The world's water problems can only be solved by better technologies and more imaginative innovations. 2

(B) However, a small-scale process has also been used to improve water availability and safety. It is a small distillation unit, which is a particularly attractive approach in rural and low-income areas, where the lack of infrastructure makes it hard to distribute water efficiently and economically. Some current projects have produced inexpensive distillation units that can remove toxic elements from any water source. A unit smaller than a dishwasher can provide daily clean water for 100 people. 0

(C) Today, the earth's population is seven billion. By 2050, the global population is expected to rise to nine billion. As a result, there have been significant calls for a change in the way the world is dealing with growing water shortages. Albert Einstein once described madness as "doing the same thing over and over again and expecting different results." 1


① A-C-B ② B-A-C ③ B-C-A ④ C-A-B ⑤ C-B-A

 


[ANSWER]
1. ④ 2. ③ 3. ② 4. ② 5. ② 6. ⑤ 7. ① 8. ⑤ 9. ③ 10. ①


11. ③ 12. ④ 13. ① 14. ① 15. ② 16. ③ 17. ④ 18. ② 19. ⑤ 20. ①


21. ② 22. ④ 23. ① 24. ⑤ 25. ② 26. ⑤ 27. ⑤ 28. ④ 29. ② 30. ②


31. ② 32. ④ 33. ② 34. ③ 35. ② 36. ⑤ 37. ③ 38. ① 39. ④ 40. ①


41. ④ 42. ② 43. ④ 44. ③ 45. ③ 

728x90
반응형